Download as pdf or txt
Download as pdf or txt
You are on page 1of 214

• Invention Related Scientist

(a) Origin of friction electricity In 600 B.C. (Thales)


(b) The word charge was given by Dr. William Gillbert
(c) Two kinds of charges Du Fay.
(d) Algebraic sign to charge Benjamin Frankline
(e) Like charges repels and unlike attracts Dr. William Gillbert
(f) Numerical value of force between two charges Coulomb by (torsion balance experiment)
(g) Methods of charging Thomos Brown
(h) Amount of induced charge Faraday ice pail experiment
(i) e/m of electron J.J. Thomson (1887)
(j) Charge and mass (indirectly) of electron and quanta of charge. R.A. millikan (by oil drop exp.)
(k) Concept of line of force M. Faraday

• Order of size of atom and nucleus are 10–10 m. (Å) and • Amount of induced charge inducing charge, equality
10–15 m. (Fermi) respectively. Order of density of nucleus holds only for metal.
= 1017 kg/m3.
• If a body is charged either positive or negative, its volume
• Classical radius of electron ~ _ 2.8 × 10–15 m.
increases and density decreases.
• Millikan calculate quanta of charge by 'Highest common
• Mass, length and time depends on state of rest or motion
factor' (H.C.F.) method and it is = charge of electron.
(Theory of relativity) while charge (also phase) not
• Smallest unit of charge is Frankline and largest is Faraday
depends.
1 C = 3 × 109 St. C, 1 Ab C = 10 C, 1 Faraday = 96500 C.
• If quark particles are exist in nature, quantization of charge
• Charge is invariant (it does not depends on speed).
• Attraction may take place between two likely charged e
objects, generally it happens when difference in their still valid, but new quanta changes to .
3
charges is so large.
COULOMB LAWAND FACTORS AFFECTING IT
• Gold leaf electroscope (G.L.E.)
• Electric force between two charges not depends on
neighbouring charges.
• If a dielectric slab (er) of thickness 't' is placed between
two charges (distance d), force decreases.

Q1Q 2
F= 2
where r = d – t + t r
4 0r

(a) Any charged body (+ or –) come in closed to uncharged • When two charges (Q1, Q2) are placed some distance
GLE, leaves diverges. apart. Neutral point is nearer to smaller charge and in
(b) If a charged body come in close to charged GLE, leaves between Q1 and Q2 if charges are like and away from
diverges if like object and converges if unlike object. charge if charges are unlike.
(c) If X–ray incident on GLE, leaves always collapses, • System of following charges, is in equilibrium, if
whatever be the charge on leaves.
Q

a a
Q Q1 Q Q Q
(a) Q1 (b) Q2 Q2
a a 4 3
Q a Q

• For a three charge system it is not possible that all charges are in stable equilibrium.
• Coulombs law is similar to newton's gravitation law.
q
• If Fg = Fe for two identical charges then 4 0G
m
ELECTRIC FIELD
• For measuring E practically a test charge (+ve) of magnitude much less than the source charge should be used.
• Electric force on a charge in uniform E is constant and hence acceleration is constant, so equations of motion can be used
qE
(acceleration a )
m
• Electric field due to linear charge distribution
(a) Finite wire (b) Infinite wire ( = 180°) (c) Charged arc

+ +
+ + R
+ +
M+ P + P O
+ R + R
+ +
+

2k 2k 2k
Ep = sin Ep = EP = sin
R 2 R R 2
(d) Charged ring of radius R

Q kQ
x >> R EP =
O P at an axial point E = kQx x2
2 3/2 ; if
x P 2
(R +x ) kqx
R x << R EP =
R3

R
• As x increases : E due to ring first then and at x it is maximum.
2
2k 1 2
• Force between two parallel wires is F (on unit length) =
d
• A small metal ball is suspended in a uniform E . If X–ray beam falls on it, The ball will be deflected in the direction of E .
• If both E and g (gravity) presents, then use resultant acceleration.
qE
(a) FE and Fg same direction a g
m
qE
(b) FE and Fg are opposite a g
m
2
qE
(c) FE and Fg are perpendicular a2 = g2 + m

L
e.g. Time period of simple pendulum T 2 (a = net acceleration)
a
• If identical charges are placed on each vertices of a regular polygon, then E at centre = zero.
• Strength of E [Separation between electric lines]–1
Electric lines of electrostatic field have following properties
(i)
Imaginary
(ii)
Can never cross each other
(iii)
Can never be closed loops
(iv)The number of lines originating or terminating on a charge is proportional to the magnitude of charge. In rationalised MKS
system (1/ o) electric lines are assoicated with unit charge, so if a body encloses a charge q, total lines of force associated
with it (called flux) will be q/ o.
(v) Total lines of force may be fractional as lines of force are imaginary.

+ A B–

qA>qB

(vi) Lines of force ends or starts normally at the surface of a conductor.


(vii) If there is no electric field there will be no lines of force.
(viii) Lines of force per unit area normal to the area at a point represents magnitude of intensity, crowded lines represent strong
field while distant weak field.
(ix) Tangent to the line of force at a point in an electric field gives the direction of intensity. So a positive charge free to move
follow the line of force.

• Lines of force starts from (+ve) charge and ends on (–ve) charge.
• Lines of force starts and ends normally on the surface of a conductor.

+ + – –
+
+ – E=0
E=0 + –
+ –
–– – – – – + –
fixed point charge near
infinite metal plate edge effect

• Lines of force do not exist inside a conductor (as field inside a conductor is zero) the field between the plates is as shown.

Q –Q' Q' –Q Q –Q' Q' –Q


+ – + – + – + –
+ – + – + – r + –
+ – E=0 + – + – + – Q'=Q 1– 1
r
+ – + – + – E<E0 + –
+ – + – + – + –
metal dielectric

• The lines of force never intersect each other due to superposition principle.
• The property that electric lines of force contract longitudinally leads to explain attraction between opposite charges.
• The property that electric lines of force exert lateral pressure on each other leads to explain repulsion between like charges.
for open surface 0 = d = E.ds

c = o E.ds (by defination of) flux


for closed surface
q net
c = or 4 kq net (Gauss's law )
0

c = E.S = E S cos

S = always normal to surface and pointed out wards.


Electric flux :
(i) It is a real scalar quantity
(ii) Units (V–m) and N – m2/C Dimensions : [ML3T–3A–1]
(iii) The value of does not depend upon the distribution of charges and the distance between them inside the closed surface.
The value of is zero in the following circumstances :
(a) If a dipole is (or many dipoles are) enclosed by a closed surface
(b) Magnitude of (+ve) and (–ve) charges are equal inside a closed surface
(c) If no charge is enclosed by the closed surface
(d) Incoming flux (– ve) = out going flux (+ve)

ds
E
E
R in
= – R2 E and = R2 E
out total
=0
ds ds
E

E
R
in
= fcircular = – R2 E and out
= curved
= R2 E r total
=0
E

y
ds E

a
ds
ds
in
= –a2 E and out
= a2 E total
=0
x
a
a
z

E 1 1
in – R2 E and out R2 E total
=0
2 2
R

E
kq q q
E and 2 R2 2
R 2 4 0R
2 0
R q
Note : here electric field is radial

q q q q
= =
Total
0
hemisphere 2 0

q
q
q q
Total
= cylinder
=
0 R 2 0
R

q q q q
= =
Total
0
cube 2 0

q q
Total
= = 8
0 0
q
q

q q
q q
qq Total
= = 4
0 0
GAUSS THEOREM
1
The total flux linked with a closed surface is times the charge enclosed by the closed surface (gaussian surface) i.e.
0
q
E.ds
0
This law is suitable for symmertrical charge distribution and valid for all vector fields obeying inverse square law.

(i) Is imaginary surface


(ii) Is spherical for a point charge, conducting and non - conducting spheres
(iii) Is cylindrical for infinite sheet of charge, conducting charge surfaces, infinite line of charges, charged cylindrical conductors
etc.
We select a Gaussian surface such
that determination of E.ds can be
done in symplest way (by symmetry)

• No. of electric lines (ELF) related to 1 C (unit charge) = 1/ 0.


• Electric lines contracts longitudinally and repels laterally.
Flux ( )

If enclosed charge (qen) is given then If external E is given then use


qen
use = E . ds
0
• For an imaginary cube

Position of charge Main centre Face centre Side center Corner


q q q q
flux from cube 2 4 8
• Gauss theorem is applicable to all conservative field obeys inverse square law and can be derived from coulombs law. E
calculated from it depends on charges inside and outside the closed surface.

Charge distribution Point charge, sphere Wire, cylinder Infinite plates


•
Gaussion surface Spherical Cylindrical Cylindrical

• E inside a conducting (solid, shell) sphere or cylinder is zero and outside, it is same as of nonconducting.
• If temperature increases r
decreases. (as Eind and r
= E0/(E0 – Eind) ; where E0 = main E , E ind = induced E )
•
E due to nonconducting
Sphere Cylinder
Shell Solid Solid Hollow

kQr Ein r 2k r
Ein = 0 Ein = Ein = Ein = 0
R3 R
2

kQ kQ 2k 2k
Es = Es = 2 Es = Es =
R2 R R R
kQ kQ k 2k
Eout = Eout = Eout = Eout =
r2 r2 r r
E E E E

r r r r
x=R x=R x=R x=R
• For a dipole, potential is zero at equator, while at any finite point E 0
• In a uniform E , dipole may feal a torque but not a force.
• If a dipole placed in a field generated by a point charge, then torque on dipole may be zero.
•
Distribution Point charge Dipole Quadrapole Octapole
Potential proportional to r–1 r–2 r–3 r–4
E proportional to r–2 r–3 r–4 r–5

•
Force between Point charge Dipole andpoint charge Dipole-dipole
–2 –3
Proportional to r r r–4

So, resultant electric field due to one element = dE cos


So, electric field due to whole ring -
dq
Suppose a ring of radius a have positive charge q. This charge E dE cos or E 2
cos
4 0r
q is uniformly distributed over its circumference.
q x
q Charge [ dq d and cos ]
2a r
Linear charge density =
2 a Length or
q 1 x q x
E dl. 3
. . 3
2 a 4 0r r 2 a 4 0r
d
q x qx
d . .2 a
r 2 a 4 0r3 4 0r
3

dE sin dE qx
E [
4 2
x2 )3 2 r a2 x2 ]
0 (a
x P 2dE cos
dE sin dE

For outside point (r > R)


d
Using Gauss's theorem
q
E.ds
0
Let a very small element AB of length dl have charge
E
q dA
dq = dl = d + P
2 a
r
Electric field intensity due to this charge (dq) at P,

dq O R
dE 2
4 0r

An another element of length d but dimetrically opposite also +

produce electric field intensity dE.


The vertical components will cancel each other and horizontal Gaussian surface
components will be added. So net electric field at point P, due
at every point on the gaussian surface E ds
to whole ring (take small elements at different place on ring)
can be calculated by adding all horizontal components. E.ds = E ds cos 0° = E ds
q Gaussian surface of radius r
E.ds [E is const. over the gaussian surface] q q
0
E(4 r 2 ) E .............(i)
q q 0 4 r2 0
E 4 r2 Ep = 2
0 4 0r As the sphere is uniformly charged, the volume charge density
For surface point r = R (charge/volume) is constant throughout the sphere
q q
ES charge enclosed in gaussian surface
4 0 R2 4
R3
For Inside point (r < R) 3
4 3 q 4 3
Because charge inside the conducting sphere or hollow is zero. q . r = .
3 3
r
3 (4 3 ) R
(i.e. q = 0) So
q qr 3
E .ds or q put this value in equation (i)
=0 Ein = 0 R3
0
1 qr
E in
4 0 R3
Outside (r > R)
From Gauss's theorem
q Let a wire of infinte length is uniformly charged having a
E.ds constant linear charge density .
s 0
P is the point where electric field is to be calculated.
q q Let us draw a coaxial Gaussian cylendrical surfaces of
E 4 r2 EP 2
0 4 0r length .
From Gauss's theorem
E
P ds q
E .dS1 E .dS 2 E .dS 3
0
r s s s

O
R E dS1 so E.dS1 0 and E dS 2 so E.dS 2 0

ds1

Gaussian surface Gaussian


E + surface
+
At surface (r = R) : +
+
+
q +
ES Put r = R +
ds3
4 0 R2 +
+ E
+
Inside (r < R) : +
+
+
From Gauss's theorem
E
q
E.ds ds2
s 0
Where Sq charge contained with in
q
E 2 r [ E dS3 ]
E
0
P dS
r charge enclosed in the Gaussian surface q .
O R
2k
so E 2 r E E
0 2 0r
r

1
where k =
Gaussian surface 4 0
At a point on equatorial line of dipole :
kq
Suppose at any instant, the dipole makes an angle with the Electric field due to +q charge E1
x2
electric field. The torque acting on dipole. d 2 . sin
E1 sin
= qEd = (q 2 sin )E = pE sin E1
The work done in rotating dipole from 1
to 2

2 2
E P
W d pE sin d
1 1
E2
r
B qE x x
+q
2 d

qE qA C E
–q q
O
W = pE (cos 1
– cos 2) = U2 – U1 ( U = – pE cos )

kq
At a point on the axis of a dipole : Electric field due to –q charge E2
x2
kq Vertical component of E1 and E2 will cancel each other and
Electric field at P due to +q charge E1 =
(r )2 horizontal components will be added
–q O q P So net electric field at P
E2 E1 E = E1cos + E2cos [ E1 = E2]
r
2 kq
E = 2E1 cos = cos
kq x2
Electric field at P due to –q charge E2 =
(r )2
Net electric field at P E = E1 – E2 [ cos and x r2 2 ]
x
kq kq
E= 2 2kq 2kq kp
(r ) (r )2 E E
3 2 2 3 2 or 2 2 3 2
kq 4 r x (r ) (r )
E= 2 2 2 [ p = q × 2 = Dipole moment]
(r )
kp
2 kpr If r > > > then E
E= 2 2 2 r3
(r )
2 kp
If r >>> E= 3
r
1. Two small balls having equal charges Q, are suspended
1920
from a hook with two insulating threads each of length L. = Coul.
127
This arrangement is carried in the space, where there is no
gravitation. The tension in each string will be- Separation between pieces = 1 cm = 10–2 m
One piece of copper has positive charge and the other
negative charge, therefore force of attraction between
F T T F
A 180° B
1 q1q 2
the pieces =
L L 4 0 r2

kQ 2 1920 1920
(A) (B) 0 9 10 9
4L2 127 127
= 2 2
2
(10 )
kQ kQ
(C) (D) = 2.057 × 1016N
4L2 L2
3. A thin conducting ring of radius r has an electric charge
kQ 2 +Q. What would be the increase in the tension of wire, if
Sol. (A) F = (electrostatic force acting between two a point charge +q is placed at the centre of the ring?
4L2
charged balls) Q
Sol. Charge on a small element dl of the ring dQ = d
2 r
kQ 2
F=T= Outward electric force on this element
4L2
Note :-In the above problem, the angle between the 1 Qd q
'threads will be 180°.' Because in the absence of gravity, Fe =
4 0 2 r r2
the tension in the threads will be only due to columb-
repulsion. Therefore the angle between the threads will Let the tension be increased by T, to balance this force Fe.
be 180°.
Fe
2. A copper atom consists of copper nucleus surrounded
by 29 electrons. The atomic weight of copper is T
d
63.5 g/mole. Let us now take two pieces of copper each d
wieghing 10g. Let us transfer one electron from one piece +Q
d +q O Charge
to another for every 1000 atoms in that piece. What will be
2 on ring
the coulomb force between the two pieces after the transfer
of electron if they are 1cm. apart.
[Avogadro number N = 6 × 1023 /g. mole charge on an T
electron = –1.6 × 10–19 coulomb] The increase in tension is given by
Sol. 63.5 g copper contains N = 6 × 1023 copper atoms. = 2T sin (d /2) = 2T d /2 T d /r
Therefore number of copper atoms in 10g copper
Arc
23
( d = rd , Angle = )
6 10 radius
= × 10
63.5
d 1 Qd q Qq
As only one electron is transferred for every 1000 atoms, Hence T T=
r 4 0 2 r r 2 8 2 0r2
therefore the number of electron transferred
4. (a) Two similar point charges q1 and q2 are placed at a
6 10 23 10 distance 'r' apart in air. If a dielectric slab of thickness 't'
n= ,
63.5 1000 (< r) and dielectric constant 'K' is placed between the
Magnitude of charge charges, calculate the coulomb force of repulsion (b). If
the thickness of slab covers half the distance, between
6 10 23 10 the charges, the coulomb repulsive force is reduced in the
q = ne = × 1.6 × 10–19 Coul. ratio 9 : 4, calculate the dielectric constant of slab.
63.5 1000
Sol. (a) The repulsive force between the charges in air is
1 q1q 2
F0 = 4 ....(A) E
0 r2
If the space is completely filled with medium of dielectric Tcos
constant K, the repulsive force becomes T
Tsin qE
1 q1q 2
F= 4 ....(B) mg
0 Kr 2
Now let us suppose that the repulsive force is F in air if qE 39.2 10 10 20 10 3
separation between charges becomes r', so tan = =
mg 8 10 6 9.8
1 q1q 2 tan = 1
F= 4 ...(C)
0 r12 = 450
6. A simple pendulum is oscillating between the plates of a
Equating (B) & (C) r1' = r k capacitor as shown in the fig. If the bob and the capacitor
This gives equivalent air separation due to presence of both are charged, what will be the effect on the time-period
dielectric of dielectric constant 'K' and thickness r. If there of the pendulum?
exists a slab of thickness t and dielectric constant K, the S
effective air separation between the charges will be (r – t)
+t k . Hence required repulsive force between charges
1 q1q 2
F= 4 2
0 (r t t k )
(b) Substituting t = r/2
1 q1q 2
Then F = 4 2 O
0 r r
r k Sol. Suppose the mass of the bob is m and the length of its
2 2
thread is . When the bob and the capacitor both are
1 q1q 2
= 4
0 (1 k )2 r 2 uncharged, then time period is given by T = 2p
g
.

q1q 2 Suppose a charge +q is given to the bob. On charging the


1 capacitor, the equilibrium position of the bob will change
Then force in air will be Fair = 4 r2
0 from O to O and thread of the pendulum will now make an
qE
F 4 angle with the vertical, such that tan = where qE is
= mg
Fair (1 k )2 the electric force and mg is the gravitational force. On
displacing the bob from the position O', it will oscillate
F 4 un der the effective acceleration g', while mg'
Given F =
air 9 = (mg)2 (qE) 2
K=4
5. The bob of a pendulum carries an electric charge of
39.2 × 10–10 coulomb in an electric field of 20 × 103 v/m
and it is at rest. The angle made by the pendulum with the
vertical will be, if the mass of pendulum is 8 × 10–6 kg and
g = 9.8-
(A) 27º
(B) 45º
(C) 87º
(D) 127º
Sol. (B) T sin = qE
T cos = mg
2 Sol. (A) Force on the charge placed at B, due to charges at A,
2 qE
g' = g
m kq 2
C, & D are F1 = ,
a2
Hence the new time period of the pendulum is T = 2 .
g kq 2
kq 2
F2 = & F3 = respectively..
a2 2a 2
=2
[g 2 (qE / m)2 ]1/ 4
[Note BD = 2 a]
since g' > g hence T < T. i.e. time period of the pendulum
will decrease
7. A simple pendulum (length , mass of the bob m) is
suspended between the parallel plates of a capacitor as
shown in the fig. What will be the effect on its time if it is
(i) positively charged, (ii) negatively-charged? Assuming
that the electric force on the bob is less than the
gravitational force and the electric field E between the
plates is uniform.
Sol. When both the bob and the capacitor are uncharged, then Resultant of F1 & F2, F12

T=2
g kq 2
= F12 F22 2F1F2 cos 90º = 2
Time-period in case (i) is a2
F12 || F3

T1 = 2 qE [ F= 2
F12 F32 2F12 .F3 cos 0 = F12 + F3]
g
m
Resultant force on the charge placed at B
and in cae (ii) it is
1 kq 2
F = F12 + F3 = (1 + 2 2 )
qE 2 a2
T2 = 2 g
m
If resultant makes an angle of with F1
Clearly, in case (i) the time period decreases (T1 < T) and
in case (ii) it in cases (T2 > T) F1 sin
tan = F F2 cos
8. A square of side 'a' has equal charge 'q' at its corners. 1
The magnitude and direction of force at B will be-
= 45º
9. Two identical charged spheres are suspended in air by
strings of equal lengths and string make an angle of 30°
with each other. When suspended in a liquid of density
0.8 gm cm–3, the angle remains the same. What is the
dielectric constant of the liquid? (Density of the material
of the spheres = 1.6 gm cm–3)
Sol. Suppose the mass of each sphere is m kg, the distance
between them is r meter. Each sphere is in equilibrium
1 kq 2
(A) (1 + 2 2 ), 45º under the action of three forces (i) weight of the
2 a2
sphere = mg
1 k 2q 2 1 q2
(B) , 45º (ii) electrical force of repulsion F = and
2 a2 4 0 r2
(iii) tension T of the string. Resolving these forces in
4kq 2
(C) , 30º vertical and horizontal components, we have
a2
T cos 15 = mg and
(D) none of these
1 q2 2ym
T sin 15° = F = 2
t= = 3 × 10–9 s
4 0 r e.E
[Putting y = 4 × 10–2 m, m = 9.1 × 10–31 kg, e = 1.6 × 10–19C,
2
1 2q E = 5 × 104 N/C]
tan 15° = 2 ....(1)
4 K
0 mgr 11. In the following fig. a unit positive charge moves along
the path ABC in an electric field E. The potential difference
On immersing in the liquid the (effective) weight of each
between A & C will be
sphere and the force of repulsion both decrease and hence
the tension also decreases. The angle is still 30°. In the (A) 0 (B) Er
liquid, we have (C) Er/2 (D) Er/4
C B
45º

45º
A
E
Sol. (B) Because the work done by the charge does not depend
upon the path
WAC= q(VAC) = qEr ( V = W/q)
0.8 mg
(i) weight of the sphere = mg 1 = = Er ( q = 1C)
1.6 2
12.Charge 2q, –q & –q lies at the vertices of a equilateral. The
value of E and V at the centroid of the triangle will be-
1 q2
(ii) electrical force of repulsion F' = , (A) E 0 and V 0
4 0 r2
(B) E = 0 and V = 0
where K is dielectric constant of the liquid, and
(C) E 0 and V = 0
(iii) tension T (say). Now T cos 150 = mg/2and T sin 150
(D) E = 0 and V ¹ 0
1 q2 Sol. (C)
=
4 0K r2

1 2q 2
tan 150 = ....(2)
4 0K mgr 2
From eq. (1) and (2) we have

1 q2 1 2q 2
= K=2
4 0 mgr 2 4 0K mgr 2
V = V1 + V2 + V3
10. An electron falls a distance of 4 cm in a uniform electric
field of magnitude 5 x 104 N/C. The time taken by electron 2q q q
in falling will be- =k =0
a a a
(A) 2.99 × 10–7 s (B) 2.99 × 10–8 s
(C) 2.99 × 10–9 s (D) 2.99 × 10–10 s E = E1 + (E2 cos 60° + E2 cos 60°]
= E1 + E2
1 2 1 eE 2 Fe eE E 0
Sol. (C) y = at = t [ a= = ]
2 2 m m m
13. From the fig. given below, the potential energy of the 15. Two identical particles of mass m carry a charge Q each.
system will be- Initially one is at rest on a smooth horizontal plane and
the other is projected along the plane directly towards the
first particle from a large distance, with the speed v. Find
the closest distance of approach-
Sol. The masses are identical, each of mass m. Therefore we
cannot treat any particle too heavier and hence at rest
throughout. An external force on the system is zero.
Accordin g to principle of conservation of linear
momentum.
kq 2 kq2 Initial momentum = Final momentum
(A) 2 4 (B)
a a mv = mv1 + mv2
kq v = v1 + v2
(C) 0 (D) 2 4
a Where v1 and v2 are velocities of first and second particles
Sol. (A) U = U12 + U23 + U34 + U41 + U13 + U24 as the particles are identical, so by symmetry at nearest
approach
q( q) ( q)(q) (q)( q)
=k v
a a a v1 = v2 =
2
( q)(q) (q)(q) ( q)( q) This means that to conserve momentum both particles
kq2
= ( 2 4)
a a 2 a 2 a v
travel with velocity along the same straight line. If d is
14. Two particles A and B having masses equal and charges q 2
and 4q. If these are accelerated from rest through same the distance of nearest approach, then initial energy of
potential difference, then what will be the ratio in their system = Final energy at nearest approach
speeds? 2 2
1 1 v 1 v 1 (Q)(Q)
(A) 4 : 1 (B) 1 : 4 mv2 = m + m + 4
2 2 2 2 2 0 d
(C) 2 : 1 (D) 1 : 2
1
Sol. (D) E = mv2 = q × V 1 1 Q2
2 mv2 =
4 4 0 d
2qV
v= Distance of nearest approach,
m
VA q 1 1 4Q 2
v q d=
VB = 4q
=
2 4 0 mv 2
1. An arbitrary surface encloses a dipole. What is the electric charges were concentrated in two point charges separated
flux through this surface? by (i) 1 cm (~ 1 / 2 diagonal of the one paisa coin ), (ii) 100
2. A metallic spherical shell has an inner radius R1 and outer m (~ length of a long building), and (iii) 106 m (radius of
radius R2 . A charge Q is placed at the centre of the spherical the earth). Find the force on each such point charge in
cavity. What will be surface charge density on (i) the inner each of the three cases. What do you conclude from these
surface, and (ii) the outer surface? results?
3. The dimensions of an atom are of the order of an 9. Fig. represents a crystal unit of cesium chloride, CsCl.
Angstrom. Thus there must be large electric fields between The cesium atoms, represented by open circles are situated
the protons and electrons. Why, then is the electrostatic at the corners of a cube of side 0.40nm, whereas a Cl atom
field inside a conductor zero? is situated at the centre of the cube. The Cs atoms are
4. If the total charge enclosed by a surface is zero, does it deficient in one electron while the Cl atom carries an excess
imply that the electric field everywhere on the surface is electron.
zero? Conversely, if the electric field everywhere on a
surface is zero, does it imply that net charge inside is zero.
5. Sketch the electric field lines for a uniformly charged hollow
cylinder shown in Fig .

0.40 nm
+ – 1.23
Cs Cl

6. What will be the total flux through the faces of the cube (i) What is the net electric field on the Cl atom due to
(Fig) with side of length a if a charge q is placed at eight Cs atoms?
(ii) Suppose that the Cs atom at the corner A is missing.
What is the net force now on the Cl atom due to
seven remaining Cs atoms?
C 10. Two charges q and –3q are placed fixed on x-axis separated
D by distance ‘d’. Where should a third charge 2q be placed
such that it will not experience any force?
B 11. Fig. shows the electric field lines around three point
A charges A, B and C.

(a) A: a corner of the cube. B


(b) B: mid-point of an edge of the cube.
(c) C: centre of a face of the cube.
(d) D: mid-point of B and C.
A
7. A paisa coin is made up of Al-Mg alloy and weighs 0.75g.
It has a square shape and its diagonal measures 17 mm. It C
is electrically neutral and contains equal amounts of
positive and negative charges. Treating the paisa coins
made up of only Al, find the magnitude of equal number (a) Which charges are positive?
of positive and negative charges. What conclusion do (b) Which charge has the largest magnitude? Why?
you draw from this magnitude?
(c) In which region or regions of the picture could the
8. Consider a coin of Example 1.20. It is electrically neutral electric field be zero? Justify your answer.
and contains equal amounts of positive and negative
(i) near A, (ii) near B, (iii) near C, (iv) nowhere.
charge of magnitude 34.8 kC. Suppose that these equal
12. Five charges, q each are placed at the corners of a regular (b) Suppose the total charge on the sphere is 2e where e
pentagon of side ‘a’ (Fig.) is the electron charge. Where can two protons be
Aq embedded such that the force on each of them is
zero. Assume that the introduction of the proton does
not alter the negative charge distribution.
E B 15. Two fixed, identical conducting plates ( & ) , each of
q q
O surface area S are charged to –Q and q, respectively, where
Q > q > 0. A third identical plate ( ), free to move is
r located on the other side of the plate with charge q at a
distance d (Fig 1.13). The third plate is released and collides
Dq qC with the plate . Assume the collision is elastic and the
a time of collision is sufficient to redistribute charge g
(a) (i) What will be the electric field at O, the centre of the amongst & .
pentagon?
(ii) What will be the electric field at O if the charge from y
one of the corners (say A) is removed?
(iii) What will be the electric field at O if the charge q at A
is replaced by –q? (b) How would your answer to (a) x
be affected if pentagon is replaced by n-sided regular
polygon with charge q at each of its corners?
13. In 1959 Lyttleton and Bondi suggested that the expansion
of the Universe could be explained if matter carried a net
charge. Suppose that the Universe is made up of hydrogen
atoms with a number density N, which is maintained a
constant. Let the charge on the proton be: e = – (1 + y)e
where e is the electronic charge. d
(a) Find the critical value of y such that expansion may –Q q Q
start.
(b) Show that the velocity of expansion is proportional (a) Find the electric field acting on the plate before
to the distance from the centre. collision.
14. Consider a sphere of radius R with charge density (b) Find the charges on and after the collision.
distributed as ( r ) = kr for r R = 0 for r > R . (c) Find the velocity of the plate after the collision and
(a) Find the electric field at all points r. at a distance d from the plate .
1. The ratio of the forces between two small spheres with 8. A total charge Q is broken in two parts Q1 and Q2 and
constant charge (a) in air (b) in a medium of dielectric they are placed at a distance R from each other. The
constant K is maximum force of repulsion between them will occur, when
(A) 1 : K (B) K : 1 Q Q
(C) 1 : K2 (D) K2 : 1 (A) Q2 , Q1 Q
R R
2. A soap bubble is given a negative charge, then its radius Q 2Q
(A) Decreases (B) Q2 , Q1 Q
4 3
(B) Increases Q 3Q
(C) Remains unchanged (C) Q2 , Q1
4 4
(D) Nothing can be predicted as information is insufficient Q Q
3. Four charges are arranged at the corners of a square (D) Q1 , Q2
2 2
ABCD, as shown in the adjoining figure. The force on the 9. Three charges 4q, Q and q are in a straight line in the
charge kept at the centre O is 1
A B position of 0, and l respectively. The resultant force on
2
+q +2q q will be zero, if Q =
(A) – q (B) –2q
O q
(C) (D) 4q
–2q +q 2
D C 10. An isolated solid metallic sphere is given +Q charge. The
(A) Zero charge will be distributed on the sphere
(B) Along the diagonal AC (A) Uniformly but only on surface
(C) Along the diagonal BD (B) Only on surface but non-uniformly
(D) Perpendicular to side AB (C) Uniformly inside the volume
4. In the absence of other conductors, the surface charge (D) Non-uniformly inside the volume
density 11. Two small spheres each having the charge +Q are
(A) Is proportional to the charge on the conductor and suspended by insulating threads of length L from a hook.
its surface area This arrangement is taken in space where there is no
(B) Inversely proportional to the charge and directly gravitational effect, then the angle between the two
proportional to the surface area suspensions and the tension in each will be
(C) Directly proportional to the charge and inversely o 1 Q2 o 1 Q2
proportional to the surface area (A) 180 , (B) 90 ,
4 0 (2 L) 2 4 0 L2
(D) Inversely proportional to the charge and the surface
area o 1 Q2 o 1 Q2
(C) 180 , (D) 180 ,
5. A body can be negatively charged by 4 2L20 4 0 L2
(A) Giving excess of electrons to it 12. Two charges each of 1 coulomb are at a distance 1km
(B) Removing some electrons from it apart, the force between them is
(C) Giving some protons to it (A) 9 × 103 Newton (B) 9 × 10–3 Newton
(D) Removing some neutrons from it (C) 1.1 × 10 Newton (D) 104 Newton
–4

6. The minimum charge on an object is 13. +2 C and +6 C two charges are repelling each other with a
(A) 1 coulomb (B) 1 stat coulomb force of 12 N. If each charge is given –2 C of charge, then
(C) 1.6 × 10–19 coulomb (D) 3.2 × 10–19 coulomb the value of the force will be
7. Out of gravitational, electromagnetic, Vander Waals, (A) 4 N (Attractive) (B) 4 N (Repulsive)
electrostatic and nuclear forces; which two are able to (C) 8 N (Repulsive) (D) Zero
provide an attractive force between two neutrons 14. Dielectric constant of pure water is 81. Its permittivity will be
(A) Electrostatic and gravitational (A) 7.12 × 10–10 MKS units
(B) Electrostatic and nuclear (B) 8.86 × 10–12 MKS units
(C) Gravitational and nuclear (C) 1.02 × 1013 units
(D) Some other forces like Vander Waals (D) Cannot be calculated
15. There are two metallic spheres of same radii, but one is 22. When 1014 electrons are removed from a neutral metal
solid and the other is hollow, then sphere, the charge on the sphere becomes
(A) Solid sphere can be given more charge (A) 16 C (B) –16 C
(B) Hollow sphere can be given more charge (C) 32 C (D) –32 C
(C) They can be charged equally (maximum) 23. A force F acts between sodium and chlorine ions of salt
(D) None of the above (sodium chloride) when put 1cm apart in air. The
16. In general, metallic ropes are suspended on the carriers permittivity of air and dielectric constant of water are
which take inflammable material. The reason is 0 and K respectively. When a piece of salt is put in water
(A) There speed is controlled electrical force acting between sodium and chlorine ions
1cm apart is
(B) To keep the centre of gravity of the carrier nearer to
the earth F FK
(A) (B)
(C) To keep the body of the carrier in contact with the K 0
earth
F F 0
(D) Nothing should be placed under the carrier (C) K (D)
0 K
17. Three equal charges are placed on the three corners of a
square. If the force between q1 and q2 is F12 and that 24. A conductor has 14.4 × 10 –19 positive charge. The
F12 conductor has
between q1 and q3 is F13, the ratio of magnitudes F is (Charge on electron = 1.6 × 10–19 coulombs)
13
(A) 9 electrons in excess
1 (B) 27 electrons in short
(A) (B) 2
2 (C) 27 electrons in excess
1 (D) 9 electrons in short
(C) (D) 2
2 25. The value of electric permittivity of free space is
18. ABC is a right angled triangle in which AB = 3 cm and (A) 9 × 109 NC2 /m2
(B) 8.85 × 10–12 Nm2 /C2sec
BC = 4 cm. And ABC = . The three charges +15, +12
2 (C) 8.85 × 10–12 C2 /Nm2
and –20e.s.u. are placed respectively on A, B and C. The
force acting on B is (D) 9 × 109 C2 /Nm2
(A) 125 dynes (B) 35 dynes 26. One metallic sphere A is given positive charge whereas
another identical metallic sphere B of exactly same mass
(C) 25 dynes (D) Zero
as of A is given equal amount of negative charge. Then
19. With the rise in temperature, the dielectric constant K of
(A) Mass of A and mass of B still remain equal
a liquid
(B) Mass of A increases
(A) Increases
(C) Mass of B decreases
(B) Decreases
(D) Mass of B increases
(C) Remains unchanged
27. The force between two charges 0.06 m apart is 5N. If each
(D) Charges erratically
charge is moved towards the other by 0.01 m, then the
20. Two charges q1 and q2 are placed in vacuum at a distance
force between them will become
d and the force acting between them is F. If a medium of
(A) 7.20 N (B) 11.25 N
dielectric constant 4 is introduced around them, the force
now will be (C) 22.50 N (D) 45.00 N
(A) 4 F (B) 2 F 28. Two charged spheres separated at a distance d exert a
force F on each other. If they are immersed in a liquid of
F F dielectric constant 2, then what is the force (if all conditions
(C) (D)
2 4 are same)
21. Force of attraction between two point charges Q and – Q
F
separated by d metre is Fe. When these charges are placed (A) (B) F
2
on two identical spheres of radius R = 0.3 d whose centres
are d metre apart, the force of attraction between them is (C) 2F (D) 4F
(A) Greater than Fe 29. Two point charges +3 C and +8 C repel each other with
(B) Equal to Fe a force of 40N. If a charge of –5 C is added to each of
them, then the force between them will become
(C) Less than Fe
(A) –10 N (B) +10 N
(D) Less than Fe
(C) +20 N (D) –20 N
30. When 1019 electrons are removed from a neutral metal 2Q Q Q Q
plate, the electric charge on it is (A) 2
, 2 (B) 2
,
4 b 4 c 4 b 4 c2
(A) – 1.6 C (B) + 1.6 C
Q
(C) 10+19 C (D) 10–19 C (C) 0, (D) None of the above
31. Electric charges of 1 C,–1 C and 2 C are placed in air at 4 c2
the corners A, B and C respectively of an equilateral 39. Three charges each of magnitude q are placed at the
triangle ABC having length of each side 10 cm. The corners of an equilateral triangle, the electrostatic force
resultant force on the charge at C is on the charge placed at the center is (each side of triangle
(A) 0.9 N (B) 1.8 N is L)
(C) 2.7 N (D) 3.6 N 1 q2
32. Charge on -particle is (A) Zero (B)
4 0 L2
(A) 4.8 × 10–19 C (B) 1.6 × 10–19 C
(C) 3.2 × 10–19 C (D) 6.4 × 10–19 C 1 3q 2 1 q2
(C) 2 (D)
33. Two small conducting spheres of equal radius have 4 L 0 12 0 L2
charges +10 C and –20 C respectively and placed at a 40. Two charges placed in air repel each other by a force of
distance R from each other experience force F1. If they are 10–4 N. When oil is introduced between the charges, the
brought in contact and separated to the same distance, force becomes 2.5 × 10–5 N. The dielectric constant of
they experience force F2. The ratio of F1 to F2 is oil is
(A) 1 : 8 (B) – 8 : 1 (A) 2.5 (B) 0.25
(C) 1 : 2 (D) – 2 : 1 (C) 2.0 (D) 4.0
34. Two charges each equal to 2 C are 0.5m apart. If both of 41. Three charges are placed at the vertices of an equilateral
them exist inside vacuum, then the force between them is triangle of side ‘a’ as shown in the following figure. The
(A) 1.89 N (B) 2.44 N force experienced by the charge placed at the vertex A in
(C) 0.144 N (D) 3.144 N a direction normal to BC is
35. Two charges are at a distance ‘d’ apart. If a copper plate A
+Q
d
(conducting medium) of thickness is placed between
2
them, the effective force will be
F
(A) 2F (B) –Q +Q
2 B a C
(C) 0 (D) 2F
Q2 – Q2
36. Two electrons are separated by a distance of 1Å. What is (A) 2 (B) 2
4 0a 4 0a
the coulomb force between them
(A) 2.3 × 10–8 N (B) 4.6 × 10–8 N
Q2
(C) 1.5 × 10–8 N (D) None of these
(C) Zero (D) 2 2
37. Two copper balls, each weighing 10g are kept in air 10 cm 0a
apart. If one electron from every 106 atoms is transferred 42. Two particle of equal mass m and charge 16 cm are placed
from one ball to the other, the coulomb force between at a distance of 16 cm. They do not experience any force.
them is (atomic weight of copper is 63.5) q
The value of is
(A) 2.0 × 1010 N (B) 2.0 × 104 N m
(C) 2.0 × 108 N (D) 2.0 × 106 N
38. A solid conducting sphere of radius a has a net positive 0
(A) l (B)
charge 2Q. A conducting spherical shell of inner radius b G
and outer radius c is concentric with the solid sphere and
has a net charge – Q. The surface charge density on the G
(C) 4 (D) 4 0G
inner and outer surfaces of the spherical shell will be 0

43. When a glass rod is rubbed with silk, it


(A) Gains electrons from silk
a
b (B) Gives electrons to silk
c (C) Gains protons from silk
(D) Gives protons to silk
44. An electron is moving round the nucleus of a hydrogen 52. Two spherical conductors B and C having equal radii and
atom in a circular orbit of radius r. The coulomb force F carrying equal charges in them repel each other with a
1 force F when kept apart at some distance. A third spherical
between the two is (Where K = ) conductor having same radius as that of B but uncharged
4 0
is brought in contact with B, then brought in contact with
e2 e2 C and finally removed away from both. The new force of
(A) K rˆ (B) K 3 r
r3 r repulsion between B and C is

e2 e2 F 3F
(C) –K 3 r (D) K 2 rˆ (A) (B)
4 4
r r
45. A body has – 80 micro coulomb of charge. Number of F 3F
(C) (D)
additional electrons in it will be 8 8
(A) 8 × 10–5 (B) 80 × 10–17 53. When a body is earth connected, electrons from the earth
(C) 5 × 10 14 (D) 1.28 × 10–17 flow into the body. This means the body is ..
46. Two point charges placed at a certain distance r in air (A) Unchanged (B) Charged positively
exert a force F on each other. Then the distance r at (C) Charged negatively (D) An insulator
which these charges will exert the same force in a medium 54. The charges on two sphere are +7 C and – 5 C
of dielectric constant k is given by respectively. They experience a force F. If each of them is
r given and additional charge of – 2 C, the new force of
(A) r (B)
k attraction will be
r F
(C) (D) r k (A) F (B)
k 2
47. Dielectric constant for metal is
F
(A) Zero (B) Infinite (C) (D) 2F
(C) 1 (D) Greater than 1 3
48. A charge of Q coulomb is placed on a solid piece of metal 55. The ratio of electrostatic and gravitational forces acting
of irregular shape. The charge will distribute itself between electron and proton separated by a distance
(A) Uniformly in the metal object 5 × 10–11 m, will be (Charge on electron = 1.6 10–19 C,
(B) Uniformly on the surface of the object mass of electron = 9.1 × 10 –31 kg, mass of proton
(C) Such that the potential energy of the system is = 1.6 × 10–27 kg., G = 6.7 × 10–11 Nm2/kg2)
minimised (A) 2.36 × 1039 (B) 2.36 × 1040
(D) Such that the total heat loss is minimised (C) 2.34 × 1041 (D) 2.34 × 1042
49. Five balls numbered 1 to 5 are suspended using separate 56. A charge q is placed at the centre of the line joining two
threads. Pairs (1, 2), (2, 4) and (4, 1) show electrostatic equal charges Q. The system of the three charges will be
attraction, while pair (2, 3) and (4, 5) show repulsion. in equilibrium, if q is equal to
Therefore ball 1 must be
Q Q
(A) Positively charged (B) Negatively charged (A) (B)
(C) Neutral (D) Made of metal 2 4
50. Equal charges q are placed at the four corners A, B, C, D Q Q
(C) (D)
of a square of length a. The magnitude of the force on the 4 2
charge at B will be 57. The electric charge in uniform motion produces
3q 2 4q 2 (A) An electric field only
(A) 2 (B) 2
4 0a 4 0a (B) A magnetic field only
1 2 2 q2 1 q2 (C) Both electric and magnetic field
(C) (D) 2 (D) Neither electric nor magnetic field
2 4 0a2 2 4 0a2
51. Two identical conductors of copper and aluminium are 58. The electric field inside a spherical shell of uniform surface
placed in an identical electric fields. The magnitude of charge density is
induced charge in the aluminium will be (A) Zero
(A) Zero (B) Constant, less than zero
(B) Greater than in copper (C) Directly proportional to the distance from the centre
(C) Equal to that in copper
(D) None of the above
(D) Less than in copper
59. Electric lines of force about negative point charge are 65. In the electric field of a point charge q, a certain charge is
(A) Circular, anticlockwise carried from point A to B, C, D and E. Then the work done
(B) Circular, clockwise A
(C) Radial, inward
(D) Radial, outward
10
60. Charges of 10 9 C are placed at each of the four +q
3
corners of a square of side 8 cm. The potential at the B E
intersection of the diagonals is C D
(A) 150 2 volt (B) 1500 2 volt (A) Is least along the path AB
(B) Is least along the path AD
(C) 900 2 volt (D) 900 volt
(C) Is zero along all the paths AB, AC, AD and AE
61. A uniform electric field having a magnitude E0 and (D) Is least along AE
direction along the positive X-axis exists. If the potential
66. The magnitude of electric field intensity E is such that, an
V is zero at x = 0, then its value at X = + x will be
electron placed in it would experience an electrical force
(A) Vx = +xE0 (B) Vx = –xE0 equal to its weight is given by
(C) Vx = +x2E0 (D) Vx = –x2E0
mg
62. Three charges 2q,– q, – q are located at the vertices of an (A) mge (B)
equilateral triangle. At the centre of the triangle e
(A) The field is zero but potential is non-zero e e2
(B) The field is non-zero but potential is zero (C) (D) g
mg mg 2
(C) Both field and potential are zero
67. An electron and a proton are in a uniform electric field, the
(D) Both field and potential are non-zero
ratio of their accelerations will be
63. Figure shows the electric lines of force emerging from a
(A) Zero
charged body. If the electric field at A and B are EA and EB
respectively and if the displacement between A and B is r (B) Unity
then (C) The ratio of the masses of proton and electron
(D) The ratio of the masses of electron and proton
68. Two parallel plates have equal and opposite charge. When
the space between them is evacuated, the electric field
A r B between the plates is 2 × 105 V/m. When the space is filled
with dielectric, the electric field becomes 1 × 105 V/m. The
dielectric constant of the dielectric material

(A) EA > EB (B) EA < EB 1


(A) (B) 1
2
B B
(C) E A = (D) E A = 2 (C) 2 (D) 3
r r
69. The insulation property of air breaks down at
64. ABC is an equilateral triangle. Charges +q are placed at
E = 3 × 106 volt/metre. The maximum charge that can be
each corner. The electric intensity at O will be
given to a sphere of diameter 5m is approximately (in
+q coulombs)
A
(A) 2 × 10–2 (B) 2 × 10–3
r (C) 2 × 10–4 (D) 2 × 10–5
r r 70. The distance between the two charges 25 C and 36 C is
O 11 cm At what point on the line joining the two, the
+q +q
B a C intensity will be zero
1 q q 1 (A) At a distance of 5 cm from 25 C
(A) 4 (B) 4 (B) At a distance of 5 cm from 36 C
0 r2 0 r
(C) At a distance of 10 cm from 25 C
1 3q
(C) Zero (D) 4 (D) At a distance of 11 cm from 36 C
0 r2
71. A charge particle is free to move in an electric field. It will 78. Two point charges Q and – 3Q are placed at some distance
travel apart. If the electric field at the location of Q is E then at
(A) Always along a line of force the locality of –3Q, it is
(B) Along a line of force, if its initial velocity is zero E
(C) Along a line of force, if it has some initial velocity in (A) – E (B)
3
the direction of an acute angle with the line of force
–E
(D) None of the above (C) – 3E (D)
3
72. If E is the electric field intensity of an electrostatic field,
79. The number of electrons to be put on a spherical
then the electrostatic energy density is proportional to
conductor of radius 0.1 m to produce an electric field of
(A) E (B) E 2 0.036 N/C just above its surface is
1 (A) 2.7 × 105 (B) 2.6 × 105
(C) (D) E 3
E2 (C) 2.5 × 10 5 (D) 2.4 × 105
73. A metallic sphere has a charge of 10 C. A unit negative 80. The intensity of the electric field required to keep a water
charge is brought from A to B both 100 cm away from the drop of radius 10–5 cm just suspended in air when charged
sphere but A being east of it while B being on west. The with one electron is approximately
net work done is (A) 260 volt/cm (B) 260 newton/coulomb
2 (C) 130 volt/cm (D) 130 newton/coulomb
(A) Zero (B) joule (g = 10 newton / kg,e = 1.6 × 10–19 coulomb)
10
2 1 81. Conduction electrons are almost uniformly distributed
(C) joule (D) joule within a conducting plate. When placed in an electrostatic
10 10
74. Two charges +4e and +e are at a distance x apart. At what field E , the electric field within the plate
distance, a charge q must be placed from charge +e so (A) Is zero
that it is in equilibrium (B) Depends upon E
x 2x (C) Depends upon E
(A) (B)
2 3
(D) Depends upon the atomic number of the conducting
x x element
(C) (D)
3 6 82. The electric field near a conducting surface having a
75. An uncharged sphere of metal is placed in between two uniform surface charge density is given by
charged plates as shown. The lines of force look like
+ + ++ ++ + + + ++ ++ + (A) and is parallel to the surface
0
2
(B) and is parallel to the surface
–– ––––– –– ––––– 0
A B
+ + ++ ++ + + + ++ ++ + (C) and is normal to the surface
0
2
(D) and is normal to the surface
0
– – – – – – – – – – – – – – 83. There is an electric field E in X-direction. If the work done
C C on moving a charge 0.2 C through a distance of 2 m along
(A) A (B) B a line making an angle 60° with the X-axis is 4.0, what is
(C) C (D) D the value of E
76. The intensity of electric field required to balance a proton
(A) 3 N/C (B) 4 N/C
of mass 1.7 × 10–27 kg and charge 1.6 × 10–19 C is nearly
(A) 1 × 10–7 V/m (B) 1 × 10–5 V/m (C) 5 N/C (D) None of these
7
(C) 1 × 10 V/m (D) 1 × 105 V/m 84. Deutron and – particle are put 1 Å apart in air. Magnitude
77. On rotating a point charge having a charge q around a of intensity of electric field due to deutron at – particle is
charge Q in a circle of radius r. The work done will be (A) Zero
q× 2 Q (B) 2.88 × 1011 newton/coulomb
(A) q × 2 r (B)
r (C) 1.44 × 1011 newton/coulomb
Q (D) 5.76 × 1011 newton/coulomb
(C) Zero (D) 2 r
0
85. Below figures (1) and (2) represent lines of force. Which 92. What is the magnitude of a point charge due to which the
is correct statement electric field 30 cm away has th e magnitude
2 newton/coulomb
1 9 2 2
0 9 10 Nm / C
4
(2) (A) 2 × 10–11 coulomb (B) 3 × 10–11 coulomb
(1)
(C) 5 × 10–11 coulomb (D) 9 × 10–11 coulomb
(A) Figure (1) represents magnetic lines of force 93. Two charge +q and –q are situated at a certain distance.
(B) Figure (2) represents magnetic lines of force At the point exactly midway between them
(C) Figure (1) represents electric lines of force (A) Electric field and potential both are zero
(D) Both figure (1) and figure (2) represent magnetic lines (B) Electric field is zero but potential is not zero
of force (C) Electric field is not zero but potential is zero
86. The unit of electric field is not equivalent to (D) Neither electric field nor potential is zero
(A) N/C (B) J/C 94. Two positive charges of 20 coulomb and Q coulomb are
(C) V/m (D) J/C – m situated at a distance of 60 cm The neutral point between
87. A flat circular disc has a charge +Q uniformly distributed them is at a distance of 20 cm from the 20 coulomb charge.
on the disc. A charge +q is thrown with kinetic energy E Charge Q is
towards the disc along its normal axis. The charge q will (A) 30 C (B) 40 C
(A) Hit the disc at the centre (C) 60 C (D) 80 C
(B) Return back along its path after touching the disc 95. In the figure the charge Q is at the centre of the circle.
(C) Return back along its path without touching the disc Work done is maximum when another charge is taken from
(D) Any of the above three situations is possible point P to L
depending on the magnitude of E P
K
88. At a certain distance from a point charge the electric field
is 500 V/m and the potential is 3000 V. What is this distance
(A) 6 m (B) 12 m L Q
(C) 36 m (D) 144 m
89. The magnitude of electric field E in the annular region of
M
a charged cylindrical capacitor N
(A) Is same throughout (A) K (B) L
(B) Is higher near the outer cylinder than near the inner (C) M (D) N
cylinder 96. A mass m = 20 g has a charge q = 3.0 mC. It moves with a
velocity of 20 m/s and enters a region of electric field of
1
(C) Varies as , where r is the distance from the axis 80 N/C in the same direction as the velocity of the mass.
r
The velocity of the mass after 3 seconds in this region is
1 (A) 80 m/s (B) 56 m/s
(D) Varies as , where r is the distance from the axis
r2 (C) 44 m/s (D) 40 m/s
90. A metallic solid sphere is placed in a uniform electric field. 97. Equal charges q are placed at the vertices A and B of an
The lines of force follow the path(s) shown in figure as equilateral triangle ABC of side a. The magnitude of
1 1 electric field at the point C is
2 2 q 2q
3 3 (A) 2 (B) 2
4 0a 4 0a
4 4
(A) 1 (B) 2 3q q
(C) 3 (D) 4 (C) 2 (D) 2
4 0a 2 0a
91. The distance between a proton and electron both having
98. The unit of intensity of electric field is
a charge 1.6 × 10–19 coulomb, of a hydrogen atom is
(A) Newton/Coulomb
10–10 metre. The value of intensity of electric field
produced on electron due to proton will be (B) Joule/Coulmb
(A) 2.304 × 10–10 N/C (B) 14.4 V/m (C) Volt – metre
(C) 16 V/m (D) 1.44 × 1011 N/C (D) Newton/metre
99. The figure shows some of the electric field lines 106. Four charges are placed on corners of a square as shown
corresponding to an electric field. The figure suggests in figure having side of 5 cm. If Q is one microcoulomb,
then electric field intensity at centre will be
Q –2Q
A B C

(A) EA > EB > EC (B) EA = EB = EC


(C) EA = EC > EB (D) EA = EC < EB
100. Two spheres of radius a and b respectively are charged –Q +2Q
and joined by a wire. The ratio of electric field of the (A) 1.02 × 107 N/C upwards
spheres is
(B) 2.04 × 107 N/C downwards
(A) a/b (B) b/a (C) 2.04 × 107 N/C upwards
2
(C) a /b 2 (D) b2/a 2 (D) 2.04 × 107 N/C downwards
101. A particle of mass m and charge q is placed at rest in a 107. Point charges +4q, –q and +4q are kept on the x-axis at
uniform electric field E and then released. The kinetic points x = 0, x = a and x = 2a respectively, then
en ergy attained by th e particle after moving a (A) Only q is in stable equilibrium
distance y is (B) None of the charges are in equilibrium
(A) qEy2 (B) qE2y (C) All the charges are in unstable equilibrium
(C) qEy (D) q2 Ey (D) All the charges are in stable equilibrium
102. A hollow insulated conducting sphere is given a positive 108. Two point charges of 20 C and 80 C are 10 cm apart.
charge of 10 C. What will be the electric field at the centre Where will the electric field strength be zero on the line
of the sphere if its radius is 2 meters joining the charges from 20 C charge
(A) Zero (B) 5 Cm–2 (A) 0.1 m (B) 0.04 m
(C) 20 Cm –2 (D) 8 Cm–2 (C) 0.033 m (D) 0.33 m
103. An electron of mass me initially at rest moves through a 109. What is the magnitude of a point charge which produces
certain distance in a uniform electric field in time t1. A an electric field of 2 N/coulomb at a distance of 60 cm
proton of mass mp also initially at rest takes time t2 to 1
move through an equal distance in this uniform electric 9 2 2
0 9 10 N m / C
4
t2
field. Neglecting the effect of gravity, the ratio of t is (A) 8 × 10–11 C (B) 2 × 10–12 C
1 –11
(C) 3 × 10 C (D) 6 × 10–10 C
nearly equal to
110. The electric field due to a charge at a distance of 3 m from
1 it is 500 N/coulomb. The magnitude of the charge is
mp 2
(A) 1 (B) 1 N - m2
me 9 109
4 0 coulomb2
1 (A) 2.5 micro-coulomb (B) 2.0 micro-coulomb
me 2
(C) 1.0 micro-coulomb (D) 0.5 micro-coulomb
(C) (D) 1836
mp 111. Electric field strength due to a point charge of 5 C at a
distance of 80 cm from the charge is
104. A cube of side b has a charge q at each of its vertices. The
electric field due to this charge distribution at the centre (A) 8 × 104 N/C (B) 7 × 104 N/C
4
(C) 5 × 10 N/C (D) 4 × 104 N/C
of this cube will be
112. Three identical point charges, as shown are placed at the
q q vertices of an isosceles right angled triangle. Which of
(A) 2 (B)
b 2b2 the numbered vectors coincides in direction with the
32q electric field at the mid-point M of the hypotenuse
(C) (D) Zero 3
b2 2
105. A charged water drop whose radius is 0.1 m is in
equilibrium in an electric field. If charge on it is equal to 4
charge of an electron, then intensity of electric field will M
1
be (g = 10 ms–1)
(A) 1.61 N/C (B) 26.2 N/C (A) 1 (B) 2
(C) 262 N/C (D) 1610 N/C (C) 3 (D) 4
113. The displacement of a charge Q in the electric field 2q 2q 2q q
E = e1iˆ + e2 ˆj + e3kˆ is rˆ = aiˆ + bjˆ . The work done is
q q q 2q
2 2
(A) Q(ae1 + be2) (B) Q ae1 be2
2q 2q 2q q
(C) Q e1 e2 a 2
b 2 (D) Q e12 e22 a b (3) (4)
(A) 1 (B) 2
114.The potential at a point, due to a positive charge of 100 C
(C) 3 (D) 4
at a distance of 9m, is
121. An electron is moving towards x-axis. An electric field is
(A) 104 V (B) 105 V
6
along y-direction then path of electron is
(C) 10 V (D) 107 V
(A) Circular (B) Elliptical
115. Two charges +5 C and +10 C are placed 20 cm apart.
(C) Parabola (D) None of these
The net electric field at the mid-Point between the two
charges is 122. An electron enters in an electric field with its velocity in
the direction of the electric lines of force. Then
(A) 4.5 × 106 N/C directed towards +5 C
(A) The path of the electron will be a circle
(B) 4.5 × 106 N/C directed towards +10 C
(B) The path of the electron will be a parabola
(C) 13.5 × 106 N/C directed towards +5 C
(C) The velocity of the electron will decrease
(D) 13.5 × 106 N/C directed towards +10 C
(D) The velocity of the electron will increase
116. Which of the following is deflected by electric field
(A) X-rays (B) -rays 1 2
123. The dimension of 0 E ( 0 : permittivity of free
(C) Neutrons (D) -particles 2
117. Consider the points lying on a straight line joining two space; E : electric field) is
fixed opposite charges. Between the charges there is (A) MLT–1 (B) ML2T–2
(A) No point where electric field is zero (C) ML–1 T–2 (D) ML2 T–1
(B) Only one point where electric field is zero 124. A cube of a metal is given a positive charge Q. For the
(C) No point where potential is zero above system, which of the following statements is true
(D) Only one point where potential is zero (A) Electric potential at the surface of the cube is zero
118. A charged particle of mass 5 × 10–5 kg is held stationary in (B) Electric potential within the cube is zero
space by placing it in an electric field of strength (C) Electric field is normal to the surface of the cube
107 NC–1 directed vertically downwards. The charge on (D) Electric field varies within the cube
the particle is 125. If q is the charge per unit area on the surface of a
(A) –20 × 10–5 C (B) –5 × 10–5 C conductor, then the electric field intensity at a point on
(C) 5 × 10–5 C (D) 20 × 10–5 C the surface is
119. In the given figure distance of the point from A where the q
electric field is zero is (A) normal to surface
0
A B
q
(B) 2 0 normal to surface
10 C 20 C
80 cm
(A) 20 cm (B) 10 cm q
(C) tangential to surface
(C) 33 cm (D) None of these 0
120. Figures below show regular hexagons, with charges at q
the vertices. In which of the following cases the electric (D) tangential to surface
2 0
field at the centre is not zero
q q q –q 126. Electric field intensity at a point in between two parallel
sheets with like charges of same surface charge densities
( ) is
q q q q
(A) 2 (B)
q q q –q 0 0

(1) (2) 2
(C) Zero (D)
0
127. Consider two point charges of equal magnitude and 134. Two point charges +9e and +e are at 16 cm away from
opposite sign separated by a certain distance. The neutral each other. Where should another charge q be placed
point due to them between them so that the system remains in equilibrium
(A) Does not exist (A) 24 cm from +9e (B) 12 cm from +9e
(B) Will be in mid way between them (C) 24 cm from +e (D) 12 cm from +e
(C) Lies on the perpendicular bisector of the line joining 135. The distan ce between ch arges 5× 10 –11 C an d
the two –2.7 × 10–11C is 0.2 m. The distance at which a third
(D) Will be closer to the negative charge charge should be placed in order that it will not experience
128. Two small spherical balls each carrying a charge Q = 10 C any force along the line joining the two charges is
(10 micro-coulomb) are suspended by two insulating (A) 0.44 m (B) 0.65 m
threads of equal lengths 1m each, from a point fixed in the (C) 0.556 m (D) 0.350 m
ceiling. It is found that in equilibrium threads are separated 136. An electron having charge ‘e’ and mass ‘m’ is moving in a
by an angle 60° between them, as shown in the figure. uniform electric field E. Its acceleration will be
What is the tension in the threads
e2 E 2e
1 (A) (B)
(Given: = 9 × 109 NM/C2) m m
4 0 eE mE
(C) (D)
m e
60° 137. Cathode rays travelling from east to west enter into region
of electric field directed towards north to south in the
Q Q plane of paper. The deflection of cathode rays is towards
(A) 18 N (B) 1.8 N (A) East (B) South
(C) 0.18 N (D) None of the above (C) West (D) North
129. The acceleration of an electron in an electric field of 138. A simple pendulum of period T has a metal bob which is
magnitude 50 V/cm, if e/m value of the electron is negatively charged. If it is allowed to oscillate above a
1.76 × 1011 C/kg, is positively charged metal plate, its period will
(A) 8.8 × 1014 m/sec2 (B) 6.2 × 1013 m/sec2 (A) Remains equal to T (B) Less than T
12
(C) 5.4 × 10 m/sec 2 (D) Zero (C) Greater than T (D) Infinite
130. A positively charged particle moving along x-axis with a 139. A charged particle of mass m and charge q is released
certain velocity enters a uniform electric field directed from rest in a uniform electric field E. Neglecting the effect
along positive y-axis. Its of gravity, the kinetic energy of the charged particle after
(A) Vertical velocity changes but horizontal velocity ‘t’ second is
remains constant
Eq 2 m 2E 2t 2
(B) Horizontal velocity changes but vertical velocity (A) (B)
remains constant 2t 2 mq

(C) Both vertical and horizontal velocities change E 2 q 2t 2 Eqm


(C) (D)
2m t
(D) Neither vertical nor horizontal velocity changes
140. A proton is about 1840 times heavier than an electron.
131. A hollow sphere of charge does not produce an electric
When it is accelerated by a potential difference of 1 kV, its
field at any
kinetic energy will be
(A) Point beyond 2 metres
1
(B) Point beyond 10 metres (A) 1840 keV (B) keV
(C) Interior point 1840
(C) 1 keV (D) 920 keV
(D) Outer point
141. A conducting sphere of radius R = 20cm is given a charge
132. A drop of 10–6 kg water carries 10–6 C charge. What electric
field should be applied to balance its weight (assume Q = 16 C. What is E at centre
g = 10m/s2) (A) 3.6 × 106 N/C (B) 1.8 × 106 N/C
(A) 10 V/m upward (B) 10 V/m downward (C) Zero (D) 0.9 × 106 N/C
(C) 0.1 V/m downward (D) 0.1 V/m upward 142. A point charge is kept at the centre of a metallic insulated
133. A charged particle of mass 0.003 gm is held stationary in spherical shell. Then
space by placing it in a downward direction of electric (A) Electric field out side the sphere is zero
field of 6 × 104 N/C. Then the magnitude of the charge is (B) Electric field inside the sphere is zero
(A) 5 × 10–4 C (B) 5 × 10–10 C (C) Net induced charge on the sphere is zero
(C) –18 × 10–6 C (D) –5 × 10–9 C (D) Electric potential inside the sphere is zero
143. An electron moving with the speed 5 × 106 per sec is 150. Charges q, 2q, 3q and 4q are placed at the corners A, B, C
shooted parallel to the electric field of intensity and D of a square as shown in the following figure. The
1 × 103 N/C Field is responsible for the retardation of direction of electric field at the centre of the square is
motion of electron. Now evaluate the distance travelled along
by the electron before coming to rest for an instant (mass D C
of e = 9 × 10–31 Kg. charge = 1.6 × 10–19 C)
4q 3q
(A) 7 m (B) 0.7 mm
(C) 7 cm (D) 0.7 cm
O
144. Three point charges are placed at the corners of an
equilateral triangle. Assuming only electrostatic forces q 2q
are acting A B
(A) The system can never be in equilibrium (A) AB (B) CB
(B) The system will be in equilibrium if the charges rotate (C) BD (D) AC
about the centre of the triangle 151. Infinite charges of magnitude q each are lying at x =1, 2, 4,
(C) The system will be in equilibrium if the charges have 8... meter on X-axis. The value of intensity of electric field
different magnitudes and different signs at point x = 0 due to these charges will be
(D) The system will be in equilibrium if the charges have (A) 12 × 109q N/C (B) Zero
the same magnitudes but different signs
(C) 6 × 109q N/C (D) 4 × 109q N/C
145. If an insulated non-conducting sphere of radius R has
152. A pendulum bob of mass 30.7 × 10–6 kg and carrying a
charge density . The electric field at a distance r from the
charge 2 × 10–8 C is at rest in a horizontal uniform electric
centre of sphere (r < R) will be
field of 20000 V/m. The tension in the thread of the
R r pendulum is (g = 9.8 m/s2)
(A) 3 (B)
0 0 (A) 3 × 10–4 N (B) 4 × 10–4 N
r 3 R (C) 5 × 10–4 N (D) 6 × 10–4 N
(C) 3 (D)
0 0 153. An infinite line charge produce a field of 7.182 × 108 N/C
146. Two plates are at potentials –10 V and +30 V. If the at a distance of 2 cm. The linear charge density is
separation between the plates be 2 cm. The electric field (A) 7.27 × 10–4 C/m (B) 7.98 × 10–4 C/m
between them is (C) 7.11 × 10–4 C/m (D) 7.04 × 10–4 C/m
(A) 2000 V/m (B) 1000 V/m
154. An electron experiences a force equal to its weight when
(C) 500 V/m (D) 3000 V/m placed in an electric field. The intensity of the field will be
147. The wrong statement about electric lines of force is
(A) 1.7 × 10–11 N/C (B) 5.0 × 10–11 N/C
(A) These originate from positive charge and end on
(C) 5.5 × 10–11 N/C (D) 56 N/C
negative charge
(B) They do not intersect each other at a point 155. The dielectric strength of air at NTP is 3 × 106 V/m then
the maximum charge that can be given to a spherical
(C) They have the same form for a point charge and a
conductor of radius 3 m is
sphere
(D) They have physical existence (A) 3 × 10–4 C (B) 3 × 10–3 C
148. A charge produces an electric field of 1 N/C at a point (C) 3 × 10–2 C (D) 3 × 10–1 C
distant 0.1 m from it. The magnitude of charge is 156. A charged ball B hangs from a silk thread S, which makes
(A) 1.11 × 10–12 C an angle with a large charged conducting sheet P, as
(B) 9.11 × 10–12 C shown in the figure. The surface charge density of the
sheet is proportional to
(C) 7.11 × 10–6 C
(D) None of these
P
149. A charged particle is suspended in equilibrium in a uniform
vertical electric field of intensity 20000 V/m. If mass of the
particle is 9.6 × 10–16 kg, the charge on it and excess
S
number of electrons on the particle are respectively
(g = 10 m/s2) B
(A) 4.8 × 10–19 C, 3 (B) 5.8 × 10–19 C, 4 (A) sin (B) tan
(C) 3.8 × 10–19 C, 2 (D) 2.8 × 10–19 C, 1 (C) cos (D) cot
157. Two point charges +8q and –2q are located at x = 0 and 162. Charges 4Q, q and Q and placed along x-axis at positions
x = L respectively. The location of a point on the x-axis at 1
which the net electric field due to these two point charges x = 0, x = and x = 1, respectively. Find the value of q so
2
is zero is that force on charge Q is zero
(A) 8 L (B) 4 L Q
(A) Q (B)
L 2
(C) 2 L (D) Q
4 (C) (D) – Q
158. Three infinitely long charge sheets are placed as shown 2
in figure. The electric field at point P is 163. An electric dipole consisting of two opposite charges of
2 × 10–6 C each separated by a distance of 3 cm is placed
Z
in an electric field of 2 × 105 N/C. The maximum torque on
Z = 3a the dipole will be
P
–2 (A) 12 × 10–1 N m (B) 12 × 10–3 N m
Z=a –1
(C) 24 × 10 N m (D) 24 × 10–3 N m
x
– Z = –a 164. An electric dipole of moment p is placed normal to the
2 2 lines of force of electric intensity E , then the work done
(A) kˆ (B) kˆ in deflecting it through an angle of 180° is
o o
(A) pE (B) + 2pE
4 4
(C) kˆ (D) kˆ (C) –2pE (D) Zero
o o 165. The distance between the two charges +q and –q of a
159. Two infinitely long parallel conducting plates having dipole is r. On the axial line at a distance d from the centre
surface charge densities + and – respectively, are of dipole, the intensity is proportional to
separated by a small distance. The medium between the
q qr
plates is vacuum. If 0is the dielectric permittivity of (A) (B)
2
vacuum, then the electric field in the region between the d d2
plates is q qr
(C) 3 (D)
d d3
(A) 0 volts/meter (B) 2 volts/meter
0 166. An electron and a proton are at a distance of 1 Å. The
2 moment of this dipole will be (C × m)
(C) volts/meter (D) volts/meter
0 0 (A) 1.6 × 1019 (B) 1.6 × 10–29
160. Four point +ve charges of same magnitude (Q) are placed (C) 3.2 × 1019 (D) 3.2 × 1029
at four corners of a rigid square frame as shown in figure. 167. The electric field due to a dipole at a distance r on its
The plane of the frame is perpendicular to z-axis. If a –ve axis is
point charge is placed at a distance z away from the above (A) Directly proportional to r3
frame (z<<L) then (B) Inversely proportional to r3
Q Q (C) Directly proportional to r2
(D) Inversely proportional to r2
L 168. Two charges +3.2 × 10–19 and –3.2 × 10–19 C placed at
2.4 Å apart form an electric dipole. It is placed in a uniform
electric field of intensity 4 × 105 volt/m. The electric dipole
Z-axis
Q Q moment is
(A) –ve charge oscillates along the z-axis. (A) 1.5 × 36 × 10–29 coulomb × m
(B) It moves away from the frame (B) 1.5 × 36 × 10–19 coulomb × m
(C) It moves slowly towards the frame and stays in the (C) 7.68 × 10–29 coulomb × m
plane of the frame (D) 7.68 × 10–19 coulomb × m
(D) It passes through the frame only once. 169. An electric dipole of moment p is placed at the origin
161. At a point 20 cm from the centre of a uniformly charged along the x-axis. The electric field at a point P, whose
position vector makes an angle with the x-axis, will make
dielectric sphere of radius 10 cm, the electric field is
100 V/m. The electric field at 3 cm from the centre of the 1
an angle ..... with the x-axis, where tan tan
sphere will be 2
(A) 150 V/m (B) 125 V/m (A) (B)
(C) 120 V/m (D) Zero (C) + (D) + 2
170. An electric dipole is placed along the x-axis at the 177. A point Q lies on the perpendicular bisector of an electrical
origin O. A point P is at a distance of 20 cm from this origin dipole of dipole moment p. If the distance of Q from the
dipole is r (much larger than the size of the dipole), then
such that OP makes an angle with the x-axis. If the electric field at Q is proportional to
3
(A) p–1 and r–2
electric field at P makes an angle with the x-axis, the
(B) p and r–2
value of would be
(C) p2 and r–3
1 3 (D) p and r–3
(A) (B) tan
3 3 2 178. If the magnitude of intensity of electric field at a
distance x on axial line and at a distance y on equatorial
2 1 3 line on a given dipole are equal, then x : y is
(C) (D) tan
3 2
(A) 1 : 1 (B) 1 : 2
171. Electric charges q, q, –2q are placed at the corners of an
(C) 1 : 2 (D) 3
2 :1
equilateral triangle ABC of side l. The magnitude of electric
dipole moment of the system is 179. An electric dipole in a uniform electric field experiences
(A) ql (B) 2ql (When it is placed at an angle with the field)
(A) Force and torque both
(C) 3ql (D) 4ql (B) Force but no torque
172. The torque acting on a dipole of moment P in an electric (C) Torque but no force
field E is (D) No force and no torque
180. The electric intensity due to a dipole of length 10 cm and
(A) P . E (B) P E
having a charge of 500 C, at a point on the axis at a
(C) Zero (D) E P distance 20 cm from one of the charges in air, is
173. The electric field at a point on equatorial line of a dipole (A) 6.25 × 107 N/C (B) 9.28 × 107 N/C
11
(C) 13.1 × 11 N/C (D) 20.5 × 107 N/C
and direction of the dipole moment
(A) Will be parallel 181. Electric potential at an equatorial point of a small dipole
(B) Will be in opposite direction with dipole moment P (r, distance from the dipole) is
(C) Will be perpendicular P
(D) Are not related (A) Zero (B) 2
4 0r
174. Two opposite and equal charges 4 × 10–8 coulomb when
P 2P
placed 2 × 10–2 cm away, form a dipole. If this dipole is
(C) 3 (D) 3
placed in an external electric field 4 × 108 newton/coulomb, 4 0r 4 0r
the value of maximum torque and the work done in rotating 182. The distance between H+ and Cl– ions in HCl molecule is
it through 180° will be 1.28 Å. What will be the potential due to this dipole at a
(A) 64 × 10–4 Nm and 64 × 10–4 J distance of 12 Å on the axis of dipole
(B) 32 × 10–4 Nm and 32 × 10–4 J (A) 0.13 V
(C) 64 × 10–4 Nm and 32 × 10–4 J (B) 1.3 V
(D) 32 × 10–4 Nm and 64 × 10–4 J (C) 13 V
175. If Ea be the electric field strength of a short dipole at a (D) 130 V
point on its axial line and Ee that on the equatorial line at 183. The potential at a point due to an electric dipole will be
the same distance, then maximum and minimum when the angles between the axis
(A) Ee = 2Ea of the dipole and the line joining the point to the dipole
are respectively
(B) Ea = 2Ee
(A) 90° and 180° (B) 0° and 90°
(C) Ea = Ee
(C) 90° and 0° (D) 0° and 180°
(D) None of the above
184. The value of electric potential at any point due to any
176. An electric dipole is placed in an electric field generated
electric dipole is
by a point charge
(A) The net electric force on the dipole must be zero p r p r
(A) k . 2 (B) k .
(B) The net electric force on the dipole may be zero r r3
(C) The torque on the dipole due to the field must be zero p.r p.r
(D) The torque on the dipole due to the field may be zero (C) k . 2 (D) k .
r r3
185. An electric dipole has the magnitude of its charge as q 193. A cube of side l is placed in a uniform field E, where
and its dipole moment is p. It is placed in a uniform electric E Eiˆ . The net electric flux through the cube is
field E. If its dipole moment is along the direction of the (A) Zero (B) l2E
field, the force on it and its potential energy are respectively
(C) 4l2E (D) 6l2E
(A) 2q . E and minimum (B) q . E and p . E
(C) Zero and minimum (D) p . E and maximum 194. Eight dipoles of charges of magnitude e are placed inside
186. Intensity of an electric field E due to a dipole, depends on a cube. The total electric flux coming out of the cube
distance r as will be
1 1 8e
(A) E 4 (B) E 3 (A)
r r 0
1 1 16e
(C) E 2 (D) E (B)
r r 0
187. The ratio of electric fields on the axis and at equator of an
electric dipole will be e
(C)
(A) 1 : 1 (B) 2 : 1 0
(C) 4 : 1 (D) None of these (D) Zero
188. For a dipole q = 2 × 10–6 C and d = 0.01 m. Calculate the 195. A point charge +q is placed at the centre of a cube of
maximum torque for this dipole if E = 5 × 105 N/C side L. The electric flux emerging from the cube is
(A) 1 × 10–3 Nm–1
q
(B) 10 × 10–3 Nm–1 (A) (B) Zero
(C) 10 × 10–3 Nm 0

(D) 1 × 102 Nm2 6qL2 q


189. A molecule with a dipole moment p is placed in an electric (C) (D)
0 6 L2 0
field of strength E. Initially the dipole is aligned parallel to
the field. If the dipole is to be rotated to be anti-parallel to 196. A charge q is placed at the centre of the open end of
the field, the work required to be done by an external cylindrical vessel. The flux of the electric field through
agency is the surface of the vessel is
(A) – 2pE (B) – pE (A) Zero
(C) pE (D) 2pE q
(B)
190. An electric dipole of moment P placed in a uniform electric 0
field E has minimum potential energy when the angle q
between P and E is (C) 2
0

(A) Zero (B) 2q


2 (D)
0
3
(C) (D) 197. It is not convenient to use a spherical Gaussian surface to
2
find the electric field due to an electric dipole using Gauss’s
191. A region surrounding a stationary electric dipoles has
theorem because
(A) Magnetic field only
(A) Gauss’s law fails in this case
(B) Electric field only
(B) This problem does not have spherical symmetry
(C) Both electric and magnetic fields
(C) Coulomb’s law is more fundamental than Gauss’s law
(D) No electric and magnetic fields
(D) Spherical Gaussian surface will alter the dipole
192. Two electric dipoles of moment P and 64 P are placed in moment
opposite direction on a line at a distance of 25 cm. The
198. According to Gauss’ Theorem, electric field of an infinitely
electric field will be zero at point between the dipoles
long straight wire is proportional to
whose distance from the dipole of moment P is
1
25 (A) r (B)
(A) 5 cm (B) cm r2
9
1 1
4 (C) 3 (D)
(C) 10 cm (D) cm r r
13
199. Electric charge is uniformly distributed along a long 204. A charge q is placed at the centre of a cube. Then the flux
straight wire of radius 1mm. The charge per cm length of passing through one face of cube will be
the wire is Q coulomb. Another cylindrical surface of q q
radius 50 cm and length 1m symmetrically encloses the (A) (B) 2
wire as shown in the figure. The total electric flux passing 0 0

through the cylindrical surface is q q


+ (C) 4 (D) 6
0 0
+
+ 205. If a spherical conductor comes out from the closed surface
of the sphere then total flux emitted from the surface
will be
1m
1
(A) (the charge enclosed by surface)
0
+
+ 50cm (B) 0 × (charge enclosed by surface)
+
1
Q 100Q (C) 4 (charge enclosed by surface)
(A) (B) 0
0 0
(D) 0
10Q 100Q
(C) (D) 206. If the electric flux entering and leaving an enclosed surface
0 0 respectively is 1 and 2 the electric charge inside the
200. The S.I. unit of electric flux is surface will be
(A) Weber (B) Newton per coulomb
(A) ( 1 + 1) 0
(C) Volt ´ metre (D) Joule per coulomb
(B) ( 2 + 1) 0
201. q1, q2, q3 and q4 are point charges located at points as
shown in the figure and S is a spherical Gaussian surface (C) ( 1 + 2) 0
of radius R. Which of the following is true according to (D) ( 2 + 1) 0
the Gauss’s law 207. A charge q is located at the centre of a cube. The electric
S flux through any face is
4 q
q1 R (A) 6 4
0
q4
q2 q3 q
(B) 6 4
0

q1 q2 q3 q
(A) E1 E 2 E 3 .d A (C) 6 4
s 2 0 0
q1 q2 q3 2 q
(B) E1 E 2 E 3 .d A
s 2 0 (D) 6 4
0
q1 q2 q3 q4 208. Shown below is a distribution of charges. The flux of
(C) E1 E 2 E 3 .d A
s electric field due to these charges through the surface S is
0
(D) None of the above
202. Gauss’s law should be invalid if
S
(A) There were magnetic monopoles +q
+q
(B) The inverse square law were not exactly true
(C) The velocity of light were not a universal constant
(D) None of these
203. The inward and outward electric flux for a closed surface +q
in units of N-m2/C are respectively 8 × 103 and 4 × 103.
Then the total charge inside the surface is [where 0 =
permittivity constant] 3q 2q
(A) (B)
(A) 4 × 103 C (B) –4 × 103 C 0 0
3 q
4 10
(C) C (D) –4 × 103 0C
(C) (D) Zero
0
209. Consider the charge configuration and spherical Gaussian 211. An electric dipole is put in north-south direction in a sphere
surface as shown in the figure. When calculating the flux filled with water. Which statement is correct
of the electric field over the spherical surface the electric (A) Electric flux is coming towards sphere
field will be due to (B) Electric flux is coming out of sphere
q2 (C) Electric flux entering into sphere and leaving the
+q1 sphere are same
(D) Water does not permit electric flux to enter into sphere
–q1 212. Two infinite plane parallel sheets separated by a distance
d have equal and opposite uniform charge densities .
Electric field at a point between the sheets is
(A) q 2
(A) Zero
(B) Only the positive charges
(C) All the charges (B)
(D) +q1 and –q1 0

210. Gauss’s law is true only if force due to a charge varies as


(C) 2
(A) r–1 (B) r–2 0
(C) r –3 (D) r–4 (D) Depends upon the location of the point
(Previous Year AIPMT Examination Questions)
1. A hollow cylinder has a charge q coulomb within it. If f is 5. Two positive ions each carrying a charge q, are separted
the electric flux in units of voltmeter associated with the by a distance d. If F is the force of repulsion between the
curved surface B, the flux linked with the plane surface A ions, the number of electrons missing from each ion will
in units of voltmeter will be [AIPMT 2007] be (e being the charge on an electron) [AIPMT 2010]
B 2 2
4 0 Fd 4 0 Fd
(A) 2 (B) 2
C A e d
2 2
4 0 Fd
4 0 Fd
(C) 2 (D) 2
q 1 q e q
(A) (B) 2 6. A square surface of side L metre in the plane of the paper
0 0
is placed in a uniform electric field E (volt/m) acting along
q the same place at an angle q with the horizontal side of
(C) 2 (D)
0 3 the square as shown in the figure. The electric flux linked
2. Three point charges +q, –2q and +q are placed at points (x = to the surface in uint of V- m is. [AIPMT 2010]
0, y = a, z = 0), (x = 0, y = 0, z = 0) and (x = a, y = 0,
z = 0) respectively. The magnitude and direction of the electric E
dipole moment vector of this charge assembly are :–
[AIPMT 2007]
(A) 2 qa along + x direction
(B) (A) EL2 (B) EL2 cos q
2 qa along + y direction
(C) EL2 sin q (D) 0
(C) 2 qa along the line joining points 3R
(x = 0, y = 0, z = 0) and (x = a, y = a, z = 0) 7. The electic filed at a distance from the center of a
2
(D) qa along the line joining points charged conducting spherical shell of radius R is E. The
(x = 0, y = 0, z = 0) and (x = a, y = a, z = 0) R
elecrtric field at a distance from the center of the sphere
3. A thin conducing ring of radius R is given a charge +Q. 2
The electric field at the centre O of the ring due to the is [AIPMT 2010]
charge on the part AKB of the ring is E. The electric field E
at the centre due to the charge on the part ACDB of the (A)
2
ring is [AIPMT 2008] (B) zero
A
(C) E
K
E
(D)
4
C B 8. A charge Q is enclosed by a Gaussian spherical suface of
O
radius R. If the radius is doubled, then the outward electric
flux will [AIPMT 2011]
(A) be reduced of half (B) remain the same
D
(C) be doubled (D) increase four times
(A) 3E along KO (B) E along OK
9. The electric potential V at any point (x,y, z), all in metres in
(C) E along KO (D) 3 E along OK
space is given by V = 4x2 volt. The electric field at the
4. The mean free path of electrons in a metal is 4 × 10–8 m.
point (1,0,2) in volt/ metre is [AIPMT 2011]
The electric field which can give on an average 2 eV energy
(A) 8 along positive X-axis
to an electron in the metal will be in unit of Vm–1
[AIPMT 2009] (B) 16 along negative X-axis
(A) 8 × 10 –7 (B) 5 × 10 –11 (C) 16 along positive X-axis
(C) 8 × 10–11 (D) 5 × 107 (D) 8 along negative X-axis
10. Three charges each +q, are placed at the corners of an
isosceles triangle ABC of sides BC and AC,2a, D and E are
the mid-points of BC and CA the work done in taking a y y/ 2

charge Q from D to E is [AIPMT 2011]


A r r'

2
(A) 1 (B) r
E
3
2 2
B C 2r 2r
D (C) (D)
3 3
eqQ
(A) 8 15. The electric field in a certain region is actin g
0a radially outward and is given by E = Ar .A charge
contained in a sphere of radius ‘a’ centred at the origin of
qQ the filed will be given by [AIMPT 2015]
(B) 4 2 2
0a (A) 4 0Aa (B) 4 0a
(C) 4 0Aa3 (D) 0Aa 2
(C) zero
16. Two identical charged spheres suspended from a common
3 qQ point by two massless strings of lengths l, are initially at
(D) 4 a distance d (d << l ) apart because of their mutual
0a repulsion. The charges begin to leak from both the spheres
11. An electric dipole of moment ‘P’ is placed in an electric at a constant rate. As a result, the spheres approach each
field of intensity ‘E’. The dipole acquires a positon such other with avelocity v. Then v varies as a function of the
that the axis of the dipole makes an angle q with the distance x between the spheres, as : [AIPMT 2016]
direction of the field. Assuming that the potential energy 1
of the dipole to be zero when q = 90°, the torque and the (A) v x2 (B) v x
potential energy of the dipole will respectively be 1
– –1
[AIPMT 2012] (C) v x 2 (D) v x
(A) pE cos , – pE sin q = 0 17. An electric dipole is placed at an angle of 30° with an
(B) pE sin , – pE cos q = 0 electric field intensity 2 × 105 N/C. It experiences a torque
(C) pE sin , – 2pE cos q = 0 equal to 4 Nm. The charge on the dipole, if the dipole
(D) pE sin , 2pE cos q = 0 length is 2 cm, is :- [NEET- II(2016)]
12. What is the flux through a cube of side ‘a’ if a point charge (A) 5 C (B) 7 C
of q is at one of its corner ? [AIPMT 2012] (C) 8 C (D) 2 C
18. Suppose the charge of a proton and an electron differ
(A) 6a 2 (B) slightly. One of them is – e, the other is (e + e). If the net
0 0 of electrostatic force and gravitational force between two
hydrogen atoms placed at a distance d (much greater than
atomic size). Apart is zero, then e is of the order of
(C) 8 (D)
0 0 [Given mass of hydrogen mh = 1.67 × 10–27 kg]
13. Two metallic sphere of radii 1 cm and 3 cm are given charges (NEET - (2017)
(A) 10 C–20 (B) 10 C–23
of –1 × 10–2 C and respectively. If these are connected by
a conducting wire, the final charge on the bigger sphere is (C) 10–37 C (D) 10–47 C
[AIMPT 2012] 19. An electron falls from rest through a vertical distance
(A) 2 × 10–2 C (B) 3 × 10–2 C h in a uniform and vertically upward directed electric
(C) 4 × 10–2 C (D) 1 × 10–2 C field E. The direction of electric field is now reversed,
14. Two pitch balls carrying equal charges are suspended keeping its magnitude the same. A proton is allowed to
from a common point by stringes of equal length, the fall from rest in it through the same vertical distance h.
equlibrium separation between them is r. Now the strings The time of fall of the electron, is comparison to the time
are rigidly clamped at half the height. The equilibrium of fall of the proton is [NEET-2018]
separation between the balls now become (A) 10 times greater (B) 5 times greater
[AIPMT 2013] (C) smaller (D) equal
20. An electron of mass m with an initial velocity 23. A cylindrical conductor of radius R is carrying a constant
current. The plot of the magnitude of the magnetic field,
V V0 ˆi(V0 0) enters and electric field E –E0 iˆ (E0
B with the distance, d, from the centre of the conductor,
= constant > 0) at t = 0. If 0 is its de-Broglie wavelength is correctly represented by the figure:
initially, then its de-Broglie wavelength at time t is [NEET-2019]
[NEET-2018]
B B
E0
(A) t (B) 0 1 t
0 mV0
(A) (B)

0
(C) (D) 0t
eE0 R d R d
1 t
mV0

B B
21. A toy car with charge q moves on a frictionless
horizontal plane surface under the influence of a uniform
electric field E. Due to the force qE, its velocity (C) (D)
increases from 0 to 6 m/s in one second duration. At
that instant the direction of the field is reversed. The car R d R d
continues to move for two more seconds under the
24. Two point charges A and B, having charges + Q and
influence of this field. The average velocity and the
– Q respectively, are placed at certain distance apart
average speed of the toy car between 0 to 3 seconds are
and force acting between them is F. If 25% charge of A
respectively
is transferred to B, then force between the charges
[NEET-2018] becomes: [NEET-2019]
(A) 1 m/s, 3.5 m/s (B) 1 m/s, 3 m/s
9F
(C) 2 m/s, 4 m/s (D) 1.5 m/s, 3 m/s (A)
16
22. Two parallel infinite line charges with linear charge
densities + C/m and – C/m are placed at a distance of 16F
2R in free space. What is the electric field mid–way (B)
9
between the two line charges? [NEET-2019]
4F
2 N/C (C)
(A) N/C (B) 3
0 R 0 R
(D) F

(C) N/C (D) Zero


2 0 R
Exercise – I
1. (B) 2. (B) 3. (C) 4. (C) 5. (A) 6. (C) 7. (C) 8. (D) 9. (A) 10. (A)
11. (A) 12. (A) 13. (D) 14. (A) 15. (C) 16. (C) 17. (B) 18. (C) 19. (B) 20. (D)
21. (A) 22. (A) 23. (A) 24. (D) 25. (C) 26. (D) 27. (B) 28. (A) 29. (A) 30. (B)
31. (B) 32. (C) 33. (B) 34. (C) 35. (C) 36. (A) 37. (C) 38. (A) 39. (A) 40. (D)
41. (C) 42. (D) 43. (B) 44. (C) 45. (C) 46. (C) 47. (B) 48. (C) 49. (C) 50. (C)
51. (C) 52. (D) 53. (B) 54. (A) 55. (A) 56. (B) 57. (C) 58. (A) 59. (C) 60. (B)
61. (B) 62. (B) 63. (A) 64. (C) 65. (C) 66. (B) 67. (C) 68. (C) 69. (B) 70. (A)
71. (B) 72. (B) 73. (A) 74. (C) 75. (C) 76. (A) 77. (C) 78. (B) 79. (C) 80. (B)
81. (A) 82. (C) 83. (D) 84. (C) 85. (A) 86. (B) 87. (D) 88. (A) 89. (C) 90. (D)
91. (D) 92. (A) 93. (C) 94. (D) 95. (A) 96. (B) 97. (C) 98. (A) 99. (C) 100. (B)
101. (C) 102. (A) 103. (B) 104. (D) 105. (C) 106. (A) 107. (C) 108. (C) 109. (A) 110. (D)
111. (B) 112. (B) 113. (A) 114. (C) 115. (A) 116. (D) 117. (A, D) 118. (B) 119. (C) 120. (B)
121. (C) 122. (C) 123. (C) 124. (C) 125. (A) 126. (C) 127. (A) 128. (B) 129. (A) 130. (A)
131. (C) 132. (A) 133. (B) 134. (B) 135. (C) 136. (C) 137. (D) 138. (B) 139. (C) 140. (C)
141. (C) 142. (C) 143. (C) 144. (A) 145. (C) 146. (A) 147. (D) 148. (A) 149. (A) 150. (B)
151. (A) 152. (C) 153. (B) 154. (C) 155. (B) 156. (B) 157. (C) 158. (B) 159. (C) 160. (A)
161. (C) 162. (D) 163. (B) 164. (D) 165. (D) 166. (B) 167. (B) 168. (C) 169. (C) 170. (B)
171. (C) 172. (B) 173. (B) 174. (D) 175. (B) 176. (D) 177. (D) 178. (D) 179. (C) 180. (A)
181. (A) 182. (A) 183. (D) 184. (D) 185. (C) 186. (B) 187. (B) 188. (C) 189. (D) 190. (A)
191. (B) 192. (A) 193. (A) 194. (D) 195. (A) 196. (C) 197. (B) 198. (D) 199. (B) 200. (C)
201. (B) 202. (B) 203. (D) 204. (D) 205. (A) 206. (B) 207. (A) 208. (B) 209. (C) 210. (B)
211. (C) 212. (B)
Exercise – II
(Previous Year AIPMT Examination Questions)
1. (B) 2. (B) 3. (B) 4. (D) 5. (C) 6. (D) 7. (B) 8. (B) 9. (D) 10. (C)
11. (B) 12. (C) 13. (B) 14. (B) 15. (C) 16. (C) 17. (D) 18. (C) 19. (C) 20. (C)
21. (B) 22. (B) 23. (B) 24. (A)

Exercise - I 7. (C) Gravitational force and nuclear force both are


attractive in nature.
q1q2 q1q2
1. (C) Fa 2
, Fb
2 Fa : Fb = K : 1 Q1Q2
4 0r K4 0r 8. (D) Q1 + Q2 = Q ..... (i) and F k .....(ii)
r2
2. (B) Due to mutual repulsion of charges distributed on
the surface of bubble. From (i) and (ii) F kQ1 (Q Q1 )
3.(C) We put a unit positive charge at O. Resultant force r2
due to the charge placed at A and C is zero and dF Q
resultant charge due to B and D is towards D along For F to be maximum dQ 0 Q1 Q2
1 2
the diagonal BD.
1 4q q
q 9. (A) The force between 4q and q; F1
4. (C) Surface charge density 4 0 l2
A
5. (A) Excess of electron gives the negative charge on body. 1 Q q
The force between Q and q; F2
6. (C) All other charges are its integral multiple. 4 0 (l / 2) 2
19. (B) With temperature rise, dielectric constant of liquid
4q 2 4Qq
We want F1 + F2 = 0 or decreases.
l2 l2 1
Q = –q 20. (D) In the presence of medium force becomes times.
K
10. (A) The charge given to a sphere will be distributed 21. (A) Separation between the spheres is not too large as
uniformly over the surface. compared to their radius so due to induction effect
11. (A) The position of the balls in the satellite will become redistribution of charge takes place. Hence effective
as shown below charge separation decreases so force increases.
180° 22. (A) Q = ne = 1014 × 1.6 × 10–19
L L
+Q +Q Q = 1.6 × 10–5 C = 16 C
1 Q2 Electrons are removed, so charge will be positive.
Thus angle = 180° and Force 23. (A) When put 1 cm apart in air, the force between Na and
4 0 (2 L)2 Cl ions = F. When put in water, the force between
F
kQ 2 1 Na and Cl ions
12. (A) F 9 109 12 9 103 N K
2
r (1000)2 24. (D) Positive charge shows the deficiency of electrons.
13. (D) Resultan t charges after adding the – 2C be 14.4 10 19
(–2 + 2) = 0 and (–2 + 6) = +4C Number of electrons 9
1.6 10 19
k Q1Q2 0 4 26. (D) Negative charge means excess of electron which
F 2
0 k
r r2 increases the mass of sphere B.
14.(A) = K 0 = 81 × 8.854 × 10–12 F1 r2
2
1
= 7.17 × 10–10 MKS units 27. (B) F
r2 F2 r1
15. (C) Because in case of metallic sphere either solid or
2
hollow, the charge will reside on the surface of the 50.04
F2 11.25 N
sphere. Since both spheres have same surface area, F20.06
so they can hold equal maximum charge. 1 Fmedium
16. (C) For providing path to charge induced on the surface 28. (A) F i.e. F K
K air
of the carriers which take inflammable material.
29. (A) In second case, charges will be –2 C and +3 C
1 q2 1 q2
17. (B) F12 an d F13 F Q1Q2
4 0 a2 4 0 (a 2) 2 Since F Q1 Q2 i.e. F Q1 Q2
F12
2 40 3 8
F13 4 F = 10 N (Attractive)
F 2 3
30. (B) By using Q = ne Q = 1019 × 1.6 × 10–19 = +1.6 C.
18. (C) Net force on B Fnet FA2 FC2
31. (B) FA = force on C due to charge placed at A

A +15 esu 10 6 2 10 6
9 10 9 1.8 N
(10 10 2 )2
FB = force on C due to charge placed at B
3 cm 6 6
10 2 10
9 109 1.8 N
B FC C (0.1) 2
–20 esu FA
+12 esu 4 cm +2 C
C 120°
FA Fnet FA2 FC2
FB

15 12 12 20 –
FA 20 dyne FC 15 dyne +1 C
2 , 2
3 4 A 10 cm B
Net force on C
Fnet FA2 FC2 (20)2 (15) 2 25 dyne
Fnet ( FA )2 ( FB ) 2 2FA FB cos120o 1.8 N
32. (C) By using Q = ne Q = +2e = +3.2 × 10–19 C q
A
F1 Q1Q2 10 20 8
33. (B) F Q1 Q2
F2 Q1'Q2' 5 5 1
O
2 FB FC
34. (C) By using F Q
9 109.
r2 FA
q q
C B
(2 10 6 )2
F 9 109. 0.144 N Fa 10 4
(0.5) 2 40. (D) By using K K 4
5
Fm 2.5 10
35. (C) Effective air separation between them becomes
infinite so force becomes zero. Q2
41. (C) | FB | | FC | k .
a2
Q2
36. (A) F 9 109 FC
r2 FC sin
FC cos 60° 60°
(1.6 10 19 )2 FB cos 60° A
60° FB sin
9 199 2.3 10 8 N
(10 10 )2 FB 60°

10
37. (C) Number of atoms in given mass 6.02 10 23
63.5 60° 60°
–Q +C
= 9.48 ´ 10 22
B a C
Hence force experienced by the charge at A in the
+ – direction normal to BC is zero.

A 42. (D) They will not experience any force if | FG | | Fe |


B
10 cm
m2 1 q2
9.48 10 22 G .
Transfer of electron between balls (16 10 2 ) 2 4 0 (16 10 2 ) 2
106
= 9.48 ´ 1016 q
4 0G
Hence magnitude of charge gained by each ball. m
43. (B) On rubbing glass rod with silk, excess electron
Q = 9.48 1016 1.6 10–19 = 0.015 C
transferred from glass to silk. So glass rod becomes
Force of attraction between the balls
positive and silk becomes negative.
(0.015)2
F 9 109 2 108 N . r
(0.1)2 e2 e2 rˆ
44. (C) F k rˆ k. r
r2 r3 r
Charge
38. (A) Surface charge density ( )
Surface area Q 80 10 6
45. (C) By Q = Ne or N N 5 1014
–Q + 2 Q e 1.6 10 19
Q1Q2 Q1Q2 r
–2Q 46. (C) F = F or 4 r 2 r'
0 4 0 r '2 K K
a b
c 47. (B) Dielectric constant k
+2Q 0

Permittivity of metals ( ) is assumed to be very high.


48. (C) Potential energy depends upon the charge at peaks
2Q Q of irregularities. Since every event in the universe
So inner
4 b 2 and
Outer
4 c2 leads to the minimisation of energy.
49. (C) Let us consider 1 ball has any type of charge. 1 and
39. (A) In the following figure since | FA | | FB | | FC | and 2 must have different charges, 2 and 4 must have
they are equally inclined with each other, so their different charges i.e. 1 and 4 must have same charges
resultant will be zero. but electrostatics attraction is also present in (1, 4)
which is impossible.
50. (C) After following the guidelines mentioned above 1 ( 7 10 6 ) ( 5 10 6 )
FC 54. (A) F
4 r2
FD 0

+Q 1 35 1012
+Q FAC N
A FA 4 0 r2
B
1 ( 5 10 6 )( 7 10 6 )
F'
4 0 r2
1 35 1012
C N
D +Q 4 0 r2

Fnet FAC FD FA2 FC2 FD Gme m p


55. (A) Gravitational force FG
r2
kq 2 kq 2
Since FA FC and FD 6.7 10 11
9.1 10 31 1.6 10 27
a2 (a 2) 2 FG
(5 10 11 ) 2
2kq 2 kq 2 kq 2 1
Fnet 2 = 3.9 × 10–47 N
a2 2a 2 a2 2
1 e2
q2 1 2 2 Electrostatic force Fe
4 0 r2
4 0 a2 2
9 109 1.6 10 19 1.6 10 19
51. (C) Since both are metals so equal amount of charge will Fe
induce on them. (5 10 11 ) 2
= 9.22 10–8 N
Q2
52. (D) Initially F k.
(fig. A). Finally when a third Fe 9.22 10 8
r2 So, 2.36 1039
spherical conductor comes in contact alternately FG 3.9 10 47
with B and C then removed, so charges on B and C 56. (B) Suppose in the following figure, equilibrium of charge
Q 3Q B is considered. Hence for it’s equilibrium
are and respectively (fig. B)
2 4 | FA | | FC |
Q 3
1 Q2 1 qQ Q
q
B C 4 0 4x2 4 0 x2 4
r
QA = Q q QB = Q
(A) FC
FA
Q/2 3Q/4 A C B
x1 x2

B C x
r Short Trick : For such type of problem the
(B) magnitude of middle charge can be determined if
either of the extreme charge is in equilibrium by using
Q 3Q the following formula.
Now force F ' 3 2 4 If charge A is in equilibrium then
k. F 2
r 8 2
53. (B) When a positively charged body connected to earth, x1
q = – QB
electrons flows from earth to body and body x
becomes neutral. If charge B is in equilibrium then
2
+ x2
+ q QA
+ + e
–
x
+
+ +
+ If the whole system is in equilibrium then use either
+ of the above formula.
57. (C) A movable charge produces electric field and Total potential V = V2q + V–q + V–q = 0
magnetic field both. 63. (A) In non-uniform electric field. Intensity is more, where
58. (A) All charge resides on the outer surface so that the lines are more denser.
according to Gauss law, electric field inside a shell is
zero.
59. (C) Electric lines force due to negative charge are radially
inward. O
EC EB
64. (C)
EA

Enet = 0

EC = E EBC = E
EB = E
60. (B) Potential at the centre O,
120°
1 Q
V 4 . 120° 120°
4 0 a/ 2
10 EA = E EA = E
where Q 10 9 C and a = 8 cm = 8 × 10–2 m
3
+q +q 65. (C) ABCDE is an equipotential surface, on equipotential
surface no work is done in shifting a charge from
A B
one place to another. Hence work done is zero, along
a O path AB, AC, AD & AE
2 66. (B) According to the question, eE = mg
mg
C E
D +q e
+q
qE ae mp
10 67. (C) a ap me
10 9 m
So V 5 9 10 9 3 1500 2 volt
8 10 2 Ewithout dielectric 2 105
68. (C) K 2
2 Ewith dielectric 1 105

1 q q
dV 69. (B) E 9 109
61. (B) E Vx = –xE0 4 0 r2 r2
dX
E r 2 3 10 6 (2.5) 2
62. (B) Obviously, from charge configuration, at the centre q
9 109 9 109
electric field is non-zero. Potential at the centre due = 2.0833 × 10–3
2q q should be less than 2.0833 10–3. In the given set
to 2q charge V2 q
r of options 2 10–3 is the maximum charge which is
smaller than 2.0833 10–3.
2q 70. (A) Suppose electric field is zero at point N in the figure
then
r Q1 = 25 C E2 N E1 Q2 = 36 C

r r
E–q E2q E–q x1 x2
–q –q x = 11 cm
At N |E1| = |E2|
and potential due to – q charge
x 11
q which gives x1 5cm
V (r = distance of centre point) Q2 36
q
r 1 1
Q1 25
71.(B) Because E points along the tangent to the lines of mg
force. If initial velocity is zero, then due to the force, 80. (B) For balance mg = eE E
e
it always moves in the direction of E. Hence will
always move on some lines of force. 4 3 4 22
Also m r d (10 7 )3 1000 kg
3 3 7
dU 1
72. (B) Electrostatic energy density K 0E2 4 / 3 22 / 7 (10 7 ) 3 1000 10
dV 2 E
1.6 10 19
dU
E2 = 260 N/C
dV
q0 81. (A) Electric field inside a conductor is zero.
82. (C) Electric field near the conductor surface is given by

10 and it is perpendicular to surface.


73. (A) 0

A B 83. (D) W = qV = qE.D


100 cm 100 cm 4 = 0.2 × E × (2 cos 60o)
Since VA = VA So WA B = 0 = 0.2 E (2 × 0.5)
74. (C) For equilibrium of q |F1| = |F2|
Q1 = +4 e F2 q F1 Q2 = + e
2m
x1 x2
x 60° E
X
x x x d
Which gives x2
Q1 4e 3 4
1 1 E 20 NC 1
Q2 e 0.2
75. (C) Electric lines of force never intersect the conductor. 84. (C) Due to deutron, intensity of electric field at 1 Å
They are perpendicular and slightly curved near the distance,
surface of conductor. 1 e
E .
mg 1.7 10 9.8 27
4 r2
76. (A) Since qE = mg or E 19
0

q 1.6 10 9 10 9
1.6 10 19
= 10.0 × 10–8 = 1 × 10–7 V/m 1.44 1011 N / C .
10 20
77. (C) Since charge Q moving on equipotential surface so 85. (A) Magnetic lines of force always makes a closed loop.
work done is zero.
V 3000
78. (B) The field produced by charge – 3Q at A, this is E as 88. (A) V = E × r r 6m
mentioned in the Example. E 500
89. (C) The magnitude of electric field in the annular region
3Q of a charged cylindrical capacitor is given by
E
x2 1
E
(along AB directed towards negative charge) 2 0 r where is the charge per unit length
A B and r is the distance from the axis of the cylinder.
Q –3 Q 1
x Thus E
r
Now field at location of – 3Q i.e. field at B due to 90. (D) The electric field is always perpendicular to the
Q E surface of a conductor. On the surface of a metallic
charge Q will be E' (along AB directed
x2 3 solid sphere, the electrical field is oriented normally
away from positive charge) (i.e. directed towards the centre of the sphere).
1 ne Er 2 q
79. (C) E 4
.
r2
n .4 91. (D) E
0 e
0
4 0 r2
0.036 0.1 0.1 360 1.6 10 19
n 105 9 109 1.44 1011 N / C
9 109 1.6 10 19 144 (10 10 ) 2
= 2.5 ´ 105 N/C.
q 101. (C) Kinetic energy = Force × Displacement = qEy
92. (A) Electric field due to a point charge E 2 102. (A) The intensity of electric field inside a hollow
4 0 r
conducting sphere is zero.
2
2 1 30 eE eE
q E 4 0r 2 t12 , For proton s t22
9 109 100 103. (B) For electron s me mp
= 2 10–11 C
1/ 2
93. (C) At O, E 0, V = 0 t22 mp t2 mp mp
+q O –q t12 me t1 me me
E+ E– 104. (D) Due to symmetric charge distribution.
r r 4 3
94. (D) At neutral point 105. (C) In balance condition QE mg r g
3
20 Q 4 (3.14) (0.1 10 6 )3 103 10
k k Q = 80 C E = 262 N/C
(20 10 2 ) 2 (40 10 2 ) 2 3 1.6 10 19
95. (A) Work done in moving a charge from P to L, P to M 106. (A) Side a = 5 × 10–2 m
and P to N is zero. a
Half of the diagonal of the square r
QE 3 10 80 3
2
96. (B) a 12 m / sec 2
m 20 10 3 Electric field at centre due to charge q
Hence v = u + at v = 20 + 12 × 3 = 56 m/s. kq
E
q a
2
97. (C) | E A | | EB | k . 2
a 2
So, Enet E A2 EB2 2 E A EB cos 0o q –2q q –2q
2E E
EB Enet E 2E 90° E
A r O
C
E E
a 60° a –q +2q –q +2q
kq
q q Now field at O 2 2 . 2
a B
E E E 2 a
2
A
2
3 k. q
a2 9 109 10 6 2 2
= 1.02 × 107 N/C
3q (5 10 )2 2
Enet
4 0 a2
(upward)
F 107. (C) Force on each charge is zero. But if any of the charge
98. (A) E q0
Newton / Coulomb
is displaced, the net force starts acting on all of them.
99. (C) At A and C, electric lines are equally spaced and 108. (C) Let neutral point be obtained at a distance x from
dense that’s why EA = EC > EB 20 C charge. Hence at neutral point
100. (B) Joined by a wire means they are at the same potential.
20 80
kQ1 kQ2 x = + 0.033 m
For same potential a ( x)2 (10 x )2
1 a2
Q1 a 1 Q Q
Q2 b 109. (A) E . 2 9 109
4 0 r2 (0.6) 2
Further, the electric field at the surface of the sphere
Q = 8 10–11 C
kQ
having radius R and charge Q is 2 . Q Q
R 9 109 9 109
110. (D) E 500
E1 kQ1 / a 2 Q1 b 2 b r2 (3)2
E2 kQ2 / b2 Q2 a 2 a Q = 0.5 C
Q mg 5 10 5 10
111. (B) E 9 109. 118. (B) QE = mg Q
r2 E 107
5 10 6 = 5 × 10–5 C.
9 109 7 10 4 N /C Since electric field is acting downward so for balance
(0.8) 2
charge must be negative.
112. (B) EA = Electric field at M due to charge placed at A
119. (C) Suppose electric field is zero at N. Hence |EA| = |EB|
EB = Electric field at M due to charge placed at B
Q1 = 10 C EB N EA Q2 = 20 C
EC = Electric field at M due to charge placed at C
C A
x1 x2
EB EA
x = 80 cm
x 80
EC Which gives x1 33 cm
M Q2 20
1 1
A Q1 10
B
120. (B) Electric field at a point due to positive charge acts
As seen from figure | EB | | EC |, so net electric field
away from the charge and due to negative charge it
at M, Enet = EA ; in the direction of vector 2.
act’s towards the charge.
113. (A) By using W Q ( E. r ) q q
E
W Q [(e1iˆ e2 ˆj e3 kˆ).(aiˆ bjˆ)] E E
q q Enet= 0
Q (e1a e2 b) E
E E
Q q q
114. (B) By using V 9 109
r
q
9100 10 6
9 10 105 V E E 2E
9 q q 120°
115. (A) From following figure, E 2E
E E
EA = Electric field at mid point M due to + 5 C charge E 2E
q –q
5 10 6 Enet = 0
9 109 45 10 5 N /C
(0.1) 2 2q 2q
EB = Electric field at M due to +10 C charge 2E
E 2E
q q Enet= 0
9 5 10 6 5 2E E
9 10 45 10 N /C 2E
(0.1) 2 2q 2q
+5 C M + 10 C 2q q
E
2E E E
A EB EA B q 2q
2E E 2E 2E
20 cm
E 2E
2q q
Net electric field at
Enet = 0
M | E B | | E A | = 45 × 105 N/C = 4.5 × 106 N/C, 121. (C) When charge enters perpendicularly in electric field,
in the direction of EB i.e. towards + 5 C charge it describe parabolic path.
122. (C) Because electric field applies the force on electron
116. (D) -particles are charged particles, so they can deflect
in the direction opposite to it’s motion.
by electric field.
117.(a,d) When two opposite charge separated by a certain Energy
123. (C) Energy density so it’s dimensions are
distance then at two points potential is zero. One Volume
point exist between the charges and other exists ML2T 2
outside them. Also no point, exists between the [ML 1T 2 ]
L3
opposite charges. Where electric field is zero. 124. (C) Electric lines of force are always normal to metallic
body.
126. (C) Electric field between sheets x 16
1 Which gives x1 12 cm
E ( ) 0 Q2 e
1 1
2 0 Q1 9e
Q1 =+9 e F2 q F1 Q2 = + e
+ +
+ + x1 x2
+ +
E 0 + E=0 + E 0 x = 16cm
+ + 135. (C) If two opposite charges are separated by a certain
+ + distance, then for it’s equilibrium a third charge
+ +
should be kept outside and near the charge which is
smaller in magnitude.
128. (B) In the following figure, in equilibrium Fe = T sin 30°, Here, suppose third charge q is placed at a distance
r = 1m x from – 2.7 × 10–11C then for it’s equilibrium |F1| = |F2|
Q1 Q2 F2 q F1
30° 30°
1m 1m x
0.2 m
T 30° T cos 30° kQ1q kQ2 q
( x 0.2) 2 x2
+10 C
T sin 30° +10 C
r 1
mg Here k and Q1 5 10 11 C , Q2 2.7 10 11 C
4 0
Q2 9 1 F eE
9 10 . 2 = T 136. (C) a
r 2 m m
(10 10 6 ) 2 1 137. (D) Cathode rays (stream of negatively charged particles)
9 109. T deflect in opposite direction of field i.e. towards
12 2
T = 1.8 N north.
138. (B) When a negatively charged pendulum oscillates over
eE a positively charged plate then effective value of g
129. (A) a a 1.76 1011 50 10 2
m l
= 8.8 × 1014 m/sec2 increases so according to T 2
, T decreases.
g
130. (A) Vertical velocity changes due to electric field, but no 139. (C) When charge q is released in uniform electric field E
change in horizontal velocity.
qE
then its acceleration a (is constant)
m
E So its motion will be uniformly accelerated motion
qE
and its velocity after time t is given by v = at t
+Q m
X 1 2 1 qE q2 E 2t 2
2

131. (C) Inside the hollow charged spherical conductor KE mv t


2 2 m 2m
electric field is zero.
132. (A) By using QE = mg 140. (C) KE = QV = e × 10 V = 1KeV.
3

141. (C) Electric field inside a conductor is always zero.


mg 10 6 10
E 10V /m; QE
Q 10 6 143.(C) Electric force qE = ma a
m
upward because charge is positive.
1.6 10 19 1 103 1.6
133. (B) By using QE = mg a 1015
9 10 31 9
mg 0.003 10 3 10 u = 5 × 10 and v = 0
6
Q = 5 × 10–10 C
E 6 10 4 u2
From v2 = u2 – 2as s
134. (B) Suppose q is placed at a distance x from +9e, then 2a
for equilibrium net force on it must be zero i.e.
(5 10 6 )2 9
|F1| = |F2| Distance s = 7 cm. (approx)
2 1.6 1015
k .Qr r Q
145. (C) For non-conducting sphere Ein 155. (B) By using E 9 109
R3 3 0 R2
V 30 ( 10) Q
146. (A) E 2000V /m. 3 106 9 109 Q = 3 10–3 C
d (2 10 2 ) (3) 2
156. (B) T sin = qE
Q and T cos = mg
148. (A) E 9 109
r2 +
E r 1 (0.1)2 2 +
12
Q 9
1.11 10 C +
9 10 9 109
+
149. (A) In equilibrium T T cos
+
mg 9.6 10 16 10 + qE
QE = mg n 3 + T sin
Ee 20, 000 1.6 10 19
D C D C mg

qE q
EB tan
EA mg mg 2 0

EC O 2E 2E
150. (B) tan
ED
157. (C) The net field will be zero at a point outside the
A B B charges and near the charge which is smaller in
A
magnitude.
EA = E, EB = 2E, EC = 3 E, ED = 4 E
151. (A) Net field at origin +8q
–2q P
q 1 1 1
E .... x=0 x=L
4 0 12 22 42 L l
q 1 1 Suppose E.F. is zero at P as shown.
1 .....
4 0 4 16 8q k .(2 q)
Hence at P; k . 2 l = L.
(L l) l2
q 1 So distance of P from origin is L + L = 2L.
12 109 q N / C
4 1 2 ˆ 2
0 1 kˆ kˆ kˆ
4 158. (B) E 2 2 o
k
2
o o o

152. (C) T ( mg ) 2 (QE ) 2 159. (C) Electric field between the plates is
–
6
(30.7 10 9.8) 2 (2 10 8
20000) 2
= 5 × 10–4 N
153. (B) Relation for electric field is given by
EE
E
2 0r ( )
(Given : E = 7.182 × 108 N/C) 2 2 0
0
r = 2 cm = 2 × 10–2 m
1 9 2 2 rE volt/meter
9 10 2 rE 0
0
4 0
0
2
160. (A) The negative charge oscillates, the resultant force
1 2 10 7.182 10 2 8
acts as a restoring force and proportional to
7.98 10 4 C / m displacement. When it reaches the plane XY, the
2 9 109
resultant force is zero and the mass moves down
F mg due to inertia. Thus oscillation is set.
154. (C) E
q e 161. (C) Electric field outside of the sphere
9 10 31 9.8 11 kQ
5.5 10 N /C Eout ...(i)
1.6 10 19 r2
Electric field inside the dielectric sphere 1
kQx where tan tan
Ein 3 2 3
...(ii)
R3
1 3 1 3
r2x tan so, tan
2 3 2
From (i) and (ii), Ein Eout
R A p
3(20)2 +q
At 3 cm, E 100 120 V / m
pnet
103
162. (D) The total force on Q l l
171. (C)
Qq 4Q 2 60°
0 B C
2
4 0l 2 l
l +q –2q p
4 0
2
pnet p2 p2 2 pp cos 60
4Q q Q
3p 3 ql ( p = ql)
x= 0 x =l / x=1
Qq 4Q 2 173. (B) The direction of electric field at equatorial point A or
q Q.
l
2
4 0l 2 B will be in opposite direction, as that of direction of
4 0 dipole moment.
4
163. (B) Maximum torque = pE A
= 2 ×10–6 × 3 × 10–2 × 2 × 105 = 12 × 10–3 N-m.
164. (D) Work done – +
270
270
pE sin d [ pE cos ] 90 0 B
90

+q P
E 174. (D) Dipole moment p = 4 × 10–8 × 2 × 10–4 = 8 × 10–12 m
Maximum torque = pE = 8 × 10–12 × 4 × 108
= 32 × 10–4 Nm
–q Work done in rotating through 180o = 2pE
165. (D) Field along the axis of the dipole = 2 × 32 × 10–4 = 64 × 10–4 J
1 2p 1 2(q r ) 2kp kp
E . . ; 175. (B) We have Ea and Ee ;
4 0 d3 4 0 d3 r3 r3
Ea = 2Ee
qr
E 176. (D) Point charge produces non-uniform electric field.
d3
166. (B) p = q × (2l) = 1.6 × 10–19 × 10–10 = 1.6 × 10–29 C-m kp
177. (D) Eequatorial i.e. E p and E r–3
r3
1 2p
167. (B) E 4
.
r3
2p k. p
0 178. (D) Eaxial = Eequatorial k.
x3 y3
168. (C) Dipole moment p = q (2l)
= 3.2 × 10–19 × (2.4 × 10–10) x 21/ 3 3
2 :1
= 7.68 × 10–29 C - m y 1
179. (C) In uniform electric field dipole experience only torque,
170. (B) but no force.
E
–q +q
Y
P
180. (A) 10 cm 20 cm
25 cm
–Q +Q
/3
X 2pr
O By using E 9 109.
; where
P (r l 2 )2 2

p = (500 × 10–6) × (10 × 10–2) = 5 × 10–5 c × m,


r = 25 cm = 0.25 m, l = 5 cm = 0.05 m 196. (C) To apply Gauss’s theorem it is essential that charge
9 5 should be placed inside a closed surface. So imagine
9 10 2 5 10 0.25
E 2 2 2
another similar cylindrical vessel above it as shown
{(0.25) (0.05) } in figure (dotted).
= 6.25 × 107 N/C
p
182. (A) V 9 109.
r2
(1.6 10 19 ) 1.28 10 10
9 109 = 0.13V
(12 10 10 ) 2 q

p cos
183. (D) V If = 0° then Va = max.
r2
If = 180° then Ve = min.
184. (D) Potential due to dipole in general position is given
by
k . p cos k . p cos r k .( p.r ) 1
V V 198. (D) e E
r2 r3 r3 2 0r r
185. (C) In the given condition angle between p and E is 199. (B) Charge enclosed by cylindrical surface (length
zero. Hence potential energy U = –pE cos 0 = 100 cm) is Qenc = 100 Q. By applying Gauss’s law
–pE = min. 1 1
Also in uniform electric field Fnet = 0. (Qenc. ) (100Q )
0 0

200. (C) S.I. unit of electric flux is


2p kp Ea 2
187. (B) Ea k EE N m2 J m
3 and 3 EE 1 = volt × m.
r r C C
188. (C) = pE = q (2l) E = 2 × 10 × 0.01 × 5 × 105
–6
max
1
= 10 × 10–3 N-m 201. (B) By using E dA (Qenc )
189. (D) W = PE (1 – cos ) here = 180o 0

W = PE(1 – cos 180°) = PE [1 – (–1)] = 2 PE


190. (A) U = –PE cos 1
203. (D) By Gauss’s law (Qenclosed)
It has minimum value when = 0o 0

i.e. (U)min = –PE × cos 0° = – PE Qenclosed = 0 = (–8 × 103 + 4 × 103) 0


191. (B) Stationary electric dipole has electric field only. = –4 × 103 0 Coulomb.
192. (A) Suppose neutral point N lies at a distance x from Q
dipole of moment p or at a distance x2 from dipole of 204. (D) Net flux through the cube net ; so flux through
64 p. 0

q
p N 64 p one face face
6 0
1 2
x1 1
205. (A) surface (Qenclosed )
25 cm 0
At N |E. F. due to dipole |= |E. F. due to dipole |
+
1 2p 1 2(64 p ) + + Qenc
. .
4 x3 4 (25 x ) 3 +
0 0
Spherical +
+ Closed
1 64 conductor
x = 5 cm. + surface
3
x (25 x ) 3 + +
193. (A) As there is no charge residing inside the cube, hence 1
net flux is zero. 206. (B) net Qenc Qenc = ( 2 – 1) 0
0
q
194. (D) 0 i.e. net charge on dipole is zero. q 4 q
0 207. (A) face
6 0 6 (4 0 )
195. (A) Electric flux coming out through a closed surface
is q/ 0.
1 1 1
208. (B) Qenc (2 q) S= × 6×1 = 3m
0 0 2
209. (C) The electric field is due to all charges present whether 9
inside or outside the given surface. Average speed = = 3m/s
3
3
211. (C) In electric dipole, the flux coming out from positive Average velocity = = 1 m/s
3
charge is equal to the flux coming in at negative
charge i.e. total charge on sphere = 0. From Gauss 22. E = E1 + E2
law, total flux passing through the sphere = 0. E + E1 + E2
212. (B) According to Gauss’s applications.
E= +
2 0 R 2 0 R
Exercise - II
E= N/C
0 R
19. (C) Fe = eE FP = eE
eE id
eE : d£R 0
ae = ap = 2 R2
me mp 23. B=
0i
: d>R
1 eE 2 1 eE 2 2 d
h= te = tp
2 me 2 mp
io
te me
= te < tp 2 R
tp mp

eE
20. (C) V = V0 + t
m
d =R d
h h q –q
0 ; =
mV0 eE 24.
m V0 + t r
m A B
2
–Kq
= 0 F=
r2
eE
1+ t 25% of charge from A is transferred to B.
mV0
3q –q+q/4
21. (B) t=0 t = 1 sec 4
3m A r B
–3q
t = 3 sec t=2 =
4
V = + at
New Force
2E
6= ×1 3q –3q
m k
4 4 –9 kq 2 = 9F
2E F = = 16
a= = 6m/s 2 r2 16 r 2
m
(iv) Circular Disc
Potential difference between two points is defined as work done
by external agent in transporting unit test charge from one point
V= R2 r2 r
to another. It is given by
A
WB A
VA VB E.dr
q B
Remarks :
(i) Potential decreases in the direction of electric
field.
(ii) Potential at infinity is taken as zero. But this
convention fails when charge distribution itself (v) Uniformly Charged Spherical Body
extends to infinity like infinite line charge and
infinite charged sheet. In such cases potential at 1 Q R3
Vout = . = If r > R
any point is expressed by expression given below, 4 0 r 2 0r
where V 0 is potential assigned to charge
distribution. 1 R2
Vsurface = . = If r = R
4 0 2 0r
(i) Isolated point charge
1 Q
V= .
4 0 r

(ii) Infinitely long line of charge r2


1 Q[3R 2 r2 ] R2
Vin = . = 3
V=– ln (r) + V0 = – E.dr 4 0 2R 3 6 0 R2
2 0 where = Volume charge density
(vi) Conducting spherical body
1 Q
Vout = . if r > R
4 0 r
1 Q
Vs = if r = R
4 0 R
(iii) An infinite sheet of uniformly distributed charge with
density

V= + V0

Vin = Vs = constant if r < R


KP
(i) It is a locus of all points having equal potential. (iii) Electric potential on axial point is V while at
(ii) Electric field is always perpendicular to equipotential r2
surface. equatorial point V = 0
(iii) Work done to move a charge on equipotential surface Potential of the general point due to a dipole is given by
is always zero.
Equipotential surfaces
KP cos
V
r2
(iv) Torque on dipole in uniform electric field is given by
V1 >V2 >V3
V3
P E
V2 (v) Work done to rotate a dipole in a electric field
Electric V1
field q 2
W d PE(cos 1 cos 2)

(vi) Potential energy of dipole in uniform electric field


U P.E –PE cos
Equipotential surface
Remark :
due to a point charge
(i) Fnet = 0, while may or may not be zero depending on
Equipotential surfaces orientation.
V1 >V2 >V3 (ii) In non uniform electric field Fnet 0 , while may or
V1 V2 V3
may not be zero.
(iii) For an ideal dipole q 0 such that qa is finite.

Electric
field Total work done in making an assembly of charges by bringing
them from infinity to their respective places is known as the
potential energy of the system.
(a) For system of two charges, W 1 q1q 2
4 0 r
Equipotential surface
(b) For system of three charges
due to uniform electric field
q1q 2 q 2q3 q3 q1
Electric Dipole W
4 0 r12 4 0 r23 4 0 r31
(i) It is a configuration in which two equal and opposite
charges are lying very close to each other. (c) For system of N- charges
N N q jq k
1 1
W
2 j 1 k 1 4 0 rjk
j k

(d) P.E. of charge q in potential field U = qV. Interaction


energy of a system of two charges
U = q1V2 = q2V1
(i) Dipole moment P q.2a , it is a vector pointing from (e) For a continuous charge distribution
–q to +q charge (unit C–m) 1
W qdV
2KP 2
(ii) Electric field on axial point of a dipole is E axial
r3
KP (i) Potential energy of a system of charges is a unique
while at an equatorial point Eequatorial 3
r property of the system.
Electric field on the general point due to a dipole is given (ii) Since electric field is conservative (like gravitation field).
by Law of conservation of mechanical energy is applicable
KP 1 3 cos 2 K U 0, K Change in KE
E
3 U Change in PE
r
(c) If V1 V2 then Final (EPE) < initial (EPE) and energy
loss (Heat)
1. When a capacitor (C), charged upto V volt is discharged C1C 2
by means of any resistance then heat loss H = CV2/2 = 2[C C ] . [V1 – V2]2
1 2
(Not depends on R)
2. Potential energy stored in electric field of a conducting Above energy is lost in connecting wires and heat loss
sphere is U = CV2/2 = Q2/2C = QV/2 = kQ2/2R and for non not depends on resistance of wires.
conducting sphere is U = (3/5)kQ2/R (M.T.R.) 2. Sharing of charges not take place, if only one plate is
connected together as shown in figure or both the
3. Capacity not depends on the potential, charge given
capacitors have equal potential
and nature of metal.
3. If a charged capacitor of energy E is connected to an
4. If a sphere is charged with a constant rate then its identical uncharged capacitor. Then final electrostatic
potential rises linearly with time [As Q = CV so dQ/dt = potential energy of system is will be E/2. Heat loss will
C.dV/dt] be E/2 and energy of each capacitor will be E/4.
5. Work done to charge a sphere from V1 to V2 is Wby
= change in electrostatic potential energy = C[V22 – V12]/2.
6. Work done by battery Wb = EMF × charge given by
battery = E ( Q)
7. Application of capacitors (i) for tuning in radio (ii) Filters
(iii) Elimination of sparking (iv) timing circuits
(v) storing charge, for use in research. 1. First successful capacitor is Leyden jar (1745)
8. Atmospheric electricity : 2. Capacity of a conductor also depends on, conductors
(i) Dielectric strength : The maximum electric field, that a placed nearer to it and increase, if unlike charged
medium can withstand. Above which medium ionizes. conductor placed near to it
3. If an uncharged object B, come in close to charged object
Medium Vacuum Air W ater A, then charge of A remains same but its potential
Dielectric strength 6
3 × 10 V/m – decreases.
Dielectric co ns tant 1 1.005 80 4. If capacity of a conductor (Radius R) = The capacity of
a parallel plate capacitor having circular (Radius a) plates
(ii) At ground level, there is a downward vertical E of about then distance between the plates d = a2/4R.
100 V/m all over the earth and gets weak as we go to 5. For a parallel plate capacitor (PPC)
higher altitudes. (At 50 km altitude it is negligible) (i) E (Between the plates) = / 0 = V/d (uniform)
(iii) There is a potential difference of about 4 × 105 V. between
(ii) Force = CV2/2d = QE/2 (iii) Pressure = 2
/2 0
50 km altitude and ground level of earth .
(iv) Order of on earth surface = – 0 × 100 (by using E (iv) EPE = CV2/2 = 0 AV
V 2
/2d
= / 0) ~ – 10–9 C/m2 ( – ve is due to direction of E). 6. For any capacitor (PPC, spherical, cylindrical), if
Thus total charge q ~ – 0.5 × 106 cb. separation between the surfaces increases, then, capacity
(v) Vehicles carrying inflammable materials usually have decreases.
metallic chains touching the ground. 7. A test charge q experiences a force F, when placed
between the plates of PPC. Now if one of the plates is
removed, then the force on the test charge will become
1. To solve the problems related to connection of two
F/2 because electric field reduced to half.
spheres or capacitors without any source, use theory of
8. Capacity of PPC not depends on nature of metal and
sharing of charges. In which
thickness of the plates.
(a) Potential of any sphere (After connection = A.C.) or
9. If one of the plates of PPC slides relatively then C
common potential
decreases [As overlapping area decreases]
10. If both the plates of PPC are touched each other resultant
Vcm = charge and potential becomes zero.
11. A material suitable for dielectric use in capacitor must
(b) To calculate charge A.C., devide total charge in the have high r (dielectric constant) and high X (dielectric
ratio of radii or capacity. strength)
7. N identical capacitors are connected in parallel to a
1. If nothing is mentioned then assume battery is battery of emf V. Now they are disconnected from battery
disconnected and Q is constant. and capacitors connected in series then potential
2. A PPC is connected to battery (V - constant) and a slab difference between the free plates (Or for combination)
of dielectric constant K is inserted between the plates is NV. and E.P.E. remains same.
then total energy given by battery is devided into two
parts.
(i) Half is used to insert the slab. (Work is done by field) 1. Maximum capacity obtained from N plates is C e =
(ii) Half is stored in form of E.P.E. (N – 1)C. It is also used to calculate minimum number of
plates to obtain desired capacity e.g. to obtain 10 C
minimum number of plates required = 11.
1. For a given voltage (Source), to store maximum energy
2. If space between the plates is devided equally in two
capacitors should be connected in parallel.
parts, use mean method e.g.
2. If N identical capacitors are connected
(i) Distance division (Dielectric is parallel to plates)
(i) In series then CS = C/N and (ii) In parallel Cp = NC
Ce = (H.M. of K1 and K2) × C
also Cp/CS = N2
3. If N resistances are connected in (i) series RS = NR. and (ii) Area division Ce = (A. M. of K1 and K2) × C
(ii) Parallel RP = R/N. Also RS/RP = N2 where C = capacity of PPC when no dielectric is used.
4. You have N identical capacitors [Capacity of each = Ce] 3. Distance division Area devision
each can withstand upto voltage Ve. Then to design a
system which can tolerate upto voltage V and whose
Ce Ve2 CV 2
capacity is C use N. 2 = similarly for
2
resistance use N[Ie2 Re] = I2R C = when no dielectric is used also C2 > C1 > C3
5. If N identical capacitors each having breakdown voltage
2K K 1
V are joined in (i) Series, then the breakdown voltage C1 = .C C2 = .C C3 = C
of the combination is = NV. (ii) Parallel, then the K 1 2
breakdown voltage of the combination is V. 4. A slab of thickness 't' is placed between the plates of
6. Two capacitors are connected in series with a battery. PPC then C = 0 A/[d – t + t/K], which not depends on
Now battery is removed and loose wires connected
position of slab and C increases as K increases
together (figure) then final charge on each capacitors
will be zero. 5. Capacity of spherical systems.

System Inner has given charge Inner is earthed Connected Connected spheres
(outer is earthed) and outer has given Spheres (far away)
charge

Diagram

Q'=(– a/b) Q
Capacity C=4p 0 ab/[b–a] C=4 0b
2
/[b–a] C=4 0b C = C1 + C2

= capacity of [M.T.R.] C=4 0 (a + b)


spherical MTR
capacitor + 4 0b

[M.T.R.]
6. Capacity of cylindrical capacitror is C = 2 0 L/ln(b/a) M.T.R.

1. (A) CHARGING (B) DISCHARGING


q = CV, i = V/R VC + VR = 0
VC + VR = V ......(1) VC VR .......(1)
q = CV
^^^^^
R C ^^^^^
R C
V
V

Charging means q increases. So if q Discharging means q decreases.


then VC increases. So from eqn. (1) So VC decreases. So VR
VR decreases and hence i decreases and hence i decrease

2. In discharging all the parameters (q, VC, VR, i) B exponentially


3. In both charging and discharging current decreases exponentially i.e. capacitor blocks D.C.
4. In time t = RC = (time constant) any parameter changes by 63%
5. If time constant ( ) of a circuit is very less, then initial changes in any parameter (q, VC, VR, i) is so rapid.
1. Sphere of radius 4 cm is suspended inside a hollow If the particle of negative charge q' is situated at a distance
spherical conductor of radius 6 cms. The hollow sphere is x, then the electric force on the particle is
charged upto 3 e.s.u and the outer surface is connected
1 qq '
to earth. The potential difference between the spheres F = – q'E = – x.
3
will be- 4 0 R
(A) 36 e.s.u. (B) 54 e.s.u. Since the ring has positive charge and the particle has
(C) 30 e.s.u. (D) 0.25 e.s.u. negative charge, hence the force F will be of attraction
(towards the ring). Under the action of this force the
Kq Kq
Sol. (D) V = Vsmall – Vlarge = particle will perform periodic motion in line on with side
r R of the centre of the ring. The acceleration of the particle is
given by
1 1
= Kq .....(1) F
r R
a=
m
K = 1 dyne cm2/stat. coulomb2
q = 3 e.s.u. r = 4 cm 1 qq '
R = 6 cm =– 4 x =– 2x.
0 mR 3
1 1
From (1) V = 3 = 0.25 e.s.u. 1 qq '
4 6
where = 4 0 mR 3
2. A sphere of radius 5 cm has electric field 5 × 106 V/m on its
surface. What will be the force acting on a charge of 5 ×
10–8C placed at distance of 20cm from the centre of sphere- (1 10 5 )(1 10 6 )
= (9 10 9 ) = 10
(A) 1.5 × 10–2 N (B) 40 N (0.9 10 3 )(1)3
(C) 4 N (D) 0 N
a x
kq kq Since the acceleration a is directly proportional to the
Sol. (A) E0 = , Es =
r2 R2 displacement x and is directed towards the centre of the
qE s R 2 ring, hence the motion of the particle is simple harmonic.
F = qE0 =
r2 The time period of the particle is
[q = 5 × 10–8 C, Es = 5 × 106 v/m, R = 0.05 m, r = 0.20 cm]
2 2
= 1.5 × 10–2N T= T= = sec
10 5
3. A thin stationary ring of radius 1m has a positive charge
of 1 × 10–5 coulomb uniformly distributed over it. A particle 4. A circular ring of radius R with uniform positive charge
of mass 0.9 gm and having a negative charge of 1 × 10–6 density ' ' per unit length is located in the y-z plane with
coulomb is placed on the axis at a distance of 1 cm from its centre at the origin O. A particle of mass m and positive
the centre of the ring. Show that the motion of the charge q is projected from the point P ( 3 R, 0, 0) on the
negatively-charged particle is approximately simple
positive x-axis directly towards O, with an initial speed v.
harmonic. Calculate the time-period of oscillation.
Find the smallest (non-zero) value of speed v such that
Sol. The electric field on the axis of a charged ring of radius R
the particle does not return P.
meter at a distance x from its centre is given by
Sol. The electric field at the centre of ring is zero. Therefore
1 qx the force on charged particle at the centre of ring is zero.
E= 4 where q is the charge on the
0 (R 2 x 2 )3 / 2 Hence if the particle reaches at the centre O of the ring,
ring. If the distance x is very small compared to radius R, then it will not return to P. For minimum value of velocity
then x2 can be neglected in R2 + x2. In this position, we v, the speed of particle at the centre must be zero. By the
principle of conservation of energy.
1 qx
have E = 4 . (K.E. + P.E.) at P = (K.E. + P.E.) at O
3
0 R
The gain in K.E. of electron K = e (V2 – V1)

1 q q q q
=e4 r2 b r1 b
0

1 1 1
= 4 eq r r1
0 2

= 9 × 109 × 1.6 × 10–19 ×

4 1 1
× 10–8 2 2
9 2 10 3 10

32
1 1 (2 R )q = × 10–19 Joule
mv2 + 4 0
3
2 ( 3R) 2 R2
1 32
mv2 = × 10–19 joule
1 (2 R )q 2 3
=0 + 4 R
0 v = 1.54 × 107 m/s [ m = 9 × 10–31 kg]
6. A charge Q is distributed over two concentric hollow
1 1 (2 R )q (2 R )q spheres of radii r and R (> r) such that the surface densities
mv2 = 4 R 2R
2 0 are equal. Find the potential at the common centre.
Sol. Suppose the charges on the spheres of radii r and R are
q Qr, and QR respectively, Then Q = Qr + QR
v = 2 0m Qr QR
Let the surface density be . Then = 2 =
4 r 4 R2
5. The radii of internal and external spheres of concentric
spherical air capacitor are 1 cm and 4 cm respectively. A Qr r2
or Q + 1 = 2 + 1
potential difference of 3000 volts is applied between the R R
spheres. What velocity will be imparted to an electron.
Qr QR r2 R2
when it approaches from a distance of r1 = 3 cm to r2 = 2 or =
QR R 2
cm as measured from the centre of spheres.
Sol. The potential differences between spherical conductors Q r2 R2
or Q =
1 q q R R2
Vab = 4 a b
0 R2
QR = Q ,
Here a = 1cm = 10-2 m, r2 R2
b = 4 cm = 4 × 10-2 m
R2
1 1 Similarly, Qr = Q
r2 R2
3000 = 9 × 109 q 2 2
10 4 10
Suppose the potentials at the common centre due to QR
and Qr be VR and Vr respectively. Then
4
q= × 10–8 coul.
9 QR QR 2
VR = 4 = 2
The potential at, a point distant 3 cm from centre, 0R 4 0 R(r R2)

1 q q
V1 = 4 Qr Qr 2
0 r1 b Vr = 4 = 2
0r 4 0 r(r R2)
The potential at a point distant 2 cm from centre,
Q R r
1 q q V = VR + Vr = 2
V2 = 4
4 0 r R2
0 r2 b
7. A point particle of mass M is attached to one end of a 8. The capacitor of capacitance 4 F and 6 F are connected
massless rigid non-conducting rod of length l. Another in series. A potential difference of 500 volts is applied to
point particle of the same mass is attached to the other the outer plates of the two capacitor system. Then the
end of the rod. The two particles carry charges +q and –q charge on each capacitor is numerically-
respectively. This arrangement is held in a region of a (A) 6000 C (B) 1200C
uniform electric field E such that the rod makes angle (C) 1200 C (D) 6000 C
(say of 50) with the field direction. Find an expression for
the minimum time needed for the rod to become parallel to C1C 2
Sol. [B] CR = C C = 2.4 F
the field after it is set free. 1 2
Sol. The rod comes to equilibrium position due to a torque. Charge flown through the circuit
The moment of inertia of system about O,
= 2.4 × 500 × 10–6C = 1200 C
2 2 9. Three capacitors are connected to D.C. source of 100 volts
M 2
I=M +M = as shown in the adjoining figure. If the charge accumulated
2 2 2
on plates of C1, C2 and C3 are qa , qb , qc , qd , qe and qf
respectively, then

The electric force qE and –qE act on A and B, along and

opposite to the direction of electric field E respectively..


Therefore net electric force on system 100
= qE – qE = 0 (A) qb + qd + qf = C
2
These two force form a couple of moment
(B) qb + qd + qf = 0
= (qE) sin (C) qa + qc + qe = 50 C
Restoring couple = – qE sin (D) qb = qd = qf
If is angular acceleration, then from relation = I , we Sol. [D] In series combination, charge is same on capacitor.
have 10. Three capacitors each of capacitance 1 F are connected
I = –qE sin = –qE in parallel. To this combination, a fourth capacitor of
[ sin = , for small value of ] capacitance 1 F is connected in series. The resultant
capacitance of the system is-
qE (A) 4 F (B) 2 F
Angular acceleration =
I
4 3
– (C) F (D) F
3 4
i.e. motion is angular S.H.M. for which standard equation
is = – 2 Sol. [D] Resultant of parallel combination
= 3 × 10–6 F
qE 2qE
2= = Total capacitance of combination is
I M
1 1 4
=
C r 3 10 6 = × 106
T 2 / M 3
t= = = = 3
4 4 2 2 2qE
Cr = F
4
11. In the circuit diagram shown in the adjoining figure, the 13. Two capacitors each of 0.5 F capacitance are connected
resultant capacitance between P and Q is- in parallel and are then charged by 200 volts. D.C. supply.
The total energy of their charges (in joules) is-
(A) 0.01 (B) 0.02
(C) 0.04 (D) 0.06

1
Sol. (B) CR = 1 F, ER = C V2
2 R

1
(A) 47 F (B) 3 F ER = × 1 × 10–6 × 200 × 200
2
(C) 60 F (D) 10 F
= 2 × 10–2 = 0.02J
Sol. [B] The given circuit can be drawn as
14. An infinite number of identical capacitors each of
capacitance 1 F are connected as in adjoining figure. Then
the equivalent capacitance between A and B is-

Where C = (3 + 2) F = 5 F

1 1 1 1 20 1
= =
CPQ 5 20 12 60 3
(A) 1 F (B) 2 F
CPQ = 3 F
1
12. Four condensers each capacity 4 F are connected as (C) F (D)
2
shown in figure VP – VQ = 15 volts. The energy stored in
the system is- Sol. [B] This combination forms a G.P.,
(A) 2400 ergs (B) 1800 ergs 1 1 1
Sum = S = 1 + + + +...........
(C) 3600 ergs (D) 5400 ergs 2 4 8
Sol. [B] Total capacitance of given system] a
Sum of infinite G.P. is S =
1 1 r
5 8
=
CR 8 CR = F 1
5 Here a = first term = 1 and r = common ratio =
2
1
S= = 2 CR = 2 F
1 1/ 2
15. Four capacitors are connected as shown in the adjoining
figure. The potential difference between A and B is 1500
volts. The energy stored in 2 F capacitance will be-

1
Energy stored = C V2
2 R

1 8
= × × 10–6 × 225
2 5
= 180 × 10–6 joule
= 180 × 10–6 × 107 ergs (A) 5.06J (B) 0.506J
= 1800 ergs (C) 50.6J (D) none of the above
Sol. [D] C = 2 + 3 = 5 F
q q q q
VAB =
12 20 5 3
q
1500 = q = 4500 C
3
4500
Hence potential difference across 2 F capacitor =
5 (A) (C1 + C2)/C1 (B) C1 / (C1 + C2)
= 900 V
(C) C1C2 (D) C1/C2
1
Energy stored = × 2 × 10–6 (900)2 = 0.81 J q2
2 Sol. [A] Energy =
16. Four capacitors of each capacity 3 F are connected as 2C
shown in the adjoining figure. The ratio of equivalent Ebefore q 2 / 2C1
capacitance between A and B and between A and C will = 2
Eafter q /[2(C1 C 2 )]
be-
( q remains same)
E B C1 C 2
EA = C1
18. Four plates of the same area of cross-section are joined as
shown in the figure. The distance between each plate is d.
The equivalent capacity across AB will be-

(A) 4 : 3 (B) 3 : 4
(C) 2 : 3 (D) 3 : 2
2 0A 3 0A
(A) (B)
3 3 3 d d
Sol. [A] CAB = 3 + = 4, CAC = + = 3
3 2 2
3 0A 0A
CAB : CAC = 4 : 3 (C) (D)
2d d
17. A capacitor of capacity C1 is charged to the potential of
V0. On disconnecting with the battery, it is connected Sol. [B] The arrangement shown in the figure is equivalent to
with a capacitor of capacity C2 as shown in the adjoining three capacitors in parallel hence resultant capacitance
figure. The ratio of energies before after the connection 3 0A
of switch S will be- =
d
1. Consider two conducting spheres of radii R1 and R2 with 12. Two-point charges of magnitude +q and –q are placed at
R1> R2. If the two are at the same potential, the larger
d d
sphere has more charge than the smaller sphere. State , 0, 0 and , 0, 0 , respectively. Find the equation
whether the charge density of the smaller sphere is more 2 2
or less than that of the larger one. of the equipotential surface where the potential is zero.
2. Do free electrons travel to region of higher potential or 13. A parallel plate capacitor is filled by a dielectric whose
lower potential? relative permittivity varies with the applied voltage (U) as
3. Can there be a potential difference between two adjacent = U where = 2V–1.A similar capacitor with no dielectric
conductors carrying the same charge? is charged to U0 = 78 V. It is then connected to the
4. Can the potential function have a maximum or minimum in uncharged capacitor with the dielectric. Find the final
free space? voltage on the capacitors. 4. A capacitor is made of two
circular plates of radius R each, separated by a distance
5. A test charge q is made to move in the electric field of a
d< constant voltage. A thin conducting disc of radius r<
point charge Q along two different closed paths (Fig.).
thickness t< the minimum voltage required to lift the disc
First path has sections along and perpendicular to lines
if the mass of the disc is m.
of electric field. Second path is a rectangular loop of the
same area as the first loop. How does the work done 14. (a) In a quark model of elementary particles, a neutron is
compare in the two cases? 2
made of one up quarks [charge e] and two down
3
1
quarks [charges – e]. Assume that they have a
3
Q
triangle configuration with side length of the order of
10.5 m. Calculate electrostatic potential energy of
neutron and compare it with its mass 939 MeV.
(b) Repeat above exercise for a proton which is made of
two up and one down quark.
a 15. Two metal spheres, one of radius R and the other of radius
b
2R, both have same surface charge density . They are
6. Prove that a closed equipotential surface with no charge brought in contact and separated. What will be new surface
within itself must enclose an equipotential volume. charge densities on them?
7. A capacitor has some dielectric between its plates, and 16. In the circuit shown in Fig. initially K1 is closed and K2 is
the capacitor is connected to a DC source. The battery is open. What are the charges on each capacitors. Then K1
now disconnected and then the dielectric is removed. was opened and K2 was closed (order is important), what
State whether the capacitance, the energy stored in it, will be the charge on each capacitor now? [C = 1 F]
electric field, charge stored and the voltage will increase,
C1= 6C k1 k2
decrease or remain constant.
8. Prove that, if an insulated, uncharged conductor is placed
near a charged conductor and no other conductors are
present, the uncharged body must be intermediate in E = 9V
potential between that of the charged body and that of C2= 3C C3= 3C
infinity.
9. Calculate potential energy of a point charge –q placed
17. Calculate potential on the axis of a disc of radius R due to
along the axis due to a charge +Q uniformly distributed
a charge Q uniformly distributed on its surface.
along a ring of radius R. Sketch P.E. as a function of axial
distance z from the centre of the ring. Looking at graph, 18. Two charges q1 and q2 are placed at (0, 0, d) and (0, 0, –d)
can you see what would happen if –q is displaced slightly respectively. Find locus of points where the potential a
from the centre of the ring (along the axis)? zero.
10. Calculate potential on the axis of a ring due to charge Q 19. Two charges –q each are separated by distance 2d. A
uniformly distributed along the ring of radius R. third charge + q is kept at mid point O. Find potential
energy of + q as a function of small distance x from O due
11. Find the equation of the equipotentials for an infinite
to – q charges. Sketch P.E. v/s x and convince yourself
cylinder of radius r0, carrying charge of linear density .
that the charge at O is in an unstable equilibrium.
1. Inside a hollow charged spherical conductor, the potential 8. Two spheres A and B of radius 4cm and 6cm are given
(A) Is constant charges of 80 C and 40 C respectively. If they are
(B) Varies directly as the distance from the centre connected by a fine wire, the amount of charge flowing
(C) Varies inversely as the distance from the centre from one to the other is
(D) Varies inversely as the square of the distance from (A) 20 Cfrom A to B (B) 16 C from A to B
the centre (C) 32 C from B to A (D) 32 C from A to B
2. Two small spheres each carrying a charge q are placed r 9. Two plates are 2 cm apart, a potential difference of 10 volt
metre apart. If one of the spheres is taken around the is applied between them, the electric field between the
other one in a circular path of radius r, the work done will plates is
be equal to (A) 20 N/C (B) 500 N/C
(A) Force between them × r (C) 5 N/C (D) 250 N/C
(B) Force between them × 2 r 10. Three particles, each having a charge of 10 C are placed
(C) Force between them / 2 r at the corners of an equilateral triangle of side 10 cm. The
(D) Zero electrostatic potential energy of the system is (Given
3. Two charged spheres of radii 10 cm and 15 cm are 1
9 109 N m 2 / C 2 )
connected by a thin wire. No current will flow, if they have 4 0
(A) The same charge on each
(A) Zero (B) Infinite
(B) The same potential
(C) 27 J (D) 100 J
(C) The same energy
11. Four equal charges Q are placed at the four corners of a
(D) The same field on their surfaces
square of each side is ‘a’. Work done in removing a charge
4. The electric potential V at any point O (x, y, z all in metres) – Q from its centre to infinity is
in space is given by V = 4x2 volt. The electric field at the
point (1m, 0, 2m) in volt/metre is 2Q2
(A) 0 (B)
(A) 8 along negative X - axis 4 0a
(B) 8 along positive X - axis 2Q2 Q2
(C) 16 along negative X - axis (C) (D)
0a 2 0a
(D) 16 along positive Z - axis
12. A particle A has charge +q and a particle B has charge +4q
5. A hollow metal sphere of radius 5 cm is charged so that with each of them having the same mass m. When allowed
the potential on its surface is 10 V. The potential at the to fall from rest through the same electric potential
centre of the sphere is
(A) 0 V vA
difference, the ratio of their speed v will become
(B) 10 V B
(C) Same as at point 5 cm away from the surface (A) 2 : 1 (B) 1 : 2
(D) Same as at point 25 cm away from the surface (C) 1 : 4 (D) 4 : 1
6. If a unit positive charge is taken from one point to another 13. Angle between equipotential surface and lines of force is
over an equipotential surface, then (A) Zero (B) 180°
(A) Work is done on the charge (C) 90° (D) 45°
(B) Work is done by the charge 14. Four identical charges +50 C each are placed, one at
(C) Work done is constant each corner of a square of side 2m. How much external
(D) No work is done energy is required to bring another charge of +50 C from
7. A conductor with a positive charge infinity to the centre of the square
(A) Is always at +ve potential 1 Nm2
Given 9 109
(B) Is always at zero potential 4 C2
0
(C) Is always at negative potential
(A) 64 J (B) 41 J
(D) May be at +ve, zero or –ve potential
(C) 16 J (D) 10 J
15. In Millikan’s oil drop experiment an oil drop carrying a 21. An alpha particle is accelerated through a potential
charge Q is held stationary by a potential difference difference of 106 volt. Its kinetic energy will be
2400 V between the plates. To keep a drop of half the (A) 1 MeV (B) 2 MeV
radius stationary the potential difference had to be made (C) 4 MeV (D) 8 MeV
600 V. What is the charge on the second drop 22. A charge of 5 C is given a displacement of 0.5 m. The work
Q Q done in the process is 10 J. The potential difference
(A) (B)
4 2 between the two points will be
3Q (A) 2 V (B) 0.25 V
(C) Q (D) (C) 1 V (D) 25 V
2
16. A charge of 5 C experiences a force of 5000 N when it is 23. The electric potential V is given as a function of distance
kept in a uniform electric field. What is the potential x (metre) by V (5x 2 10x 9) volt . Value of electric
difference between two points separated by a distance of
1 cm field at x = 1 is
(A) 20 V/m (B) 6 V/m
(A) 10 V (B) 250 V
(C) 11 V/m (D) –23 V/m
(C) 1000 V (D) 2500 V
24. Two metal pieces having a potential difference of 800 V
17. Two insulated charged conducting spheres of radii 20 cm
are 0.02 m apart horizontally. A particle of mass
and 15 cm respectively and having an equal charge of 10
C are connected by a copper wire and then they are 1.96 10 15 kg is suspended in equilibrium between the
separated. Then
plates. If e is the elementary charge, then charge on the
(A) Both the spheres will have the same charge of 10 C particle is
(B) Surface charge density on the 20 cm sphere will be (A) e (B) 3e
greater than that on the 15 cm sphere
(C) 6e (D) 8e
(C) Surface charge density on the 15 cm sphere will be
25. A sphere of radius 1 cm has potential of 8000 V, then
greater than that on the 20 cm sphere
energy density near its surface will be
(D) Surface charge density on the two spheres will be
equal (A) 64 105 J / m3 (B) 8 103 J / m3
18. Two equal charges q are placed at a distance of 2a and a,
(C) 32 J / m3 (D) 2.83J / m3
third charge –2q is placed at the midpoint. The potential
energy of the system is 26. How much kinetic energy will be gained by an – particle
is going from a point at 70 V to another point at 50 V
q2 6q 2
(A) (B) (A) 40 eV (B) 40 keV
8 0a 8 0a
(C) 40 MeV (D) 0 eV
7q 2 9q 2 27. If a charged spherical conductor of radius 10 cm has
(C) (D) potential V at a point distant 5 cm from its centre, then the
8 0a 8 0a
potential at a point distant 15 cm from the centre will be
19. Two point charges 100 C and 5 C are placed at points
A and B respectively with AB = 40cm. The work done by 1 2
external force in displacing the charge 5 C from B to C, (A) V (B) V
3 3
where BC = 30 cm, angle ABC and
2 3
(C) V (D) 3V
1 9 2 2 2
9 10 Nm / C
4 0 28. Two unlike charges of magnitude q are separated by a
81 distance 2d. The potential at a point midway between
(A) 9 J (B)J them is
20
(A) Zero
9 9
(C) J (D) J 1
25 4 (B) 4
20. Equal charges are given to two spheres of different radii. 0
The potential will 1 q
(C) 4 .
(A) be more on the smaller sphere
0 d
(B) be more on the bigger sphere
1 2q
(C) be equal on both the spheres (D) 4 .
(D) depend on the nature of the materials of the spheres 0 d2
29. What is the potential energy of the equal positive point (A) 9 10 4 V (B) 18 10 4 V
charges of 1 C each held 1 m apart in air
(A) 9 10 3 J (B) 9 10 3 eV (C) 36 10 4 V (D) 36 10 4 V
38. Ten electrons are equally spaced and fixed around a circle
(C) 2eV / m (D) Zero
of radius R. Relative to V = 0 at infinity, the electrostatic
30. An oil drop having charge 2e is kept stationary between potential V and the electric field E at the centre C are
two parallel horizontal plates 2.0 cm apart when a potential
difference of 12000 volts is applied between them. If the (A) V 0 and E 0 (B) V 0 and E 0
density of oil is 900 kg/m3, the radius of the drop will be
(C) V 0 and E 0 (D) V 0 and E 0
(A) 2.0 10 6 m (B) 1.7 10 6 m
39. Two positive point charges of 12 C and 8 C are 10cm
(C) 1.4 10 6 m (D) 1.1 10 6 m apart. The work done in bringing them 4 cm closer is
31. The ratio of momenta of an electron and an -particle (A) 5.8 J (B) 5.8 eV
which are accelerated from rest by a potential difference (C) 13 J (D) 13 eV
of 100 volt is 40. There is a solid sphere of radius ‘R’ having uniformly
2me distributed charge. What is the relation between electric
(A) 1 (B) field ‘E’ (inside the sphere) and radius of sphere ‘R’ is
m
(A) E R 2 (B) E R 1
me me
(C) m (D) 2m
(C) E (D) E R2
32. A proton is accelerated through 50,000 V. Its energy will R3
increase by 41. As shown in the figure, charges +q and –q are placed at
(A) 5000 eV (B) 8 10 15 J the vertices B and C of an isosceles triangle. The potential
at the vertex A is
(C) 5000 J (D) 50,000 J
33. When a proton is accelerated through 1V, then its kinetic A
energy will be
(A) 1840 eV (B) 13.6 eV a
(C) 1 eV (D) 0.54 eV
b b
34. An electron enters between two horizontal plates B C
separated by 2mm and having a potential difference of +q –q
1000V. The force on electron is
1 2q
(A) 8 10 12 N (B) 8 10 14 N (A) 4 . (B) Zero
2
0 a b2
(C) 8 109 N (D) 8 1014 N
1 q 1 ( q)
35. Two metal spheres of radii R1 and R2 are charged to the (C) 4 . (D) 4 .
same potential. The ratio of charges on the spheres is 0 a2 b2 0 a2 b2

(A) R1 : R 2 (B) R1 : R 2 42. Three charges Q, +q and +q are placed at the vertices of a
right-angled isosceles triangle as shown. The net
(C) R12 : R 22 (D) R13: R 32 electrostatic energy of the configuration is zero if Q is
equal to
36. Electric charges of 10 C, 5 C, 3 C and 8 C are
Q
placed at the corners of a square of side 2m . the
potential at the centre of the square is
(A) 1.8 V (B) 1.8 106 V

(C) 1.8 105 V (D) 1.8 104 V +q +q


a
37. Two charges of 4 C each are placed at the corners A and
B of an equilateral triangle of side length 0.2 m in air. The q 2q
(A) (B)
1 2 2 2
1 N-m 2
electric potential at C is 4 9 109 (C) –2q (D) +q
0 C2
43. Two electric charges 12 C and –6 C are placed 20 cm 1 Qq 1 Qq
apart in air. There will be a point P on the line joining these (A) 4 (B) 4
0 l 0 l2
charges and outside the region between them, at which
the electric potential is zero. The distance of P from –6 C 1
(C) 4 Qql (D) Zero
charge is 0
(A) 0.10 m (B) 0.15 m 50. A particle of mass ‘m’ and charge ‘q’ is accelerated through
(C) 0.20 m (D) 0.25 m a potential difference of V volt, its energy will be
44. An electron of mass m and charge e is accelerated from (A) qV (B) mqV
rest through a potential difference V in vacuum. The final
speed of the electron will be q q
(C) V (D)
m mV
(A) V e / m (B) eV / m
51. Two spheres A and B of radius ‘a’ and ‘b’ respectively are
(C) 2eV / m (D) 2eV / m at same electric potential. The ratio of the surface charge
45. The radius of a soap bubble whose potential is 16V is densities of A and B is
doubled. The new potential of the bubble will be a b
(A) 2V (B) 4V (A) (B)
b a
(C) 8V (D) 16V
a2 b2
46. In the rectangle, shown below, the two corners have (C) (D)
b2 a2
charges q1 5 C and q 2 2.0 C . The work done in 52. Potential at a point x-distance from the centre inside the
moving a charge 3.0 C from B to A is (take conducting sphere of radius R and charged with charge Q is

1/ 4 1010 N-m 2 / C2 ) Q Q
0 (A) (B)
R x
q1
A
Q
(C) (D) xQ
15 cm x2
53. In an hydrogen atom, the electron revolves around the
B
q2 nucleus in an orbit of radius 0.53 10 10 m . Then the
15 cm
(A) 2.8 J (B) 3.5 J electrical potential produced by the nucleus at the position
(C) 4.5 J (D) 5.5 J of the electron is
47. A hollow conducting sphere of radius R has a charge (A) – 13.6 V (B) – 27.2 V
(+Q) on its surface. What is the electric potential within (C) 27.2 V (D) 13.6 V
54. A ball of mass 1 g and charge 10–8 C moves from a point A.
R
the sphere at a distance r from its centre where potential is 600 volt to the point B where potential
3 is zero. Velocity of the ball at the point B is 20 cm/s. The
1 Q velocity of the ball at the point A will be
(A) Zero (B) 4 (A) 22.8 cm/s (B) 228 cm/s
0 r
(C) 16.8 m/s (D) 168 m/s
1 Q 1 Q 55. Three charges Q, (+q) and (+q) are placed at the vertices
(C) 4 (D) 4 of an equilateral triangle of side l as shown in the figure. If
0 R 0 r2
the net electrostatic energy of the system is zero, then Q
48. A spherical conductor of radius 2m is charged to a is equal to
potential of 120 V. It is now placed inside another hollow Q
spherical conductor of radius 6m. Calculate the potential
to which the bigger sphere would be raised
(A) 20 V (B) 60 V l l
(C) 80 V (D) 40 V
49. A charge (–q) and another charge (+Q) are kept at two +q l +q
points A and B respectively. Keeping the charge (+Q) fixed q
at B, the charge (–q) at A is moved to another point C (A) (B) (–q)
2
such that ABC forms an equilateral triangle of side l. The
(C) (+q) (D) Zero
network done in moving the charge (–q) is
64. A hollow conducting sphere is placed in an electric field
56. Electric potential at any point is V 5x 3y 15z ,
produced by a point charge placed at P as shown in figure.
then the magnitude of the electric field is
Let VA ,VB , VC be the potentials at points A, B and C
(A) 3 2 (B) 4 2
respectively. Then
(C) 5 2 (D) 7
57. The work done in bringing a 20 coulomb charge from
A
point A to point B for distance 0.2m is 2J. The potential C P
difference between the two points will be (in volt)
(A) 0.2 (B) 8 B
(C) 0.1 (D) 0.4 (A) VC VB (B) VB VC
20
58. If 4 10 eV energy is required to move a charge of 0.25 (C) VA VB (D) VA VC
coulomb between two points. Then what will be the
65. An electron enters in high potential region v2 from lower
potential difference between them
potential region v1 then its velocity
(A) 178 V (B) 256 V
(A) will increase
(C) 356 V (D) None of these
(B) will change in direction but not in magnitude
59. Kinetic energy of an electron accelerated in a potential
(C) no change in direction of field
difference of 100 V is
(D) no change in direction perpendicular to field
(A) 1.6 10 17 J (B) 1.6 10 21 J 66. The electric potential at the surface of an atomic nucleus
(C) 1.6 10 29 J (D) 1.6 10 34 J (Z = 50) of radius 9.0 × 10–13 cm is
(A) 80 volts (B) 8 × 106 volts
60. If 3 charges are placed at the vertices of equilateral triangle
(C) 9 volts (D) 9 × 105 volts
of charge ‘q’ each. What is the net potential energy, if the
side of equilateral is l cm 67. A pellet carrying charge of 0.5 coulombs is accelerated
through a potential of 2,000 volts. It attains a kinetic energy
1 q2 1 2q 2 equal to
(A) (B)
4 0 l 4 0 l (A) 1000 ergs (B) 1000 joules
2 (C) 1000 kWh (D) 500 ergs
1 3q 1 4q 2
(C) (D) 68. A particle has a mass 400 times than that of the electron
4 0 l 4 0 l and charge is double than that of a electron. It is
61. If identical charges (–q) are placed at each corner of a accelerated by 5V of potential difference. Initially the
cube of side b, then electric potential energy of charge particle was at rest, then its final kinetic energy will be
(+q) which is placed at centre of the cube will be (A) 5 eV (B) 10 eV
8 2q 2 8 2q 2 (C) 100 eV (D) 2000 eV
(A) (B)
4 0b 0b 69. An electron (charge = 1.6 10 19 coulomb) is accelerated

4q 2 through a potential of 1,00,000 volts. The energy required


4 2q 2
(C) (D) by the electron is
0b 3 0b
(A) 1.6 10 24 joule (B) 1.6 10 14
62. An -particle is accelerated through a potential difference erg
of 200V. The increase in its kinetic energy is
(C) 0.53 10 14 joule (D) 1.6 10 14 joule
(A) 100 eV (B) 200 eV
(C) 400 eV (D) 800 eV 70. The charge given to a hollow sphere of radius 10 cm is
63. A thin spherical conducting shell of radius R has a charge 3.2×10–19 coulomb. At a distance of 4 cm from its centre,
q. Another charge Q is placed at the centre of the shell. the electric potential will be
(A) 28.8 10 9 (B) 288 volts
R volts
The electrostatic potential at a point p a distance from
2 (C) 2.88 volts (D) Zero
the centre of the shell is 71. Work done in moving a positive charge on an equipotential
(q Q) 2 2Q surface is
(A) 4 0 R (B) 4 (A) Finite, positive but not zero
0R
(B) Finite, negative but not zero
2Q 2q 2Q q
(C) 4 (D) 4 (C) Zero
0R 4 0R 0R 4 0R
(D) Infinite
72. A charge of 10 e.s.u. is placed at a distance of 2 cm from a
charge of 40 e.s.u. and 4 cm from another charge of 20
e.s.u. The potential energy of the charge 10 e.s.u. is (in
ergs)
A B
(A) 87.5 (B) 112.5 r
(C) 150 (D) 250
73. A table tennis ball which has been covered with
conducting paint is suspended by a silk thread so that it
hang between two plates, out of which one is earthed and 1 q 1 q
other is connected to a high voltage generator. This ball (A) W (B) W
4 o r 4 0 r2
(A) Is attracted towards high voltage plate and stays
1 q
there (C) W (D) W = zero
(B) Hangs without moving
4 r2 0

(C) Swing backward and forward hitting each plate in 80. When one electron is taken towards the other electron,
turn then the electric potential energy of the system
(D) Is attracted to earthed plate and stays there (A) Decreases (B) Increases
74. A sphere of 4 cm radius is suspended within a hollow (C) Remains unchanged (D) Becomes zero
sphere of 6 cm radius. The inner sphere is charged to 81. A hollow metal sphere of radius 5cm is charged such that
potential 3 e.s.u. and the outer sphere is earthed. The the potential on its surface is 10V. The potential at a
charge on the inner sphere is distance of 2cm from the centre of the sphere
(A) Zero (B) 10 V
1
(A) 54 e.s.u. (B) e.s.u. (C) 4 V (D) 10/3 V
4
82. The work done in carrying a charge of 5 C from a point A
(C) 30 e.s.u. (D) 36 e.s.u. to a point B in an electric field is 10mJ. The potential
75. State which of the following is correct
difference (VB VA ) is then
(A) Joule = coulomb × volt
(A) + 2kV (B) – 2 kV
(B) Joule = coulomb/volt
(C) + 200 V (D) – 200 V
(C) Joule = volt × ampere
83. Value of potential at a point due to a point charge is
(D) Joule = volt/ampere
(A) Inversely proportional to square of the distance
76. When a positive q charge is taken from lower potential to
a higher potential point, then its potential energy will (B) Directly proportional to square of the distance
(A) Decrease (B) Increases (C) Inversely proportional to the distance
(C) Remain unchanged (D) Become zero (D) Directly proportional to the distance
77. When a negative charge is taken at a height from earth’s 84. Electric potential of earth is taken to be zero because earth
surface, then its potential energy is a good
(A) Decreases (A) Insulator (B) Conductor
(B) Increases (C) Semiconductor (D) Dielectric
(C) Remains unchanged 85. There is 10 units of charge at the centre of a circle of
radius 10m. The work done in moving 1 unit of charge
(D) Will become infinity
around the circle once is
78. When a charge of 3 coulombs is placed in a uniform electric
(A) Zero (B) 10 units
field, it experiences a force of 3000 Newton. Within this
field, potential difference between two points separated (C) 100 units (D) 1 unit
by a distance of 1 cm is 86. Two parallel plates separated by a distance of 5 mm are
(A) 10 volts kept at a potential difference of 50 V. A particle of mass
(B) 90 volts 10 15 kg and charge 10 11
C enters in it with a velocity
(C) 1000 volts
(D) 3000 volts 107 m / s. The acceleration of the particle will be
79. There are two equipotential surface as shown in figure. (A) 108 m / s 2 (B) 5 105 m / s2
The distance between them is r. The charge of –q coulomb
is taken from the surface A to B, the resultant work done (C) 105 m / s2 (D) 2 103 m / s 2
will be
87. The electric potential inside a conducting sphere 95. Figure shows three points A, B and C in a region of uniform
(A) Increases from centre to surface electric field E . The line AB is perpendicular, and BC is
(B) Decreases from centre to surface parallel to the field lines. Then which of the following
(C) Remains constant from centre to surface holds good. Where VA,VB and VC represent the electric
(D) Is zero at every point inside potential at points A, B and C respectively
88. The potential at a distance R/2 from the centre of a
A
conducting sphere of radius R will be
Q
(A) 0 (B) 8 B C
0R

Q Q
(C) 4 (D) 2 (A) VA VB VC
0R 0R

89. Four charges Q, Q, Q, Q are placed at the corners (B) VA VB VC


of a square taken in order. At the centre of the square (C) VA VB VC
(A) E 0, V 0 (B) E 0, V 0
(D) VA VB VC
(C) E 0, V 0 (D) E 0, V 0 96. In a certain charge distribution, all points having zero
90. The radius of nucleus of silver (atomic number = 47) is potential can be joined by a circle S. Points inside S have
positive potential and points outside S have negative
3.4 10 14 m . The electric potential on the surface of potential. A positive charge, which is free to move, is
nucleus is (e 1.6 10 19
C) placed inside S
(A) It will remain in equilibrium
(A) 1.99 106 volt (B) 2.9 10 6 volt (B) It can move inside S, but it cannot cross S
(C) It must cross S at some time
(C) 4.99 106 volt (D) 0.99 106 volt
(D) It may move, but will ultimately return to its starting
91. Point charge q1 2 C and q 2 1 C are kept at points point
x = 0 and x = 6 respectively. Electrical potential will be zero 97. A square of side ‘a’ has charge Q at its centre and charge
at points ‘q’ at one of the corners. The work required to be done in
moving the charge ‘q’ from the corner to the diagonally
(A) x = 2 and x = 9 (B) x = 1 and x = 5
opposite corner is
(C) x = 4 and x = 12 (D) x = –2 and x = 2
92. Equipotential surfaces associated with an electric field Qq
which is increasing in magnitude along the x-direction are (A) Zero (B) 4
0a
(A) Planes parallel to yz-plane
(B) Planes parallel to xy-plane Qq 2 Qq
(C) (D) 2
(C) Planes parallel to xz-plane 4 0a 0a
(D) Coaxial cylinders of increasing radii around the x-axis
98. As per this diagram a point charge +q is placed at the
93. A bullet of mass 2 gm is having a charge of 2 C. Through origin O. Work done in taking another point charge –Q
what potential difference must it be accelerated, starting from the point A [co-ordinates (0, a] to another point B
from rest, to acquire a speed of 10 m/s [co-ordinates (a, 0)] along the straight path AB is
(A) 5 kV (B) 50 kV Y
(C) 5 V (D) 50 V A
94. The points resembling equal potentials are
S

P Q O B X
qQ 1
(A) Zero (B) 2a
4 0 a2
R
(A) P and Q (B) S and Q qQ 1 a qQ 1
(C) (D) 2a
(C) S and R (D) P and R 4 0 a2 2 4 0 a2
99. To charges q1 and q2 are placed 30cm apart, shown in the 104. Eight drops of mercury of equal radii possessing equal
figure. A third charge q3 is moved along the arc of a circle charges combine to form a big drop. Then the capacitance
of radius 40 cm from C to D. The change in the potential of bigger drop compared to each individual small drop is
q3 (A) 8 times (B) 4 times
energy of the system is 4 k , where k is (C) 2 times (D) 32 times
0
105. A condenser of capacity 50 F is charged to 10 volts. Its
q3 energy is equal to
C 3 4
(A) 2.5 10 joule (B) 2.5 10 joule

40 (C) 5 10 2 (D) 1.2 10 8


joule joule
106. The potential gradient at which the dielectric of a
q2
q1 D condenser just gets punctured is called
A B 30cm (A) Dielectric constant
(A) 8q2 (B) 8q1 (B) Dielectric strength
(C) 6q2 (D) 6q1 (C) Dielectric resistance
100. Two thin wire rings each having a radius R are placed at a (D) Dielectric number
distance d apart with their axes coinciding. The charges 107. A parallel plate condenser has a capacitance 50 F in air
on the two rings are +q and –q. The potential difference and 110 F when immersed in an oil. The dielectric constant
between the centres of the two rings is ‘k’ of the oil is
(A) Zero (A) 0.45 (B) 0.55
(C) 1.10 (D) 2.20
Q 1 1 108. Separation between the plates of a parallel plate capacitor
(B) 4 R 2
0 R d2 is d and the area of each plate is A. When a slab of material
of dielectric constant k and thickness t (t < d) is introduced
(C) QR / 4 2 between the plates, its capacitance becomes
0d
0A 0A
Q 1 1 (A) d t 1 1 (B) d t 1 1
(D) 2 R 2 2 k k
0 R d
0A 0A
101. If an electron moves from rest from a point at which
potential is 50 volt to another point at which potential is (C) d t 1 1 (D) d t 1 1
70 volt, then its kinetic energy in the final state will be k k
(A) 3.2 × 10–10 J (B) 3.2 × 10–18 J 109. The capacity of parallel plate condenser depends on
(C) 1 N (D) 1 dyne (A) The type of metal used
102. In the following diagram the work done in moving a point (B) The thickness of plates
charge from point P to point A, B and C is respectively as (C) The potential applied across the plates
WA, WB and WC , then (D) The separation between the plates
C A 110. The energy of a charged capacitor resides in
P
(A) The electric field only
(B) The magnetic field only
B
(C) Both the electric and magnetic field
(A) WA = WB = WC (B) WA = WB = WC = 0
(D) Neither in electric nor magnetic field
(C) WA > WB > WC (D) WA < WB < WC
111. No current flows between two charged bodies connected
103. A hollow metallic sphere of radius R is given a charge Q.
together when they have the same
Then the potential at the centre is
1 Q Q
. (A) Capacitance or ratio
(A) Zero (B) 4 V
0 R
(B) Charge
1 2Q 1 Q
(C) 4 . (D) 4 . (C) Resistance
0 R 0 2R
Q
(D) Potential or ratio
C
112. The capacity of a parallel plate condenser is C. Its capacity 121. The radius of two metallic spheres A and
when the separation between the plates is halved will be B are r1 and r2 respectively (r1 > r2). They are connected
(A) 4C (B) 2C by a thin wire and the system is given a certain charge.
The charge will be greater
C C
(C) (D) (A) On the surface of the sphere B
2 4
(B) On the surface of the sphere A
113. Eight small drops, each of radius r and having same charge (C) Equal on both
q are combined to form a big drop. The ratio between the
(D) Zero on both
potentials of the bigger drop and the smaller drop is
122. The capacity of a spherical conductor in MKS system is
(A) 8 : 1 (B) 4 : 1
(C) 2 : 1 (D) 1 : 8 R 4 0
(A) (B)
114. 1000 small water drops each of radius r and charge q 0 R
coalesce together to form one spherical drop. The potential
(C) 4 2
of the big drop is larger than that of the smaller drop by a 0R (D) 4 0R
factor of 123. Can a metal be used as a medium for dielectric?
(A) 1000 (B) 100 (A) Yes
(C) 10 (D) 1 (B) No
115. A parallel plate condenser is immersed in an oil of dielectric (C) Depends on its shape
constant 2. The field between the plates is
(D) Depends on dielectric
(A) Increased proportional to 2
124. The area of each plate of a parallel plate capacitor is 100
1 cm2 and the distance between the plates is 1mm. It is filled
(B) Decreased proportional to with mica of dielectric 6. The radius of the equivalent
2
capacity of the sphere will be
(C) Increased proportional to 2 (A) 47.7 m (B) 4.77 m
1 (C) 477 m (D) None of the above
(D) Decreased proportional to 125. The respective radii of the two spheres of a spherical
2
condenser are 12 cm and 9 cm. The dielectric constant of
116. The capacitance of a spherical condenser is 1 F. If the the medium between them is 6. The capacity of the
spacing between the two spheres is 1 mm, then the radius condenser will be
of the outer sphere is (A) 240 pf (B) 240 F
(A) 30 cm (B) 6 m (C) 240 F (D) None of the above
(C) 5 cm (D) 3 m 126. A parallel plate condenser is connected with the terminals
117. If the dielectric constant and dielectric strength be denoted of a battery. The distance between the plates is 6 mm. If a
by k and x respectively, then a material suitable for use as glass plate (dielectric constant K = 9) of 4.5 mm is
a dielectric in a capacitor must have introduced between them, then the capacity will
(A) High k and high x (B) High k and low x become
(C) Low k and low x (D) Low k and high x (A) 2 times (B) The same
118. When air in a capacitor is replaced by a medium of dielectric (C) 3 times (D) 4 times
constant K, the capacity 127. The radii of two metallic spheres P and Q are r1 and r2
(A) Decreases K times (B) Increases K times respectively. They are given the same charge. If r 1 > r2
(C) Increases K2 times (D) Remains constant then on connecting them with a thin wire, the charge will
119. 64 drops each having the capacity C and potential V are flow
combined to form a big drop. If the charge on the small (A) From P to Q
drop is q, then the charge on the big drop will be (B) From Q to P
(A) 2q (C) Neither the charge will flow from P to Q nor from Q
(B) 4q to P
(C) 16q (D) The information is incomplete
(D) 64 q 128. A capacitor of capacity C has charge Q and stored energy
120. The capacity of a parallel plate capacitor increases with is W. If the charge is increased to 2Q, the stored energy
the will be
(A) Decrease of its area (B) Increase of its distance (A) 2W (B) W/2
(C) Increase of its area (D) None of the above (C) 4W (D) W/4
129. Between the plates of a parallel plate condenser, a plate of 134. The true statement is, on increasing the distance between
thickness t1 and dielectric constant k1 is placed. In the the plates of a parallel plate condenser
rest of the space, there is another plate of thickness t2 and (A) The electric intensity between the plates will decrease
dielectric constant k2. The potential difference across the (B) The electric intensity between the plates will increase
condenser will be (C) The electric intensity between the plates will remain
unchanged
Q t1 t2 0Q t1 t2
(A) A (B) (D) The P.D. between the plates will decrease
0 k1 k2 A k1 k2
135. There is an air filled 1 pF parallel plate capacitor. When
the plate separation is doubled and the space is filled with
Q k1 k2 0Q
wax, the capacitance increases to 2 pF. The dielectric
(C) A (D) (k1t1 k 2 t 2 ) constant of wax is
0 t1 t2 A
(A) 2 (B) 4
130. The distance between the plates of a parallel plate (C) 6 (D) 8
condenser is 4mm and potential difference is 60 volts. If
136. The capacity and the energy stored in a parallel plate
the distance between the plates is increased to 12mm,
condenser with air between its plates are respectively Co
then
and Wo. If the air is replaced by glass (dielectric constant
(A) The potential difference of the condenser will become = 5) between the plates, the capacity of the plates and the
180 volts energy stored in it will respectively be
(B) The P.D. will become 20 volts
Wo
(C) The P.D. will remain unchanged (A) 5Co, 5Wo (B) 5Co ,
5
(D) The charge on condenser will reduce to one third
Co Co Wo
(C) , 5Wo (D) ,
131. The two metallic plates of radius r are placed at a distance 5 5 5
r 137. Force of attraction between the plates of a parallel plate
d apart and its capacity is C. If a plate of radius and capacitor is
2
thickness d of dielectric constant 6 is placed between the
q2 q2
plates of the condenser, then its capacity will be (A) (B)
2 0 AK 0 AK
7C 3C
(A) (B)
2 7 q q2
(C) 2 A (D)
7C 9C 0 2 0 A2 K
(C) (D)
3 4 138. A capacitor of capacity C is connected with a battery of
132. The distance between the plates of a parallel plate potential V in parallel. The distance between its plates is
condenser is 8 mm and P.D. 120 volts. If a 6mm thick slab reduced to half at once, assuming that the charge remains
of dielectric constant 6 is introduced between its plates, the same. Then to charge the capacitance upto the
then potential V again, the energy given by the battery will be
(A) The charge on the condenser will be doubled
CV2
(B) The charge on the condenser will be reduced to half (A)
4
(C) The P.D. across the condenser will be 320 volts
(D) The P.D. across the condenser will be 45 volts CV 2
(B)
133. In a parallel plate condenser, the radius of each circular 2
plate is 12 cm and the distance between the plates is 5 mm. 3CV 2
There is a glass slab of 3mm thick and of radius 12 cm with (C)
4
dielectric constant 6 between its plates. The capacity of
(D) CV2
the condenser will be
139. N identical spherical drops charged to the same potential
(A) 144 10 9 F V are combined to form a big drop. The potential of the
new drop will be
(B) 40pF
(A) V (B) V/N
(C) 160 pF
(C) V × N (D) V × N2/3
(D) 1.44 F
140. One plate of parallel plate capacitor is smaller than other, 146. The capacitance of a parallel plate condenser does not
then charge on smaller plate will be depend on
(A) Less than other (A) Area of the plates
(B) More than other (B) Medium between the plates
(C) Equal to other (C) Distance between the plates
(D) Will depend upon the medium between them (D) Metal of the plates
141. A 6 F capacitor is charged from 10 volts to 20 volts. 147. Between the plates of a parallel plate condenser there is
Increase in energy will be 1mm thick paper of dielectric constant 4. It is charged at
100 volt. The electric field in volt/metre between the plates
(A) 18 10 4 J (B) 9 10 4 J of the capacitor is
(C) 4.5 10 4 J (D) 9 10 6 J (A) 100 (B) 100000
(C) 25000 (D) 4000000
142. As shown in the figure, a very thin sheet of aluminium is
148. The electric field between the two spheres of a charged
placed in between the plates of the condenser. Then the
spherical condenser
capacity
(A) Is zero
(B) Is constant
(C) Increases with distance from the centre
Al strip
(D) Decreases with distance from the centre
149. The distance between the plates of a parallel plate
d
(A) Will increase capacitor is d. A metal plate of thickness is placed
2
(B) Will decrease
between the plates. The capacitance would then be
(C) Remains unchanged
(A) Unchanged (B) Halved
(D) May increase or decrease
(C) Zero (D) Doubled
143. Twenty seven drops of water of the same size are equally
150. An uncharged capacitor is connected to a battery. On
and similarly charged. They are then united to form a
charging the capacitor
bigger drop. By what factor will the electrical potential
(A) All the energy supplied is stored in the capacitor
changes
(B) Half the energy supplied is stored in the capacitor
(A) 9 times (B) 27 times
(C) The energy stored depends upon the capacity of the
(C) 6 times (D) 3 times
capacitor only
144. The capacitance of a metallic sphere will be 1 F, if its
(D) The energy stored depends upon the time for which
radius is nearly
the capacitor is charged
(A) 9 km (B) 10 m
151. A capacitor is kept connected to the battery and a dielectric
(C) 1.11 m (D) 1.11 cm
slab is inserted between the plates. During this process
145. A parallel plate capacitor of plate area A and plate
(A) No work is done
separation d is charged to potential V and then the battery
(B) Work is done at the cost of the energy already stored
is disconnected. A slab of dielectric constant k is then
in the capacitor before the slab is inserted
inserted between the plates of the capacitors so as to fill
the space between the plates. If Q, E and W denote (C) Work is done at the cost of the battery
respectively, the magnitude of charge on each plate, the (D) Work is done at the cost of both the capacitor and
electric field between the plates (after the slab is inserted) the battery
and work done on the system in question in the process 152. The capacitance of an air capacitor is 15 F the separation
of inserting the slab, then state incorrect relation from the between the parallel plates is 6mm. A copper plate of 3mm
following thickness is introduced symmetrically between the plates.
The capacitance now becomes
2
0 AV 0 AV (A) 5 F
(A) Q (B) W
d 2kd (B) 7.5 F
2
(C) 22.5 F
V 0 AV 1 (D) 30 F
(C) E (D) W 1
kd 2d k
153. An air capacitor is connected to a battery. The effect of 158. The expression for the capacity of the capacitor formed
filling the space between the plates with a dielectric is to by compound dielectric placed between the plates of a
increase parallel plate capacitor as shown in figure, will be (area of
(A) The charge and the potential difference plate = A)
(B) The potential difference and the electric field d1 d3
(C) The electric field and the capacitance
(D) The charge and the capacitance
154. A light bulb, a capacitor and a battery are connected K1 K2 K3
together as shown here, with switch S initially open. When
the switch S is closed, which one of the following is true?

d2

0A
d1 d2 d3
(A)
K1 K2 K3
S 0A
(A) The bulb will light up for an instant when the capacitor d1 d 2 d 3
starts charging (B)
K1 K 2 K3
(B) The bulb will light up when the capacitor is fully
charged 0 A(K1K 2 K 3 )
(C) d1d 2d3
(C) The bulb will not light up at all
(D) The bulb will light up and go off at regular intervals AK1 AK 2 AK 3
155. A parallel plate capacitor has a capacity C. The separation (D) 0
d1 d2 d3
between the plates is doubled and a dielectric medium is
introduced between the plates. If the capacity now becomes 159. The intensity of electric field at a point between the plates
2C, the dielectric constant of the medium is of a charged capacitor
(A) 2 (B) 1 (A) Is directly proportional to the distance between the
(C) 4 (D) 8 plates
156. The diameter of each plate of an air capacitor is 4 cm. To (B) Is inversely proportional to the distance between the
make the capacity of this plate capacitor equal to that of plates
20 cm diameter sphere, the distance between the plates (C) Is inversely proportional to the square of the distance
will be between the plates
(D) Does not depend upon the distance between the
(A) 4 10 3 m
plates
(B) 1 10 3 m 160. The capacity of a condenser in which a dielectric of
dielectric constant 5 has been used, is C. If the dielectric
(C) 1 cm
is replaced by another with dielectric constant 20, the
(D) 1 10 3 cm capacity will become
157. A spherical condenser has inner and outer spheres of C
radii a and b respectively. The space between the two is (A) (B) 4C
4
filled with air. The difference between the capacities of
two condensers formed when outer sphere is earthed and C
when inner sphere is earthed will be (C) (D) 2 C
2
(A) Zero 161. In a spherical condenser radius of the outer sphere is R.
(B) 4 0a The different in the radii of outer and inner sphere is x. Its
capacity is proportional to
(C) 4 0b
xR x(R x)
b (A) (B)
(R x) r
(D) 4 0a
b a
R(R x) R
(C) (D)
x x
162. A capacitor when filled with a dielectric K = 3 has charge 169. A parallel plate capacitor is first charged and then a
Q0, voltage V0 and field E0. If the dielectric is replaced dielectric slab is introduced between the plates. The
with another one having K = 9 the new values of charge, quantity that remains unchanged is
voltage and field will be respectively (A) Charge Q (B) Potential V
(A) 3Q0 , 3V0 , 3E 0 (B) Q0 , 3V0 , 3E 0 (C) Capacity C (D) Energy U
170. A 2 F capacitor is charged to 100 volt and then its plates
V0 V0 E 0 are connected by a conducting wire. The heat produced
(C) Q0 , , 3E 0 (D) Q0 , ,
3 3 3 is
163. A charge of 10–9 C is placed on each of the 64 identical (A) 1 J (B) 0.1 J
drops of radius 2cm. They are then combined to form a (C) 0.01 J (D) 0.001 J
bigger drop. Find its potential 171. The force between the plates of a parallel plate capacitor
(A) 7.2 × 103 V (B) 7.2 × 102 V of capacitance C and distance of separation of the plates
2
(C) 1.44 × 10 V (D) 1.44 × 103 V d with a potential difference V between the plates, is
164. 125 identical drops each charged to the same potential of
50 volts are combined to form a single drop. The potential CV 2 C2 V 2
(A) (B)
of the new drop will be 2d 2d 2
(A) 50 V (B) 250 V
(C) 500 V (D) 1250 V C2 V 2 V 2d
(C) 2 (D)
165. The plates of a parallel plate capacitor of capacity 50 C d C
are charged to a potential of 100 volts and then separated
172. Two metal spheres of capacitance C1 and C2 carry some
from each other so that the distance between them is
charges. They are put in contact and then separated. The
doubled. How much is the energy spent in doing so?
final charges Q1 and Q2 on them will satisfy
(A) 25 10 2 J (B) 12.5 10 2 J
Q1 C1 Q1 C1
(C) 2 (D) 12.5 10 2 J (A) Q C2 (B) Q C2
25 10 J 2 2
166. Two spherical conductors each of capacity C are charged
to potentials –V and V. These are then connected by Q1 C1 Q1 C2
(C) Q C2 (D) Q C1
means of a fine wire. The loss of energy will be 2 2

1 173. A parallel plate condenser with oil between the plates


(A) Zero (B) CV 2
2 (dielectric constant of oil K = 2) has a capacitance C. If the
oil is removed, then capacitance of the capacitor becomes
(C) CV2 (D) 2CV2
167. The area of the plates of a parallel plate condenser is A (A) 2C (B) 2C
and the distance between the plates is 10mm. There are
two dielectric sheets in it, one of dielectric constant 10 C C
(C) (D)
and thickness 6mm and the other of dielectric constant 5 2 2
and thickness 4mm. The capacity of the condenser is 174. What is the area of the plates of a 3F parallel plate capacitor,
12 if the separation between the plates is 5mm
(A) 0A
35
(A) 1.694 109 m 2
2
(B) 0A
3 (B) 4.529 109 m 2
5000
(C) 0A (C) 9.281 109 m 2
7
(D) 1500 0 A
(D) 12.981 109 m 2
168. An air capacitor of capacity C 10 F is connected to a
175. A parallel plate capacitor has circular plates of 0.08m radius
constant voltage battery of 12V. Now the space between and 1.0 × 10–3 m separation. If a P.D. of 100 volt is applied,
the plates is filled with a liquid of dielectric constant 5. the charge will be
The charge that flows now from battery to the capacitor is
(A) 1.8 10 10 (B) 1.8 10 8 C
(A) 120 C (B) 699 C C
(C) 480 C (D) 24 C (C) 1.8 10 20 (D) None of these
C
176. The capacity of a parallel plate condenser is 10 F without 185. When we touch the terminals of a high voltage capacitor,
dielectric. Dielectric of constant 2 is used to fill half the even after a high voltage has been cut off, then the
distance between the plates, the new capacitance in F is capacitor has a tendency to
(A) 10 (B) 20 (A) Restore energy (B) Discharge energy
(C) 15 (D) 13.33 (C) Affect dangerously (D) Both (B) and (C)
177. The energy stored in the condenser is 186. In a capacitor of capacitance 20 F, the distance between
the plates is 2mm. If a dielectric slab of width 1mm and
1 dielectric constant 2 is inserted between the plates, then
(A) QV (B) QV
2 the new capacitance is
(A) 2 F (B) 15.5 F
1 1Q
(C) C (D) (C) 26.6 F (D) 32 F
2 2C
187. A metallic sheet is inserted between the plates of a parallel
178. The capacitance of an air filled parallel plate capacitor is plate capacitor. The capacitance of the capacitor
10 F. The separation between the plates is doubled and
(A) Increases
the space between the plates is then filled with wax giving
(B) Is independent of the position of the sheet
the capacitance a new value of 40 10 12 farads . The
(C) Is maximum when the metal sheet in the middle
dielectric constant of wax is
(D) Is maximum when the metal sheet touches one of the
(A) 12.0 (B) 10.0 capacitor plates
(C) 8.0 (D) 4.2
188. The capacity of a parallel plate capacitor with no dielectric
179. Two identical charged spherical drops each of capacitance substance but with a separation of 0.4 cm is 2 F. The
C merge to form a single drop. The resultant capacitance separation is reduced to half and it is filled with a dielectric
is substance of value 2.8. The final capacity of the capacitor
(A) Equal to 2C is
(B) Greater than 2C (A) 11.2 F (B) 15.6 F
(C) Less than 2C but greater than C (C) 19.2 F (D) 22.4 F
(D) Less than C 189. Two insulated metallic spheres of 3 F and 5 F
180. A condenser having a capacity 2.0 micro farad is charged capacitances are charged to 300 V and 500V respectively.
to 200 volts and then the plates of the capacitor are The energy loss, when they are connected by a wire is
connected to a resistance wire. The heat produced in
(A) 0.012 J (B) 0.0218 J
joules will be
(C) 0.0375 J (D) 3.75 J
(A) 4 104 J (B) 4 1010 J 190. Two conducting spheres of radii 5 cm and 10 cm are given
a charge of 15 C each. After the two spheres are joined
(C) 4 10 2 J (D) 2 10 2 J
by a conducting wire, the charge on the smaller sphere is
181. The radius of a metallic sphere if its capacitance is 1/9F, is
(A) 5 C
(A) 106 m (B) 107 m
(B) 10 C
(C) 109 m (D) 108 m
(C) 15 C
182. The ratio of charge to potential of a body is known as
(D) 20 C
(A) Capacitance (B) Conductance
191. In a parallel plate capacitor of capacitance C, a metal sheet
(C) Inductance (D) Resistance
is inserted between the plates, parallel to them. If the
183. If the capacity of a spherical conductor is 1 picofarad,
thickness of the sheet is half of the separation between
then its diameter, would be
the plates. The capacitance will be
(A) 1.8 10 3 m (B) 18 10 3 m C 3C
(A) (B)
2 4
(C) 1.8 10 5 m (D) 18 10 7 m
(C) 4C (D) 2C
184. The capacity of a parallel plate condenser is 15 F, when 192. While a capacitor remains connected to a battery and
the distance between its plates is 6 cm. If the distance dielectric slab is applied between the plates, then
between the plates is reduced to 2 cm, then the capacity
(A) Potential difference between the plates is changed
of this parallel plate condenser will be
(B) Charge flows from the battery to the capacitor
(A) 15 F (B) 30 F
(C) 45 F (D) 60 F (C) Electric field between the plates increases
(D) Energy store in the capacitor decreases
193. A body of capacity 4 F is charged to 80 V and another 201. Work done by an external agent in separating the parallel
body of capacity 6 F is charged to 30V. When they are plate capacitor is
connected the energy lost by 4 F capacitor is
1 2
(A) 7.8 mJ (B) 4.6 mJ (A) CV (B) C V
2
(C) 3.2 mJ (D) 2.5 mJ
194. The capacity of the conductor does not depend upon 1
(C) CV 2 (D) None of these
(A) Charge (B) Voltage 2
(C) Nature of the material (D) All of these 202. A parallel plate capacitor has an electric field of 105 V/m
195. A solid conducting sphere of radius R1 is surrounded by between the plates. If the charge on the capacitor plate is
another concentric hollow conducting sphere of radius 1 C, the force on each capacitor plate is
R2. The capacitance of this assembly is proportional to (A) 0.5 N (B) 0.05 N
(C) 0.005 N (D) None of these
R 2 R1 R 2 R1
(A) (B) 203. A parallel plate capacitor has plate area A and separation
R1R 2 R1R 2 d. It is charged to a potential difference V0. The charging
battery is disconnected, and the plates are pulled apart to
R1R 2 R1R 2 three times the initial separation. The work required to
(C) R R (D) R
1 2 2 R1 separate the plates is
196. Two spherical conductors A and B of radius a and b (b >
3 0 AV02 2
0 AV0
a) are placed in air concentrically B is given charge + Q (A) (B)
coulomb and A is grounded. The equivalent capacitance d 2d
of these is 2 2
0 AV0 0 AV0
ab (C) (D)
(A) 4 (B) 4 3d d
0 0 (a b)
b a 204. The electric field between the plates of a parallel plate
2 capacitor when connected to a certain battery is E0. If the
b
(C) 4 0b (D) 4 0 space between the plates of the capacitor is filled by
b a introducing a material of dielectric constant K without
197. The capacity of a parallel plate condenser is 10 F, when disturbing the battery connections, the field between the
the distance between its plates is 8 cm. If the distance plates shall be
between the plates is reduced to 4 cm, then the capacity (A) KE0 (B) E0
of this parallel plate condenser will be
E0
(A) 5 F (B) 10 F (C) (D) None of the above
K
(C) 20 F (D) 40 F
198. A capacitor is used to store 24 watt hour of energy at 1200 205. If the distance between parallel plates of a capacitor is
volt. What should be the capacitance of the capacitor halved and dielectric constant is doubled then the
capacitance
(A) 120 mF (B) 120 F
(A) Decreases two times (B) Increases two times
(C) 24 F (D) 24 mF
199. The mean electric energy density between the plates of a (C) Increases four times (D) Remain the same
charged capacitor is (here q = charge on the capacitor and 206. Putting a dielectric substance between two plates of
A = area of the capacitor plate) condenser, capacity, potential and potential energy
respectively
q2 q (A) Increase, decrease, decrease
(A) (B)
2 0A 2 2 0 A2 (B) Decrease, increase, increase
(C) Increase, increase, increase
q2 (D) Decrease, decrease, decrease
(C) (D) None of the above 207. A thin metal plate P is inserted half way between the plates
2 0A
of a parallel plate capacitor of capacitance C in such a
200. A charge of 40 C is given to a capacitor having way that it is parallel to the two plates. The capacitance
capacitance C = 10 F. The stored energy in ergs is now becomes
(A) 80 × 10–6 (B) 800 (A) C (B) C/2
(C) 80 (D) 8000 (C) 4C (D) None of these
208. If there are n capacitors in parallel connected to V volt (A) Electric field between the plates
source, then the energy stored is equal to (B) Potential difference across the plates
(A) CV (C) Charge on the plates
(D) Energy stored in the capacitor
1
(B) nCV 2 218. Capacitance of a parallel plate capacitor becomes 4/3 times
2
its original value if a dielectric slab of thickness t = d/2 is
(C) CV2 inserted between the plates (d is the separation between
1 the plates). The dielectric constant of the slab is
(D) CV 2 (A) 8 (B) 4
2n
(C) 6 (D) 2
209. If n drops, each of capacitance C, coalesce to form a single
219. A 10 micro-farad capacitor is charged to 500 V and then
big drop, then the ratio of the energy stored in the big
its plates are joined together through a resistance of 10
drop to that in each small drop will be
ohm. The heat produced in the resistance is
(A) n : 1 (B) n1/3 : 1 (A) 500 J (B) 250 J
(C) n5/3 : 1 (D) n2 : 1 (C) 125 J (D) 1.25 J
210. A conducting sphere of radius 10cm is charged 10 C. 220. The unit of electric permittivity is
Another uncharged sphere of radius 20 cm is allowed to (A) Volt/m2 (B) Joule/coulomb
touch it for some time. After that if the sphere are separated, (C) Farad/m (D) Henry/m
then surface density of charges, on the spheres will be in
221. The work done in placing a charge of 8 × 10–18 coulomb
the ratio of
on a condenser of capacity 100 micro-farad is
(A) 1 : 4 (B) 1 : 3
(A) 32 10 32 (B) 16 10 32
(C) 2 : 1 (D) 1 : 1 Joule Joule
211. 64 small drops of mercury, each of radius r and charge q 26 10
coalesce to form a big drop. The ratio of the surface density (C) 3.1 10 Joule (D) 4 10 Joule
of charge of each small drop with that of the big drop is 222. 64 drops of mercury each charged to a potential of 10V.
(A) 1 : 64 (B) 64 : 1 They are combined to form one bigger drop. The potential
(C) 4 : 1 (D) 1 : 4 of this drop will be (Assume all the drops to be spherical)
212. Capacitance (in F) of a spherical conductor with radius (A) 160 V (B) 80 V
1m is (C) 10 V (D) 640 V
(A) 1.1 10 10 (B) 10–6 223. A spherical drop of mercury having a potential of 2.5 V is
obtained as a result of merging 125 droplets. The potential
(C) 9 10 9 (D) 10–3 of constituent droplets would be
213. A condenser has a capacity 2 F and is charged to a voltage (A) 1.0 V (B) 0.5 V
of 50 V. The energy stored is (C) 0.2 V (D) 0.1 V
(A) 25 × 105 Joule (B) 25 Joule 224. A parallel plate capacitor of capacity C0 is charged to a
(C) 25 × 10 erg (D) 25 × 103 erg potential V0
214. The energy required to charge a capacitor of 5 F by (i) The energy stored in the capacitor when the battery
connecting a d.c. source of 20 kV is is disconnected and the separation is doubled E1
(A) 10 kJ (B) 5 kJ (ii) The energy stored in the capacitor when the charging
(C) 2 kJ (D) 1 kJ battery is kept connected and the separation between
215. The capacitance of a parallel plate capacitor is 12 F. If the E1
distance between the plates is doubled and area is halved, the capacitor plates is doubled is E2 Then E value
2
then new capacitance will be
(A) 8 F (B) 6 F is
(C) 4 F (D) 3 F (A) 4 (B) 3/2
216. A capacitor of capacitance 6 F is charged upto 100 volt. (C) 2 (D) 1/2
The energy stored in the capacitor is 225. A parallel plate capacitor carries a charge q. The distance
(A) 0.6 Joule (B) 0.06 Joule between the plates is doubled by application of a force.
The work done by the force is
(C) 0.03 Joule (D) 0.3 Joule
217. A parallel plate air capacitor is charged and then isolated. q2
(A) Zero (B)
When a dielectric material is inserted between the plates C
of the capacitor, then which of the following does not
q2 q2
change? (C) (D)
2C 4C
226. As in figure shown, if a capacitor C is charged by 233. On increasing the plate separation of a charged condenser,
connecting it with resistance R, then energy is given by the energy
the battery will be (A) Increases (B) Decreases
C (C) Remains unchanged (D) Becomes zero
234. The energy stored in a condenser is in the form of
(A) Kinetic energy (B) Potential energy
(C) Elastic energy (D) Magnetic energy
R 235. When a dielectric material is introduced between the plates
of a charge’s condenser, then electric field between the
V plates
1 2 1 2 (A) Remain constant
(A) CV 7 (B) More than CV
2 2 (B) Decreases
1 (C) Increases
(C) Less than CV 2 (D) Zero
2 (D) First increases then decreases
227. A capacitor is charged to 200 volt it has 0.1 coulomb 236. When a lamp is connected in series with capacitor, then
charge. When it is discharged, energy will be (A) Lamp will not glow
(A) 1 J (B) 4 J (B) Lamp will burst out
(C) 10 J (D) 20 J (C) Lamp will glow normally
228. If eight identical drops are joined to form a bigger drop, (D) None of these
the potential on bigger as compared to that on smaller 237. If the potential of a capacitor having capacity of 6 F is
drop will be increased from 10 V to 20 V, then increase in its energy will be
(A) Double (B) Four times (A) 12 × 10–6 J (B) 9 × 10–4 J
(C) Eight times (D) One time (C) 4 × 10 J–6 (D) 4 ×10–9 J
229. If a dielectric substance is introduced between the plates 238. A 4 F condenser is charged to 400 V and then its plates
of a charged air-gap capacitor. The energy of the capacitor are joined through a resistance. The heat produced in the
will resistance is
(A) Increase (A) 0.16 J (B) 0.32 J
(B) Decrease (C) 0.64 J (D) 1.28 J
(C) Remain unchanged 239. Five capacitors of 10 F capacity each are connected to a
(D) First decrease and then increase d.c. potential of 100 volts as shown in the adjoining figure.
The equivalent capacitance between the points A and B
230. A 40 F capacitor in a defibrillator is charged to 3000 V.
will be equal to
The energy stored in the capacitor is sent through the
patient during a pulse of duration 2ms. The power 10 F 10 F
delivered to the patient is
(A) 45 kW (B) 90 kW A B
10 F
(C) 180 kW (D) 360 kW
231. A spherical drop of capacitance 1 F is broken into eight 10 F 10 F
drops of equal radius. Then, the capacitance of each small
drop is ......
100 Volt
1 (A) 40 F (B) 20 F
(A) F (B) 8 F
8 (C) 30 F (D) 10 F
240. Three capacitors of capacitances 3 F, 9 F and 18 F are
1 1 connected once in series and another time in parallel. The
(C) F (D) F
2 2
232. An air filled parallel plate capacitor has capacity C. If Cs
ratio of equivalent capacitance in the two cases Cp
distance between plates is doubled and it is immersed in a
liquid then capacity becomes twice. Dielectric constant
will be
of the liquid is
(A) 1 : 15 (B) 15 : 1
(A) 1 (B) 2
(C) 1 : 1 (D) 1 : 3
(C) 3 (D) 4
241. Four condensers each of capacity 4 F are connected as 246. The capacitor of capacitance 4 F and 6 Fare connected
shown in figure. VP – VQ = 15 Votts. The energy stored in in series. A potential difference of 500 volts is applied to
the system is the outer plates of the two capacitor system. The potential
4 F difference across the plates of capacitor of 4 F capacitance
is
4 F 4 F F (A) 500 volts (B) 300 volts
P Q (C) 200 volts (D) 250 volts
4 F
247. Two capacitances of capacity C1 and C2 are connected in
series and potential difference V is applied across it. Then
(A) 2400 ergs (B) 1800 ergs the potential difference across C1 will be
(C) 3600 ergs (D) 5400 ergs C2 C1 C2
242. Two capacitors each of 1 F capacitance are connected in (A) V C (B) V C
1 1
parallel and are then charged by 200 volts d.c. supply. C2 C1
The total energy of their charges (in joules) is (C) V C C (D) V C C
1 2 1 2
(A) 0.01 (B) 0.02
248. Three capacitances of capacity 10 F, 5 F and 5 F are
(C) 0.04 (D) 0.06
connected in parallel. The total capacity will be
243. In an adjoining figure are shown three capacitors C1, C2 (A) 10 F (B) 5 F
and C3 joined to a battery. The correct condition will be
(C) 20 F (D) None of the above
(Symbols have their usual meanings)
249. Three capacitors of capacity C1, C2, C3 are connected in
V2 C2 Q2 series. Their total capacity will be
V1 C1 Q1
1
(A) C1 C2 C3 (B) (C C C )
1 2 3
V3 Q3
C3
(C) (C1 1 C2 1 C3 1 ) 1 (D) None of these
+ –
V 250. Two capacitors of equal capacity are first connected in
(A) Q1 Q2 Q3 and V1 V2 V3 V parallel and then in series. The ratio of the total capacities
in the two cases will be
(B) Q1 Q2 Q3 and V V1 V2 V3 (A) 2 : 1 (B) 1 : 2
(C) 4 : 1 (D) 1 : 4
(C) Q1 Q2 Q3 and V V1 V2
251. Two capacitors connected in parallel having the capacities
(D) Q 2 Q3 and V2 V3 C1 and C2 are given ‘q’ charge, which is distributed among
them. The ratio of the charge on C1 and C2 will be
244. In the circuit diagram shown in the adjoining figure, the
C1 C2
resultant capacitance between P and Q is (A) C (B) C
2 1
12 F
1
P (C) C1 C2 (D) C C
1 2
2 F 3 F 252. Two capacitors of capacities C1 and C2 are charged to
voltages V1 and V2 respectively. There will be no exchange
of energy in connecting them in parallel, if
Q
(A) C1 C2 (B) C1V1 C2V2
20 F
(A) 47 F (B) 3 F C1 C2
(C) 60 F (D) 10 F (C) V1 V2 (D) V V2
1
245. Two condensers of capacity 0.3 F and 0.6 F respectively
253. If three capacitors each of capacity 1 F are connected in
are connected in series. The combination is connected
such a way that the resultant capacity is 1.5 F, then
across a potential of 6 volts. The ratio of energies stored
(A) All the three are connected in series
by the condensers will be
(B) All the three are connected in parallel
1 (C) Two of them are in parallel and connected in series to
(A) (B) 2
2 the third
1 (D) Two of them are in series and then connected in
(C) (D) 4 parallel to the third
4
254. A capacitor of capacity C1 is charged to the potential of 258. Three condensers each of capacitance 2F are put in series.
Vo. On disconnecting with the battery, it is connected The resultant capacitance is
with a capacitor of capacity C2 as shown in the adjoining
figure. The ratio of energies before and after the 3
(A) 6F (B) F
connection of switch S will be 2

2
S (C) F (D) 5 F
3
C1 V0 C2
259. Two condensers of capacities 1 F and 2 F are connected
in series and the system is charged to 120 volts. Then the
P.D. on 1 F capacitor (in volts) will be
(A) (C1 C2 ) / C1 (B) C1 /(C1 C2 )
(A) 40 (B) 60
(C) C1C 2 (D) C1 / C2 (C) 80 (D) 120
255. Four capacitors of each of capacity 3 F are connected as 260. Four condensers are joined as shown in the adjoining
shown in the adjoining figure. The ratio of equivalent figure. The capacity of each is 8 F. The equivalent
capacitance between A and B and between A and C will be capacity between the points A and B will be
A B

B
C
(A) 32 F (B) 2 F
(A) 4 : 3 (B) 3 : 4
(C) 8 F (D) 16 F
(C) 2 : 3 (D) 3 : 2
261. The capacities and connection of five capacitors are
256. The capacities of two conductors are C1 and C2 and their shown in the adjoining figure. The potential difference
respective potentials are V1 and V2. If they are connected between the points A and B is 60 volts. Then the equivalent
by a thin wire, then the loss of energy will be given by capacity between A and B and the charge on 5 f
capacitance will be respectively
C1C2 (V1 V2 ) C1C2 (V1 V2 )
(A) 2(C1 C2 ) (B) 2(C1 C2 )
5 F 9 F
A
C1C2 (V1 V2 ) 2 (C1 C2 )(V1 V2 )
(C) (D) C1C2 12 F 10 F 8 F
2(C1 C2 )

257. A parallel plate condenser is filled with two dielectrics as


B
shown. Area of each plate is A metre2 and the separation
is t metre. The dielectric constants are k 1 and k 2
(A) 44 F ; 300 C (B) 16 F ; 150 C
respectively. Its capacitance in farad will be
(C) 15 F ; 200 C (D) 4 F ; 50 C
262. Three equal capacitors, each with capacitance C are
connected as shown in figure. Then the equivalent
k1 k2
capacitance between A and B is

A C C C B
0A 0A k1 k2
(A) (k1 k2 ) (B) .
t t 2
(A) C (B) 3C
2 0A 0A k k
(C) (k1 k2 ) (D) . 1 2 C 3C
t t 2 (C) (D)
3 2
263. Four plates of the same area of cross-section are joined as 269. In the connections shown in the adjoining figure, the
shown in the figure. The distance between each plate is d. equivalent capacity between A and B will be
The equivalent capacity across A and B will be

A 6 F 12 F
B
A B
9 F 24 F
2 0A 3 0A
(A) (B)
d d
18 F
3 0A 0A
(C) (D) (A) 10.8 F (B) 69 F
2d d
(C) 15 F (D) 10 F
264. In the adjoining figure, four capacitors are shown with
their respective capacities and the P.D. applied. The charge 270. 2 F capacitance has potential difference across its two
and the P.D. across the 4 F capacitor will be terminals 200 volts. It is disconnected with battery and
then another uncharged capacitance is connected in
parallel to it, then P.D. becomes 20 volts. Then the capacity
20 F of another capacitance will be
(A) 2 F (B) 4 F
300 Volt
4 F 4 F (C) 18 F (D) 10 F
12 F 271. The resultant capacitance between A and B in the following
figure is equal to

(A) 600 C; 150 volts 3 F 3 F 3 F


(B) 300 C ; 75 volts A

(C) 800 C; 200 volts


2 F 2 F 3 F
(D) 580 C; 145 volts
265. Three identical capacitors are combined differently. For
the same voltage to each combination, the one that stores B
the greatest energy is 3 F 3 F 3 F
(A) Two in parallel and the third in series with it
(A) 1 F (B) 3 F
(B) Three in series
(C) 2 F (D) 1.5 F
(C) Three in parallel
272. In the following circuit, the resultant capacitance between
(D) Two in series and third in parallel with it
A and B is 1 F. Then value of C is
266. Two capacitors each of capacity 2 F are connected in
parallel. This system is connected in series with a third C 1 F
capacitor of 12 F capacity. The equivalent capacity of A
the system will be 8 F
(A) 16 F (B) 13 F
6 F 4 F
(C) 4 F (D) 3 F
267. A 4 F condenser is connected in parallel to another 2 F
2 F 12 F
condenser of 8 F. Both the condensers are then B
connected in series with a 12 F condenser and charged
32
to 20 volts. The charge on the plate of 4 F condenser is (A) F
11
(A) 3.3 C (B) 40 C
(C) 80 C (D) 240 C 11
(B) F
268. A capacitor having capacitance C is charged to a voltage 32
V. It is then removed and connected in parallel with another 23
identical capacitor which is uncharged. The new charge (C) F
32
on each capacitor is now
(A) CV (B) CV / 2 32
(D) F
(C) 2 CV (D) CV / 4 23
273. Two dielectric slabs of constant K1 and K2 have been 277. In the circuit shown in the figure, the potential difference
filled in between the plates of a capacitor as shown below. across the 4.5 F capacitor is
What will be the capacitance of the capacitor? 3 F

4.5 F
k1 6 F
d
k2
12V

2 0A 8
(A) ( K1 K2 ) (A) volts
2 3
(B) 4 volts
2 0 A K1 K 2 (C) 6 volts
(B) 2 K1 K 2 (D) 8 volts
278. Minimum number of capacitors of 2 F capacitance each
2 0 A K1 K 2 required to obtain a capacitor of 5 F will be
(C) 2 K1 K 2 (A) Three (B) Four
(C) Five (D) Six
2 0 A K1 K 2 279. 100 capacitors each having a capacity of 10 F are
(D) d K1 K 2 connected in parallel and are charged by a potential
difference of 100 kV. The energy stored in the capacitors
274. What is the equivalent capacitance between A and B in and the cost of charging them, if electrical energy costs
the given figure (all are in farad)? 108 paise per kWh, will be
8 4
(A) 107 joule and 300 paise

A B (B) 5 106 joule and 300 paise


12 4 16
(C) 5 106 joule and 150 paise
13 48 (D) 107 joule and 150 paise
(A) F (B) F
18 13 280. Three capacitors of 2.0, 3.0 and 6.0 F are connected in
1 240 series to a 10 V source. The charge on the 3.0 F capacitor
(C) F (D) F
31 71 is
275. A condenser having a capacity of 6 F is charged to 100 V (A) 5 C (B) 10 C
and is then joined to an uncharged condenser of 14 F (C) 12 C (D) 15 C
and then removed. The ratio of the charges on 6 F and 281. Four capacitors are connected as shown in the figure.
14 F and the potential of 6 F will be Their capacities are indicated in the figure. The effective
6 capacitance between points x and y is (in F)
(A) and 50 volt
14 1 F 1 F
14
(B) and 30 volt
6
x 1 F y
6
(C) and 30 volt
14
14
(D) and 0 volt 2 F
6
276. 0.2 F capacitor is charged to 600 V by a battery. On 5 7
removing the battery, it is connected with another parallel (A) (B)
6 6
plate condenser of 1F. The potential decreases to
(A) 100 volts (B) 120 volts 8
(C) (D) 2
(C) 300 volts (D) 600 volts 3
282. In the circuit shown here C1 6 F , C2 3 F and 286. A 10 F capacitor and a 20 F capacitor are connected in
series across a 200 V supply line. The charged capacitors
battery B 20V . The switch S1 is first closed. It is then
are then disconnected from the line and reconnected with
opened and afterwards S2 is closed. What is the charge their positive plates together and negative plates together
finally on C2? and no external voltage is applied. What is the potential
C2 3 F difference across each capacitor?
400 800
C1 S2 (A) V (B) V
6 F 9 9
(C) 400 V (D) 200 V
S1
287. Two condensers C1 and C2 in a circuit are joined as shown
B = 20 V in figure. The potential of point A is V1 and that of B is V2.
(A) 120 C (B) 80 C The potential of point D will be
(C) 40 C (D) 20 C A D B
283. The effective capacitance between the points P and Q of V1 V2
the arrangement shown in the figure is C1 C2
2 F 1 C2V1 C1V2
(A) (V1 V2 ) (B)
2 C1 C2
2 F 1 F
P Q C1V1 C2V2 C2V1 C1V2
2 F 5 F (C) C1 C2 (D) C1 C2
288. To obtain 3 F capacity from three capacitors of 2 F
2 F 1 F
each, they will be arranged
1 (A) All the three in series
(A) F (B) 1 F
2 (B) All the three in parallel
(C) 2 F (D) 1.33 F (C) Two capacitors in series and the third in parallel with
284. A capacitor of capacitance 5 F is connected as shown in the combination of first two
the figure. The internal resistance of the cell is 0.5 . The (D) Two capacitors in parallel and the third in series with
amount of charge on the capacitor plate is the combination of first two
289. A 10 F capacitor is charged to a potential difference of
1 1
50 V and is connected to another uncharged capacitor in
parallel. Now the common potential difference becomes
5 F 20 volt. The capacitance of second capacitor is
2
(A) 10 F (B) 20 F
(C) 30 F (D) 15 F
2.5V
290. What is the effective capacitance between points X and
+ – Y?
(A) 0 C (B) 5 C
(C) 10 C (D) 25 C C1 = 6 F
285. Choose the incorrect statement from the following: When
two identical capacitors are charged individually to C3 = 6 F C2 = 6 F
different potentials and connected parallel to each other B
X Y
after disconnecting them from the source A C C = 20 F D
5
(A) Net charge equals the sum of initial charges
(B) The net energy stored in the two capacitors is less
than the sum of the initial individual energies C4 = 6 F
(C) The net potential difference across them is different (A) 24 F
from the sum of the individual initial potential
(B) 18 F
difference
(C) 12 F
(D) The net potential difference across them equals the
sum of the individual initial potential differences (D) 6 F
291. The combined capacity of the parallel combination of two 296. In the given network capacitance, C1 10 F , C2 5 F
capacitors is four times their combined capacity when
connected in series. This means that and C3 4 F . What is the resultant capacitance
(A) Their capacities are equal between A and B?
(B) Their capacities are 1 F and 2 F
A
(C) Their capacities are 0.5 F and 1 F C2
C1
(D) Their capacities are infinite
292. The charge on a capacitor of capacitance 10 F connected
as shown in the figure is C3
2 B
(A) 2.2 F (B) 3.2 F
3 10 F
(C) 1.2 F (D) 4.7 F
297. The equivalent capacitance between A and B is
1 F 1 F
2V 1 F
(A) 20 F (B) 15 F B
A
(C) 10 F (D) Zero
293. The resultant capacitance of given circuit is
1 F 1 F
P
2C (A) 2 F (B) 3 F
2C
(C) 5 F (D) 0.5 F
2C 298. The capacitance between the points A and B in the given
C circuit will be
C C 1.5 F
Q
(A) 3 C (B) 2 C 3 F 3 F
C A B
(C) C (D)
3
294. Three plates A, B, C each of area 50 cm2 have separation
1.5 F
3mm between A and B and 3mm between B and C. The
energy stored when the plates are fully charged is (A) 1 F (B) 2 F
(C) 3 F (D) 4 F
299. The equivalent capacitance of three capacitors of
A capacitance C1, C2 and C3 are connected in parallel is 12
B
C 12V units and product C1 .C2 .C3 48 . When the capacitors
C1 and C2 are connected in parallel, the equivalent
capacitance is 6 units. Then the capacitance are
(A) 1.6 10 9 (A) 2, 3, 7 (B) 1.5, 2.5, 8
J
9
(C) 1, 5, 6 (D) 4, 2, 6
(B) 2.1 10 J
300. In the circuit shown in figure, each capacitor has a capacity
(C) 5 10 9
J of 3 F. The equivalent capacity between A and B is

(D) 7 10 9 J
295. A capacitor of 20 F is charged to 500 volts and connected
A B
in parallel with another capacitor of 10 F and charged to
200 volts. The common potential is
(A) 200 volts
(B) 300 volts 3
(C) 400 volts (A) F (B) 3 F
4
(D) 500 volts (C) 6 F (D) 5 F
301. What is the effective capacitance between A and B in the 306. Two capacitors A and B are connected in series with a
following figure? battery as shown in the figure. When the switch S is
closed and the two capacitors get charged fully, then
2 F 2 F
2 F 3 F

A B
1 F
A B
(A) 1 F (B) 2 F
(C) 1.5 F (D) 2.5 F S
10V
302. A potential difference of 300 volts is applied to a
combination of 2.0 F and 8.0 F capacitors connected in
(A) The potential difference across the plates of A is 4V
series. The charge on the 2.0 F capacitor is
and across the plates of B is 6V
(A) 2.4 10 4C (B) 4.8 10 4C (B) The potential difference across the plates of A is 6V
and across the plates of B is 4V
(C) 7.2 10 4 C (D) 9.6 10 4 C
(C) The ratio of electrical energies stored in A and B is 2 : 3
303. Ten capacitor are joined in parallel and charged with a (D) The ratio of charges on A and B is 3 : 2
battery up to a potential V. They are then disconnected 307. In the figure, three capacitors each of capacitance 6pF are
from battery and joined again in series then the potential connected in series. The total capacitance of the
of this combination will be combination will be
(A) V (B) 10V
C1 C2 C3
(C) 5V (D) 2V
304. In the circuit here, the steady state voltage across capacitor
C is a fraction of the battery e.m.f. The fraction is decided by
R1

C
V
R2
R3 (A) 9 10 12 (B) 6 10 12
F F

(C) 3 10 12 (D) 2 10 12
F F
(A) R1 only (B) R1 and R2 only
(C) R1 and R3 only (D) R1, R2 and R3 308. Equivalent capacitance between A and B is
305. A parallel plate capacitor of area A, plate separation d and
capacitance C is filled with three different dielectric
materials having dielectric constants k1,k2 and k3 as 4 F 4 F
shown. If a single dielectric material is to be used to have
the same capacitance C in this capacitor, then its dielectric
constant k is given by 4 F
A = Area of parallel plate capacitor
A B
A/2 A/2 4 F 4 F
K1 K2 d/2
(A) 8 F (B) 6 F
d
(C) 26 F (D) 10 / 3 F
K3
309. Two capacitors of 10 F and 20 F are connected in series
A with a 30V battery. The charge on the capacitors will be,
respectively
1 1 1 1 1 1 1
(A) k (B) k (A) 100 C, 200 C
k1 k2 2k3 k1 k2 2k3
(B) 200 C, 100 C
k1k2 (C) 100 C, 100 C
(C) k k1 k2
2 k3 (D) k k1 k2 2k3
(D) 200 C, 200 C
310. In the figure a capacitor is filled with dielectrics. The 314. Two capacitors C1 2 F and C2 6 F in series, are
resultant capacitance is
connected in parallel to a third capacitor C3 4 F . This
A/2 A/2 arrangement is then connected to a battery of e.m.f. = 2V,
as shown in the figure. How much energy is lost by the
d/2 K1 battery in charging the capacitors ?
K3 d C1 C2
K2

2 0A 1 1 1 0A 1 1 1
(A) C3
d k1 k2 k3 (B) d k1 k2 k3
+ –
2 0A
(C) k1 k2 k3 (D) None of these 2V
d
311. Three capacitors of capacitance 3 F, 10 F and 15 F are (A) 22 10 6
J
connected in series to a voltage source of 100V. The charge
on 15 Fis (B) 11 10 6
J
(A) 50 C (B) 100 C
(C) 200 C (D) 280 C 32 6
(C) 10 J
312. Consider a parallel plate capacitor of 10 F (micro-farad) 3
with air filled in the gap between the plates. Now one half
of the space between the plates is filled with a dielectric of 16 6
(D) 10 J
dielectric constant 4, as shown in the figure. The capacity 3
of the capacitor changes to 315. A 20F capacitor is charged to 5V and isolated. It is then
connected in parallel with an uncharged 30F capacitor.
The decrease in the energy of the system will be
(A) 25 J (B) 200 J
K=4
(C) 125 J (D) 150 J
316. A parallel plate capacitor has capacitance C. If it is equally
filled with parallel layers of materials of dielectric constants
(A) 25 F (B) 20 F K1 and K2 its capacity becomes C1. The ratio of C1 to C is
(C) 40 F (D) 5 F (A) K1 K 2
313. The combination of capacitors with
K1 K 2
and C3 2 F is charged by (B) K K
1 2
connecting AB to a battery. Consider the following
statements K1 K 2
I. Energy stored in C1 = Energy stored in C2 + Energy (C) K1 K 2
stored in C3
II. Charge on C1 = Charge on C2 + Charge on C3 2K1K 2
III. Potential drop across C1 = Potential drop across C2 = (D) K K
1 2
Potential drop across C3
317. The equivalent capacitance in the circuit between A and B
Which of these is/are correct?
will be
C3
1 F 1 F 1 F
C1
A B A B
C2
(A) 1 F (B) 2 F
(A) I and II (B) II only
1
(C) I and III (D) III only (C) 3 F (D) F
3
318. The equivalent capacitance between A and B is 323. Two capacitors of capacitances 3 F and 6 F are charged
to a potential of 12 V each. They are now connected to
each other, with the positive plate of each joined to the
C C C C
negative plate of the other. The potential difference across
A each will be
(A) 6 volt (B) 4 volt
B (C) 3 volt (D) Zero
324. Two identical capacitors, have the same capacitance C.
C 3C One of them is charged to potential V1 and the other to V2.
(A) (B)
4 4 The negative ends of the capacitors are connected
together. When the positive ends are also connected, the
C 4C
(C) (D) decrease in energy of the combined system is
3 3
1 1
319. The effective capacity between A and B in the figure given (A) C (V12 V22 ) 2
(B) C (V1 V2 )
2
4 4
is
1 2 1 2
2 F 3 F (C) C V1 V2 (D) C V1 V2
4 4
A 325. A capacitor of 10 F charged up to 250 volts is connected
in parallel with another capacitor of 5 F charged up to
4 F 4 F 100 volts. The common potential is
(A) 500 V (B) 400 V
B (C) 300 V (D) 200 V
2 F 3 F 326. Two capacitors of 1 F and 2 F are connected in series,
the resultant capacitance will be
43 24 2
(A) F (B) F (A) 4 F (B) F
24 43 3
3
43 12 (C) F (D) 3 F
(C) F (D) F 2
12 43
327. The charge on any one of the 2 F capacitors and 1 F
320. In the given figure the capacitors C1 , C3 , C4 , C5 have a capacitor will be given respectively (in C) as
capacitance4 F each if the capacitor C2 has a capacitance 2 f 2 f
10 F, then effective capacitance between A and B will be
C4
1 g
C1 C2 C3
2v
(A) 1, 2 (B) 2, 1
A C5 B
(C) 1, 1 (D) 2, 2
328. When two identical capacitors are in series have 3 F
(A) 2 F (B) 4 F capacitance and when parallel 12 F. What is the
capacitance of each?
(C) 6 F (D) 8 F
(A) 6 F (B) 3 F
321. Two capacitors C1 and C2 = 2C1 are connected in a circuit
with a switch between them as shown in the figure. Initially (C) 12 F (D) 9 F
the switch is open and C1 holds charge Q. The switch is 329. In the circuit as shown in the figure the effective
closed. At steady state, the charge on each capacitor will be capacitance between A and B is
4 F
(A) Q, 2Q (B) Q / 3, 2Q / 3
A
(C) 3Q / 2, 3Q (D) 2Q / 3, 4Q / 3
322. Three capacitors of 2 F, 3 F and 6 F are joined in series 4 F 2 F 2 F
and the combination is charged by means of a 24-volt 4 F
battery. The potential difference between the plates of
the 6 F capacitor is B
(A) 4 volt (B) 6 volt (A) 3 F (B) 2 F
(C) 8 volt (D) 10 volt (C) 4 F (D) 8 F
330. Four equal capacitors, each of capacity C, are arranged as 334. A capacitor of capacity C1 is charged upto V volt and
shown. The effective capacitance between A and B is then connected to an uncharged capacitor of capacity
C C2. Then final potential difference across each will be

C C2V C2
(A) C C (B) 1 V
A C B 1 2 C1

C1V C2
C (C) C C (D) 1 V
1 2 C1

335. A series combination of three capacitors of capacities


5 1 F, 2 f and 8 F is connected to a battery of e.m.f. 13
(A) C
8 volt. The potential difference across the plates of 2 F
capacitor will be
3
(B) C (A) 1 V (B) 8 V
5
13
5 (C) 4 V (D) V
(C) C 3
3
336. Two capacitors of capacitance 2 F and 3 F are joined in
(D) C
series. Outer plate first capacitor is at 1000 volt and outer
331. In the figure shown, the effective capacitance between plate of second capacitor is earthed (grounded). Now the
the points A and B, if each has capacitance C, is potential on inner plate of each capacitor will be
(A) 700 Volt (B) 200 Volt
C (C) 600 Volt (D) 400 Volt
B 337. In the figure a potential of + 1200 V is given to point A and
point B is earthed, what is the potential at the point P?
C
C C 4 F

3 F
C B
A
A P

2 F
C
(A) 2C (B) (A) 100 V (B) 200 V
5
(C) 400 V (D) 600 V
C 338. All six capacitors shown are identical, each can withstand
(C) 5C (D)
2 maximum 200 volts between its terminals. The maximum
voltage that can be safely applied between A and B is
332. Three capacitors each of capacity 4 F are to be connected
in such a way that the effective capacitance is 6 F. This
can be done by
(A) Connecting them in parallel
(B) Connecting two in series and one in parallel
(C) Connecting two in parallel and one in series
A B
(D) Connecting all of them in series
333. Three capacitors of capacitance 3 F are connected
parallel in a circuit. Then their maximum and minimum
capacitances will be
(A) 9 F, 1 F (B) 8 F, 2 F (A) 1200 V (B) 400 V
(C) 9 F, 0 F (D) 3 F, 2 F (C) 800 V (D) 200 V
(Previous Year AIPMT Examination Questions)
1. Charge +q and –q are placed at points A and B respectively 6. Three capacitors each of capacitance C and of breakdown
which are a distance 2L apart, C is the midpoint between A voltage V are joined in series. The capacitance and
and B. The work done in moving a charge +Q along the breakdown voltage of the combination will be
semicircle CRD is [AIPMT 2007] [AIPMT 2009]
R C V V
(A) , (B) 3C,
3 3 3

A C B D
C
(C) , 3V (D) 3C, 3V
3
qQ qQ 7. The electric potential at a point (x,y,z) is given by
(A) 4 (B) 2
0L 0L [AIPMT 2009]
V = – x2y – xz3 + 4
qQ qQ
(C) 6 (D) – 6 The electric field E at that point is
0L 0L

2. Two condensers, one of capacity C and the other of ˆ 3 ˆ 2 ˆ 2


(A) E = i 2xy + z + j x + k 3xz
capacity C/2, are connected to a V volt battery, as shown.
[AIPMT 2007] (B) E = ˆi 2 xy + ˆj ( x 2 y2 ) kˆ (3xz – y2 )

C (C) E = ˆi z3 Jˆ xyz kˆ z 2
V C
2
(D) E = iˆ 2xy – z3 + ˆj xy 2 + kˆ 3z 2 x
1
(A) 2CV2 (B) CV 2 8. A series combination of n1 capacitors, each of value C1,
4 is charged by a source of potential difference 4V. When
3 1 another parallel combination of n2 capacitors, each of value
(C) CV 2 (D) CV 2
4 2 C2, is charged by a source of potential difference V, it has
3. The electric potential at a point in free sapce due to a the same (total) energy stored in it, as the first combination
charge Q coulomb is Q × 1011 volts. The electric field at has, The value of C2, in terms of C1, is then
that point is [AIPMT 2008] [AIPMT 2010]
22
(A) 4 0 Q×10 V/m (B) 12 0 Q×1020 V/m
20
(C) 4 0 Q×10 V/m (D) 12 0 Q×1022 V/m 2C1 2 n
(A) n n (B) 16 n C1
4. The energy required to charge a parallel plate condenser 1 2 1

of plate separation d and plate area of cross-section A


such that the uniform electric field between the plates is E 1 n 16C1
(C) 2 n C1 (D) n n
is [AIPMT 2008] 1 1 2

1 9. A parallel plate condenser has a uniform electric field E


2
0E / Ad
(A) (B) 2
2 0E /Ad (V/m) in the space between the plates. If the distance
between the plates is d (m) and area of each plate is A(m2)
1 2 the energy (joule) stored in the condenser is
(C) 0E2Ad (D) 0E Ad
[AIPMT 2011]
2
5. Three concentric spherical shells have radii a, b and c 1 2
(a < b < c) and have surface charge densities ,– and (A) 0E (B) 0EAd
2
respectively. If VA, VB and VC denote the potentials of the
three shells,then for c = a + b, we have 1 2
[AIPMT 2009] (C) 0E Ad (D) E2Ad/ 0
2
(A) VC=VA VB (B) VC=VB VA
(C) VC VB VA (D) VC= VB= VA
10. Four electric charges +q, +q, –q are placed at the corners 14. Two thin dielectric slabs of dielectric constants K1 and K2
of a square of side 2L (see figure). The electric potential at (K1< K2) are inserted between plates of a parallel plate
point A, mid-way between the two charge +q and +q, is capacitor, as shown in the figure. The variation of electric
[AIPMT 2011] field E between the plates with distance d as measured
–q from plate P is correctly shown by [AIPMT 2014]
+q
P+ –Q
+ –
L + –
+ –
A + –
+ –
+ –
L + –
K1 K2

+q 2L –q
E E
1 2q 1
(A) 1+ (A) (B)
4 0 L 5
0 0
1 2q 1 d d
(B) 1–
4 0 L 5
E E
(C) Zero
1 2q (C) (D)
(D) 4 1+ 5
0 L
0 0
d d
11. The potential energy of a particle in a force field is : 15. A conducting sphere of radius R is given a charge Q. The
A B electric potential and the electric field at the centre of the
U= – , where A and B are positive constants and sphere respectively are [AIPMT 2014]
r2 r
Q
r is the distance of particle from the centre of the field. For (A) Zero and
stable equilibrium the distance of the particle is 4 0R 2
[AIPMT 2012] Q
(B) 4 and Zero
(A) B/A (B) B/2A 0R
(C) 2A/B (D) A/B
Q Q
12. Four point charges –Q,–q, 2q and 2Q are placed, one at (C) 4 and
0R 4 0R 2
each corner of the square. The relation between Q and q
for which the potential at the centre of the square is zero (D) Both are zero
is [AIPMT 2012] 16. In a region, the potential is represented by V(x,y,z) = 6x –
8xy – 8y + 6yz, where V is in volts and x, y, z are in meters.
1 The electric force experienced by a change of 2 coulomb
(A) (B) Q = –q
q situated at point (1,1,1) is
[AIPMT 2014]
1
(C) Q – (D) Q = q (A) 6 5 N (B) 30 N (C) 24 N (D) 4 35 N
q
17. A parallel plate air capacitor of capacitance C is connected
13. A, B and C, are three points in a uniform electric field. The to a cell of emf V and then disconnected from it. A dielectric
electric potential is [AIPMT 2013] slab of dielectric constant K, which can just fill the air gap
of the capacitor, is now inserted in it. Which of the
B A following is incorrect ? [AIPMT 2015]
E
C (A) The potential difference between the plates decreases
K times
(A) Maximum at A (B) The energy stored in the capacitor decrease K times
(B) Maximum at B
1 1
(C) Maximum at C (C) The change in energy stored is CV 2 –1
2 K
(D) Same at all the three points A, B and C
(D) The charge on the capacitor is not conserved
18. A parallel plate air capacitor has capacity C distance of 23. The diagrams below show regions of equipotentials.
separation between plates is d and potential difference V [NEET – 2017]
is applied between the plates force of attraction between
the plate air capacitor is [AIPMT 2015] 20 V 40 V 20 V 40 V 10 V 30 V 40 V
2 2 20 V
CV C2 V2
(A) (B)
2d 2 2d A B A BA B A B
CV 2 CV 2 10 V
(C) (D)
2d d 10 V 30 V 10 V 30 V 20 V 40 V 30 V
19. If potential (in volts) in a region is expressed as V(x, y, z)
(a) (b) (c) (d)
= 6xy – y + 2yz, the electric field (in N/C) at point (1,1,0) is
[AIPMT 2015] A positive charge is moved from A to B in each diagram.
(A) – 6 ˆi + 9 ˆj+ kˆ (B) – 3i +5 j+3kˆ
ˆ ˆ (A) Maximum work is required to move q in figure (c).
(B) In all the four cases the work done is the same.
(C) – 6 ˆi + 5 ˆj+ 2kˆ (D) – 2 ˆi +3 ˆj+ kˆ
(C) Minimum work is required to move q in figure (a).
20. A capacitor of 2mF is charged as shown in the diagram. (D) Maximum work is required to move q in figure (b).
When the switch S is turned to position 2, the percentage 24. A capacitor is charged by a battery. The battery is removed
of its stored energy dissipated is [AIPMT 2016] and another identical uncharged capacitor is connected
1 2 in parallel. The total electrostatic energy of resulting
system : [NEET – 2017]
S
(A) increases by a factor of 4
V (B) decreases by a factor of 2
(C) remains the same
(D) increases by a factor of 2
(A) 0% (B) 20% (C) 75% (D) 80% 25. The electrostatic force between the metal plates of an
21. A small signal voltage V(t) = V0 sin ? t is applied across isolated parallel plate capacitor C having a charge Q
an ideal capacitor C :- [AIPMT 2016] and area A, is [NEET-2018]
(A) Current I (t), lags voltage V(t) by 90°. (A) proportional to the square root of the distance
(B) Over a full cycle the capacitor C does not consume between the plates.
any energy from the voltage source.
(B) linearly proportional to the distance between the
(C) Current I (t) is in phase with voltage V(t). plates.
(D) Current I (t) leads voltage V(t) by 180°
(C) independent of the distance between the plates.
22. A parallel-plate capacitor of area A, plate separation d and
capacitance C is filled with four dielectric materials having (D) inversely proportional to the distance between the
dielectric constants k1,k2, k3 and k4 as shown in the figure plates.
below. If a single dielectric material is to be used to have 26. Which of the following acts as a circuit protection
the same capacitance C in this capacitor, then its dielectric device? [NEET-2019]
constant k is given by :- [NEET – II (2016)] (A) inductor
(B) switch
(C) fuse
(D) conductor
27. A parallel plate capacitor of capacitance 20 F is being
charged by a voltage source whose potential is changing
at the rate of 3 V/s. The conduction current through the
connecting wires, and the displacement current through
2 3 1 the plates of the capacitor, would be, respectively :
(A) k = k + k + k + k [NEET-2019]
1 2 3 4
1 1 1 1 3 (A) 60 A, 60 A
(B) k = k + k k 3 2k 4
1 2 (B) 60 A, zero
(C) k = k1 + k2 + k3 = 3k4
(C) zero, zero
2
(D) k k1 k 2 k 3 (D) 60 A
3
(Previous Year AIIMS Questions)
1. If an electron is brought toward another electron,the 8. Seven capacitors each of capacitance 2 F areto be
electrostatic potential energy of the system (1995)
10
(A) becomes zero (B) increases connected to obtain a capacitance of F . Which of
11
(C) remains the same (D) decreases
the following combination is possible? (2000)
2. If a point charge moves round in a circle about a charge q,
(A) 5 in parallel, 2 in series
then work done by the charge, is (1997)
(B) 4 in parallel, 3 in series
1 q (C) 3 in parallel, 4 in series
(A) zero (B) 4 ×
0 r (D) 2 in parallel, 5 in series.
9. Potential energy of equal +ve charges1 C held 1 m apart
1 q2 1 q2 in air is (2000)
(C) × (D) ×
4 0 r 4 0 r2 (A) 1 J (B) 9× 10–3eV
(C) zero (B) 9× 10–3J
3. What is the area of the plates of a 3 F parallel plate capacitor,
10. Electric potential V at any point x, y, z in space is given by
if the separation between the plates is 5 mm? (1998)
9 2 V= 6z2.The value of the electric field at the point (2, –1,3)
(A) 9.281 ×10 m
is (2000)
(B) 4.529 × 109m2
(A) 24 (B) –12
(C) 1.694 × 109m2
(C) –36 (D) 12
(D) 12.981 × 109m2
11. From the figure find the capacitance of the capacitor?
4. Electric potential of earth is taken to be zero, because
earth is a good (1998) K1 K2
P
(A) semiconductor
(B) conductor
(C) insulator A/2 A/2
(D) dielectric
5. Two identical conductors of copper and aluminium are Q
placed in an identical electric fields. The magnitude of
induced charge in the aluminum will be (2000) (2001)
(A) zero A K1 + K 2
0
(B) greater than in copper (A) C = d 2
(C) equal to that in copper
(D) less than in copper A K1 K 2
0
6. Given a number of capacitors labelled as 8 F, 250 V. Find (B) C = 2d K1 + K 2
the minimum number of capacitors needed to get an
arrangement equivalent to16 F, 1000 V is (2000)
0 A K1
(A) 4 (C) C = d K2
(B) 16
(C) 32
(D) 64 0 A 2K1K 2
(D) C = d K1 K 2
7. An insulated charged of a hollow metal sphere of radius
5 cm has a potential of 10 V at the surface. What is the 12. If a unit positive charge is taken from one point to another
potential at centre? (2000) over an equipotential surface, then (2001)
(A) 10V (A) work is done on the charge
(B) zero (B) work is done by the charge
(C) same as that at 5 cm from the surface (C) work done is constant
(D) same as that at 25 cm from the surface (D) no work is done
13. In the given figure, the capacitance C1, C3, C4, C5 have a 18. Two infinitely long parallel conducting plates having
capacitance 4 F each. If the capacitor C2 has a capacitance surface charge densities + and – respectively, are
10 F, then effective capacitance between A and B will be separated by a small distance.The medium between the
plates is vacuum. If 0 is the dielectric permittivity of
C4 vacuum, then the electric field in the region between the
plates is (2005)
(A) 0 volt/meter (B) /2 0 volt/meter
b c d (C) / 0 volt/meter (D) 2 / 0 volt/meter
B
A a C1 C2 C3 19. Two concentric conducting thin spherical shells A and B
having radii r A and rB (rB > rA) are charged to QA and
– QB(|QB| < |QA|). The electrical potential along a line,
C5
(passing through the centre is)

(2002)
(A) 2 F (B) 6 F
(C) 4 F (D) 8 F V
14. A conducting sphere of radius 10 cm is charged with10 C. (A)
Another uncharged sphere of radius 20 cm is allowed to 0 rA rB
touch it for some time. After that if the spheres are X

separated, then surface densityof charges on the spheres


will be in the ratio of (2002)
(A) 1: 4 (B) 2: 1
(C) 1: 3 (D) 1: 1
15. Figure shown is a distribution of charges. The flux of
V
electric field due to these charges through the surface S is (B)
0 rA rB
X
S

+q –q

V
+q
(C)
0 r r
(2003) A B
X
(A) 3q/ 0 (B) 2q/ 0
(C) q/ 0 (D) zero
16. Equipotential surfaces associated with an electric field
which is increasing in magnitude along the x-direction are
(2004) V
(A) planes parallel to yz-plane (D)
(B) planes parallel to xy-plane
0 r r
X
A B

(C) planes parallel to xz-plane


(D) coaxial cylinders of increasing radii around the x-axis.
20. The voltage of clouds is 4 × 106 volt with respect to ground.
17. A 40 F capacitor in a defibrillator is charged to 3000 V.
In a lightening strike lasting 100 msec, a charge of 4
The energy stored in the capacitor is sentthrough the
coulombs is delivered to the ground. The power of the
patient during a pulse of duration 2 ms. The power
lightening strike is (2006)
delivered to the patient is (2004)
(A) 160 MW (B) 80 MW
(A) 45 kW (B) 90 kW
(C) 20 MW (D) 500 MW
(C) 180 kW (D) 360 kW
21. Five capacitors, each of capacitance value C are connected 24. What would be the voltage across C3? (2010)
as shown in the figure. The ratio of capacitance between
P and R, and the capacitance between Pand Q if the
battery is connected to P and R first and then Pand Q C1
(2006)
P C2
C C

Q T C3 V

C C C1 C2 V
(A)
C1 C2 C3
R S
C
(A) 3 : 1 (B) 5 : 2 C1V
(B) C C C
(C) 2 : 3 (D) 1 : 1 1 2 3
22. A conducting sphere of radius R carrying charge Q lies
inside an uncharged conducting shell of radius 2R. If they C2 V
(C) C C
are joined by a metal wire, the amount of heat that will be 1 2 C3
produced is (2009)
C3 V
1 Q2 (D) C C C
(A) . 1 2 3
4 0 4R
25. An electric charge 10–3 C is placed at the origin (0,0) of
1 Q2 (x–y) co-ordinate system. Two points A and B are situated
(B) .
4 0 2R
at 2, 2 and (2, 0) respectively. The potential
1 Q2
(C) . difference between the points A and B will be (2010)
4 0 R
(A) 4.5 volt
2 Q2 (B) 9 volt
(C) .
4 0 3R (C) zero
23. What is the energy stored in the capacitor between (D) 2 volt
terminals a and b of the network shown in the figure ?
26. In figure a particle having mass m = 5 g and charge q' =
(Capacitance of each capacitor C = 1 F) (2009)
2 ×10–9 C starts from rest at point a and moves in a straight
10 V line to point b. What is its speed v at point b? (2010)
–9
3 × 10 C a b –9
–3 × 10 C
a
C C
c 1 cm 1 cm 1 cm
d
C b C
(A) 2.65 cms–1
(B) 3.65 cms–1
C
(C) 4.65 cms–1
(A) 12.5 J (B) Zero
(D) 5.65 cms–1
(C) 25 J (D) 50 J
(Assertion & Reason Type Questions)

Read the assertion and reason carefully to mark the correct 8. Assertion : Dielectric breakdown occurs under the
option out of the options given below : influence of an intense light beam.
(A) If both assertion and reason are true and the reason is Reason : Electromagnetic radiations exert pressure.
the correct explanation of the assertion. 9. Assertion : When charges are shared between any two
(B) If both assertion and reason are true but reason is not bodies, no charge is really lost, but some
the correct explanation of the assertion. loss of energy does occur.
(C) If assertion is true but reason is false. Reason : Some energy disappears in the form of heat,
(D) If the assertion and reason both are false. sparking etc.
1. Assertion : The coulomb force is the dominating force 10. Assertion : Surface of a symmetrical conductor can be
in the universe. treated as equipotential surface.
Reason : The coulomb force is weaker than the
Reason : Charges can easily flow in a conductor.
gravitational force.
11. Assertion : The capacity of a given conductor remains
[AIIMS 2003]
same even if charge is varied on it.
2. Assertion : If three capacitors of capacitance C1 < C2 <
C3 are connected in parallel then their Reason : Capacitance depends upon nearly medium
equivalent capacitance Cp > Cs as well as size and shape of conductor.
12. Assertion : Charge is invariant.
1 1 1 1
Reason : C [AIIMS 2002] Reason : Charge does not depends on speed of frame
p C1 C2 C3 of reference.
3. Assertion : A metallic shield in form of a hollow shell 13. Assertion : Mass of ion is slightly differed from its
may be built to block an electric field. element.
Reason : In a hollow spherical shield, the electric field Reason : Ion is formed, when some electrons are
inside it is zero at every point. removed or added so mass changes.
[AIIMS 2001] 14. Assertion : Charge is quantized
4. Assertion : Electrons move away from a low potential Reason : Charge, which is less than 1 C is not
to high potential region. possible
Reason : Because electrons has negative charge 15. Assertion : If a point charge q is placed in front of an
[AIIMS 1999] infinite grounded conducting plane surface,
5. Assertion : If the distance between parallel plates of a the point charge will experience a force.
capacitor is halved and dielectric constant
Reason : This force is due to the induced charge on
is made three times, then the capacitor
the conducting surface which is at zero
becomes 6 times.
potential.
Reason : Capacity of the capacitor does not depend
upon the nature of the material. 16. Assertion : The surface charge densities of two
spherical conductors of different radii are
[AIIMS 1997]
equal. Then the electric field intensities near
6. Assertion : A parallel plate capacitor is connected their surface are also equal.
across battery through a key. A dielectric
slab of constant K is introduced between Reason : Surface charge density is equal to charge
the plates. The energy which is stored per unit area.
becomes K times. 17. Assertion : Three equal charges are situated on a circle
Reason : The surface density of charge on the plate of radius r such that they form on equilateral
remains constant or unchanged. triangle, then the electric field intensity at
[AIIMS 1996] the centre is zero.
7. Assertion : If a proton and an electron are placed in the Reason : The force on unit positive charge at the
same uniform electric field. They experience centre, due to the three equal charges are
different acceleration. represented by the three sides of a triangle
Reason : Electric force on a test charge is independent taken in the same order. Therefore, electric
of its mass. [AIIMS 1994] field intensity at centre is zero.
18. Assertion : On going away from a point charge or a 26. Assertion : The tyres of aircraft's are slightly
small electric dipole, electric field decreases conducting.
at the same rate in both the cases. Reason : If a conductor is connected to ground, the
Reason : Electric field is inversely proportional to extra charge induced on conductor will flow
square of distance from the charge or an to ground.
electric dipole. 27. Assertion : A bird perches on a high power line and
19. Assertion : The whole charge of a conductor cannot nothing happens to the bird.
be transferred to another isolated Reason : The level of bird is very high from the
conductor. ground.
Reason : The total transfer of charge from one to 28. Assertion : In the absence of an externally applied
another is not possible. electric field, the displacement per unit
20. Assertion : Conductors having equal positive charge volume of a polar dielectric material is
and volume, must also have same potential. always zero.
Reason : Potential depends only on charge and Reason : In polar dielectrics, each molecule has a
volume of conductor. permanent dipole moment but these are
21. Assertion : At a point in space, the electric field points randomly oriented in the absence of an
towards north. In the region, surrounding externally applied electric field. (1994)
this point the rate of change of potential 29. Assertion : Two adjacent spherical conductors,carrying
will be zero along the east and west. the same positive charge have a potential
Reason : Electric field due to a charge is the space difference between them.
around the charge. Reason : The potential to which a conductor is raised
22. Assertion : A point charge is brought in an electric field. depends upon the charge. (1999)
The field at a nearby point will increase, 30. Assertion : Electrons move away from a region of lower
whatever be the nature of the charge. potential to a region of higher potential.
Reason : The electric field is independent of the Reason : Since an electron has a negative charge.
nature of charge. (1999)
23. Assertion : The force with which one plate of a parallel 31. Assertion: Electric potential of earth is taken zero.
plate capacitor is attracted towards the Reason : No electric field exists on earth surface.
other plate is equal to square of surface (2009)
density per per unit area. 32. Assertion : An electric field is preferred in comparison
Reason : The electric field due to one charged plate to magnetic field for detecting the electron
of the capacitor at the location of the other beam in a television picture tube.
is equal to surface density per . Reason : Electric field requires low voltage.
24. Assertion : The lightening conductor at the top of high (2010)
building has sharp pointed ends. 33. Assertion : Lines of force are perpendicular to
Reason : The surface density of charge at sharp conductor surface.
points is very high resulting in setting up Reason : Generally electric field is perpendicular to
of electric wind. equipotential surface. (2016)
25. Assertion : Circuit containing capacitors should be
handled cautiously even when there is no
current.
Reason : The capacitors are very delicate and so
quickly break down.
Exercise – I
1. (A) 2. (D) 3. (B) 4. (A) 5. (B) 6. (D) 7. (D) 8. (D) 9. (B) 10. (C)
11. (C) 12. (B) 13. (C) 14. (A) 15. (B) 16. (A) 17. (C) 18. (C) 19. (D) 20. (A)
21. (B) 22. (A) 23. (A) 24. (B) 25. (D) 26. (A) 27. (B) 28. (A) 29. (A) 30. (B)
31. (D) 32. (B) 33. (C) 34. (B) 35. (B) 36. (C) 37. (C) 38. (B) 39. (C) 40. (C)
41. (B) 42. (B) 43. (C) 44. (C) 45. (C) 46. (A) 47. (C) 48. (D) 49. (D) 50. (A)
51. (B) 52. (A) 53. (C) 54. (A) 55. (A) 56. (D) 57. (C) 58. (B) 59. (A) 60. (C)
61. (D) 62. (C) 63. (D) 64. (D) 65. (A) 66. (B) 67. (B) 68. (B) 69. (D) 70. (A)
71. (C) 72. (D) 73. (C) 74. (D) 75. (A) 76. (B) 77. (B) 78. (A) 79. (D) 80. (B)
81. (B) 82. (A) 83. (C) 84. (B) 85. (A) 86. (A) 87. (C) 88. (C) 89. (A) 90. (A)
91. (C) 92. (A) 93. (B) 94. (C) 95. (B) 96. (C) 97. (A) 98. (A) 99. (A) 100. (D)
101. (B) 102. (B) 103. (B) 104. (C) 105. (A) 106. (B) 107. (D) 108. (C) 109. (D) 110. (A)
111. (D) 112. (B) 113. (B) 114. (B) 115. (B) 116. (D) 117. (A) 118. (B) 119. (D) 120. (C)
121. (B) 122. (C) 123. (B) 124. (B) 125. (A) 126. (C) 127. (D) 128. (C) 129. (A) 130. (A)
131. (D) 132. (D) 133. (C) 134. (C) 135. (B) 136. (B) 137. (A) 138. (D) 139. (D) 140. (C)
141. (B) 142. (C) 143. (A) 144. (A) 145. (B) 146. (D) 147. (B) 148. (D) 149. (D) 150. (B)
151. (C) 152. (D) 153. (D) 154. (A) 155. (C) 156. (B) 157. (C) 158. (A) 159. (D) 160. (B)
161. (C) 162. (D) 163. (A) 164. (D) 165. (A) 166. (C) 167. (C) 168. (C) 169. (A) 170. (C)
171. (A) 172. (B) 173. (D) 174. (A) 175. (B) 176. (D) 177. (B) 178. (C) 179. (C) 180. (C)
181. (C) 182. (A) 183. (B) 184. (C) 185. (D) 186. (C) 187. (A) 188. (A) 189. (C) 190. (B)
191. (D) 192. (B) 193. (A) 194. (D) 195. (D) 196. (D) 197. (C) 198. (A) 199. (A) 200. (B)
201. (C) 202. (B) 203. (D) 204. (B) 205. (C) 206. (A) 207. (A) 208. (B) 209. (C) 210. (C)
211. (D) 212. (A) 213. (D) 214. (D) 215. (D) 216. (C) 217. (C) 218. (D) 219. (D) 220. (C)
221. (A) 222. (A) 223. (D) 224. (A) 225. (C) 226. (B) 227. (C) 228. (B) 229. (B) 230. (B)
231. (C) 232. (D) 223. (A) 234. (B) 235. (B) 236. (A) 237. (B) 238. (B) 239. (D) 240. (A)
241. (B) 242. (C) 243. (C) 244. (B) 245. (B) 246. (B) 247. (C) 248. (C) 249. (C) 250. (C)
251. (A) 252. (C) 253. (D) 254. (A) 255. (A) 256. (C) 257. (B) 258. (C) 259. (C) 260. (A)
261. (D) 262. (B) 263. (B) 264. (D) 265. (C) 266. (D) 267. (B) 268. (B) 269. (D) 270. (C)
271. (A) 272. (D) 273. (D) 274. (D) 275. (C) 276. (A) 277. (D) 278. (B) 279. (C) 280. (B)
281. (C) 282. (C) 283. (B) 284. (C) 285. (D) 286. (B) 287. (C) 288. (C) 289. (D) 290. (D)
291. (A) 292. (A) 293. (A) 294. (B) 295. (C) 296. (B) 297. (D) 298. (A) 299. (D) 300. (D)
301. (B) 302. (B) 303. (B) 304. (B) 305. (B) 306. (B) 307. (D) 308. (A) 309. (D) 310. (D)
311. (C) 312. (A) 313. (B) 314. (B) 315. (D) 316. (D) 317. (C) 318. (D) 319. (B) 320. (B)
321. (B) 322. (A) 323. (B) 324. (C) 325. (D) 326. (B) 327. (D) 328. (A) 329. (C) 330. (C)
331. (A) 332. (B) 333. (A) 334. (C) 335. (C) 336. (D) 337. (C) 338. (B)
Exercise – II
(Previous Year AIPMT Examination Questions)
1. (D) 2. (C) 3. (A) 4. (C) 5. (A) 6. (C) 7. (A) 8. (D) 9. (C) 10. (B)
11. (C) 12. (B) 13. (B) 14. (C) 15. (B) 16. (D) 17. (D) 18. (C) 19. (C) 20. (D)
21. (B) 22. (A) 23. (B) 24. (B) 25. (C) 26. (C) 27. (A)
Exercise – III
(Previous Year AIIMS Questions)
1. (B) 2. (A) 3. (C) 4. (B) 5. (D) 6. (C) 7. (A) 8. (A) 9. (D) 10. (C)
11. (A) 12. (D) 13. (C) 14. (B) 15. (D) 16. (A) 17. (B) 18. (C) 19. (C) 20. (B)
21. (C) 22. (A) 23. (A) 24. (A) 25. (C) 26. (C) 27. (A) 28. (B) 29. (A) 30. (A)
Exercise – IV
1. (D) 2. (C) 3. (A) 4. (A) 5. (B) 6. (C) 7. (B) 8. (B) 9. (B) 10. (A)
11. (A) 12. (A) 13. (A) 14. (C) 15. (A) 16. (B) 17. (A) 18. (D) 19. (D) 20. (D)
21. (B) 22. (D) 23. (D) 24. (A) 25. (C) 26. (B) 27. (C) 28. (A) 29. (B) 30. (A)
31. (C) 32. (D) 33. (A)

Exercise - I
1. (A) Inside the hollow sphere, at any point the potential
1 10 10 6 10 10 6
is constant. USystem
2. (D) The force is perpendicular to th e 4 0 10 /100
displacement.
10 10 6 10 10 6
10 10 6 10 10 6
3. (B) Because current flows from higher potential to lower
potential. 10 /100 10 /100
4. (A) The electric potential V (x, y, z) = 4x2 volt
100 10 12 100
3 9 109 27J
Now E ˆi V ˆj V kˆ
V 10
x y z 11. (C) Potential at centre O of the square
V V V Q Q
Now 8x, 0 and 0
x y z
Hence E 8xiˆ , so at point (1m, 0, 2m) O
a
a 2
E 8iˆ volt/metre or 8 along negative X-axis.
5. (B) Since potential inside the hollow sphere is same as
that on the surface.
6. (D) On the equipotential surface, electric field is normal Q Q
to the charged surface (where potential exists) so
that no work will be done.
7. (D) May be at positive, zero or negative potential, it is Q
according to the way one defines the zero potential. VO 4
a
8. (D) Total charge Q = 80 + 40 = 120 C. By using the 4 0
2
r1
formula Q1' Q . New charge on sphere A · Work done in shifting (–Q) charge from centre to
r1 r2 infinity
rA 4 W = –Q(V – V0) = QV0
'
is QA Q
rA rB
120 48 C .
4 6
4 2 Q2 2Q 2
Initially it was 80 C i.e. 32 C charge flows from A to B.
4 0a 0a
V 10
9. (B) E 2
500 N / C 2QV
d 2 10 12. (B) Using v v Q
m
1 qq
10. (C) For pair of charge U . 1 2
4 0 r vA QA q 1
vB QB 4q 2
13. (C) Lines of force is perpendicular to the equipotential A
surface. Hence angle = 90o 100 C
14. (A) Potential at the centre of square
40cm 50cm

9 109 50 10 6
V 4 90 2 104 V
2
B C
2 +5 C 30cm
Work done in bringing a charge (q = 50 C) from
to centre (O) of the square is W = q(Vo – V ) = qV0 100 10 6 9
VB 9 109 106 V
0.4 4
W 50 10 6 90 2 104 64J
15. (B) In balance condition 100 10 6 9
and VC 9 109 106 V
V 4 3 0.5 5
QE = mg Q r g
d 3 6 9 9 9
So W 5 10 106 106 J
3 5 4 4
r3 Q1 r1 V2
Q kq 1
V Q2 r2 V1 20. (A) V i.e. V
R R
3 Potential on smaller sphere will be more.
Q r 600 Q 2e 106
2 Q2 21. (B) K qV 2e 106 J eV 2MeV
Q2 r 2400 2 e
2 22. (A) Since W = qV 20 = 5 × V V = 2 volts
QV 5 V dV d
16. (A) F QE 5000 V = 10 volt 23. (A) E (5x 2 10x 9) 10x 10
d 10 2 dx dx
17. (C) After redistribution, charges on them will be different, (E)x = 1 = –10 × 1 – 10 = –20 V/m
but they will acquire common potential 24. (B) For equilibrium mg = qE
Q1 Q2 Q1 r1 15 800
i.e. k k 1.96 10 9.8 q
r1 r2 Q2 r2 0.02
Q 1 Q1 r22 1 r2 1.96 10 15 9.8 0.02
As q
4 r2 2 Q2 r12 2 r1 800

1 1.96 10 15 9.8 0.02


n 1.6 10 19
r 800
i.e. surface charge density on smaller sphere will be n = 3.
more. 25. (D) Energy density
1 (q) ( 2q) 1 2
18. (C) Usystem 1 2 1 12 V
4 a 4 ue 0E 8.86 10
0 0 2 2 r
( 2q) (q) 1 (q) (q) = 2.83 J/m3
a 4 0 2a 26. (A) KE = q(V1 – V2) = 2 × 1.6 × 10–19 × (70 – 50) = 40eV
7 q2 27. (B) Potential inside the sphere will be same as that on its
Usystem surface i.e.
8 0a
19. (D) Work done in displacing charge of 5 C from B to C q q
V Vsurface stat volt , Vout stat volt
is 10 15
W 5 10 6 (VC VB ) where Vout 2 2
Vout V
V 3 3
kq k( q) Potential at the centre
28. (A) Potential at mid point O, V 0
d d V 9 109
+q O –q 6 6 6 6
10 10 5 10 3 10 8 10
1 1 1 1
d d
= 1.8 × 105 V
2d 6
4 10
QQ 37. (C) Potential at C 9 109 2
29. (A) By using U 9 109 1 2 0.2
r
= 36 × 104 V
10 6 10 6
9 3
U 9 10 9 10 J C
1
30. (B) In equilibrium QE = mg
V 4 3 0.2m 0.2m
Q. mg r g
d 3
19 12000 4 3
2 1.6 10 r 900 10
2 10 2 3 +4 C 0.2m +4 C
r = 1.7 × 10 m –6
38. (B) At centre E = 0, V 0
31. (D) Momentum p 2mK ; where K = kinetic energy
1 1
= Q.V 39. (C) W Uf Ui 9 109 Q1 Q2
r2 r1
pe m e Qe
p 2mQV p mQ W 9 10
9
12 10
6
8 10
6
p m Q
1 1
me 2 2
4 10 10 10
2m
= 12.96 J 13 J
32. (B) Kinetic energy K = Q.V K = (+e) (50000 V) 1 Qr 1
40. (C) E .
3
E
= 50000 eV = 50000 × 1.6 × 10–19 J = 8 10–15 J 4 R 0 R3
33. (C) KE = qV = eV = e × 1 = 1eV 41. (B) Potential at A = Potential due to (+q) charge
V + Potential due to (– q) charge
34. (B) Force on electron F = QE Q
d 1 q 1 ( q)
. 0
1000 4 a0 b 0 2 2 4 a2 b2
19 14
F (1.6 10 ) 8 10 N
2 10 3 42. (B) Net electrostatic energy
35. (B) Spheres have same potential kQq kq 2 kQq
U 0
Q Q Q1 R1 a a a 2
i.e. k 1 k 2
R1 R2 Q2 R2
kq Q 2q
Q q 0 Q
36. (C) Length of each side of square is 2 m so distance a 2 2 2
2 43. (C) Point P will lie near the charge which is smaller in
of it’s centre from each corner is 1m. magnitude i.e. – 6 C. Hence potential at P
2
P –6 C 12 C

x 20cm
O
2m 1 ( 6 10 6 ) 1 (12 10 6 )
V 0
1m 4 0 x 4 0 (0.2 x )
x = 0.2 m
1 2eV Q ( 1.6 10 19 )
44. (C) Kinetic energy K mv 2 eV v 53. (C) V 9 109 9 109 = 27.2V
2 m r 0.53 10 10
1 V 1
45. (C) Potential V V' 8V 54. (A) By using m(v12 v22 ) QV
r 2 2
46. (A) Work done W = 3 × 10–6 (VA – VB); where 1
10 3 {v12 (0.2)2 } 10 8 (600 0)
2
v1 = 22.8 cm/s
( 5 10 6 ) 2 10 6
1
VA 1010 106 volt 55. (A) Potential energy of the system
2 2 15
15 10 5 10
Qq kq 2 kqQ
and U k 0
l l l
kq q
6 6 (Q q Q) 0 Q
(2 10 ) 5 10 13 l 2
VB 1010 106 volt
15 10 2
5 10 2 15 dV dV
56. (D) E x ( 5) 5; E y 3
dx dy
1 13
W 3 10 6 106 106 = 2.8 J dV
15 15 and E z 15
dz
47. (C) In side a conducting body, poten tial is same
everywhere and equals to the potential of its surface E net E 2x E 2y E 2z (5)2 ( 3) 2 ( 15) 2
48. (D) If charge acquired by the smaller sphere is Q then 7
kQ 2
it’s potential 120 ... (i) 57. (C) By using W = Q. V 0.1volt V
2 20
Also potential of the outer sphere 58. (B) By using, KE = QV 4 × 1020 × 1.6 × 10–19
kQ = 0.25 × V V = 256 volt
V ...(ii) 59. (A) By using KE = QV KE = 1.6 × 10–19 × 100
6
From equation (i) and (ii) V = 40 volt = 1.6 × 10–17 J
49. (D) According to figure, potential at A and C are equal. 1 Q1Q 2
60. (C) U . ; net potential energy
Hence work done in moving – q charge from A to C is 4 0 r
zero.
q2 1
A U net 3 .
4 0 l
–q
61. (D) Length of the diagonal of a cube having each side b
l is 3 b. So distance of centre of cube from each
l
3b
vertex is .
B C 2
l Hence potential energy of the given system of charge is
+Q
50. (A) KE = qV
51. (B) Given electric potential of spheres are same i.e. 1 ( q) (q) 4q 2
U 8 .
VA = VB 4 3b 03 0b
Q Q Q1 a 2
1 1
. 1 . 2 ...(i) 62. (C) KE = QV KE = (2e) 200 V = 400 eV
4 0 a 4 0 b Q2 b 63. (D) Electric potential at P
Q
As surface charge density
4 r2 q R
P
Q1 b 2 a b2 b Q
1 R/2
2 Q2 a 2 b a 2 a
52. (A) Potential at any point inside the charged spherical
conductor equals to the potential at the surface of k.Q k.q
V
R R
Q
the conductor i.e. . 2
R
2Q q
4 0R 4 0R
64. (D) Conducting surface behaves as equipotential 81. (B) At a point inside the sphere, the potential is same
surface. everywhere and is equal to that of the surface.
65. (A) Electron is moving in opposite direction of field so W
field will produce an accelerating effect on electron. 82.(A) Work done W = Q(VB – VA) (VB – VA) =
Q
50 1.6 10 19 10 10 3
66. (B) V 9 109 8 106 V J / C 2 kV
9 10 15 5 10 6
67. (B) Energy = 0.5 × 2000 = 1000 J 1 Q 1
68. (B) E = 2e × 5V = 10eV Final kinetic energy 83. (C) V 4 .
r
V
0 r
= 10eV 85. (A) The work done in moving a charge on equipotential
69. (D) Energy = 1.6 × 10–19 × 100000 = 1.6 × 10–14 J surface is zero.
70. (A) Potential is to be determined at a distance of 4 cm 11
qE q V 10 50
from centre of sphere i.e. inside the sphere. 86.(A) a 108 m / sec 2
m m d 15 3
71. (C) Work done = ( V) Q 10 5 10
For an equipotential surface. 87. (C) Electric potential inside a conductor is constant and
it is equal to that on the surface of conductor.
10 40 10 20 88. (C) Potential inside the conducting sphere is same as
72. (D) Energy 250 erg
2 4
Q
73. (C) Firstly being a conductor it is attracted by the high that of surface i.e.
voltage plate, when charge is shared, ball is repelled 4 0R
until it goes to other plate and whole of the charge is 89.(A) At centre
transferred to the earth and the process is repeated. E=0
74. (D) Suppose charge on inner sphere is +Q as shown. and V = 0
Potential on inner sphere
A B
+Q –Q
–Q
4 E E
+Q 6cm
E O E

–Q +Q
D C
Q Q
V
4 6 19
1 (Ze) 47 1.6 10
90.(A) V . 9 109 14
4 0 r 3.4 10
1 1
3 Q Q = 36 e.s.u.
4 6 1.99 10 6 V
91. (C) Potential will be zero at two points
q1 = 2 C M q2 = –1 C N
76. (B) P.E. = Work done by external agent
= (Vfq – Viq) Vf > Vi P.E. > 0 i.e.P.E. will increase O
x=0 x=4 x=6 x = 12
77. (B) It is assumed that charge on earth is 106 C hence by l l
6
taking away a negative charge from the earth,
potential energy will increase. At internal point (M) :

3000 1 2 10 6 ( 1 10 6 )
78. (A) V Ed 10 2
10V 0 l=2
3 4 0 (6 l) l
So distance of M from origin; x = 6 – 2 = 4
79. (D) The work done is given by = q(V2 – V1) = 0
At exterior point (N) :
80. (B) Potential energy of the system will be given by
1 2 10 6 ( 1 10 6 )
0
( e) ( e) e2 4 0 (6 l ') l'
4 0r 4 0r l’ = 6
As r decreases, potential energy increases. So distance of N from origin, x = 6 + 6 = 12
92. (A) 100. (D) Potential at the centre of rings are
+q –q
Y

R R
Equipotential E
surfaces X O1 O2

Z
d
1 k.q k( q)
93. (B) By using mv2 QV VO1 ,
2 R R2 d2
1 6
2 10 (10)2 2 10 6
V V = 50 kV k( q) kq
2 VO2
R R2 d2
95. (B) In the direction of electric field potential decreases. 1 1
96. (C) A free positive charge move from higher (positive) VO1 VO2 2kq
R R2 d2
potential to lower (negative) potential. Hence, it must
cross S at some time.
q 1 1
97. (A) q q 2 R
0 R 2 d2
101. (B) K.E. = q0 (VA – VB) = 1.6 × 10–19 (70 – 50)
= 3.2 × 10–18 J
Q 102. (B) According to the figure, there is no other charge. A
single charge when moved in a space of no field,
q q does not experience any force. No work is done.
a WA = WB = WC = 0
From symmetry of the figure all corner have same 103. (B) Potential V any where inside the hollow sphere,
electric potential. Therefore work done in moving 1 Q
including the centre is V .
the charge q from the corner to the diagonally 4 0 r
opposite corner is zero. 104. (C) Volume of 8 small drops = Volume of big drop
98. (A) Since A and B are at equal potential so potential
difference between A and B is zero. Hence W = Q. V 4 3 4 3
8 r R R = 2r
=0 3 3
99. (A) Change in potential energy ( U) = Uf – Ui As capacity is r, hence capacity becomes 2 times.
q3 1 1
105. (A) U CV 2 50 10 6
(10) 2 2.5 10 3 J
2 2

Cmedium 110
40 cm 107. (D) Cmedium K Cair K 2.20
50 cm C air 50
q2 108. (C) Potential difference between the plates
q1 V = Vair + Vmedium
D
30 cm 10 cm
40 cm
1 q1q 3 q 2 q3 q1q3 q 2 q3
U A
4 0 0.4 0.1 0.4 0.5
1 q3
U [8q 2 q 3 ] (8q 2 )
4 0 4 0
k = 8q2 t
117.(A) High K means good insulating property and high x
(d t) t means able to withstand electric field gradient to a
0 K 0
higher value.
t 118. (B) Cmedium = K × Cair
V (d t )
0 K 119. (D) By using Q = nq Q = 64 q
Q t 0A
(d t ) 120. (C) Capacity of parallel plate capacitor C
A 0 K d
C A
Q Q 121. (B) After connection of wire, potential becomes equal
Hence capacitance C
V Q t
d t Q1 Q2 Q1 r1
A 0 K
r1 r2 Q2 r2 when r1 > r2, then Q1 > Q2
0A 0A
t 1 123. (B) Because metals are good conductor of electricity.
d t d t 1
k k
0 AK
K 0A 124. (B) C 4 0r
109. (D) C d
d r = Radius of sphere of equivalent capacity
110. (A) Stationary charge produces electric field only.
AK 100 10 4 6 15
r 4.77 m
4 d 3 3.14
1 10 4 3.14
0A 0A
112. (B) C .C ' C' 2C 4
d d ab 1 12 9 10
125. (A) C 4 0K .6
2 b a 9 109 3 10 2

2 = 24 × 10–11 = 240 pF
2
VBig 4
113. (B) By using Vbig = n vsmall (8) 3
3
vsmall 1 1
126. (C) C
2 2 d
114. (B) VBig n 3 vsmall (1000) 3 vsmall 100 vsmall C medium d 6 6
3
Cair t 4.5 2
E air E d t 6 4.5
115. (B) E medium K 9
K 2 127. (D) Since charge flows from high potential to lower
116. (D) Given : (b – a) = 1 × 10–3 m ... (i)
potential.
If positive charge is given, then V1 < V2 as r1 > r2
So positive charge flows from Q P
If negative charge is given, then V1 > V2
b a
So negative charge flows from P Q.
Since it is not given that whether the charge given is
positive or negative, hence the information is
ab incomplete.
6
and C 4 0 1 10
b a Q2
128. (C) W W’ = 4W
6 1 ab 2C
1 10 129. (A) Potential difference across the condenser
9 10 10 3 9

ab = 9 ... (ii)
V V1 V2 E1t1 E2 t 2 t1 t2
From equations (i) and (ii) K1 0 K2 0
9 1
b 1000 b2 – b – 9000 = 0 t1 t2 Q t1 t2
b 1000 V
0 K1 K2 A 0 K1 K2
1 ( 1)2 4(1000) ( 9000) 130. (A) For capacitor
b
2 1000
{Solving of quadratic equation} V1 d1 V1 d 2 60 12
V2 180 V
V2 d2 d1 4
6 6
1 36 10 36 10
b 3m.
2000 2000
131. (D) Area of the given metallic plate A = r2
q2
2 Hence C’ = 5C0. Energy stored W0
r A 2C0
Area of the dielectric plate A '
2 4
q2 q2 W0
Uncovered area of the metallic plates A” = A – A’ W' W'
2C ' 2 5C0 5
A 3A
A 137. (A) Force on one plate due to another is
4 4
q q2
C F = qE = q 2 0K
q
2AK 0 2AK 0
K
A/4
A
C (where 2 K is the electric field produced by one
0
plate at the location of other).
138.(D) Extra charge Q = (2CV – CV) = CV flows through
The given situation is equivalent to a parallel potential V of the battery.
combination of two capacitor. One capacitor (C’) is
Thus W = QV = CV2
A
filled with a dielectric medium (K = 6) having area 139. (D) If the drops are conducting, then
4
1
while the other capacitor (C’’) is air filled having area 4 3 4 3
R N r R N 3 r.
3A 3 3
4 Final charge Q = Nq
A 3A Q
K 0 0 So final potential V
4 4 R
Hence Ceq C ' C"
d d 2
Nq
V N3
0A K 3 0A 6 3 9 1
C
d 4 4 d 4 4 4 N3r
140. (C) Because the charges are produced due to induction
0A
C and moreover the net charge of the condenser should
d be zero.
132. (D) If nothing is said, it is considered that battery is
1 2 2
disconnected. Hence charge remain the same 141. (B) E E Final E Initial C(VFinal VInitial )
2
t
Also Vair d and Vmedium (d t ) 1
0 0 k 6 (202 102 ) 10 6
2
t 6 = 3 × (400 – 100) × 10–6 = 3 × 300 × 10–6 = 9 × 10–4 J
(d t ) (8 6 )
Vm k Vm 6 142. (C) Since aluminum is a metal, therefore field inside this
Va d 120 8 will be zero. Hence it would not affect the field in
Vm = 45V q q
between the two plates, so capacity
V Ed
0A 1 (0.12)2 remains unchanged.
133. (C) C .
t 4 9 109 1 3 2 2
d t 2 10
K 2 143. (A) VBig n 3 .vsmall VBig (27) 3 .vsmall 9 vsmall
2 144 10 10 144. (A) 4 0r = 1 × 10–6 r = 10–6 × 9 × 109 = 9 km
160pF
36 5 145. (B) After inserting the dielectric slab
134. (C) Electric field between the plates of a parallel plate K 0A
Q New capacitance C ' K.C
capacitor E i.e. E d° d
0 A 0 V
New potential difference V '
0A K 0A K
135. (B) C 1pF and C ' 2pF K = 4.
d 2d 0 AV
136. (B) When a dielectric K is introduced in a parallel plate New charge Q ' C'V '
d
condenser its capacity becomes K times.
V' V (2) 2 ]
New electric field E ' 0A 0[ 0 4
d Kd C2
d d d
Work done (W) = Final energy – Initial energy
4 0
2 Hence C1 = C2 40 d = 10–3 m
0
1 1 1 V 1 d
W C 'V ' 2 CV 2 (KC) CV 2
2 2 2 K 2 157. (C) Capacity when outer sphere is earthed
1 1 1 1 ab
CV 2 1 CV 2 1 C1 4 0
2 K 2 K b a
Capacity when inner sphere is earthed
2 2
0 AV 1 0 AV 1
1 so | W | 1 . 4 0 ab b2
2d K 2d K C2 4 0b 4 0
b a b a

V 100 Difference in capacity = C2 – C1 = 4 b


0
147. (B) E 10, 0000 V / m
d 10 3
159. (D) Electric field between the plates of parallel plate
148. (D) The electric field between the spheres of a charged
capacitor is non-uniform and it decreases with capacitor is uniform and it doesn’t depend upon
distance.
1
distance from the centre as E . K 0A C1 K1 C 5
r2 160. (B) C ;
d C2 K2 C2 20
0A A
149. (D) C 2 0 C2 = 4C
d d
d ab a = R – x, b = R so, R(R x)
2 161. (C) C ; C
b a x
150. (B) In charging of capacitor half of the supplied energy
162. (D) When there is no battery, charge remains same while
is stored in the capacitor. potential difference and electric field decreases
151. (C) In this process capacity increases, so battery V0 3 V0
supplies additional charge to capacitor. i.e. Q ' Q0 , V ' and
9 3
0A
E0 3 E0
0A E'
152. (D) By using Cair , C medium t 9 3
d d t 2
K 10 9
163. (A) V n3v V (64) 2 / 3 9 109
2
0A (2 10 )
For K = C medium = 7.2 × 10 V 3
d t
2 2
Cm d Cm 6 164. (D) V n3 v V 125 3 50 1250V
Ca d t Cm = 30 C
15 6 3 1 1
165. (A) Wext C 'V '2 CV 2
2 2
0 KA
153.(D) C 1 C 1 1
d (2V)2 CV 2 CV 2
2 2 2 2
C K,
1
Q = CV Q C ( V = constant) Wext 50 10 6 (100) 2 25 10 2
J
154. (A) Initially when key is closed, the capacitor acts as 2
short-circuit, so bulb will light up. But finally the 1 C C
166. (C) V | V ( V) |2 CV 2
capacitor becomes fully charged, so it will act as 2 (C C)
open circuit, so bulb will not glow.
167. (C) C 0A 0A 5000
A A 3 3 0A
t1 t2 6 10 4 10 7
155. (C) C1 0
d1 and
C2 K 0
d2 k1 k2 10 5
C1 1 Cd2 1 2d 168. (C) Initially charge on the capacitor Q = 10 × 12 = 120 C
C2 K d1 K=4
2C K d Finally charge on the capacitor Q’ = (5 × 10) × 12 =
156. (B) Capacity of spherical conductor of 20 cm diameter 600 C
C1 = 4 0r = 4 0 × 10 So charge supplied by the battery later = Q’ – Q =
Capacity of parallel plate air capacitor 480 C
170. (C) Heat produced = Energy of charged capacitor
t
1 d t
CV 2 C K
2 C' d
1
(2 10 6 ) (100) 2 0.01J 1 10 3
2 2 10 3 1 10 3
20 2
172. (B) Potential of both spheres will be same. C' 2 10 3
C medium C C’ = 26.6 F
173. (D) Cair
K 2 187. (B)
x
Cd 3 5 10 3 d–x
174. (A) We have C 0A A
d 12
0 8.85 10
= 1.7 × 109 m2
(0.08)2 0
175. (B) Q CV 100 1.8 10 8 C d
3
1 10
A 0 On placing or inserting thin metal sheet between the
176. (D) C 10 F
d plates of a parallel plate capacitor.
A A A 4 A 0 A 0 A 0
C1 0 0 0
. C 1C 2 0A
t d d d 1 3 d C1 , C2 , Ceq
d t d 1 x d x C1 C 2 d
k 2 2k 2 2
So, C remains unchanged.
4 Hence, C is independent of the position of the sheet.
C1 10 13.33 F
3 C1 K1 d2
0 KA
1 188. (A) C C2 K2 d1
177. (B) The energy stored QV d
2
0.4
0A K 0A
2 1 2
178. (C) C1 and C2 C2 = 11.2 F
d 2d
C2 2.8 (0.4)
C2 K 12
40 10 K
C1 2 12 K=8 1 C1C2 (V2 V1 )2
10 10 2 189. (C) U
2 (C1 C2 )
1 1
12
179. (C) C ' n 3C C' 23 C 2C < C’ > C (3 5) 10 (500 300) 2
6
1 1 (3 5) 10
180. (C) U CV 2 2 10 6
(200) 2 2
4 10 J
2 2 15 10 12
4 104
0.0375J
C 1 8 10 6
181. (C) C = 4 0R, R R ( ) 9 109 10 m9
4 0 9 190. (B) Charge on smaller sphere
r1 5
183. (B) C = 4 R = Total charge r r 30 10 C
0
1 2 5 10
C
R 9 109 10 12 9 10 3 m
4 0A 0A 2 0A
0 191. (D) C and C '
Diameter = 2R = 2 × 9 × 10–3 = 18 × 10–3 m d d d
d 2
1 C1 d2 15 2 d
184. (C) C C2 = 45 F 2
d C2 d1 C2 6

C’ = 2C
0A 0A 192. (B) By inserting the dielectric slab. Capacitance (i.e.
186. (C) C and C '
d t ability to hold the charge) increases. In the presence
d t
K of battery more charge is supplied from battery.
193. (A) Initial energy of body of capacitance 4 F is 1 1
1 208. (B) U Ceq V 2 (nC)V 2
Ui (4 10 6 ) (80)2 0.0128J 2 2
2 5
Final potential on this body after connection is
209. (C) U Big n 3 usmall
4 80 6 30
V 50V. 210. (C) After redistribution new charges on spheres are
4 6
So final energy on it 10 10
Q '1 10 C
10 20 3
1
Uf 4 10 6 (50)2 0.005 J 20 20
2
and Q '2 10 C
Energy lost by this body = Ui – Uf = 7.8 mJ 10 20 3
1 Q1' r2 2
195. (D) Capacitance of the given assembly Ratio of charge densities
2 Q'2 r12
R1R 2 R1R 2 10
C 4 C 2
0
(R 2 R1 ) 3 20 2 Q
R 2 R1
20 10 1
4 r2
3
1 C1 d2 1
C 8
197. (C) C so 2 C2 20 F q R2 q (n 3 r)2
d C2 d1 10 4 211. (D) small
Big Q r2 (nq) r2
1 1
198. (A) U CV 2 so 24 60 60 C(1200) 2 1 1
2 2 1
n3 (64) 3
C = 120 mF 4
199. (A) Energy density 1
212. (A) C 4 0 R 1 1.1 10 10 F
2 9
1 1 2
q2 9 10
2
0E 0 1 1
2 2 0 2 0 2 0A2 213. (D) U CV 2 2 10 6
(50) 2 25 10 4
J
2 2
Q2 (40 10 6 )2 16 10 10
200. (B) U 8 10 5 J = 25 × 103 erg
2C 2 10 6 10 2 10 5
1 1
= 8 × 10–5 × 107 = 800 erg 214. (D) U CV 2 5 10 6 (20 103 ) 2 1kJ
2 2
0A 1
215. (D) C As A times and d 2 times
CV 2 Q E 10 6 105 d 2
202. (B) F 0.05N
2d 2 2 1 1 12
So C times i.e. C C 3 F
203. (D) Work done W = Uf – Ui 4 4 4
1 1
216. (C) U CV 2 6 10 6 (100) 2 0.03J
1 1 (C) 1 2 2
Ui CV02 and U f .(3V0 )2 3 CV0 2
2 2 3 2 217. (C) Because there is no source of charge.

80AV02 0A
So W 218.(D) Cair , with dielectric slab
d d
204. (B) In the presence of battery potential difference 0A
C’=
t
V d t
remains constant. Also E , so E remains same. K
d
K 0A
205. (C) Capacitance with dielectric C medium Given C
4 0A 4 0A
C
d 3 t 3 d
d t
K K
Cmedium
d
d
1 4
3 CV0 2 K
4t 2
2
2 4t d d
207. (A) Thin metal plates doesn’t affect the capacitance. 4 d
2
1 1 1 Q2
219. (D) U CV 2 10 10 6
(500) 2 = 1.25 J 233. (A) Energy U for a charged capacitor charge Q
2 2 2 C
Farad m F is constant and with the increase in separation C will
220. (C) C 0A Cd
0 0 2 1
d A m m
decrease C , So overall U will increase.
d
Q2 (8 10 18 2
) 32
221. (A) W 32 10 J
2C 6
2 100 10
235. (B) In general electric field between the plates of a
2 2
222. (A) V n3v (64) 3 10 160 volt charged parallel plate capacitor is given by E
0K
2 2
2.5 236. (A) When a lamp is connected to D.C. line with a capacitor.
223. (D) V n3v 2.5 (125) 3 v V 0.1volt If will form an open circuit. Hence, the lamp will not
25
glow.
1 E
224. (A) Let E C0 V0 2 then E1 2E and E 2 237. (B) The increase in energy of the capacitor
2 2
1 1
U C(V22 V12 ) (6 10 6 ) (20 2 10 2 )
E1 4 2 2
So E 1
2 = 3 × 10–6 × 300 = 9 × 10–4 J
225. (C) Work done appears in the form of energy which is
1 1
238. (B) U CV 2 4 10 6
(400) 2 0.32J
q2 2 2
given by
2C 239. (D) In the given system, no current will flow through the
226. (B) Some energy lost in the form of heat in resistance branch CD so it can be removed
also. C
227. (C) Given V = 200 volt, Q = 0.1 C 5 F
10 10
QV 0.1 200
As energy U ,U = 10 Joule
2 2
A B
2 2
228. (B) V n3v (8) 3 v 4v i.e. 4 times. 10 10 5 F

Q2 D
229. (B) U ; in given case C increases so U will
2C Effective capacitance of the system = 5 + 5 = 10 F
decrease.
1 1 1 1 1
230. (B) Power 240. (A) Cs = 2 F
Cs 3 9 18 2
1
CV 2 Cs 2 1
2 1 40 10 6 (3000) 2 Cp = 3 + 9 + 18 = 30 F
90 kW Cp 30 15
t 2 2 10 3
1 8
C C C 1 241. (B) Total capacitance of given system Ceq F
231. (C) Using C n 3c c F 5
1 1 2 2
n3 (8) 3 1 1 8
U Ceq V 2 10 6
225 180 10 6 J
2 2 5
0A
232. (D) C ...(i) = 180 × 10–6 × 107 erg = 1800erg
d
1 1
0 KA 242. (C) U CV 2 2 (200) 2 10 6
0.04J
C' ...(ii) 2 2
2d
From equation (i) and (ii) 243. (C) Q1 = Q2 + Q3 because in series combination charge is
same on both the condenser and V = V1 + V2 because
C' K K in parallel combination V2 = V3.
2 K 4
C 2 2 Hence V = V1 + V2
244. (B) The given circuit can be drawn as Equivalent capacitance

12 F 1 3
Ceq 1 F
2 2

q2 1
5 F 254. (A) Energy (U) . q remains same so U
2C C

U Before C1 C2
U After C1
20 F

where C = (3 + 2) F = 5 F 3 3 3
255. (A) CAB 3 4 F CAC 3 F
3 2 2
1 1 1 1 20 1
CAB : CAC = 4 : 3
C PQ 5 20 12 60 3
1 1
CPQ = 3 F 256. (C) Initial energy Ui C1V12 C2 V22 ,
2 2
245. (B) In series combination Q is constant, hence according
to 1
Final energy U f (C1 C2 )V 2
2
Q2 1 U1 C 2 0.6 2
U U C1V1 C 2 V2
2C C U 2 C1 0.3 1 (where V )
246. (B) Potential difference across 4 F capacitor C1C2

6 Hence energy loss


V 500 300 volt
4 6 C1C 2
U Ui Uf (V1 V2 ) 2
2(C1 C 2 )
C1C 2
247. (C) Charge flowing V . So potential difference 257. (B) The two capacitors are in parallel so
C1 C 2
0A
C1C 2 V 1 C2 V C (k1 k 2 )
across C1 t 2
C1 C2 C1 C1 C2
1 1 1 1 2
248. (C) In parallel, C = C1 + C2 + C3 = 20 F 258. (C) C F
C 2 2 2 3
1 1 1 1 259. (C) Charges developed are same so C1V1 = C2V2
249. (C) C C1 C2 C3
R
V1
1 1 1 1 2
CR (C1 C2 C3 ) V2

C C V1 + V2 = 120 V1 = 80 volts
250. (C) C1 = 2C and C2 , so 1 4 :1 260. (A) Given circuit can be drawn as
2 C2
251. (A) In parallel combination V1 = V2 8 F
q1 q2 q1 C1
or
C1 C2 q2 C2
8 F
253. (D) The circuit can be drawn as follows
A B
8 F
1 F
1 F 1 F 2
8 F
A B

Equivalent capacitance = 4 × 8 = 32 F
1 F
261. (D) The given circuit can be redrawn as follows 266. (D) 4 F
2 F
10 F
12 F

12 F 5 F 8 F A B
A B 2 F

Q Q
9 F

4 12
CAB 3 F
60V 4 12
267. (B) Equivalent capacitance of the circuit Ceq = 6 F
A B
Charge supplied from source Q = 6 × 20 =120 C
Equivalent capacitance of the circuit CAB = 4 F
Charge given by the battery 12 F
Q = Ceq V = 4 × 60 = 240 C 4 F
Charge in 5 F capacitor
5 12 F
Q' 240 50 C
(10 5 9)
Q 8 F
262. (B) The given circuit can be redrawn as follows
Q
C
20V
C
B Hence charge on the plates of 4 F capacitor
A
C 4
Q' 120 40 C
(4 8)
268. (B) Charge flows to second capacitor until the potential
CAB = 3C
263. (B) The given arrangement is equivalent to the parallel V CV
is same i.e. . So new charge =
combination of three identical capacitors. Hence 2 2
0A
269. (D) Given circuit can be drawn as follows. It is a balance
equivalent capacitance 3C 3 whetstone bridge type network, hence 24 F
d
capacitor can be neglected
1 1 1 1 120
264. (D) Total capacitance C F 4 F
C 20 8 12 31 6 F 12 F
120
Total charge Q CV 300 1161 C
31 A B
1161
Charge, through 4 F condenser 580 C
2 9 F 18 F 6 F

580
and potential difference across it 145V Equivalent capacitance between A and B = 4 + 6
4
= 10 F.
1
265. (C) U CV 2 C1V1 C 2 V2
2 270. (C) By using, common potential V
C1 C2
Now if V is constant, then U is greatest when 'Ceq' is
maximum. This is when all the three are in 2 200 C 2 0
parallel. 20 C2 = 18 F
2 C2
271. (A) The given circuit can be redrawn as follows
1 F
3 F 3 F 3 F 3 3 3
3 F
A A
2 F 3 F 2 1
2 F 2

B B
3 F 3 F 3 F 3 3

On further solving the network in similar manner equivalent capacitance obtained between A and B will be 1 F.
272. (D) 12 F and 6 F are in series and again are in parallel with 4 F.
12 6
Therefore, resultant of these three will be 4 4 4 8 F
12 6
This equivalent system is in series with 1 F.
8 1 8
Its equivalent capacitance F ...(i)
8 1 9
4 8 32 8
Equivalent of 8 F, 2 F and 2 F F ...(ii)
4 8 12 3
(i) and (ii) are in parallel and are in series with C

32
C
8 8 32 9 32
and Ceq 1 C F
9 3 9 32 23
C
9
273. (D) The two capacitors formed by the slabs may assumed to be in series combination.
274. (D) The given circuit can be simplified as follows
8F 20 F
3
3

12 8 4 12 8/3
A 4 16 B A 4 16 B

Hence equivalent capacitance between A and B


1 1 1 1 240
C AB F
C AB 12 20 16 71
3
275. (C) Let q1, q2 be the charges on two condensers
q1 q2 q1 6 3
V
6 14 q2 14 7
14 600
Also q1 + q2 = 600 q1 q1 600 q1 6
6 20
q1 600
V 30 volt
6 20
0.2 600
276. (A) By using charge conservation 0.2 × 600 = (0.2 + 1)V V 100 V
1.2
277. (D) The given circuit can be redrawn as follows potential
6 20 3 0 120
difference across 4.5 F capacitor 282. (C) Common potential V Volt
(6 3) 9
9
V 12 = 8 V
9 So, charge on 3 F capacitor
9
2
6 120
Q2 3 10 40 C
9 9
4.5 F F
2 9 F 283. (B) The given circuit can be redrawn as follows
2 F

2 F 5 F 2 F 1 F 2 F 2 F
Q P Q
P

2 F 1 F
12V
278. (B) The possible arrangement may be
2 F CPQ = 1 F
284. (C) In steady state condition. No current flows through
2.5
line (1). Hence total current i 1A
(1 1 0.5)
A 2 F B
2 F 2 F
1 1
Line (2)
1 5 F
279. (C) Energy stored in the capacitor CV 2 100 2
2 Line (1)
1 6
10 10 (100 103 ) 2 100 5 106 J
2
Electric energy costs = 108 Paise per KWH 2.5 V
108 Paise
Potential difference across line (2)
3.6 106 J = potential difference across capacitor
Total cost of charging = 1 × 2 = 2 Volt
5 106 108 So, charge on capacitor = 5 × 2 = 10 C
150 Paise 286. (B) Initially potential difference a cross each capacitor
3.6 106
1 20 400
280. (B) Net capacitance 1 F V1 200 V
1 1 1 (10 20) 3
2 3 6
10 200
Total charge = CV = 1 F × 10 V = 10 C and V2 200 V
(10 20) 3
Total charge on every capacitor in series system is
same. So charge on 3 F is 10 C. C1V1 C 2 V2
281. (C) The given circuit can be simplified as follows Finally common potential V C1 C2
2 F
2/3 F
400 200
1 F 1 F 2 F 1 F 10 20
3 3 800
V V
(10 20) 9
X Y X Y
1 F 287. (C) Charge on C1 = charge on C2
2 F 2 F C1(VA – VD) = C2(VD – VB)
C1V1 C 2 V2
C1(V1 – VD) = C2(VD – V2) VD
2 8 C1 C2
Cxy 2 F
3 3
2 2 296. (B)
288. (C) C 2 3 F C1
2 2
C3
2 F 2 F A B

2 F C2

(C1 C 2 ) C3 (5 10) 4 60
C 3.2 F
(C1 C 2 ) C3 5 10 4 19
2 F 1 F 2 F
297. (D) A B
C1V1 C 2 V2 10 50 C2 0
289. (D) V 20
C1 C2 10 C2
1 1 1 1 1 2 1 4
200 + 20C2 = 500 C2 = 15 F 2 CAB = 0.5 F
C 2 1 2 2 2
290. (D) The given figure is equivalent to a balanced
Wheatstone's bridge, hence Ceq = 6 F
3 F 3 F 3 F
C1C 2 298. (A) A B
291. (A) CP = 4Cs (C1 C 2 ) 4
(C1 C 2 )
(C1 – C2)2 = 0 C1 = C2 1 1 1 1
292. (A) In steady state potential difference, across capacitor 1 CAB = 1 F
C AB 3 3 3
= 2V.
So charge on capacitor Q = 10 × 2 = 20 C 299. (D) C1 + C2 + C3 =12 ...(i)
C1C2C3 = 48 ...(ii)
293. (A) 2C P P C1 + C2 = 6 ...(iii)
2C
From equation (i) and (iii)
2C 2C
2C 2C C C3 = 6 ...(iv)
C From equation (ii) and (iv) C1C2 = 8
C C 2C Q
Q Also (C1 – C2)2 = (C1 + C2)2 – 4C1C2
C + C = 2C 2C/2 = C (C1 – C2)2 = (6)2 – 4 × 8 = 4
C 1 – C2 = 2 ...(v)
P On solving (iii) and (v) C1 = 4, C2 = 2
2C P 2C
300. (D)
2C
2C CPQ = 3C
3 F 3 F
CC 2C
Q Q
3 F 3 F 3 F 6 F
C + C = 2C A B A B

294. (B) There are two capacitors parallel to each other. 3 F

2 0A 3 F
Total capacitance
d
CAB = 5 F
1 2 0A 2 301. (B) The given circuit can be redrawn as shown below
Energy stored V
2 d
2 F 2 F 1 F

8.86 10 12 50 10 4 122
2.1 10 9 J 1 F
3 10 3 1 F
A B A B
V1C1 V2 C2 500 20 200 10
295. (C) V 400 V CAB = 2 F
C1 C2 20 10
302. (B) In series combination charge Q is same. So charge
C 6 10 12
on 2 F capacitor is 307. (D) C 2 10 12 F
n 3
2 8
Q C eq V 300 10 6 = 4.8 × 10–4 C 308. (A)
2 8
303. (B) In series V’ = nV = 10 V 4 F 4 F 2 F

304. (B) In steady state potential difference across capacitor


4 F 4 F
V2 = potential difference across resistance
R2 4 F 4 F 2 F
R2 V
R1 R 2
A B A B
Hence V2 depends upon R2 and R1
CAB = 8 F
R2
309. (D) In series combination of capacitor charge on each
R1 capacitor is same Q1 = Q2 = Q = Ceq V
C R3
V1 10 20 200
Ceq V 30 30 200 C
V2 10 20 30
A
K1 0 K1 0 A
310. (D) C1 2
+ – d d
V
2
A
K1 0 K1 0 A
2 A
305. (B) C1 K2 0
d d 2 K 2 0A
C2
2 d d
A 2
K2 0 K2 0A
C2 2 K3 0A K3 0 A
d d and C3
2d 2d
2
C1C 2
K3 0A 2K3 0 A Now, Ceq C3
and C3 C1 C 2
d d
2 K3 K1K 2 A
. 0
2 K1 K 2 d
1 1 1
Ceq C1 C2 C3 1 1 1 1
311. (C) C Ceq 2 F
1 1 eq 3 10 15
0A 0 Charge on each capacitor
(K1 K 2 ) 2K 3
d d Q = Ceq × V 2 × 100 = 200 C
1 d 1 1 A A A
0 K 0 0
C eq 0 A K1 K 2 2K 3 4 2 4
312. (A) C1 , C2 , C3
1 d d d
1 1 A
Ceq . 0 A/4 A/2 A/4
K1 K 2 2K3 d
1
1 1 K
So K eq d
K1 K 2 2K 3
1 2 3
306. (B) In series combination of capacitors, voltage
distributes on them, in the reverse ratio of their
V 3
K 1 0A
capacitance i.e. A ...(i) Ceq C1 C2 C3
2 d
VB 2
Also, VA + VB = 10 ...(ii) 4 1
10 25 F
On solving (i) and (ii) VA = 6V, VB = 4V 2
313. (B) 320. (B) Given circuit is a balanced Whetstone bridge.
C1C 2 2 6 321. (B) In steady state charge on C1 is
314. (B) Ceq C3 4 5.5 F
C1 C2 2 6 C1 Q
Q1 Q
Energy supplied (E) = QV = CV = 22 × 10 J 2 –6
C1 C2 3
P.E. stored and charge on C2 is
1 1
(U) Ceq V 2 5.5 (2)2 11 10 6
J C2 2
2 2 Q2 .Q Q
C1 C2 3
Energy lost = E – U = 11 × 10–6 J
C1C 2 1 1 1 1
U (V1 V2 )2 322. (A) Ceq 1 F
315. (D) 2(C1 C2 ) Ceq 2 3 6
Total charge Q = Ceq.V = 1 × 24 = 24 C
20 30
(5 0)2 150 J So p.d. across 6 F capacitor
2(20 30)
24
K1 0 A K2 0A = 4 volt
316. (D) C A , CB 6
d d
2 2 C1V1 C2 V2 6 12 3 12
323. (B) V 4 volt
C1 C 2 3 6
CA CB
324. (C) Initial energy of the system
1 1
A K1 K2 K2K3 Ui CV12 CV2 2
2 2
When the capacitors are joined, common potential
CV1 CV2 V1 V2
d/2 d/2 V
2C 2
C1 2K1K 2 Final energy of the system
Ceq
C2 K1 K 2 2
1 1 V V2
C A CB 2K1K 2 Uf (2C)V 2 2C 1
0A 2 2 2
CA CB K1 K 2 d
1
C(V1 V2 )2
0A 4
C
d 1
Decrease in energy = Ui Uf C(V1 V2 )2
317. (C) All capacitors are in parallel 4
So, Ceq = 1 F + 1 F + 1 F = 3 F C1V1 C 2 V2 10 250 5 100
325. (D) V 200 volt
318. (D) C1 C2 10 5
C C
1 1 1 2
326. (B) Ceq F
Ceq 1 2 3
C C C C/3
327. (D) Potential difference across both the lines is same i.e.
A B
2 V. Hence charge flowing in line 2
A B
C 4C 2
Ceq C Q 2 2 C
3 3 2
4 F 2 F 2 F
Line (2)
319. (B) 2 F 3 F 3 F 2 F 1 F
4 F
Line (1)
A B
6/5 F 8 F 6/5 F
A B 2V
So charge on each capacitor in line (2) is 2 C
1 5 1 5 20 3 20 24
Ceq F
Ceq 6 8 6 24 43
C 332. (B) The given circuit can be drawn as follows
328. (A) In series C i.e. C = nC’ = 2 × 3 = 6 F
n 2 F
C 12
In parallel C’ = nC i.e. C 6 F
4 F 4 F
n 2
329. (C) The given circuit can be simplified as follows
4 F A B
4 F 4 F
A

4 F 2 F 2 F
CAB = 2 + 4 = 6 F
333. (A) Cmax = nC = 3 × 3 = 9 F,
C 3
B C min 1 F
4 F n 3
334. (C) Common potential
2 F
4 F C1V C 2 0 C1
V .V
A C1 C 2 C1 C 2
4 F 1 1 1 1 8
335. (C) Ceq F
Ceq 1 2 8 13
4 F
8
B Total charge Q Ceq V 13 8 C
4 F 13
2 F
8
Equivalent capacitance between A an d B is Potential difference across 2 F capacitor 4V
2
CAB = 4 F
2 3 6
330. (C) The given circuit can be simplified as follows 336. (D) Equivalent capacitance F
2 3 5
C
6
C
B Total charge by Q = CV 1000 1200 C
A 5
C Potential (V) across 2 F is
C Q 1200
V 600 volt
C 2
Potential on internal plates = 1000 – 600 = 400 V
2C/3
337. (C) Given circuit can be reduced as follows
C 2C
A B 3
+1200 3 F
VB = 0
A VP B
C
In series combination charge on each capacitor
Equivalent capacitance between A an d B is remain same. So using Q = CV
5 C1V1 = C2V2 3(1200 – Vp) = 6(VP – VB)
C AB C
3 1200 – Vp = 2Vp ( VB = 0)
331. (A) The given circuit can be simplified as follows 3Vp = 1200 Vp = 400 volt
C/2 338. (B) Given circuit can be reduced as follows
C
A B
C/2 B A B
3C 3C
C C
C C (C = capacitance of each capacitor)
C/2 The capacitor 3C, 3C shown in figure can with stand
A B maximum 200 V.
C A
So maximum voltage that can be applied across A
C/2 and B equally shared. Hence maximum voltage
applied cross A and B be equally shared. Hence max.
Equivalent capacitance between A and B is
voltage applied across A and B will be
CAB = 2 C
(200 + 200) = 400 volt.
Exercise - II 1 qA qB qC
VC
(Previous Year AIPMT Examination Questions) 4 0 a b c
1.(D) : W U Q(VD VC ) here VC 0 a2 b 2 a
c 2 a
q q Qq 0 c 0
Q 0
4 0 (3L) 4 0 (L) 6 0L So, VC = VA VB
2.(C) : Work done in charging fully both the condensers C
6.(C) : In series, Ceq , Veq = 3 V.
1 1 C 2 3 3
CV 2 V CV 2
2 2 2 4 7.(A) : V = –x2y – xz3 + 4
Q Q
3.(A) : Since V and E ; E V = – ˆi ˆj kˆ (–x2 – xz3 + 4)
4 0r 4 0r 2 x y z

Given V Q 1011 Q 4 0r E= (2xy + z 3 )iˆ + x 2 ˆj + 3xz 2 kˆ .

1 1 C1 1 16C1
8.(D) : (4V) 2 (n 2 C 2 ) C2
r=
11 i.e., E = V 2 n1 2 n1n 2
4 0 10 r
QV 1
E= Q × 1011 4 1011 9.(C) : U CV 2
0 2
r
4 0 Q × 10 22 volt m –1 1 0A
V2
2 d
4.(C) : Energy stored in capacitor for field 2
1 V 1 2
1 = 0 AD = 0 E AD
E CV 2 2 d 2
2
kq kq kq kq
1 0A 2 10.(B) : VA
(E.d) L L 5L 5L
2 d
2kq 1
1 2
1 .
0 AE d
L 5
2
Energy stored in capacitor + Energy loss in the dU
11.(C) : For Stable equilibrium, F = – 0
process of charging = Energy given by cell. dr
we get r = 2A/B.
A 0
V2 12. (B): Potential V at centre of the square
d
–kQ kq k2Q k2q
= 2 × Energy stored in capacitor 0 ; So Q = –q .
r r r r
Since energy stored in capacitor = Energy loss in the
13.(B) : In the direction of electric field, electric potential
process of storing the charge in capacitor.
decreases.
5.(A) : q A = 4 a 2 , VB > VC > VA
q B = – 4 b2 , 1
14.(C) : Electric field, E
K
qC = 4 c2 , c = a + b As K1 < K2 so E1 > E2.
1 qA qB qC Q
VA 15.(B) : At centre, E = 0 & V = .
4 0 a b c 4 0R
1 qA qB qC Vˆ Vˆ Vˆ
VB 16.(D) : E i– j– k
4 0 a b c x y z
a2 = –[(6 – 8y)iˆ + (–8x – 8 + 6z)jˆ + (6y)k]
ˆ
b c
0 b
At (1, 1, 1), E 2iˆ + 10jˆ – 6kˆ
2
a
a E 22 102 62 140 2 35 .
0 b
17.(D) : Charge on plate will not change.
Q2 24.(B) :
18.(C) : F
2 0A

Q = CV and C = 0A
0A = Cd
d
C2 V 2 CV 2 Charge on capacitor q = CV
So F = .
2Cd 2d When it is connected with another uncharged
capacitor.
Vˆ Vˆ Vˆ
19.(C) : E i– j– k
x y z
E = –(6y)iˆ – (6x – 1 + 2z)jˆ – (2y)kˆ
at point (1, 1, 0)
E 6iˆ – 5jˆ – 2kˆ = – 6iˆ + 5jˆ + 2kˆ .
20.(D) : Initial energy stored in the 2mF capacitor is
1 q1 q 2 q 0 V
(2 )V 2 V 2 J Vc V
2 C1 C 2 C C; c 2
Energy loss Initial energy
C1C2 1
(V1 – V2 )2 (v – 0)2 Ui CV 2
2(C1 + C 2 ) 2
5 2 Final energy
E loss V J
4 2 2
1 V 1 V CV 2
5 2 Uf C C
V 2 2 2 2 4
% loss = 4 2 100 80% .
V Loss of energy = Ui – Uf
21.(B) : Capacitor does not consume energy effectively over
full cycles CV 2
4
i.e. decreases by a factor (B).
22.(A) :
+ –
25. (C)
C2 C3
K1 K2 K3 C1

C4/3 C4/3 C4/3


K4 K4 K4
Q
E= =
2 0 2 0 A;

1 3 3d 3d 3d 1 1 Q
F = QE
C1 C4 2k1 0 A 2k 4 0 A 2 0 A k1 k4 2 0 A
26. (C)
k 0A 2 0 A k1k 4 k 2k4 k 3k 4
Ceq
d 3d k 2 +k 4 k 2 +k 4 k3 +k 4 Q
27. v=
C
2 k1k 4 k2k 4 k 3k 4
k Q = CV
3 k1 +k 4 k 2 +k 4 k 3 +k 4 .
dv v
23.(B) : Work done w = q V i=c = 20 F× 3 = 60 A
V is same in all the cases so work is done will be dt s
same in all the cases. Also, conduction current in wires is equal to
displacement current between the plates of capacitor.
Exercise - III 9.(D): The potential energy is given by
(Previous Year AIIMS Questions) 1 q1q2
UP
1.(B) : If two charges are of the same sign, they repel each 4 0 r
other. Thus if two electrons are brought closer, some
1 10 6 1 10 6
work is done against the force of repulsion. Therefore 9 109 9 10 3 J
the electrostatic potential energy of the system 1
increases. 10.(C): The electric potential at any point (x, y, z)in space is
given as V = 6z2
Aliter: (–e) r (–e) Electric field E V
Mathematically, the potential due to one electron at
V ˆ V ˆ V ˆ
Ke E i+ j+ k 12zkˆ
the position of the other electron is and hence x y z
r
the electrostatic potential energy of the system E (2, –1, 3) = –12 × 3 = –36
11.(A): The given capacitor can be viewed as a parallel
Ke( e) Ke2 combination of two capacitors C1 and C2
, where r is the distance between
r r Where
the two electrostatic. Now if r decreases, P. E.
increases. A A
0 K1 0 K2
2.(A) : Since the charge is moving in a circle or along an C1 2 ; C2 2
equipotential surface, therefore workdone will be d d
zero. where d is the separation between the plates,
3.(C) : Let the area be A The effective capacitance C = C1 + C2
The separation between the plates d = 5 mm = 0.005 m A A
0 K1 0 K2
2 2 0 A K1 K 2
0A Cd
Capacitance C A d d d 2
d 0 12.(D) : Since the potential at each point of an equipotential
A = 3 × 0.005 × 9 × 109 × 4 surface is the same, the potential does not change
A = 1.69 × 109 m2 while we move a unit positivecharge from one point
4.(B) : Earth is a good conductor. It can be given almost an to another. Therefore work done in the process is zero.
unlimited charge. Its potential is zero. 13.(C) : b
5.(D) : Induced charge on aluminium and copper will the C1 C5
equal.
6.(C) : Let m capacitors are joined in series and n such groups a d
A C2 B
are joined in parallel.
8 8 C4 C3
So, C = and Cequi n 16 or,,
m m c
n = 2m. Capacitance of capacitors C1, C3, C4,C5 = 4 F each
Potential of arrangement, mV = 1000 and capacitance of capacitor C2 = 10 F. lf a battery
1000 is applied across A and B, the points b and c will be
or, m 4. n = 2 × 4 = 8. at the same potential (since C1 = C4 = C3 = C5 = 4
250
So, total number of capacitors required F).Therefore no charge flows through C2.
= nm = 8 × 4 = 32. We have the capacitors C1 and C5 in series. Therefore
their equivalent capacitance,
7.(A) : In the case of a hollow metal sphere, the electric field
inside the shall is zero. This means that the potential C1 C5 4 4
C' 2 F.
inside the shell is constant. Therefore the potential C1 C5 4 4
at centre of the sphere is the same as that on its Similarly, C4 and C3 are in series. Therefore, their
surface i.e. 10V. equivalent capacitance,
8.(A) : 5 capacitors in parallel gives
C3 C4 4 4
5 × 2 F = 10 F. C" 2 F.
C3 C4 4 4
Then 2 capacitors in series give 11 F.
When they are connected in series, total capacitance Now C’ and C” are in parallel.
10 1 10 Therefore,effective capacitance between A and B =
will be F. C’ + C” = 2 + 2 = 4 F.
10 1 11
charge 1
14. (B) : Surface density of charge 20. (B) : Work done charge × potential difference
area of surface 2
1
C1V1 C2V2 W 4 (4 106 0) 8 10 6 J.
1 , 2 2
2
4 r1 4 r22 Power of the lightening strike, P
W
The spheres are in contact. They therefore acquire t
the same potential V 8 10 6
80 MW.
2 2 100 10 3
1 C1 r22 C1 r2 r1 r2
. 21.(C) : Between the points P and Q, a capacitor C1and series
2 r12 C2 C2 r1 r2 r1 combination of C2, C3, C4,C5 are connected in parallel.
r2 20 2 P
or 1 .
r1 10 1 C1 C5
2
15.(D) : According to Gauss’s law, the total electric flux
through a closed surface is given by Q T

Q
E.dS
C2 C4
S 0
where Q is the total charge enclosed by the surface. R S
The total charge enclosed by the given surface is C3
Q = + q – q = 0. The total flux = 0. 1 1 1 1 1
16.(A) : Any surface which has same electrostatic potential C s C2 C3 C4 C5
at every point is called an equipotentialsurface. 1 1 1 1 4 C
Electric field is always perpendicular to an or Cs .
C C C C C 4
equipotential surface. Therefore x-direction is The equivalent capacitance between P and Q is
perpendicular only to yz plane. C 5C
C ' C1 Cs C .
1 4 4
17.(B) : The energy stored in capacitor CV 2 Between the points P and R a series combination of
2
C1, C2 and a series combination of C3, C4,C5 are
1
40 10 6 (3000)2 connected in parallel.
2 1 1 1 1 1 2 C
= 20 × 9 × 106 × 10–6 = 180 J. C'
C s C1 C2 C C C or, s 2
'

The power delivered in 2 ms is


1 1 1 1 1 1 1 3
180 "
C s C3 C4 C5 C C C C
90 103 W 90 kW.
2 10 3 C
18.(C) : Electric field between the plates is Cs" .
3
2 The equivalent capacitance between P and R is
1 –
C C 5C
C " Cs' Cs"
E1 2 3 6
5C
E2 C" 6 4 2
.
C' 5C 6 3
4
22.(A): The capacitances of two are
volt/meter.
0 C1 = 4 0R and C2 = 4 0 (2R).
19.(C) : |QB| > |QA| (given). Inside the shell B, the potential is Q2
Initial energy Ei
a constant and negative. 2C1
But VA inside rA is positive and greater than that of B Q2
because rA < rB . Upto rA, one as hence positive Final energy E f
2C2
potential. Beyond rA, VA 1 , this is > V , one
B
Heat produced = Ei – Ef
r Q2 1 1 1 Q2 1
1
1 2 4 0R 2 4 0R 4 0 2R 2
gets the course . At B, the negative potential is
r 1 Q2
more therefore the potential line is still negative. (C) .
explains all these factors. 4 0 4R
23.(A) : Network is redrawn as shown in figure. r1 2iˆ 2 ˆj
b 2 2

C | r1 | r1 2 2 2
C
r2 2iˆ 0 ˆj
a C F e
10
CC or | r2 | r2 2
Potential at point A is
C
1q 1 10 3 10 6
d VA
4 0 r1 4 0 2
Potential at point B is
V 1 q 1 10 3
10 6

This is a balanced Wheatstone’s network. VB


4 0 r2 4 0 2
Equivalent capacitance Ceq= C VA – VB = 0.
Charge on capacitor between the terminals a and b 26.(C) :
Q CV –9 –9
3 × 10 C a b 3 × 10 C
2 2
Energy stored in that capacitor 1 cm 1 cm 1 cm
a e According to conservation of energy, we get
C b C Ka + Ua = Kb + Ub.
Here, Ka = 0 and the potential energies are Ua = q’Va
d and Ub = q’Vb.
C C
2 q '(Va Vb )
V or v
m
2
Q Va = (9.0 × 109 Nm2 C–2)
1
2 Q 2 C 2V 2 CV 2
3 10 9 C 3 10 9 C
2C 8C 8C 8 1350 V
Given: C = 1 F, V= 10 V 0.01 m 0.02 m
Vb = (9.0 × 109 Nm2 C–2)
(1 10 6 ) 102 100
10 6 12.5 J .
8 8 3 10 9 C 3 10 9 C
1350 V .
24.(A) : Capacitors C1 and C2 are in parallel, their equivalent 0.02 m 0.01 m
capacitance is in series with capacitor C3. Hence,
the equivalent capacitance of the given circuit is 2(2 10 9 C)(2700 V) 2
v 4.65 10 m s 1.
(C1 C2 )C3 5 10 3 kg
Ceq
C1 C2 C3 = 4.65 cm s–1.
27.(A) : Energy stored in capacitor of capacitance C with
Charge on capacitor C3 is
Q = V Ceq 1
voltage Vis, U CV 2
V (C1 C2 )C3 2
C1 C2 C3 A
but C 0
, V Ed
Voltage across C3 is d
Q V (C1 C2 ) where A is the area of capacitor plate and d is the
C3 C1 C2 C3 . distance of separation and E is the electric field.
25.(C) : Y 1 0A 2 2 1
U E d 0 E 2 Ad
A 2 d 2
0A
r1 2, 2 28.(B) : As 4 0R
d
2
6
X A 2 2 cm
d 2.25 10
(0, 0) (2 0) 4 R 4 100
r2
= 2.25 × 10–4 m
6. (C) In the given case V = V0 (constant)
F 3000 N 1
29.(A) : As E 1000 NC 1
q 3C Energy stored in the capacitor CV 2
2
dr = 1 cm = 10–2 m
C KC, so energy stored will become A times
dV = E (dr) = 1000 × 10–2 = 10V
Q = CV, so Q will become K times
30.(A) : E(4 C) D(4 C) Kq
Surface charge density K 0. '
A
7. (B) Electron and proton have same amount of charge so
O they have same coulomb force. They have different
(4 C)F C(4 C) accelerations because they have different masses
F
(a ) Therefore, both assertion and reason are
m
true and reason is the correct explanation of the
A(4 C) B(4 C) assertion.
9. (B) Charge is always conserved but energy is lost in the
As shown in the figure, O is the center of hexagon term of heat.
ABCDEF of each side 8 cm. As it is a regular hexagon 10. (A) Potential is constant on the surface of a sphere so it
OAB, OBC, etc are equilateral triangles. behaves as an equipotential surface.
OA = OB = OC = OE = OF = 8cm 11. (A) Capacitance is basically a geometrical quantity.
= 8 × 10–2 m
14. (C) Q ne and charge lesser than 1 C is possible.
The potential at O is
15. (A) This is the concept of electric image.
q If we are asked to find the force between an infinite
V 6
4 0 r earthed conductor and a point charge q placed at
perpendicular distance l from the earthed conductor
6 90 109 4 10 6
(see figure), than we proceed as follows.
2.7 10 6 V
8 10 2 Firstly, the conductor being earthed implies V = 0.

Exercise - IV
1. (D) Gravitational force is the dominating force in nature
l
and not coulomb’s force. Gravitational force is the
weakest force. Also, Coulomb’s force >> gravitational a – a
force.
Electrical
2. (C) Equivalent capacitance of parallel combination is
image of
Cp = C1 + C2 + C3.
3. (A) In a hollow spherical shield, the charge is present So, we redraw the situation in which we replace the
only on its surface but charge is zero at every point conductor and introduce an IMAGE charge –q as
inside the hollow sphere. Hence, the metallic shield shown.
in form of hollow shell may be built to block an electric The force between the two charges (object charge q
field. and image charge –q) is the electrostatic force
4. (A) Electron has negative charge, in electric field between the infinite grounded conductor and q.
negative charge moves from lower potential to higher 1 q2 1 q2
potential. So, F F
4 0 (2l ) 2 4 0 4l 2
5. (B) By the formula capacitance of a capacitor
(attractive in nature)
KA K
C1 0 16. (B) As 1 = 2 (Given)
d d
d q1 q2 q r12
C1 K1 d 2 K1 1 , or 1
Hence, 2 4 r12 4 r22
q2 r22
C2 d1 K 2 K 2 3K 6 [Let r1 and r2 be two different radii]
or C2 = 6C1
Then the ratio of electric field intensities near the
Q
Again, for capacity of a capacitor C surface of spherical conductor,
V
2
Therefore, capacity of a capacitor does not depend E1 q1 4 r
0 2 q1 r22
2 1 i.e. E1 = E2
upon the nature of the material of the capacitor. E2 4 0 1 r q2 q2 r12
17. (A) Resultant of electric intensity at O due to B and C is 26. (B) During take off and landing, the friction between
equal and opposite to that due to A. tyres and the run way may cause electrification of
A tyres. Due to conducting nature of tyre, the charge
so collected is conducted to a ground and electrical
sparking is avoided.
27. (C) When the bird perches on a single high power line,
O no current passes through its body because its body
is at equipotential surface i.e., there is no potential
B C difference. While when man touches the same line,
standing bare foot on ground the electrical circuit is
18. (D) The rate of decrease of electric field is different in completed through the ground. The hands of man
the two cases. In case of a point charge, it decreases are at high potential and his feet’s are at low
1 potential. Hence large amount of current flows
as 2 but in the case of electric dipole it decreases through the body of the man and person therefore
r
gets a fatal shock.
1
more rapidly, as E .
r3 28. (A) D 0 E P where
19. (D) The whole charge of a conductor can be transferred
E = applied electric field and
to another isolated conductor, if it is placed inside
the hollow insulated conductor and connected with P = polarisation or dipole moment per unit volume
it. np 2
20. (D) Electric potential of a charged conductor depends
not only on the amount of charge and volume but [ p is the component of dipole moment of each
also on the shape of the conductor. Hence if their molecule in direction E and n is the no. ofmolecules
shapes are different, they may have different electric per unit volume]
potential.
21. (B) Since the electric field is directed from south to north Now in the absence of an electric field ( E = 0) the
hence rate of change of potential will be along this polarisation vector P vanishes (because P is there
direction, but it is zero along east and west.
if E is there).
22. (D) Electric field at the nearby point will be resultant of
existing field and field due to the charge brought. It Therefore displacement D = 0.
may increase or decrease if the charge is positive or 29. (B) Let us consider two spherical conductors of radii r1
negative depending on the position of the point with and r2 which are possessing the same positive charge
respect to the charge brought. Q. Therefore the potential on the surface of each
23. (D) The electric field due to one charged plate at the conductor will be
location of the other is E and the force per KQ KQ
2 V1 , V2
0 r1 r2
2
unit area is F E . Since r1 r2 , V1 V2 and consequently there will
2 0 be a potential difference between the two conductors.
24. (A) A charged cloud induces opposite charge on pointed 30. (A) Suppose A and B are two regions having potentials
conductors. At sharp points of the conductor surface V1 and V2such that V1 > V2.So the electric field will be
density of charge is very high and charge begins to set up from A to B(i.e.from higher potential to lower
leak from the pointed ends by setting up oppositely potential). Now since the electron has got a negative
charged electric wind. This wind, when comes in charge, the force experienced by the electron will be
contact with the charged cloud, neutralizes some of in a direction opposite to the direction of the electric
its charge lowering the potential difference between field. Hence the electrons move from a region of
the cloud and the building. This reduces the chances lower potential to a region of higher potential due
of lightening striking the building [if the lightening totheir negative charges.
strikes the building, the charge is conducted to the 31. (C) Electric potential of earth is taken as zero,because
earth and the building remains safe]. the capacitance of earth is taken infinite.
25. (C) A charged capacitor, after removing the battery, does 32. (D) If electric field is used for detecting the electron beam,
not discharge itself. If this capacitor is touched by then very high voltage will have to be applied or
someone, he may feel shock due to large charge still very long tube will have to betaken.
present on the capacitor. Hence it should be handled
cautiously otherwise this may cause a severe shock.
(5) If n electrons pass through a point in a conductor in time
The time rate of flow of charge through any cross-section is q ne
t then current through the conductor is
called current. If Dq charge passes through a cross-section in t t
Number of electrons flowing through conductor in t
q t
time Dt then average current av and i n st a nt a n eous second is n
t e
q dq 1
current Lim For 1 ampere of current n
t 0 t dt 1.6 10 19
= 6.25 × 10 electrons/second
18

Important Points (6) The conductor remains uncharged when current flows
(1) Current is a fundamental quantity with dimension through it because the charge entering at one end per
M°L°T°A¹ second is equal to charge leaving the other end per
(2) Current is a scalar quantity with its SI unit ampere second.
Ampere : The current through a conductor is said to be one (7) For a given conductor current does not change with
ampere if one coulomb of charge is flowing per second through change in its cross-section because current is simply
a cross-section of wire. rate of flow of charge.
CGS unit of current is biot (Bi) (8) Current density : This is defined as current flowing per
1 ampere unit area held normal to direction of flow of current.

1 coulomb (1 / 10) emu of ch arg e 1 nq


current density J and current J . da
= Biot A At
sec ond sec ond 10
(3) The conventional direction of current is the direction of I1
flow of positive charge or applied field. It is opposite to I2
direction of flow of negatively charged electrons.
I E I1 = I2
+ +
+ + The direction of current density J is same as that of

electric field E . J is a vector with unit amp/m² and


dimension M° L–2 T° A¹.
I E (9) If n particles each having a charge q pass per second per
- - unit area then current associated with cross-sectional
- - q
area A is nqA .
t
(10) If there are n particles per unit volume each having a
charge q and moving with velocity v then current through
(4) The current may be constituted by motion of different
type of charge carries in different situations. q
cross-sectional area A is nqvA
t
S .No. Nature of material Caus e of current (11) If a charge q is moving in a circle of radius r with speed
1 Conductors M otion of electrons v then its time period is T = 2pr/v. The equivalent current
2 Vacu um tubes M otion of electrons q qv
3 Semiconductors M otion of holes and .
electrons T 2 r
4 Electrolytes M otion of pos itive and
negative ion s (a) Heating Effects : The phenomenon of heating of a
5 Dis charge tube M otion of pos itive and conductor by the flow of an electric current through it
negative ion s is called heating effect of electric current.
(b) Magnetic Effects : The phenomenon of production of a
magnetic field in space around a current carrying (1) Conductors are substances through which electric
conductor is called magnetic effect of electric current. charges can flow easily.
(c) Chemical Effects : The phenomenon of decompostion (2) They are characterised by presence of a large number of
of an electrolyte into ions on passing an electric current free electrons (1029 electrons per m³).
through it is called chemical effect of electric current. (3) The number density of free electrons in a conductor is
same throughout the conductor. This is because free
Heat
electrons experience repulsive force between them and
Light
conductor allows movement of free electrons. Thus, free
electrons are evenly scattered throughout the volume of
conductor.
(4) These free electrons transport electric charge so are called
as conduction electrons.
[a] Behaviour of conductor in absence of applied potential
difference :
Current-carrying coil
Filament bulb (1) The free electrons present in a conductor gain energy
Magnetic effect
Heating effect from temperature of surrounding and move randomly in
a conductor.

A C
(2) The speed gained by virtue of temperature is called as
thermal speed of an electron.

1 3 3kT
Electrolyte mv 2rms kT so thermal speed vrms
2 2 m
Voltameter where m is mass of electron
Chemical effect At room temperature T = 300 K vrms = 105 m/sec.
(3) The average distance travelled by a free electron between
(a) Direct Current : The current whose magnitude and two consecutive collisions is called as mean free path
direction does not vary with time is called direct current l. (l ~ 10 A°)
(dc). The various sources are cells, battery, dc dynamo
total distance travelled
etc. mean free path
number of collisions

–
I = const e

time
(b) Alternating Current : The current whose magnitude Random motion of electrons
continuously changes with time and periodically changes (4) The time taken by an electron between two successive
its direction is called alternating current. It has constant collisions is called as relaxation time t. (t ~ 10–14 s)
amplitude and has alternate positive and negative halves.
It is produced by ac dynamo. total time taken
relaxation time
number of collisions

+ time (5) The thermal speed can be written as vT


O T T
2 – (6) In absence of applied potential difference electrons have
random motion.The average displacement and vera ge
I = I 0sin t velocity is zero. There is no flow of current due to thermal
motion of free electrons in a conductor.
[b] Behaviour of conductor in presence of applied potential (7) The drift velocity depends on nature of metal through
difference : t, applied potential difference, length of conductor.
(1) When two ends of a conductors are joined to a battery eE eV
then one end is at higher potential and another at lower vd
m mL
potential. This produces an electric field inside the
conductor from point of higher to lower potential i.e. vd is independent of radius or area of cross-section of
a conductor.
V
E . (8) The rise of temperature causes increase in vrms and hence
L a decrease in l and relaxation time t causing a decrease
L in drift velocity.
HP - - - - LP
- - - - (9) Mobility of a charge carrier is defined as drift velocity
E acquired per unit electric field.
vd e e m
V Mobility
E m m ne 2 ne
(2) The field exerts an electric force on free electrons causing
The unit is m V s and dimensions are M–1 T2 A1
2 –1 –1
acceleration of each electron.
The mobility depends on applied potential difference,
length of conductor, number density of charge carriers,
eE
F ma e E so acceleration a current in conductor, area of cross-section of conductor.
m
Ex. Though the drift velocity for electrons is small, an
(3) The average velocity with which the free electrons are
electric bulb lights up immediately as we turn the switch
drifted towards the positive end of a conductor under
on. Why?
the influence of an external electric field is called drift
velocity vd.
Sol. When switch is made on the electric field E responsible
for setting up 8current propagates through wires at speed
Using we have v d
eE of light 3 × 10 m/s. So field is set up immediately in time
v u a t L/c causing electrons to drift and hence bulb lights up
m
(vd ~ 10–4 m/sec) immediately.
(4) The direction of drift velocity for electrons in a metal is
opposite to that of applied field E. Ohm’s Law : If the physical state i.e. temperature, nature of
(5) Relation between current and drift velocity material and dimensions of a conductor remain unchanged then
Let n be number density of free electrons and A be area the ratio of potential difference applied across its ends to current
of cross-section of conductor. flowing through it remains constant.
Number of free electrons in conductor of length L = nAL V
V I or V = I R where R is resistance of conductor..
Total charge on these free electrons Dq = neAL
Time taken by drifting electrons to cross conductor Dt = L/vd
eE ne 2 ne 2 V
q v I = n e A vd = n e A AE A
neAL d neAv d or m m m L
Current
t L
I = neAvd. V m L
The current flowing through a conductor is directly So R 2 R is resistance of conductor
ne A
proportional to the drift velocity (I vd)
(6) The current density
(1) The property of a substance due to which it opposes the
eE ne² flow of current through it is called resistance.
J nevd ne E so J E
A m m (2) It is a scalar quantity with unit volt/ampere called
ohm (W)
or J = s E
W M1L2 T 2
V
ne 2 Dimensions of R M1L2 T 3A 2
where is specific conductivity of conductor q ATA
m (3) The reciprocal of resistance is called conductance
which depends on temperature and nature of material. 1
G .
R
J E is microscopic form of ohm’s law.
The SI unit is ohm –1 or mho or siemen (s) and its 1
dimensions are M–1 L–2 T3 A2. (10) For maximum metals per °C
273
(4) The substances which obey ohm’s law are called ohmic
or linear conductor. The resistance of such conductors 273 T
is independent of magnitude and polarity of applied so R R0 1 R0 R0
potential difference. Here the graph between I and V is 273 273 273
a straight line passing through the origin. The reciprocal So R T i.e. The resistance of pure metallic conductor
of slope of straight line gives resistance
is proportional to its absolute temperature.
V 1
R = constant. 1 L L
tan (11) R L and R so R or R where r is
A A A
e.g. silver, copper, mercury, carbon, mica etc. called resistivity or specific resistance.
(12) The fractional change in resistance without change in
I
volume or mass are :
(a) When change in length is small (£ 5%) fractional change
R 2 L
O in R is
V R L
(5) The substances which do not obey ohm’s law are called (b) When change in radius is small (£ 5%) fractional change
nonohmic or non linear conductors. The I–V curve is
not a straight line. R 4 r
in R is
i.e. p n diode, transistors, thermionic valves, rectifiers R r
etc.
(c) When change in area is small (£ 5%) fractional change
I R 2 A
in R is
R A

O V
(a) In terms of microscopic quantities E = rJ so resistivity
(6) V = IR defines resistance and hence it is applicable to is numerically equal to ratio of magnitude of electric field
all ohmic and nonohmic conductors. If R is constant or to current density.
V I then it represents ohm’s law. RA
(b) so if L = 1 m, A = 1 m² then r = R. Specific
V E L
(7) The relation R is macroscopic form while
J resistivity is numerically equal to resistance of substance
is microscopic form of ohms law. having unit area of cross-section and unit length.
(8) The resistance of a conductor depends on temperature, (c) It is a scalar with unit ohm-meter (W-m) and dimensions
nature of material, length and area of cross-section. M1 L3 T–3 A–2.
(9) The temperature dependence of resistance is given by (d) The reciprocal of resistivity is called conductivity or
R = R0 (1 + a Dq), where a is temperature coefficient of specific conductance with units mho/m and dimensions
resistance and Dq is change in temperature. The variation
is graphically represented as. 1 ne 2
M–1 L–3 T3 A2. ne
The temperature coefficient of resistance m
R R0 (e) The resistivity is independent of shape and size of body
is defined as change in resistance per and depends on nature of material of body. The resistivity
R0
is the property of material while resistance is property
unit resistance at 0°C per degree rise of temperature. of object.
(f) The temperature dependence of resistivity is given by
relation r = r0 (1 + Dq) where is temperature coefficient
of resistivity and Dq is change is temperature. For metals
R0 is positive so resistivity increases with temperature
while for non-metals is negative so resistivity decreases
Temperature with temperature.
When radius is increased n times new resistance
Metal semiconductor m R´ r4 R
R´ so or R´
d 2 (nr) 4 R (nr ) 4
n4

Temperature Temperature L L A V m
(c) R 2
A A A A dA 2
superconductor
When area is increased n times new resistance

m R´ A2 R
R´ so or R´
2 2
d(nA) R (nA) n2
Tc
Temperature
A thermistor is a heat sensitive device made of semiconducting
material where resistivity changes rapidly with change of
(1) The heating element of devices like heater, geyser, press temperature. It has a large temperature coefficient. The
etc are made of nichrome because it has high resistivity temperature coefficient may be positive or negative. These can
and high melting point. It does not react with air and be used over wide range of temperature.
acquires steady state when red hot at 800°C.
(2) Fuse wire is made of tin lead alloy because it has low
(1) The thermistors are used to detect small temperature
melting point and low resistivity. The fuse is used in
changes because they respond rapidly to temperature
series, and melts to produce open circuit when current
changes.
exceeds the safety limit.
(2) The thermistors with a large negative temperature
(3) Resistances of resistance box are made of manganin or
coefficient is used in resistance thermometer in very low
constantan because they have moderate resistivity and
temperature measurement.
very small temperature coefficient of resistance. The
(3) These are used in temperature control units of industry.
resistivity is nearly independent of temperature.
(4) Thermistors are used in protection of windings of
(4) The filament of bulb is made up of tungsten because it
generators, motors and transformers.
has low resistivity, high melting point of 3300 K and
gives light at 2400 K. The bulb is filled with inert gas (5) They are used for voltage stabilisation, remote sensing
because at high temperature it reacts with air forming and safe guarding filament of picture tube of T.V.
oxide.
(5) The connection wires are made of copper because it has
low resistance and resistivity.
Ex. Discuss the effect on the resistance R of wire when mass Strip C
Colour Strip A Strip B Strip D
is kept constant and (a) length is increased n times (b) 10
Black 0 0 Tolerance
radius is increased n times (c) cross-sectional area is 10¹
Brown 1 1
increased n times.
10²
Re d 2 2
L L xL L2 L2 d 10³
Orange 3 3
Sol. (a) R where d is
A A xL V m Yellow 4 4 10 4
density, m is mass and V is volume of wire. Green 5 5 10 5
When length is increased n times the new resistance Blue 6 6 10 6
(nL) 2 d Violet 7 7 10 7

m Grey 8 8 10 8
R´ (nL) 2 White 9 9 10 9
n 2 so R´ = n² R Gold 5%
R L2 10 1
Silver 10%
L L A V m 10 2
(b) R . No Colour 20%
A A A A2 d 2r4
(3) Potential difference across each resistance is different
and is directly proportional to its resistance V R. So
V1 = I R1 ; V2 = I R2 and V3 = I R3
A B C D (4) The series combination obeys law of conservation of
The first two rings give first two significant figures of resistance. energy
The third ring indicates the decimal multiplier i.e. the number So V = V1 + V2 + V3 = I (R1 + R2 + R3) equivalent resistance
of zeroes that will follow the two significant figures. The fourth V
ring shows tolerance or percentage accuracy. Rs R1 R 2 R3
Aid to memory BBROY Great Britain Very Good Wife. (5) The equivalent resistance is equal to sum of individual
Ex. Draw a colour code for 42 k W ± 10% carbon resistance. resistances.
Sol. According to colour code colour for digit 4 is yellow, for (6) The equivalent resistance is greater than largest of
digit 2 it is red, for 3 colour is orange and 10% tolerance individual resistance.
is represented by silver colour. (7) The resistances are connected in series (a) to increase
So colour code should be yellow, red, orange and silver. the resistance and (b) to divide large potential difference
Ex. What is resistance of following resistor. across many resistances.
Violet Gold (8) In n identical resistances R are connected in series then
the equivalent resistance Rs = nR
(9) This combination is used in resistance boxes and
sometimes in decorative bulbs.
Yellow Brown (10) In resistances connected in series if one resistance get
open the current in whole circuit will become zero.
Sol. Number for yellow is 4, Number of violet is 7
(b) Parallel Combination :
Brown colour gives multiplier 101, Gold gives a tolerance
of ± 5% R1
I1
So resistance of resistor is 47 × 101 W ± 5% or 470 ± 5% W. R2
I2
Second Colour Code :
I3 R3
End Body
I

V
+ –
(1) Resistances are said to be connected in paralled between
Dot Ring two points if it is possible to proceed from one point to
(1) The colour of body gives first significant figure. another along different paths.
(2) The colour of end gives second significant figure. (2) Resistances are said to be in parallel if potential across
(3) The colour of dot gives the decimal multiplier. each resistance is same and equal to applied potential.
(4) The colour of ring gives the tolerance. (3) Current through each resistance is different and is
inversely proportional to resistance of resistor.
V V V
I 1/R. So 1 ,
R 2 and
2 3
(a) Series Combination : R1 R3
I R1 I R2 I R3 (4) The parallel combination obeys the conservation of
charge.
V1 V2 V3
1 1 1
V So I = I1 + I2 + I3 = V
– R1 R2 R3
+
(1) Resistances are said to be connected in series between Reciprocal of equivalent resistance
two points if they provide only a single path between
two points. 1 1 1 1
(2) Resistances are connected in series if same current flows Rp V R1 R2 R3
through each resistance when some potential difference
is applied across the combination. (5) The reciprocal of equivalent resistance is equal to sum
of reciprocal of individual resistances.
(6) The equivalent resistance is smaller than smallest of (1) Distance between electrodes (r d) larger is the
individual resistance. separation between electrodes more is the length of
(7) The resistances are connected in parallel to decrease electrolyte through which ions have to move so more is
resistance. internal resistance.
(8) If n identical resistances R are connected in parallel then (2) Conductivity or nature of electrolyte (r 1/ )
equivalent resistance Rp = R/n (3) Concentration of electrolyte (r c)
(9) This combination is used in household electrical (4) Temperature of electrolyte (r 1/T)
applicances. (5) Nature and area of electrodes dipped in electrolyte (r
(10) In resistances connected in parallel if one resistance 1/A)
becomes open then also all others will work as usual.

R2 The potential difference between the two electrodes of a cell


(11) In case of two resistances in parallel 1 and in a closed circuit i.e. when current is being drawn from the cell
2 R1 is called terminal potential difference.
I 1 + I2 = I (a) When cell is discharging :
When cell is discharging current inside the cell is from
R2 R1
So 1 and 2 cathode to anode.
R1 R 2 R1 R 2
E
sources of emf and kirchhoff’s laws Current or E = IR + Ir = V + Ir or V = E – Ir
r R
E r
An electrochemical cell is a device which by converting chemical
energy into electrical energy maintains the flow of charge in a
circuit. It usually consists of two electrode of different materials I I
and an electrolyte. The electrode at higher potential is called R
anode and the one at lower potential is cathode. When current is drawn from the cell potential difference
is less than emf of cell. Greater is the current drawn from
R the cell smaller is the terminal voltage. When a large
current is drawn from a cell its terminal voltage is reduced.
(b) When cell is charging :
E r

I I
ELECTROLYTE
+ –
V
When cells is charging current inside the cell is from
The emf of a cell is defined as work done by cell in moving a anode to cathode.
unit positive charge in the whole circuit including the cell once.
(1) emf E = W/q; SI unit is joule/coulomb or volt. V–E
Current or V = E + Ir
(2) emf is the maximum potential difference between the two r
electrodes of the cell when no current is drawn from the During charging terminal potential difference is greater
cell. than emf of cell.
(3) emf is the characteristic property of cell and depends on (c) When cell is in open circuit :
the nature of electrodes and electrolyte used in cell.
E
(4) emf is independent of quantity of electrolyte, size of In open circuit R = 0
electrodes and distance between the electrodes. R r
So V = E
The opposition offered by the electrolyte of the cell to In open circuit terminal potential difference is equal to
the flow of electric current through it is called the internal emf and is the maximum potential difference which a cell
resistance of the cell. The internal resistance of cell can provide.
depends on.
(d) When cell is short circuited :
2 E2R
E E P R
In short circuit R = 0 so and V = IR = 0 (r R )2
R r r
In short circuit current from cell is maximum and terminal dP
potential difference is zero. so P = Pmax if 0
dR
(e) Power transferred to load by cell :
P = Pmax if r=R
E2
Pmax Power transferred by cell to load is maximum when
4r
E2 E2
P r = R and Pmax =
4r 4R

r=R
R

DIFFERENCE BETWEEN EMF AND POTENTIAL DIFFERENCE

EMF OF A CELL POTENTIAL DIFFERENCE


1. The emf of a cell is the maximum potential The potential difference between the two points
difference between the two electrodes of a is the difference of potential between these two
cell when the cell is in the open circuit. points in a closed circuit.
2. Emf is independent of resistance of circuit This depends upon the resistance between two
and depends upon the nature of electrodes points of the circuit and current flowing through
and electrolyte. the circuit.
3. The term emf is used for source of current. Potential difference can be measured between
any two points of circuit.
4. This is a cause. It is an effect.

(1) The current inside a cell is due to motion of both positive E1 E2


and negative ions while outside it depends on nature of
circuit elements like conductors, semiconductor, gas or
electrolyte. E = E2 - E1
(2) The cell is a source of constant emf and not of constant
current because if resistance of circuit changes then E1 E2
current I = E/r + R also changes but emf remains constant.
(3) As I = E/r so more current can be drawn from a cell with
larger emf and smaller internal resistance. e.g. In lead
E = E1 - E2
acid accumulator E = 2.05 V and r min = 0.1 . (7) Capacity of a battery is equal to product of current in
(4) With use of cell its internal resistance increases ampere and time in hour for which a cell can operate. It
appreciably but emf fall slightly. The current delivering depends on the amount of electrolyte and so an size of
ability is reduced. cell.
(5) A cell neither creates nor destroys charge but maintains e.g. capacity 8Ah means we can draw 8A current for one
the flow of charge by providing required energy. hour or 2A current for 4 hours.
(6) The emf of cell is taken to be positive in a circuit if current
inside a cell is from negative to positive i.e. during
discharging otherwise negative.
The cells are said to be connected in series if negative terminal
E1 E2
first is connected to positive of second whose negative terminal
is connected to positive of third cell. The external resistance is
E = E1 + E2 connected between free terminals of first and last cells.
Let m identical cells each of emf E and internal resistance r be
E r E r r connected in parallel. The combination can be replaced by a
E
I single cell of emf E and internal resistance r/m. The current
R
E
Let n idential cells each of emf E and internal resistance r be R r/m
connected in series. The combination can be replaced by a
r/m
single cell of emf nE and internal resistance nr. The current
nE
flowing through load
R nr I
R
nE nr
E
I (1) If r/m << R then . If equivalent internal resistance
R
R r/m is less than external resistance R then current in
circuit is equal to current produced by a single cell.
nE
(1) If nr << R then . If equivalent internal resistance mE
R (2) If r/m >> R then . If equivalent internal resistance
nr is less than external resistance R then current in circuit r
is equal to n times circuit current due to single cell.
(2) If nr >> R then I = E/r. If equivalent internal resistance r
nr is greater than external resistance R then current in is greater than external resistance then current in
m
circuit is equal to short circuited current obtained from
circuit is equal to m times the current produced by a
one cell.
short circuited cell.
(3) Maximum current can be drawn from series combination
(3) Thus maximum current can be drawn from parallel
of cells if external resistance is very large as compared
combination of cells if external resistance is small as
to equivalent internal resistance.
compared to net internal resistance of cells.
(4) If in series combination of n cells p cells are reversed
than equivalent emf Let there be n identical cells in one row and m rows of cell
Eeq = (n – p) E – pE = (n – 2p)E and r eq = nr in parallel.
(n 2p)E E E E
So current r r r
nr R
(5) If unidentical cells are connected in series then
Eeq = E1 + E2 + ..... = Ei and req = r1 + r2 + ..... = ri.
Ei
So current . I
R ri R

nE nr/m
The cells are said to be connected in parallel if positive terminals
of all the cells are connected together at one point and their
negative terminals at another point. The external resistor is
connected between these two points. I
R
E r
The combination of cells can be replaced by a single cell of emf
E r nr
nE and internal resistance . The current
m
E r nE E
I nr R r
R
R m n m
where n × m = p = total number of cells.
R r E2R n2E2 m2 E 2 R
(1) The current in the circuit is maximum when is P and Pmax
n m R r
2 4R 4r 2
minimum
n m
d R r
So 0 or (3) In mixed combination of cells current in circuit and power
dm n m transferred to load become maximum under same
d mR r R r condition. This is why it is preferred over series and
0 or 0 parallel combination of cells.
dm p m p m2
R R r R r
or or
p mn m 2 n m The cells which cannot be recharged electrically are called
In mixed grouping of cells current in circuit is maximum primary cells. Here the original state of cell cannot be brought
R r nE mE back by passing electrical energy through cell from external
if and max . source after cell is discharged.
n m 2R 2r
(2) In mixed grouping of cells power transferred to the load e.g. Voltaic cell, Daniel cell, Leclanche cell, manganese-alkaline
is maximum where external resistance R is equal to total cell, mercury button cell etc.
nr R r
internal resistance or R or . The cells in which chemical process is reversible are called
m n m
This shows power transfer is maximum when current is secondary cells. Here original chemical state of cell can be
maximum. brought back by passing electrical energy through cell from
external source.
e.g. Lead acid accumulator, alkali cells etc.

Comparative Study of Various Cells :


S.No. Cell Type Anode Cathode Electrolyte Depolariser Emf
1. Voltaic P Copper Zinc Di H2SO4 - 1.08 V
2. Leclanche P Graphite Zinc NH4C Soln MnO2 1.45 V
3. Daniel P Copper Zinc Dil H2SO4 CuSO4 Soln. 1.12 V
4. Dry P Graphite Zinc Paste of MnO2 + 1.50 V
NH1 C + ZnC 2
Charcoal
powder
5. Manganese P Compressed Zinc KOH Soln - 1.50 V
alkaline MnO2
6. Mercury P Mercuric oxide Zinc KOH Soln - 1.3 V
button with graphite
7. Lead acid S Lead PbSO4 Dil H2SO4 - 2.05 V
accumulator
8. Ni-Fe cell S Nickel Iron KOH Soln - 1.35 V
or Edison cell
Polarisation : Polarisation is formation of hydrogen gas bubbles
Local Action : This is due to impurities of copper, iron carbon on the anode of cell. This causes an increase in internal
etc in commercial zinc. When zinc rod is dipped in electrolyte resistance because layer of hydrogen is a bad conductor. The
the impurities and zinc in contact form small local cells in which hydrogen ions moving towards anode are unable to reach anode
small currents are produced resulting in wastage of zinc even and transfer their charge. These positively charged ions set up
when cell is not being used. This defect is called local action.This a field from hydrogen to zinc resulting in back e.m.f. weakening
is overcome by amalgamating zinc rod by mercury. When cell the action of cell. This defect may be overcome by using a
is being used fresh zinc continues to come on surface so depolarizer i.e. oxidising agent like MnO2 or CuSO4 which
chemical reaction continues without local action. oxidises hydrogen to water.
The construction of some cells is shown below : In any electrical network, the algebraic sum of currents
A C meeting at a junction is always zero i.e. I = 0
+ I1 – I2 + I3 + I4 – I5 = 0 or I1 + I3 + I4 = I2 + I5
Cu Zn
(1) The sum of currents flowing towards a junction is equal
to sum of currents leaving the junction.
Electrolyte
Dil. H2 SO4 (2) By convention the current directed towards the junction
is positive while those directed away from the junction
Local is taken as negative.
Polarisation Zn action (3) The first law is in accordance with conservation of
Voltaic Cell charge.
Cu C (4) The charges do not accumulate at a junction. The total
crystals Zn Rod
charge entering a junction is equal to total charge leaving
A+ Cu Pot
the junction.
CuSO4. Soln. (b) Kirchhoff’s second law or voltage law or loop rule :
(depolariser) The algebraic sum of all the potential drops and emf’s
Porous pot along any closed path in a network is zero.
Dil H2SO4 i.e. V = 0
(electrolyte)
(1) The second law is in accordance with conservation of
Deniel Cell
energy.
C A
Zn rod Graphite rod (2) According to second law the electric energy given to
+
the charge by a source of emf is lost in passing through
Porous pot resistance.
×× Glass pot
× (3) The change in potential in covering a resistance in the
×× MnO2 + charcoal direction of current is negative (–IR) while in opposite
×
×
dust (depolariser)
×× direction it is positive.
××
× NH2 Cl solution
(electrolyte) A R B A R B
Leclanche Cell
A
+ Graphite rod VA - R = VB VB + R = VA
C VB - VA = R VA - VB = R
Zn pot
×× ×× Porous pot The potential falls along direction of current. The
×× ××
×× ×× MnO2 + charcoal potential fall is taken as negative while potential rise is
×× ××
×× ×× dust (depolariser) taken as positive.
×× ××
×× ×× NH2Cl + ZnCl2 (4) The emf of source is taken as positive when it is traversed
Paste (electrolyte) from negative to positive terminal while it is taken as
Dry Cell negative when it is traversed from positive to negative
irrespective of the direction of current.
(a) Kirchhoff’s first law or current law or junction rule :
A B A B
2

VA - E = VB VB + E = VA
1 VB - VA = -E VA - VB = E
3
(5) If there are n loops their will be (n – 1) equations according
to loop rule.
(6) Th e algebraic sum of products of currents and
5 resistances in a closed loop is equal to sum of emf’s
4 applied in the circuit i.e. E = IR
R
Let two cells of emf E1 and E2 and internal resistances r1 and This is in series with R (AC) = R + R/3 = 4R/3
r2 be conneted in parallel to an external resistance R. 3
This 4R/3 is in parallel with R(AD) = 4R/7
1 E1 r1
d c
2 4R 8R
So R eq
7 7
2 E2 r2
a b
Wheatstone bridge :
B
R
m n
P g Q
Applying loop rule to loop d m n c d
–IR –I1 r1 + E1 = 0 ......(1) G
A C
Applying loop rule to loop a m n b a
Kg
–IR –I2 r2 + E2 = 0 ......(2)
R S
Applying first rule at junction I = I1 + I2.
Multiply equation 1 by r2 and eqn. 2 by r1 and put I2 D
= I – I1 we get E R KC
–IR r2 – I1 r1r2 + E1 r2 = 0 and –IR r1 – (I – I1) r2r1 + E2 r1 = 0
On adding these we get E1 r2 + E2 r1 = IR (r1 + r2) + Ir1 r2 (1) It is an arrangement of four resistances devised by
Charles Wheatstone which is used to measure an
r1r2
= I(r1 + r2) R unknown resistance.
r1 r2 (2) The wheatstone bridge principle states that if four
resistances P, Q, R and S are arranged to form a bridge
(E1r2 E 2 r1 ) / r1 r2 E eq with a cell and key between A and C, and a galvanometer
or
r1r2 R req between B and D then bridge is said to be balanced
R
r1 r2 when galvanometer shows a zero deflection.
P R
Eeq req In balanced condition Ig = 0 so VB = VD or .
Q S
This is called condition of balance.
equivalent cell (3) The condition of balance depends on resistance P, Q, R
and S. This is independent of emf of battery.
R (4) In state of balance the cell and galvanometer can be
Two dissimilar cells in parallel are equivalent to a single cell of interchanged.
(5) While performing an experiment at start press the cell
r1r2
internal resistance req and emf key Kc first and then the galvanometer key Kg and at end
r1 r2 remove Kg first and then Kc to avoid induced effects.
(6) P and Q are called ratio arms, R is known resistance arm
E1r2 E 2 r1 r1r2 E1 E2
E eq and S is the unknown arm. BD and AC are called
r1 r2 r1 r2 r1 r2 conjugate arms.
(7) The resistance of balanced wheatstone bridge between
If a circuit is symmetric about a line then all points lying on line (P Q )(R S)
of symmetry are at same potential. In this method we divide the A and C is .
P Q R S
circuit in two equal parts and double the resistance of one part.
e.g. in above question DF is line of symmetry so D and F are (8) The sensitivity of bridge depends upon value of
at same potential. resistances. The sensitivity is maximum when P, Q, R
and S are of same order.
R R R /2 R (9) Unbalanced wheatstone bridge
Now is in parallel to R (CD) =
2 R R /2 3 In unbalanced bridge Ig 0, VB VD s o
P R It is a device based on wheatstone bridge. It consists of one
. meter long straight and uniform wire AC. The wire is made of
Q S
manganin or constantan because they have low temperature
P R coefficient and large resistivity. The wires are soldered to two
If then VB > VD so current flows from B to D. thick L shaped copper strips with binding terminals. Between
Q S
these there is a third copper strip. There are two fixed gaps ab
P R
If then VB < VD so current flows from D to B. and cd.
Q S
P L
(10) Wheatstone bridge is not suitable for measurement of Resistance P L and Q 100 – L. So
very small and very high resistances. Very low resistances Q 100 L
are measured by Kelvin’s double bridge while very high P R RQ (100 L)
resistances by leakage method. Since so S R
Q S P L
(1) It is used to measure unknown resistances (1 to
S 1000 ), for comparison of two unknown resistances and
specific resistance of material of wire.
A P B Q C (2) For better accuracy R must be adjusted to obtain L
1000 100 10 10 100 1000 between 40 to 60 cm. It has maximum accuracy when
L 50 cm.
(3) It is more sensitive than PO box but suffers from end
resistances.
I R Ex. An unknown resistance S is placed on the left gap and
D known resistance of 60 in right gap of meter bridge.
The null point is obtained at 40 cm from left end of bridge.
KG KC
G Find the unknown resistance?
B' A'
Post-office Box Sol. (100 L) (100 60)
S R 60 40
(1) It is so named because it has a shape of box and was L 60
designed to find resistance of electric cables and Ex. In a meterbridge the length of wire is 100 cm. At what
telegraph wires. It was used in post offices to determine point is balance point obtained if two resistances are in
resistance of transmission lines. ratio 2 : 3?
(2) It is based on principle of wheatstone bridge. P L 2
Q Sol. so 3L = 200 – 2L
(3) Unknown resistance is S Q 100 L 3
R and specific resistance
P
200
r2S or L 40 cm.
is , where r is radius and L is length of wire. 5
L Galvanometer :
(4) In PO box we first press cell key and then press
(1) These are instrumen ts used for detection and
galvanometer key to eliminate induced effects.
measurement of small currents.
(5) It is used to find unknown resistance, specific resistance
(2) The moving coil galvanometer work on the principle that
of a wire, internal resistane of cell, resistance of
when a current carrying coil is placed in a magnetic field
galvanometer etc.
it experiences a torque.
(3) The different type of moving coil galvanometers are
E K
( ) (a) Pivoted Galvanometer : It consists of a coil of fine
insulated wire wound on a metallic frame. The coil is
RB
R S mounted on two jewelled pivots and is symmetrically
D
placed between cylindrical pole pieces of a strong
a b c d permanent horse-shoe magnet.
(b) Dead beat Galvanometer : Here coil is wound over the
metallic frame to make it dead beat. On passing current
P the galvanometer shows a steady deflection without any
B Q
A L 100-L C
oscillation. The damping is produced by eddy currents.
G
(c) Ballistic Galvanometer : This is used for measurement (7) Shunt (S) : It is a low resistance connected in parallel
of charge. Here coil is wound on an insulating frame and to coil of galvanometer to convert it to ammeter. It protects
oscillates on passing current. a galvanometer from strong currents. It is also used to
(4) In moving coil galvanometer the deflection produced is alter range of an ammeter.
proportional to current flowing through galvanometer G.S
i.e. they have linear scale of measurement. (8) The resistance of converted ammeter is R A
G S
In equilibrium deflecting torque = restoring torque (9) The range of an ammeter is increased by reducing shunt
resistance S.
C
i.e. NIAB = C or or I = K where K g G
NAB If I = N Ig then S G
N g g N 1
is galvanometer constant.
(5) Current Sensitivity : This is defined as the deflection The range of an ammeter can be increased N times by
produced in galvanometer when a unit current flows reducing shunt to S = G/N–1.
through it. (10) It is possible to increase the range of an ammeter because
it lowers the resistance of ammeter further.
NAB (11) The length of the shunt required l = r2S/ where r is
Current sensitivity CS radian/ampere
C radius of shunt wire and is specific resistance of
or division/ampere material of shunt wire.
(a) The sensitivity can be increased by increasing number (12) Reducing the shunt resistance may increase range but
of turns in coil (N), area of cross-section of coil (A), it reduces the sensitivity.
magnetic field B and decreasing torsional constant (C).
(b) The reciprocal of current sensitivity is called figure of (1) A voltmeter is a high resistance galvanometer used to
merit. measure potential difference.
C (2) A voltmeter is connected in parallel to a circuit element
Figure of merit FM because when connected in parallel it draws least current
NAB from the main current. So it measures nearly accurate
(6) Voltage Sensitivity : This is defined as the deflection potential difference.
produced in galvanometer when a unit voltage is applied (3) An ideal voltmeter has infinite resistance.
across its terminals.
(4) The reading of a voltmeter is always less than actual
Voltage sensitivity VS NAB division/ value because all practical voltmeter may have large but
V R CR finite resistance.
volt, where R is resistance of coil. (5) Greater is the resistance of voltmeter more accurate is its
(7) Shunting a galvanometer reduces its current sensitivity. reading.
(6) A galvanometer is converted to a voltmeter by connecting
(1) An ammeter is a low resistance galvanometer used to a high resistance in series with coil of galvanometer.
measure strength of current in an electrical circuit.
V
(2) An ammeter is always connected in series in a circuit V G) or R G
g (R
because when an ammeter is connected in series it does g
not appreciably change the resistance of circuit and
(7) The resistance of converted voltmeter is Rv = R + G
hence the main current flowing through the circuit.
(8) The range of a voltmeter is increased by increasing the
(3) In ideal ammeter has zero resistance. series resistance.
(4) The reading of an ammeter is always less than actual
current in the circuit because all practical ammeters have N gG G
If V = NVg = NIg G then R = (N – 1) G
low finite resistance.
g
(5) Smaller is the resistance of an ammeter more accurate
will be the reading. The value of resistance required to increase range N
(6) A galvanometer can be converted to an ammeter by times is R = (N – 1)G
connecting a low resistance shunt in parallel to coil of Ex. What is the value of shunt which passes 10% of main
galvanometer. current through a galvanometer of 99 ?

g gG 10
Here Ig G = (I – Ig) S or S G Sol. S where g 0.1 .
g g 100
Here G is resistance of galvanometer and Ig is current 0.1 99 9.9
required to produced full scale deflection of current. So S 11
(1 0.1) 0.9
1. A current of 0.5 A is passing through a CuSO4 solution.
How many Cu++ ions will be deposited on cathode in 23
3kT 3 1.38 10 300
10 second? v rms 10 5 m / s
m 31
9.1 10
Sol. If n is number of copper ions then q = n × 2e
So mean free path = vrms × = 105 × 2.5 × 10–14 = 25 A°
q = It or n × 2e = It
6. Calculate the drift velocity of electrons in silver wire with
so
cross-sectional area 3.14 × 10–6 m2 carrying a current
t 0.5 10 of 20 A. Given atomic weight of Ag = 108, density of
n = 1.5625 × 1019.
2e 2 1.6 10 19 silver = 10.5 × 103 kg/m3.

2. In case of hydrogen atom an electron moves in an orbit of 6.02 3 10 26


Sol. Number of electrons per kg of silver =
radius 5 × 10–11 m with a speed of 2.2 × 106 m/s. Calculate 108
equivalent current.
Number of electrons per unit volume of silver
Sol. The equivalent current
6.023 10 26
qv 1.6 10 19
2.2 10 6 n 10.5 10 3
1.12 mA 1 08
2 r 2 3.14 5 10 11
3. How many electrons flow through the filament of a 120 vd
neA
volt, 60 W electric lamp in one second?
20
Power 60 26 3 19 6
108
Sol. Electric current or 0.5 A 6.023 10 10.5 10 1.6 10 3.14 10
voltage 120
= 6.798 × 10–4 m/sec.
.t 0.5 1 7. The I-V curves at two different temperatures T1 and T2
n 3.125 1018 are shown. (a) Is the specimen ohmic (b) At which
e 1.6 10 19
temperature is resistance higher and (c) which temperature
4. A potential difference V is applied to a copper wire of is greater?
diameter d and length L. What is effect on drift speed
Sol. (a) I-V curve is linear and passes through origin so
when V, L and d are doubled?
specimen is ohmic
eE eV V 1
Sol. Drift velocity v d
m mL (b) R . From graph 1> 2 so tan 1> tan
tan
As vd V so on doubling V drift velocity is doubled. q2 or R1 < R2. The resistance R2 at temperature T2 is
As vd 1/L so on doubling L drift velocity is halved. greater.
As vd is independent of d so no change on doubling the (c) The resistance increases with temperature so T2 > T1
diameter. because R2 > R1.
5. Calculate the relaxation time and mean free path in Cu at 8. If a copper wire is stretched to make it 0.1% longer what is
room temperature 300 K, if number density of free electrons percentage change in its resistance.
is 8.5 × 1028 /m³ and resistivity = 1.7 × 10–8 mho-m. Given
k = 1.38 × 10–23 J/K. L L2
Sol. R taking log we get log R = log + 2 log L
Sol. Relaxation time A V
m m R 2 L
2 – log V differentiating we get = 2(0.1%) =
ne ne 2 R L
0.2%.The resistance increases by 0.2%.
31
9.1 10 14 9. At what temperature would the resistance of a copper
2.5 10 second
8.5 10 28 (1.6 10 19 2
) 1.7 10 8 conductor be doubled of its value at 0°C. Does this same
temperature hold for all copper conductors regardless of
shape and size? Cu = 4.0 × 10–3/°C.
Sol. By symmetry potential at point 4 and 5 is same. Similarly
R2 R 0 [1 (t 2 0)] 1 t2 potential at point 3 and 6 is same. The equivalent circuits
Sol. R = R0 (1 + ) so R R 0 [1 (t1 0)] 1 t1 can be drawn as :
1

Given R2 = 2R1 and t1 = 0 so 2 = 1 + t2 1 R 4,5 R 8


a
R R
1 1
or t2 250 C
4 10 3

1 R R R R
Here t 2 does not include dimensions of conductor

so it is valid for all copper conductors of all shape and


size. R R
b
10. Find equivalent resistance between the points A and B? 2 R 3,6 R 7

1 R/2 4,5
2R 2R R a
A 1 3 B
2 4

R R
Sol. Point 1 and 3 are at same potential. Similarly point 2 and 4 2 2R
are at same potential. Joining resistance between 1 and 2,
2 and 3, 3 and 4 we find that they all are in parallel. R/2
b
1 1 1 1 2 2 3,6
So equivalent resistance R 2R 2R R R or
p 1
a
Rp = R/2
11. Five resistors are connected as shown. Find equivalent
resistance between the points B and C.
R 7R
Sol. Resistance of arm ADC = 3 + 7 = 10 . 5
This is in parallel to 10 of arm AC. Their parallel
combination gives equivalent resistance of 5 .
So effective resistance between B and C b
2
14 5 70 The equivalen t resistance between a and b is
R eq 3.684
14 5 19 7
12. When two resistances are joined in series their resistance R R
5 7
R eq R
is 40 and when they are joined in parallel the resistance 7 12
R R
is 7.5 . Find the individual resistances? 5
Sol. Let the two resistances be R1 and R2
14. Calculate the current shown by ammeter A in the circuit
When connected in series R1 + R2 = 40 .......(1)
shown in fig.
R1R 2 Sol. The equivalent circuits can be drawn as
When connected in parallel R R 7.5
1 2 10
s o R1 R2 = 7.5 × 40 = 300
(R1 – R2)2 = (R1 + R2)2 – 4R1 R2 = 402 – 4 × 300 = 400 10 10
So R1 – R2 = 20 .......(2)
Solving 1 and 2 we have R1 = 30 and R2 = 10 . 10 10
13. Twelve equal resistances R are used to generate shape of
a cube. Calculate equivalent resistance across the side of
A
cube?
12V
10 (1) The reading will be reduced to one half
(2) The reading will be double
(3) The reading will not be affected
10 (4) The reading will increase four fold
Sol. Since Q is connected in parallel the net resistance becomes
R/2, so the current l = 2V/R, double the value.
17. Three identical cells each of emf 2V and unknown internal
A resistance are connected in parallel. This combination is
12V connected to a 5 resistor. If the terminal voltage across
the cells is 1.5 V, what is internal resistance of each cell.
E
Sol. In parallel combination of three cells
R r/3

ER
A Terminal potential difference V R or
R r /3
12V r ER RV
3 V
10
The equivalent resistance of circuit is R eq ER RV 2 5 5 1.5
3 or r 3 3 5
V 1.5
E 12
3.6 A 18. Determine currents I1, I2 and I3?
The current R eq 10 / 3
b 1 24V a
15. The resultant resistance between the points A and B in
2
the following diagram Fig. will be-

c 2
d
1 1 1 1 27V 6
A 3

e f
2 2 2 1 4
Sol. Using II law in loop abcda –2I 1 + 24 – 27 – 6I 2 = 0
B or 2I1 + 6I2 = – 3 ...(1)
(1) 4 (2) 8 Using II law in loop cdfec we get –27 – 6I2 + 4I3 = 0
or – 6I2 + 4I3 = 27 ......(2)
(3) 6 (4) 2
Sol. From junction rule at c we getI1 = I2 + I3 ......(3)
1 From 2 and 3 we get –6I2 + 4(I1 – I2) = 27
R1 R 2 1 2 2 or 4I1 – 10I2 = 27 ......(4)
R R1 R 2 4R 3 R1 R 2 ....(1)
2 2 Solving 1 and 4 we get I1 = 3A,I2 = –1.5 A so I3 = I1 – I2
R1 = 1 , R2 = 0. R3 = 2 ...(2)
= 4.5 A
From eqs. (1) and (2)
1
Negative I2 means the direction should have been opposite
1 1 to shown in figure.
R 1 4 2 42 1 3 2
2 2 19. Calculate equivalent resistance between A and B.
16. How will reading in the ammeter A of the fig. be affected if
Sol. After distribution of current using loop rule for
an other identical bulb Q is connected in parallel to P as
Loop a–(I – I1) R – I2 R + I1 R = 0 or 2I1 – I2 = I
shown. The voltage in the mains is maintained at constant
value. Loop b–(I – I1 – I2) R + I3R + I2R = 0 or I1 + 2I2 + I3 = I
Loop c–I3R – (I – I1 – I2 + I3) R + (I1 + I2 – I3) R = 0
or 2I1 + 2I2 – 3I3 = I
Solving for I1, I2 and I3 we get
Mains
P Q
4
1 , 2 , 3
A 7 7 7
Here I2 = I3 so, no distribution of current takes place at D
2 I
so it is a pseudo junction. Solving 1 and 2 we get 1 and Ig =
5 5
The equivalent circuit becomes.
If Req is equivalent resistance between AC then
2
Resistance of triangle DCE is R parallel to (R + R) = R 4 3 7
3 V = I Req = I1 × 2R + (I1 + Ig) R = IR + IR = IR
5 5 5
2 8
Resistance of network other than AB = R R R R 7
3 3 so R eq R
5
Equivalent resistance between AB
23. Calculate the effective resistance between A and B in the
8R following network.
2R
3 8 Sol. The circuit can be redrawn as
R eq R
8R 7
2R P R 2
3 Here so bridge is balanced
Q S 3
20. A circuit has a section ABC. If the potential at points A, B
and C are V1, V2 & V3 then calculate potential at O. So the resistance between c and d is non useful.
Sol. Applying junction rule at O – I1 – I2 – I3 = 0 or Equivalent resistance = (P + Q) | | (R + S)
I1 + I 2 + I 3 = 0 (P Q)(R S) (2 3)(4 6) 5 10 10
R eq
Let V0 be potential at O then P Q R S 2 3 4 6 15 3
V0 – V1 = I1 R1,V0 – V2 = I2 R2, V0 – V3 = I3 R3
24. Calculate the effective resistance between A and B in
V0 V1 V0 V2 V0 V3 following network.
so 1 , 2 , 3
R1 R2 R3 Sol. Ratio of upper resistances 5 : 10 : 15 = 1 : 2 : 3
Ratio of lower resistances 10 : 20 : 30 = 1 : 2 : 3
adding them we get
The ratio is same so resistance in middle are nonuseful.
1 1 1 V1 V2 V3 Equivalent resistance = (5 + 10 + 15) | | (10 + 20 + 30)
V0 0
R1 R2 R3 R1 R2 R3 30 60
So R eq 20
or 30 60

V1 V2 V3 1 1 1 25. If a shunt of resistance G/n is connected to a galvanometer


V0 then find change in current sensitivity?
R1 R2 R3 R1 R2 R3
Sol. The current sensitivity of a galvanometer
21. Calculate potential difference between points d and b in IS = /I
the circuit. Current sensitivity of a shunted galvanometer I S = /I
Sol. Using voltage rule from b to d via c we get
g '
Vb – 1 × 2 + 4 × 1 = Vd or Vd – Vb = 2 volt. I and Ig So
22. Calculate effective resistance between A and C?

' ' 1 g g g
P R 1 r 2R 2 S S
Sol. Here and
Q 2R 2 S R 1
For shunted galvanometer (I – Ig) S = Ig G
P r G
So so bridge is unbalanced or ( g) gG so
Q S n
g
1 n
Applying Kirchhoff’s law to
Loop a–(I – I1)R – Ig R + I1 × 2R = 0 or 3I1 – Ig = I ......(1) ' g S
1 n This shows that I’S < IS
S S
Loop b–(I – I1 – Ig) 2R + (I1 + Ig) R + Ig R = 0
or 3I1 + 4Ig = 2I ......(2) The current sensitivity is reduced by shunting of
galvanometer.
26. A galvanometer of resistance 12 shows full scale 27. Find the resistance of galvanometer in which maximum
deflection for a current of 2.5 mA. How will you convert it deflection of 50 div is reduced to 20 div when shunted by
to ammeter of range 7.5 A and voltmeter of range 10 volt. a 12 resistance?
Find resistance of meter. Sol. The current through galvanometer after shunting by 12
Sol. Shunt resistance required for ammeter
12
resistance is g K20
3 12 G
g 2.5 10
S G 12 4m
3
g 7.5 2.5 10 12
Here Ig = 50 K So 50K 20K or 600
12 G
1 1 1
Resistance of ammeter R = 240 + 20G or G = 18 .
a G S or
28. A galvanometer whose resistance is 10 is converted to
a voltmeter of range 10V and 100V by connecting
GS 4 10 3 12 resistances R1 and R2 in series. if Ig = 10 mA find
Ra 4m
G S 4 10 3 12 R1 and R2.
Series resistance required for voltmeter Sol. The potential difference across G + R1 is 10 volt.
So Ig (G + R1) = 10 or10 × 10–3 (10 + R1) = 10 or
V 10
R G 12 3988 R1 = 990 .
g 2.5 10 3
The potential difference across G + R1 + R2 is 100 volt
Resistance of voltmeter Rv = R + G = 3988 + 12 = 4000 . So Ig (G + R1 + R2) = 100 or10 × 10–3 (10 + 990 + R2)
= 100 or R2 = 9000 .
1. Is the motion of a charge across junction momentum 11. Two conductors are made of the same material and have
conserving? Why or why not? the same length. Conductor A is a solid wire of diameter
2. The relaxation time T is nearly independent of applied E 1mm. Conductor B is a hollow tube of outer diameter 2mm
field whereas it changes significantly with temperature T. and inner diameter 1mm. Find the ratio of resistance
First fact is (in part) responsible for Ohm’s law whereas RA to RB.
the second fact leads to variation of with temperature. 12. 3.19 AB is a potentiometer wire. If the value of R is
Elaborate why? increased, in which direction will the balance point J shift?
3. What are the advantages of the null-point method in a E R
Wheatstone bridge? What additional measurements
would be required to calculate Runknown by any other
method?
J
4. What is the advantage of using thick metallic strips to A B
join wires in a potentiometer?
G
5. For wiring in the home, one uses Cu wires or Al wires.
What considerations are involved in this? 13. The circuit in Figure shows two cells connected in
6. Why are alloys used for making standard resistance coils? opposition to each other. Cell E1 is of emf 6V and internal
resistance 2 ; the cell E2 is of emf 4V and internal resistance
7. Power P is to be delivered to a device via transmission
8 . Find the potential difference between the points A
cables having resistance RC. If V is the voltage across R
and B.
and I the current through it, find the power wasted and
how can it be reduced. A B
8. A cell of emf E and internal resistance r is connected across
an external resistance R. Plot a graph showing the variation E1 E2
of P.D. across R, verses R.
9. First a set of n equal resistors of R each are connected in
series to a battery of emf E and internal resistance R. A
current I is observed to flow. Then the n resistors are 14. Two cells of same emf E but internal resistance r1 and r2
connected in parallel to the same battery. It is observed are connected in series to an external resistor R. What
that the current is increased 10 times. What is ‘n’? should be the value of R so that the potential difference
10. Let there be n resistors R1 Rn with Rmax (R1 across the terminals of the first cell becomes zero.
Rn) and Rmin {R1 .. Rn}. Show that when they are
E E B
connected in parallel, the resultant resistance Rp< Rmin
and when they are connected in series, the resultant
resistance Rs> Rmax. Interpret the result physically.

R
1. There are eight resistances of R ohm each. Two-two 5. The following diagram shows the circuit for the
resistances are connected in parallel to form couples and comparision of e.m.f. of the cells. The circuit can be
these couples are connected in series, then the total corrected by :–
resistance of this combination is :–
R E Rh
(A) (B) 2R
2
(C) 4R (D) 8 R E1
2. Reading of ammeter in ampere for the following circuit is :-
E2
+ – G
2V (A) Reversing the terminals of E
i (B) Reversing the terminals of E1
A
(C) Reversing the terminals of E2
2 2 2
(D) Reversing the current in Rh.
2V
– 6. Two similar cells are connected first in series and then in
+
parallel, the ratio of balancing length on the potentiometer
1 wire will be :–
(A) 1 (B)
2 (A) 1 : 2 (B) 2 : 1
(B) 1 : 4 (D) 4 : 1
2
(C) (D) 3 7. In the following circuit, the reading of the voltmeter will
3
be :–
3. For the following circuit the potential difference between (in volt)
x and y in volt is :–
+ –
12V Rh
100 100
V
40cm 60cm
200 V x V y
DC
100 4.8V G
100
+ –

(A) 10 (B) 50 (A) 7.2 (B) 4.8


(C) 100 (D) 0 (C) 6 (D) 4
4. The value of total resistance between x and y in ohm is :– 8. The resistance of a wire is 50 ohm. Then the graph between
B log V and log I is :–
(A) straight line
(B) parabola
(C) hyperbola
R R
R (D) circle
9. The resistance of wire is 20 ohm. The wire is stretched to
x y three times of its length. Then the resistance will be :–
R (A) 6.67 W
A R C (B) 60 W
(A) R (B) 3 R (C) 120 W
(C) 4 R (D) 5 R (D) 180 W
10. In the following diagram, the deflection in the 19. 62.5 × 1018 electrons per second are flowing through a
galvanometer in a potentiometer circuit is zero, then :– wire of area of cross-section 0.1m2, the value of current
flowing will be
+ –
(A) 1 A (B) 0.1 A
E Rh
(C) 10 A (D) 0.11 A
A B 20. A piece of wire of resistance 4 ohms is bent through 180°
E1 at its mid point and the two halves are twisted together,
then the resistance is
E2 G G (A) 8 ohms (B) 1 ohm
(C) 2 ohms (D) 5 ohms
(A) E1 > E2 (B) E2 > E1 21. When a piece of aluminium wire of finite length is drawn
(C) E1 = E2 (D) E1 + E2 = E through a series of dies to reduce its diameter to half its
11. The resistance of a wire is 20 ohms. It is so stretched that original value, its resistance will become
the length becomes three times, then the new resistance (A) Two times (B) Four times
of the wire will be (C) Eight times (D) Sixteen times
(A) 6.67 ohms (B) 60.0 ohms 22. A wire 100cm long and 2.0mm diameter has a resistance of
(C) 120 ohms (D) 180.0 ohms 0.7 ohm, the electrical resistivity of the material is
12. The resistivity of a wire (A) 4.4 × 10–6 ohm × m (B) 2..2 × 10–6 ohm × m
(A) Increases with the length of the wire (C) 1.1 × 10–6 ohm × m (D) 0.22 × 10–6 ohm × m
(B) Decreases with the area of cross-section 23. A certain wire has a resistance R. The resistance of another
(C) Decreases with the length and increases with the wire identical with the first except having twice its
cross-section of wire diameter is
(D) None of the above statement is correct (A) 2 R (B) 0.25 R
13. Ohm’s law is true (C) 4 R (D) 0.5 R
(A) For metallic conductors at low temperature
24. In hydrogen atom, the electron makes 6.6 × 1015 revolutions
(B) For metallic conductors at high temperature
per second around the nucleus in an orbit of radius
(C) For electrolytes when current passes through them
0.5 × 10–10 m. It is equivalent to a current nearly
(D) For diode when current flows
(A) 1 A (B) 1 mA
14. The example for non-ohmic resistance is
(C) 1 A (D) 1.6 × 10–19 A
(A) Copper wire (B) Carbon resistance
25. A wire of length 5m and radius 1mm has a resistance of
(C) Diode (D) Tungston wire
1 ohm. What length of the wire of the same material at the
15. Drift velocity vd varies with the intensity of electric field
same temperature and of radius 2mm will also have a
as per the relation
resistance of 1 ohm
1 (A) 1.25 m (B) 2.5 m
(A) vd E (B) vd
E (C) 10 m (D) 20 m
(C) vd = constant (D) vd E2 26. When there is an electric current through a conducting
16. On increasing the temperature of a conductor, its wire along its length, then an electric field must exist
resistance increases because (A) Outside the wire but normal to it
(A) Relaxation time decreases (B) Outside the wire but parallel to it
(B) Mass of the electrons increases (C) Inside the wire but parallel to it
(C) Electron density decreases (D) Inside the wire but normal to it
(D) None of the above 27. Through a semiconductor, an electric current is due to
17. In a conductor 4 coulombs of charge flows for 2 seconds. drift of
The value of electric current will be (A) Free electrons
(A) 4 volts (B) 4 amperes (B) Free electrons and holes
(C) 2 amperes (D) 2 volts (C) Positive and negative ions
18. The specific resistance of a wire is , its volume is 3m3 and (D) Protons
its resistance is 3 ohms, then its length will be 28. In an electrolyte 3.2 × 1018 bivalent positive ions drift to
1 3 the right per second while 3.6 × 1018 monovalent negative
(A) (B) ions drift to the left per second. Then the current is
(A) 1.6 amp to the left
1 1 (B) 1.6 amp to the right
(C) 3 (D)
3 (C) 0.45 amp to the right
(D) 0.45 amp to the left
29. A metallic block has no potential difference applied across 39. It is easier to start a car engine on a hot day than on a cold
it, then the mean velocity of free electrons is T = absolute day. This is because the internal resistance of the car
temperature of the block) battery
(A) Proportional to T (A) Decreases with rise in temperature
(B) Proportional to T (B) Increases with rise in temperature
(C) Zero (C) Decreases with a fall in temperature
(D) Finite but independent of temperature (D) Does not change with a change in temperature
30. The specific resistance of all metals is most affected by 40. 5 amperes of current is passed through a metallic
(A) Temperature conductor. The charge flowing in one minute in coulombs
(B) Pressure will be
(C) Degree of illumination (A) 5 (B) 12
(D) Applied magnetic field 1
(C) (D) 300
31. The positive temperature coefficient of resistance is for 12
(A) Carbon (B) Germanium 41. If the resistance of a conductor is 5 at 50oC and 7 at
(C) Copper (D) An electrolyte 100oC then the mean temperature coefficient of resistance
32. The fact that the conductance of some metals rises to of the material is
infinity at some temperature below a few Kelvin is called (A) 0.008/oC (B) 0.006/oC
(A) Thermal conductivity (C) 0.004/ Co (D) 0.001/oC
(B) Optical conductivity 42. The resistance of a discharge tube is
(C) Magnetic conductivity
(A) Ohmic (B) Non-ohmic
(D) Superconductivity
(C) Both (A) and (B) (D) Zero
33. Dimensions of a block are 1 cm × 1cm × 100 cm. If specific
43. We are able to obtain fairly large currents in a conductor
resistance of its material is 3 × 10–7 ohm – m, then the
because
resistance between the opposite rectangular faces is
(A) The electron drift speed is usually very large
(A) 3 × 10–9 ohm (B) 3 × 10–7 ohm
–5
(C) 3 × 10 ohm (D) 3 × 10–3 ohm (B) The number density of free electrons is very high
and this can compensate for the low values of the
34. In the above question, the resistance between the square
electron drift speed and the very small magnitude of
faces is
the electron charge
(A) 3 × 10–9 ohm (B) 3 × 10–7 ohm
(C) The number density of free electrons as well as the
(C) 3 × 10–5 ohm (D) 3 × 10–3 ohm
electron drift speeds are very large and these
35. There is a current of 20 amperes in a copper wire of
compensate for the very small magnitude of the
10–6 square metre area of cross-section. If the number of free
electron charge
electrons per cubic metre is 1029, then the drift velocity is
(A) 125 × 10–3 m/sec (B) 12.5 × 10–3 m/sec (D) The very small magnitude of the electron charge has
to be divided by the still smaller product of the
(C) 1.25 × 10–3 m/sec (D) 1.25 × 10–4 m/sec
number density and drift speed to get the electric
36. The electric intensity E, current density j and specific
current
resistance k are related to each other by the relation
44. A platinum resistance thermometer has a resistance of
(A) E = j/k (B) E = jk
50 at 20° C. When dipped in a liquid the resistance
(C) E × k/j (D) k = jE
becomes 76.8 . The temperature coefficient of resistance
37. The resistance of a wire of uniform diameter d and length
for platinum is = 3.92 × 10–31 ° C. The temperature of the
L is R. The resistance of another wire of the same material
liquid is
but diameter 2d and length 4L will be
(A) 100° C (B) 137° C
(A) 2R (B) R
(C) 167° C (D) 200° C
R R
(C) (D) 45. In a wire of circular cross-section with radius r, free
2 4 electrons travel with a drift velocity V when a current I
38. There is a current of 1.344 amp in a copper wire whose flows through the wire. What is the current in another
area of cross-section normal to the length of the wire is wire of half the radius and of the same material when the
1mm2. If the number of free electrons per cm3 is 8.4 × 1022, drift velocity is 2V
then the drift velocity would be
(A) 2I (B) I
(A) 1.0 mm/sec (B) 1.0 m/sec
(C) 0.1mm/sec (D) 0.01mm/sec I I
(C) (D)
2 4
46. The resistivity of a wire depends on its 55. 1.6 mA current is flowing in conducting wire then the
(A) Length (B) Area of cross-section number of electrons flowing per second is
(C) Shape (D) Material (A) 1011 (B) 1016
47. The conductivity of a superconductor is
(C) 1019 (D) 1015
(A) Infinite (B) Very large
56. A current I is passing through a wire having two sections
(C) Very small (D) Zero
48. In a neon discharge tube 2.9 × 1018 Ne+ ions move to the d
P and Q of uniform diameters d and respectively. If the
right each second while 1.2 × 1018 electrons move to the 2
mean drift velocity of electrons in sections P and Q is
left per second. Electron charge is 1.6 × 10–19 C.The current
denoted by vP and vQ respectively, then
in the discharge tube
1
(A) 1 A towards right (A) vP = vQ (B) vP vQ
(B) 0.66 A towards right 2
(C) 0.66 A towards left 1
(C) vP = vQ (D) vP = 2 vQ
(D) Zero 4
49. A steady current flow in a metallic conductor of non- 57. If an electric current is passed through a nerve of a man,
uniform cross-section. The quantity/ quantities constant then man
along the length of the conductor is/are (a) Begins to laugh
(A) Current, electric field and drift speed (B) Begins to weep
(B) Drift speed only (C) Is excited
(C) Current and drift speed (D) Becomes insensitive to pain
(D) Current only 58. The resistance of a coil is 4.2 at 100o C and the
50. The resistivity of alloys = R alloy; the resistivity of temperature coefficient of resistance of its material is
constituent metals Rmetal. Then, usually 0.004/o C. Its resistance at 0o C is
(A) Ralloy = Rmetal (A) 6.5 (B) 5
(B) Ralloy < Rmetal (C) 3 (D) 4
(C) There is no simple relation between Rmetal and Rmetal 59. Masses of three wires of copper are in the ratio of 1 : 3 : 5
and their lengths are in the ratio of 5 : 3 : 1 The ratio of
(D) Ralloy > Rmetal
their electrical resistances are
51. Two wires A and B of same material and same mass have
(A) 1 : 3 : 5 (B) 5 : 3 : 1
radius 2r and r. If resistance of wire A is 34 , then
resistance of B will be (C) 1 : 15 : 125 (D) 125 : 15 : 1
(A) 544 (B) 272 60. Conductivity increases in the order of
(C) 68 (D) 17 (A) Al, Ag, Cu (B) Al, Cu, Ag
52. Two rods of same material and length have their electric (C) Cu, Al, Ag (D) Ag, Cu, Al
resistance in ratio 1 : 2. When both rods are dipped in 61. A uniform wire of resistance R is uniformly compressed
water, the correct statement will be along its length, until its radius becomes n times the
(A) A has more loss of weight original radius. Now resistance of the wire becomes
(B) B has more loss of weight R R
(A) 4 (B)
(C) Both have same loss of weight n n2
(D) Loss of weight will be in the ratio 1 : 2 R
53. 20 A current flows for 30 seconds in a wire, transfer of (C) (D) nR
n
charge will be 62. The resistance of a conductor is 5 ohm at 50oC and 6 ohm
(A) 2 × 10–4 C (B) 4 × 10–4 C at 100oC. Its resistance at 0oC is
(C) 6 × 10–4 C (D) 8 × 10–4 C (A) 1 ohm (B) 2 ohm
54. 1 and 2 are the electrical conductivities of Ge and Na (C) 3 ohm (D) 4 ohm
respectively. If these substances are heated, then 63. If an electron revolves in the path of a circle of radius of
(A) Both 1 and 2 increase 0.5 × 10–10 m at frequency of 5 × 1015 cycles/s the electric
(B) 1 increases and 2 decreases
current in the circle is (Charge of an electron = 1.6 × 10–19 C)
(C) 1 decreases and 2 increases
(A) 0.4 mA (B) 0.8 mA
(D) Both 1 and 2 decrease
(C) 1.2 mA (D) 1.6 mA
64. Equal potentials are applied on an iron and copper wire of 73. The resistance of the galvanometer is 25 it gives a full-
same length. In order to have the same current flow in the scale deflection when a current of 10mA is passed through
two wires, the ratio r (iron)/r (copper) of their radii must it. The value of resistance R used in series to convert it
be (Given that specific resistance of iron = 1.0 × 10–7 ohm–m into voltmeter of range 100 volt is :–
and specific resistance of copper = 1.7 × 10–8 ohm-m) (A) 10,000 (B) 10025
(A) About 1.2 (B) About 2.4 (C) 975 (D) 9975
(C) About 3.6 (D) About 4.8 74. Which of the statement is wrong :–
65. An electron (charge = 1.6 × 10–19 coulomb) is moving in a (A) when all resistance are equal, then the sensitivity of
circle of radius 5.1 × 10–11m at a frequency of 6.8 × 1015 wheatstone bridge is maximum.
revolutions/sec. The equivalent current is approximately (B) when the galvanometer and the cell are interchanged,
then the balancing of wheat stone bridge will be
(A) 5.1 × 10–3 amp (B) 6.8× 10–3 amp
–3
effected.
(C) 1.1 × 10 amp (D) 2.2 × 10–3 amp
(C) Kirchoff’s first law for the currents meeting at the
66. A rod of a certain metal is 1.0 m long and 0.6 cm in diameter. Junctions in an electric circuit shows the conservation
Its resistance is 3.0 × 10–3 ohm. Another disc made of the of charge.
same metal is 2.0 cm in diameter and 1.0 mm thick. What is (D) Rheostat can be used as potential divider.
the resistance between the round faces of the disc 75. In a meter bridge the null point is obtained at the middle
(A) 1.35 × 10–8 ohm (B) 2.70 × 10–7 ohm point of the wire. If in one gap the resistance is 10 , then
–6
(C) 4.05 × 10 ohm (D) 8.10 × 10–5 ohm the value of resistance in the other gap is :–
67. At what temperature will the resistance of a copper wire (A) 10 (B) 5
become three times its value at 0oC (Temperature (C) (D) 500
coefficient of resistance for copper = 4 × 10–3 per oC ) 76. A new electric store house of 1.5 emf of a flash gives a
(A) 400oC (B) 450oC current of 15A, when it is connected with an electric
(C) 500 C o (D) 550oC ammeter of 0.04 , then the value of internal resistance of
68. An electron revolves 6 × 1015 times/sec in circular loop. the electric store house is :–
The current in the loop is (A) 0.04 (B) 0.06
(C) 0.10 (D) 10
(A) 0.96 mA (B) 0.96 A
77. In a torch there are two cells each of 1.45 volt and 0.15 .
(C) 28.8 A (D) None of these Each cell gives a current to filament of a lamp of 1.5 ,
69. The charge of an electron is 1.6 × 10–19 C. How many then the value of current in ampere is :–
electrons strike the screen of a cathode ray tube each (A) 16.11 (B) 1.611
second when the beam current is 16 mA (C) 0.1611 (D) 2.6
(A) 1017 (B) 1019 78. In a potentiometer experiment a voltage source is balanced
(C) 10 –19 (D) 10–17 at 60cm length where as a 3 volt battery is balanced at
70. If potential V = 100 ± 0.5 Volt and current I = 10 ± 0.2 amp 45cm length. What is the voltage of unknown voltage
are given to us. Then what will be the value of resistance source :–
(A) 10 ± 0.7 ohm (B) 5 ± 2 ohm (A) 3 V (B) 4 V
(C) 0.1 ± 0.2 ohm (D) None of these (C) 4.5 V (D) 6 V
79. A potential difference V is applied across a copper wire of
71. The specific resistance of a metal wire is 64 × 10–6 × cm.,
diameter d and length L. when only d is doubled, the drift
the length is 198 cm and the resistance is 7 . The radius velocity :–
of wire is : (A) increases two times (B) decreases times
(A) 2.4 cm (B) 0.24 cm (C) does not change (D) decreases times
(C) 0.024 cm (D) 24 cm 80. Find the potential difference between X and Y in volt is :–
72. Which of the following statement is wrong :– X
(A) the resistance of a voltmeter is high.
2 3
(B) the resistance of an ammeter is low.
(C) an ammeter is connected in parallel with a conductor 2A 2A
in the circuit.
(D) a voltmeter is connected in parallel with a resistance
in the circuit. 2
Y
(A) 1 (B) –1
(C) 2 (D) –2
81. By increasing the temperature, the specific resistance of a 90. When a current flows through a conductor its temperature
conductor and a semiconductor (A) May increase or decrease
(A) Increases for both (B) Remains same
(B) Decreases for both (C) Decreases
(C) Increases, decreases (D) Increases
(D) Decreases, increases 91. What length of the wire of specific resistance 48 × 10–8
82. Which of the following is vector quantity m is needed to make a resistance of 4.2 (diameter of
(A) Current density (B) Current wire = 0.4 mm)
(C) Wattless current (D) Power (A) 4.1 m (B) 3.1 m
83. Masses of 3 wires of same metal are in the ratio 1 : 2 : 3 (C) 2.1 m (D) 1.1 m
92. A strip of copper and another of germanium are cooled
and their lengths are in the ratio 3 : 2 : 1. The electrical
from room temperature to 80 K. The resistance of
resistances are in ratio
(A) Each of these increases
(A) 1 : 4 : 9 (B) 9 : 4 : 1 (B) Each of these decreases
(C) 1 : 2 : 3 (D) 27 : 6 : 1 (C) Copper strip increases and that of germanium
84. A current of 1 mA is flowing through a copper wire. How decreases
many electrons will pass a given point in one second (D) Copper strip decreases and that of germanium
[e = 1.6 × 10–19 Coulomb] increases
(A) 6.25 × 1019 (B) 6.25 × 1015 93. The length of a given cylindrical wire is increased by
(C) 6.25 × 10 31 (D) 6.25 × 108 100 %. Due to the consequent decrease in diameter the
85. The drift velocity of free electrons in a conductor is ‘v’ change in the resistance of the wire will be
when a current ‘i’ is flowing in it. If both the radius and (A) 300 % (B) 200 %
current are doubled, then drift velocity will be
(C) 100 % (D) 50 %
v
(A) v (B) 94. Express which of the following setups can be used to
2 verify Ohm’s law
v v
(C) (D) A
4 8
86. A wire of radius r has resistance R. If it is stretched to a V
3r (A)
radius of , its resistance becomes
4
9R 16 R
(A) (B)
16 9
81R 256 R A V
(B)
(C) (D)
256 81
87. The resistance of a conductor increases with
(A) Increase in length A
(B) Increase in temperature
(C) Decrease in cross–sectional area (C)
(D) All of these V
88. A copper wire has a square cross-section, 2.0 mm on a
V
side. It carries a current of 8 A and the density of free
electrons is 8 × 1028 m–3. The drift speed of electrons is
equal to (D)
(A) 0.156 × 10–3 m.s–1 (B) 0.156 × 10–2 m.s–1 A
–3
(C) 3.12 × 10 m.s –1 (D) 3.12 × 10–2 m.s–1 95. We have two wires A and B of same mass and same material.
89. Two wires of same material have length L and 2L and The diameter of the wire A is half of that B. If the resistance
cross–sectional areas 4A and A respectively. The ratio of of wire A is 24 ohm then the resistance of wire B will be
their specific resistance would be (A) 12 Ohm (B) 3.0 Ohm
(A) 1 : 2 (B) 8 : 1 (C) 1.5 Ohm (D) None of the above
(C) 1 : 8 (D) 1 : 1
96. In a hydrogen discharge tube it is observed that through 104. When a potential difference is applied across the ends of
a given cross-section 3.13 × 1015 electrons are moving a linear metallic conductor :–
from right to left and 3.12 × 1015 protons are moving from (A) the free electrons are accelerated continuously from
left to right. What is the electric current in the discharge the lower potential end to the higher potential end of
tube and what is its direction the conductor
(A) 1mA towards right (B) 1mA towards left (B) the free electrons are accelerated continuously from
(C) 2mA towards left (D) 2mA towards right the higher potential end to the lower potential end of
97. A steady current i is flowing through a conductor of the conductor
uniform cross-section. Any segment of the conductor has (C) the free electrons acquire a constant drift velocity
(A) Zero charge from the lower potential end to the higher potential
(B) Only positive charge end of the conductor.
(C) Only negative charge (D) the free electrons are set in motion from their position
(D) Charge proportional to current i of rest
98. The length of the wire is doubled. Its conductance will be 105. A current of 2 A is flowing through a cell of e.m.f. 5 V and
(A) Unchanged (B) Halved internal resistance 0.5 from negative to positive
1 electrode. If the potential of negative electrode is 10 V, the
(C) Quadrupled (D) of the original value
4 potential of positive electrode will be :–
99. A source of e.m.f. E = 15 V and having negligible internal (A) 5 V (B) 14 V
resistance is connected to a variable resistance so that (C) 15 V (D) 16 V
the current in the circuit increases with time as i = 1.2 t + 3. 106. 100 cells, each of e.m.f. 5 V and internal resistance 1 , are
Then, the total charge that will flow in first five second to be arranged so as to produce maximum current in a
will be 25 resistance. Each row is to contain equal number of
(A) 10 C (B) 20 C cells. The number of rows should be :–
(C) 30 C (D) 40 C
(A) 2 (B) 4
100. The new resistance of wire of R , whose radius is reduced
half, is (C) 5 (D) 10
(A) 16 R (B) 3 R 107. In the circuit shown below, the cell has an e.m.f. of 10 V
(C) 2R (D) R and internal resistance of 1 . The other resistances are
101. For two wires A and B of same material and of same mass, shown in the fig. The potential difference VA – VB is :–
the radius of A is double that of B. If the resistance of wire
A is 34 then that of B will be :– E = 10V
(A) 544 (B) 272 r=1
(C) 68 (D) 17
102. In the following circuit if VA–VB = 4V, then the value of
resistance X in ohms will be :– 4 A 2 1

10 5V 4
2 B
A 2V X B
(A) 6 V
(A) 5 (B) 10 (B) 4 V
(C) 15 (D) 20 (C) 2 V
103. For the network of resistance shown in the fig. the
equivalent resistance of the network between the points (D) – 2 V
A and B is 18 . The value of unknown resistance R is 108. Two resistance R1 and R2 are made of different materials.
R The temperature coefficient of the material of R1 is and
10 10 of the material of R2 is – . The resistance of the series
combination of R1 and R2 will not change with temp., then
10 ratio of resistance of two wire at 0°C will be :

B 10 10 10 A (A) (B)

(A) 8 (B) 10 2 2
(C) 16 (D) 24 (C) (D)
109. In the arrangment of resistances shown below. The 114. Three resistances of magnitude 2, 3 and 5 ohm are
effective resistance between points A and B is connected in parallel to a battery of 10 volts and of
negligible resistance. The potential difference across 3
5 10 15
resistance will be
A B (A) 2 volts (B) 3 volts
10 10 (C) 5 volts (D) 10 volts
10 20 30 115. A current of 2 A flows in a system of conductors as shown.
The potential difference (VA – VB) will be
(A) 20 (B) 30
A
(C) 90 (D) 110
110. A resistance of 4 and a wire of length 5 m and resistance 3
5 are joined in series and connected to a cell of e.m.f.
2A
10V and internal resistance 1 . A Parallel combination of
two identical cells is balanced across 300 cm of the wire. D C
The e.m.f. E of each cell is :
4 10V,1 2
B
(A) +2 V (B) +1 V
3m
(C) –1 V (D) –2 V
E 5, 5m
116. Referring to the figure below, the effective resistance of
F the network is
E
G r r r
r
(A) 1.5 V (B) 3.0 V
(C) 0.67 V (D) 1.33 V r r r
111. Equivalent resistance between A and B will be
(A) 2 r (B) 4 r
3 3 5r
(C) 10 r (D)
2
117. Two resistances are joined in parallel whose resultant is
3 3
3 3 6
ohm. One of the resistance wire is broken and the
8
A 3 3 B effective resistance becomes 2 . Then the resistance in
(A) 2 ohm (B) 18 ohm ohm of the wire that got broken was
(C) 6 ohm (D) 3.6 ohm 3
(A) (B) 2
112. A wire has a resistance of 12 ohm. It is bent in the form of 5
equilateral triangle. The effective resistance between any 6
two corners of the triangle is (C) (D) 3
5
(A) 9 ohms (B) 12 ohms 118. Given three equal resistors, how many different
8 combination of all the three resistors can be made
(C) 6 ohms ohms (D)
3 (A) Six (B) Five
113. The effective resistance between the points A and B in (C) Four (D) Three
the figure is 119. Lamps used for household lighting are connected in
D (A) Series (B) Parallel
3 3 (C) Mixed circuit (D) None of the above
120. The equivalent resistance of resistors connected in series
6
A is always
C
(A) Equal to the mean of component resistors
3 (B) Less than the lowest of component resistors
3
(C) In between the lowest and the highest of component
B resistors
(A) 5 (B) 2
(D) Equal to sum of component resistors
(C) (D) 4
121. A cell of negligible resistance and e.m.f. 2 volts is 127. In the network of resistors shown in the adjoining figure,
connected to series combination of 2, 3 and 5 ohm. The the equivalent resistance between A and B is
potential difference in volts between the terminals of
3 ohm resistance will be 3 3 3
A B
(A) 0.6 (B)
3
(C) 3 (D) 6
122. Four wires of equal length and of resistances 10 ohms each 3 3
are connected in the form of a square. The equivalent (A) 54 ohm (B) 18 ohm
resistance between two opposite corners of the square is (C) 36 ohm (D) 9 ohm
(A) 10 ohm (B) 40 ohm 128. A wire is broken in four equal parts. A packet is formed by
keeping the four wires together. The resistance of the
10
(C) 20 ohm (D) ohm packet in comparison to the resistance of the wire will be
4 (A) Equal (B) One fourth
123. Two resistors are connected (A) in series (B) in parallel. 1
The equivalent resistance in the two cases are 9 ohm and (C) One eight (D)
2 ohm respectively. Then the resistances of the component 16th
129. Four resistances are connected in a circuit in the given
resistors are
figure. The electric current flowing through 4 ohm and
(A) 2 ohm and 7ohm (B) 3 ohm and 6 ohm
6 ohm resistance is respectively
(C) 3 ohm and 9 ohm (D) 5 ohm and 4 ohm 4 6
124. Resistors of 1, 2, 3 ohm are connected in the form of a
triangle. If a 1.5 volt cell of negligible internal resistance is
connected across 3 ohm resistor, the current flowing 4 6
through this resistance will be
(A) 0.25 amp (B) 0.5 amp
(C) 1.0 amp (D) 1.5 amp
20V
125. Resistances of 6 ohm each are connected in the manner (A) 2 amp and 4 amp (B) 1 amp and 2 amp
shown in adjoining figure. With the current 0.5 ampere as (C) 1 amp and 1 amp (D) 2 amp and 2 amp
shown in figure, the potential difference VP – VQ is 130. An infinite sequence of resistance is shown in the figure.
6 6 6 The resultant resistance between A and B will be, when
R1 = 1ohm and R2 = 2ohm
P 6 Q R1 R1 R1 R1 R1
A
0.5 A
6 6 R2 R2 R2 R2 R2
(A) 3.6 V (B) 6.0 V
B
(C) 3.0 V (D) 7.2 V (A) Infinity (B) 1
126. The equivalent resistance of the arrangement of (C) 2 (D) 1.5
resistances shown in adjoining figure between the points 131. In the figure, the value of resistors to be connected
A and B is between C and D so that the resistance of the entire
8 circuit between A and B does not change with the number
of elementary sets used is
16 20
R R R R C
A
16
A B R R R R R
9
6
B
18 R R R R D
(A) R (B) R 3 1
(A) 6 ohm (B) 8 ohm
(C) 16 ohm (D) 24 ohm (C) 3 R (D) R 3 1
132. In the figure shown, the total resistance between A and B is I
2 C 1 1 1 1 1 (A) ,R (B) I, 2R
3
A I I
(C) , 2R (D) ,R
8 8 3 2
137. Four wires AB, BC, CD, DA of resistance 4 ohm each and
a fifth wire BD of resistance 8 ohm are joined to form a
B 2 D 1 1 1 1 1 rectangle ABCD of which BD is a diagonal. The effective
(A) 12 (B) 4 resistance between the points A and B is
(C) 6 (D) 8
(A) 24 ohm (B) 16 ohm
133. The current from the battery in circuit diagram shown is
2 A 7 4 8
(C) ohm (D) ohm
3 3
15V 138. A battery of e.m.f. 10 V is connected to resistance as
shown in figure. The potential difference VA – VB between
0.5 6 1 the points A and B is
1 A 3
3
8 B 10
(A) 1 A (B) 2 A 3 B 1
(C) 1.5 A (D) 3 A
10V
134. In the given figure, when key K is opened, the reading of
the ammeter A will be
10V
(A) –2 V (B) 2 V
+ –
20
5 (C) 5 V (D) V
11
E A D
139. Three resistances, each of 1 ohm, are joined in parallel.
4 Three such combinations are put in series, then the
A
B C resultant resistance will be
K (A) 9 ohm (B) 3 ohm
(A) 50 A (B) 2 A
10 1
A (C) 1 ohm (D) ohm
(C) 0.5 A (D) 3
9
135. In the given circuit, the potential of the point E is 140. A student has 10 resistors of resistance ‘r’. The minimum
E 1 resistance made by him from given resistors is
A
+ –
D r
8V (A) 10 r (B)
10
r r
(C) (D)
C 100 5
B
5 141. The resistance of a galvanometer is 50 and the current
(A) Zero (B) –8 V required to give full scale deflection is 100 A. In order to
4 4 convert it into an ammeter, reading upto 10 A, it is
(C) V (D) V necessary to put a resistance of :–
3 3
136. If a resistance R 2 is connected in parallel with the (A) 5 × 10–3 in parallel
resistance R in the circuit shown, then possible value of (B) 5 × 10–4 in parallel
current through R and the possible value of R2 will be (C) 105 in series
(D) 99, 950 in series
R2 142. The resistivity of a wire depends on its :–
(A) length
R (B) area of cross section
I (C) shape
(D) material
+ – A
143. The conductivity of a super conductor is :– 152. What is the equivalent resistance between A and B
(A) infinite (B) very large
(C) very small (D) zero
144. Electromotive force of a cell is basically a C
(A) force (B) power A 2R 2R D R B
(C) work (D) current capacity
145. A battery of 10 V and internal resistance 0.5 is connected
across a variable resistance R. The value of R for which 2 3
the power delivered in its maximum state, is equal to :– (A) R (B) R
3 2
(A) 0.5 (B) 1
(C) 1.5 (D) 2 3
(C) R (D) 2 R
146. A galvanometer has a resistance G and current ia flowing 2
in it, produces full scale deflection. If S1 is the value of 153. The current in the following circuit is
shunt which converts it into an ammeter of range 0 – i and
S2 is the value of the shunt for the range 0 – 2i. Then the
ratio will be
2V 3 3
(A) 1 (B) 2
1 i ia 2i ia 3
(C) 2 2i i (D) i ia
a
147. Resistance of a galvanometer coil is 8 and 2 shunt 1 2
(A) A (B) A
resistance is connected with it. If main current is 1A then 8 9
the current flow through 2 resistance will be
(A) 0.2 A (B) 0.8 A 2
(C) A (D) 1 A
(C) 0.1 A (D) 0.4 A 3
148. Which of the following is scalar :– 154. What is the equivalent resistance of the circuit

(A) current (B) velocity 4V, 1 2


+ – 2
(C) force (D) acceleration A
2
149. In a Neon discharge tube 2.9 × 1018 Ne+ ions move to the
right each second, while 1.2 × 1018 electrons move to the
left per sec., electron charge is 1.6 × 10–19 C. The current 4
in the discharge tube :–
(A) 1 A towards right
(B) 0.66 A towards right V
(C) 0.66 A towards left (A) 6 (B) 7
(D) zero (C) 8 (D) 9
150. Constanton wire is used in making standard resistances,
155. 10 wires (same length, same area, same material) are
because its :–
connected in parallel and each has 1 resistance, then
(A) specific resistance is low
the equivalent resistance will be
(B) density is high
(A) 10 (B) 1
(C) temperature coeff. of resistance is negligible
(C) 0.1 (D) 0.001
(D) melting point is high
151. What is the equivalent resistance between A and B in the 156. The equivalent resistance of the circuit shown in the
figure below if R = 3 figure is
A B 2
2 2
R R
R 2

R R
(A) 9 (B) 12
(A) 8 (B) 6
(C) 15 (D) None of these
(C) 5 (D) 4
157. In the given figure, the equivalent resistance between the 162. In the given figure, potential difference between A and B is
points A and B is
R2 = 4
10K D
A
30 V
R1 = 2 R4 = 2
10K 10K
R3 = 4
B
(A) 8 (B) 6 (A) 0 (B) 5 volt
(C) 4 (D) 2 (C) 10 volt (D) 15 volt
158. An infinite ladder network is arranged with resistances R 163. If each resistance in the figure is of 9 then reading of
and 2 R as shown. The effective resistance between ammeter is
terminals A and B is
R R R +
A
9V
–
2R 2R 2R

B A
(A) (B) R
(C) 2 R (D) 3 R (A) 5 A (B) 8 A
159. If all the resistors shown have the value 2 ohm each, the (C) 2 A (D) 9 A
equivalent resistance over AB is
164. Four resistances 10 , 5 , 7 and 3 are connected so
A B that they form the sides of a rectangle AB, BC, CD and DA
respectively. Another resistance of 10 is connected
across the diagonal AC. The equivalent resistance
between A and B is
(A) 2 (B) 5
(A) 2 ohm (B) 4 ohm
(C) 7 (D) 10
2 2
(C) 1 ohm (D) 2 ohm 165. Two wires of equal diameters, of resistivities 1 and 2
3 3
and lengths l1 and l2, respectively, are joined in series.
160. A battery of emf 10 V and internal resistance 3 is
The equivalent resistivity of the combination is
connected to a resistor as shown in the figure. If the
current in the circuit is 0.5 A. then the resistance of the 1l1 2l 2 1l2 2 l1
(A) l1 l2 (B) l1 l2
resistor will be

1l2 2 l1 1l1 2l 2
(C) l1 l2 (D) l1 l2
166. Four resistances of 100 each are connected in the form
R of square. Then, the effective resistance along the diagonal
points is
(A) 19 (B) 17 (A) 200 (B) 400
(C) 10 (D) 12 (C) 100 (D) 150
161. The potential drop across the 3 resistor is 167. Equivalent resistance between the points A and B is
3 (in )

4
A 1 1 1 1 1 B
6

1 1
3V (A) (B) 1
5 4
(A) 1 V (B) 1.5 V
1 1
(C) 2 V (D) 3 V (C) 2 (D) 3
3 2
168. Two wires of the same material and equal length are joined 174. Two resistance wires on joining in parallel the resultant
in parallel combination. If one of them has half the
6
thickness of the other and the thinner wire has a resistance resistance is ohms . One of the wire breaks, the effective
5
of 8 ohms, the resistance of the combination is equal to
resistance is 2 ohms. The resistance of the broken wire is
5 8
(A) ohms (B) ohms 3
8 5 (A) ohm (B) 2 ohm
5
3 8
(C) ohms (D) ohms 6
8 3 (C) ohm (D) 3 ohm
169. In the circuit shown here, what is the value of the unknown 5
resistor R so that the total resistance of the circuit between 175. In the circuit, the potential difference across PQ will be
points P and Q is also equal to R nearest to
10 100

3 48 V
P Q 80
3 R
100 Q
20
(A) 3 ohms (B) 39 ohms P
(C) 69 ohms (D) 10 ohms (A) 9.6 V (B) 6.6 V
(C) 4.8 V (D) 3.2 V
170. A uniform wire of resistance 9 is cut into 3 equal parts.
176. Three resistors are connected to form the sides of a triangle
They are connected in the form of equilateral triangle ABC.
ABC, the resistance of the sides AB, BC and CA are
A cell of e.m.f. 2 V and negligible internal resistance is
40 ohms, 60 ohms and 100 ohms respectively. The effective
connected across B and C. Potential difference across
resistance between the points A and B in ohms will be
AB is
(A) 1 V (B) 2 V (A) 32 (B) 64
(C) 3 V (D) 0.5 V (C) 50 (D) 200
177. Find the equivalent resistance across AB
171. The resistors of resistances 2 , 4 and 8 are connected
in parallel, then the equivalent resistance of the A
combination will be 2
2
8 7
(A) (B) 2
7 8
7 4 2 2
(C) (D)
4 9
B
172. Effective resistance between A and B is
(A) 1 (B) 2
(C) 3 (D) 4
178. The equivalent resistance between x and y in the circuit
shown is
A B

(A) 15 (B) 5
x y
5
(C) (D) 20
2
12
173. The effective resistance of two resistors in parallel is
.
7
If one of the resistors is disconnected the resistance (A) 10
becomes 4 . The resistance of the other resistor is (B) 40
(A) 4 (B) 3 (C) 20
12 7 5
(C) (D) (D)
7 12 2
179. The equivalent resistance between the points P and Q of 20V,1.5
the circuit given is
3 P 2

R R R 2 3
P Q Q
(A) zero (B) 4 volt (Vp > VQ)
(C) 4 volt (VQ > VP) (D) 2.5 volt (VQ > VP)
R R 187. Two cells, each of e.m.f. E and internal resistance r, are
(A) (B)
4 3 connected in parallel across a resistor R. The power
(C) 4 R (D) 2 R dissipated in the resistor is maximum if :
180. Two wires of the same dimensions but resistivities (A) R = r (B) R = 2r
1 and 2are connected in series. The equivalent 3r r
resistivity of the combination is (C) R (D) R =
2 2
188. A cylindrical wire is stretched such that its length gets
1 2
(A) 1 + 2 (B) doubled but volume remains same, the resistance of wire
2
becomes :–
(C) (D) 2( + (A) four times (B) remains same
1 2 1 2)
(C) half (D) becomes double
181. A car battery of e.m.f. 12V and internal resistance 189. In a network as shown in the figure the potential difference
5 × 10–2 , receives a current of 60 A from an external across the resistance 2R is (the cell has an emf of E and
source, then terminal potential difference of battery is :– has no internal resistance) :
(A) 32 V (B) 10 V
(C) 15 V (D) 50 V 4R
182. A copper wire stretched so as to make it 0.1% longer. The
R
percentage increase in the resistance of the wire is :–
2R
(A) 1.0 (B) 2.0
(C) 0.1 (D) 0.2 E
183. 10,000 electrons are passing per minute through a tube of
radius 1cm. The resulting current is : 4E
(A) 2 E (B)
(A) 10000 A (B) 0.25 × 10–16 A 7
–9
(C) 10 A (D) 0.5 × 10–19 A E
(C) (D) E
184. Seven resistances are connected as shown in the figure. 7
The equivalent resistance between A and B is :– 190. There are 8.4 × 1022 free electrons per cm3 in copper. The
current in the wire is 0.21 A (e = 1.6 × 10–19 C). Then the
10
drifts velocity of electrons in a copper wire of 1 mm2 cross
A 3 B section, will be :–
(A) 2.12 × 10–5 m/s (B) 0.78 × 10–5 m/s
10 –5
5 8 6 6 (C) 1.56 × 10 m/s (D) none of these
191. In the figure given the value of X resistance will be, when
the p.d. between B and D is zero
(A) 3 (B) 4
B
(C) 4.5 (D) 5 X
6
185. At what temperature will the resistance of a copper wire
become three times its value at 0° C [Temperature 3
8
coefficient of resistance for copper = 4 × 10–3 per °C] :– 15
A C
(A) 400° C (B) 450° C 4
15 6
(C) 500° C (D) 500° C
186. In the circuit shown below, the internal resistance of the 4
6 4
battery is 1.5 and VP and VQ are the potentials at P and
D
Q respectively, what is the potential difference between (A) 4 ohm (B) 6 ohm
the points P and Q :– (C) 8 ohm (D) 9 ohm
192. The effective resistance between points A and B is 196. Five resistances are connected as shown in the figure.
The effective resistance between the points A and B is

A B
D
A B

(A) 10
10 20
(B) 20 (A) (B)
3 3
(C) 40 (C) 15 (D) 6
(D) None of the above three values 197. In the given figure, when galvanometer shows no
193. Five resistors of given values are connected together as deflection, the current (in ampere) flowing through 5
shown in the figure. The current in the arm BD will be resistance will be
B
2
R R
2.1A
G
A 4A C

R R 5
(A) 0.5 (B) 0.6
D
(C) 0.9 (D) 1.5
198. In the Wheatstone’s bridge shown, P = 2 , Q = 3 , and
S = 8 . In order to obtain balance, shunt resistance across
‘S’ must be
(A) Half the current in the arm ABC
(B) Zero P Q
(C) Twice the current in the arm ABC
(D) Four times the current in the arm ABC
194. In the network shown in the figure, each of the resistance
is equal to 2 . The resistance between the points A and B S R
is

(A) 2 (B) 3
(C) 6 (D) 8
A 199. Five equal resistances each of value R are connected in a
form shown alongside. The equivalent resistance of the
B
network
(A) 1 (B) 4 B
(C) 3 (D) 2 R R
195. In the arrangement of resistances shown below, the
effective resistance between points A and B is R
A C
5 10 15
P R R

A 10 10 B D
(A) Between the points B and D is R
Q R
(B) Between the points B and D is
10 20 30 2
(A) 20 (B) 30 (C) Between the points A and C is R
(C) 90 (D) 110 R
(D) Between the points A and C is
2
200. In the circuit shown below the resistance of the 14 3
galvanometer is 20 . In which case of the following (A) (B)
3 14
alternatives are the currents arranged strictly in the
9 14
decreasing order (C) (D)
14 9
i1 204. In a typical Wheatstone network, the resistances in cyclic
10 100 order are A = 10 , B = 5 , C = 4 and D = 4 for the
ig bridge to be balanced
i2
G
A = 10 B=5

2 20
i
D=4 C=4
2V 0
(A) i, i1, i2, ig (B) i, i2, i1, ig (A) 10 should be connected in parallel with A
(C) i, i2, ig, i1 (D) i, i1, ig, i2 (B) 10 should be connected in series with A
201. Potential difference between the points P and Q in the (C) 5 should be connected in series with B
electric circuit shown is (D) 5 should be connected in parallel with B
i = 1.5 A 205. In the circuit shown in figure, the current drawn from the
P battery is 4A. If 10 resistor is replaced by 20 resistor,
then current drawn from the circuit will be
RA = 2 RB = 4

RD = 6 RC = 12
4A
Q
+ –
(A) 4.5 V (B) 1.2 V
(C) 2.4 V (D) 2.88 V (A) 1 A (B) 2 A
(C) 3 A (D) 0 A
202. The current between B and D in the given figure is
206. Calculate the equivalent resistance between A and B
B
3 3 3
30 30
B
A 60 C A 3 3
l
30 30 3 3 3
D 9
(A) (B) 3
30V 2
5
(C) 6 (D)
(A) 1 amp (B) 2 amp 3
207. The equivalent resistance between P and Q in the given
(C) Zero (D) 0.5 amp figure, is
203. In the given figure, equivalent resistance between A and
B will be

P Q

A B
(A) 50 (B) 40
(C) 30 (D) 20
208. If each of the resistance of the network shown in the 212. For what value of unknown resistance X, the potential
figure is R, the equivalent resistance between A and B is difference between B and D will be zero in the circuit
shown in the figure
B
R R R 1
12
R A 1
B C
R A
1
X
(A) 5 R (B) 3 R
R 6 1
(C) R (D)
2 D
209. The equivalent resistance of the following diagram A and
B is
(A) 4 (B) 6
(C) 2 (D) 5
213. Which arrangement of four identical resistances should
be used to draw maximum energy from a cell of voltage V
A B
(A)

2
(A) (B) 9
3
(C) 6 (D) None of these (B)
210. Thirteen resistances each of resistance R ohm are
connected in the circuit as shown in the figure below.
The effective resistance between A and B is
R R (C)

R R R R
(D)
A R B
F 214. An unknown resistance R1 is connected in series with a
R
R R resistance of 10 . This combinations is connected to
one gap of a metre bridge while a resistance R 2 is
R R connected in the other gap. The balance point is at 50 cm.
4R Now, when the 10 resistance is removed the balance
(A) 2R (B) point shifts to 40 cm. The value of R1 is (in ohm)
3
(A) 60 (B) 40
2R
(C) (D) R (C) 20 (D) 10
3
211. In a Wheatstone’s bridge all the four arms have equal 215. A wire has a resistance of 6 . It is cut into two parts and
resistance R. If the resistance of the galvanometer arm is both half values are connected in parallel. The new
also R, the equivalent resistance of the combination as resistance is ....
seen by the battery is (A) 12 (B) 1.5
R (C) 3 (D) 6
(A) (B) R
2
R
(C) 2 R (D)
4
216. Six equal resistances are connected between points P, Q 221. In a typical Wheatstone network the resistance in cyclic
and R as shown in the figure. Then the net resistance will order are A = 10 , B = 5 , C = 4 and D = 4 for the
be maximum between bridge to be balanced.
P

D C
Q R
(A) 10 should be connected in parallel with A
(B) 10 should be connected in series with A
(A) P and Q (B) Q and R
(C) 5 should be connected in series with B
(C) P and R (D) Any two points
(D) 5 should be connected in parallel with B
217. The total current supplied to the circuit by the battery is
(A) a, b (B) b, c
(C) a, c (D) all
2
6V 6 222. In the circuit shown here, what is the value of the unknown
3 resistor R so that the total resistance of the circuit between
points ‘P’ and ‘Q’ is also equal to R :–
1.5
10
P 3 Q
(A) 1 A (B) 2 A
3 R
(C) 4 A (D) 6 A
218. An electric current is passed through a circuit containing
two wires of the same material, connected in parallel. If (A) 3 (B) 39
4
the lengths and radii of the wires are in the ratio of and (C) 69 (D) 10
3
2 223. A battery has e.m.f. 4V and internal resistance ‘r’. When
, then the ratio of the currents passing through the wire
3 this battery is connected to an external resistance of
will be 2 ohms, a current of 1 amp. flows in the circuit. How much
1 current will flow if the terminals of the battery are connected
(A) 3 (B) directly :–
3
(A) 1 A (B) 2 A
8
(C) (D) 2 (C) 4 A (D) infinite
9
219. If a rod has resistance 4 W and if rod is turned as half 224. In the circuit shown here, E1 = E2 = E3 = 2V and R1 = R2
cycle then the resistance along diameter = 4 ohms. The current flowing between points A and B
through battery E2 is
(A) 1.56 (B) 2.44
R 2V
(C) 4 (D) 2

220. If three resistors of resistance 2 , 4 and 5 are 40cm


connected in parallel then the total resistance of the A B
combination will be
E
20 19
(A) (B)
19 20
(A) zero (B) 2 amp from A to B
19 10
(C) (D) (C) 2 amp from B to A (D) none of the above
10 19
225. AB is a potentiometer wire of length 100 cm and its 230. If specific resistance of a potentiometer wire is 10–7 m
resistance is 10 ohm. It is connected in series with a and current flow through it is 0.1 amp., cross–sectional
resistance R = 40 ohm and a battery of e.m.f. 2V and area of wire is 10–6 m2 then potential gradient will be :–
negligible internal resistance. If a source of unknown e.m.f. (A) 10–2 volt/m (B) 10–4 volt/m
E is balanced by 40 cm length of the potentiometer wire, –6
(C) 10 volt/m (D) 10–8 volt/m
the value of E is: 231. The terminal potential difference of a cell is greater than
R 2V its e.m.f. when it is
(A) Being discharged
40cm (B) In open circuit
A B (C) Being charged
(D) Being either charged or discharged
E 232. In the circuit shown, potential difference between X and
Y will be
(A) 0.8 V (B) 1.6 V 40 X Y
(C) 0.08 V (D) 0.16 V
226. Three resistance of values 2 , 3 and 6 are to be
connected to produce an effective resistance of 4 . This
can be done by connecting : 20
(A) 3 resistance in series with the parallel combination 120V
of 2 and 6 (A) Zero (B) 20 V
(B) 6 resistance in series with the parallel combination (C) 60 V (D) 120 V
of 2 and 3 233. In the above question, potential difference across the
(C) 2 resistance in series with the parallel combination 40 resistance will be
of 3 and 6 (A) Zero (B) 80 V
(C) 40 V (D) 120 V
(D) 2 resistance in parallel with the parallel combination
234. In the circuit shown, A and V are ideal ammeter and
of 3 and 6
voltmeter respectively. Reading of the voltmeter will be
227. A battery of electro motive force E is connected in series
2V
with a resistance R and a voltmeter. An ammeter is
connected in parallel with the battery
(A) neither the ammeter nor the voltmeter will be damaged.
A V
(B) both ammeter and voltmeter are likely to be damaged.
(C) only voltmeter is likely to be damaged. 1 1
(D) only ammeter is likely to be damaged.
228. Two resistance wires on joining in parallel the resultant (A) 2 V (B) 1 V
resistance is ohm. One of the wire breaks, the effective (C) 0.5 V (D) Zero
resistance is 2 ohm. The resistance of the broken wire 235. When a resistance of 2ohm is connected across the
was :– terminals of a cell, the current is 0.5 amperes. When the
resistance is increased to 5 ohm, the current is
3
(A) ohm (B) 2 ohm 0.25 amperes. The internal resistance of the cell is
5
(A) 0.5 ohm (B) 0.1 ohm
6 (C) 1.5 ohm (D) 2.0 ohm
(C) ohm (D) 3 ohm
5 236. The terminal potential difference of a cell when short-
229. The temperature coefficient of resistance of a wire is circuited is (E = E.M.F. of the cell)
0.00125 per degree celcius. At 300 K its resistance is E
1 ohm. The resistance of the wire will be 2 ohm at following (A) E (B)
2
temperature :–
E
(A) 1154 K (B) 1127 K (C) Zero (D)
3
(C) 600 K (D) 1400 K
237. A primary cell has an e.m.f. of 1.5 volts, when short- 246. A cell of internal resistance r is connected to an external
circuited it gives a current of 3 amperes. The internal resistance R. The current will be maximum in R, if
resistance of the cell is (A) R = r (B) R < r
(A) 4.5 ohm (B) 2 ohm
r
1 (C) R > r (D) R
(C) 0.5 ohm (D) ohm 2
4.5 247. To get the maximum current from a parallel combination of
238. A 50V battery is connected across a 10 ohm resistor. The n identical cells each of internal resistance r in an external
current is 4.5 amperes. The internal resistance of the resistance R, when
battery is (A) R > > r (B) R < < r
(A) Zero (B) 0.5 ohm (C) R = r (D) None of these
(C) 1.1 ohm (D) 5.0 ohm
248. Two identical cells send the same current in 2 resistance,
239. The potential difference in open circuit for a cell is whether connected in series or in parallel. The internal
2.2 volts. When a 4 ohm resistor is connected between its resistance of the cell should be
two electrodes the potential difference becomes 2 volts.
(A) 1 (B) 2
The internal resistance of the cell will be
(A) 1 ohm (B) 0.2 ohm 1
(C) (D) 2.5
(C) 2.5 ohm (D) 0.4 ohm 2
240. A new flashlight cell of e.m.f. 1.5 volts gives a current of 249. The internal resistances of two cells shown are 0.1 and
15 amps, when connected directly to an ammeter of 0.3 . If R = 0.2 , the potential difference across the cell
resistance0.04 . The internal resistance of cell is 2V, 0.1 2V, 0.3
(A) 0.04 (B) 0.06
(C) 0.10 (D) 10 A B
241. A cell whose e.m.f. is 2 V and internal resistance is 0.1 , is
connected with a resistance of 3.9 The voltage across
the cell terminal will be 0.2
(A) 0.50 V (B) 1.90 V
(C) 1.95 V (D) 2.00 V (A) B will be zero
242. The reading of a high resistance voltmeter when a cell is (B) A will be zero
connected across it is 2.2 V. When the terminals of the cell (C) A and B will be 2V
are also connected to a resistance of 5 the voltmeter (D) A will be > 2V and B will be < 2V
reading drops to 1.8 V. Find the internal resistance of the 250. A torch battery consisting of two cells of 1.45 volts and
cell an internal resistance 0.15 , each cell sending currents
(A) 1.2 (B) 1.3 through the filament of the lamps having resistance
(C) 1.1 (D) 1.4 1.5ohms. The value of current will be
243. When cells are connected in parallel, then (A) 16.11 amp (B) 1.611 amp
(A) The current decreases (C) 0.1611 amp (D) 2.6 amp
(B) The current increases 251. The electromotive force of a primary cell is 2 volts. When
(C) The e.m.f. increases it is short-circuited it gives a current of 4 amperes. Its
(D) The e.m.f. decreases internal resistance in ohms is
244. The internal resistance of a cell depends on (A) 0.5 (B) 5.0
(A) The distance between the plates (C) 2.0 (D) 8.0
(B) The area of the plates immersed
252. The figure shows a network of currents. The magnitude
(C) The concentration of the electrolyte of currents is shown here. The current i will be
(D) All the above
245. n identical cells each of e.m.f. E and internal resistance r 15A
3A
are connected in series. An external resistance R is
connected in series to this combination. The current 8A
through R is
nE nE
(A) (B)
R nr nR r 5A
E nE (A) 3 A (B) 13 A
(C) (D)
R nr R r (C) 23 A (D) – 3 A
253. A battery of e.m.f. E and internal resistance r is connected 258. The current in the arm CD of the circuit will be
to a variable resistor R as shown here. Which one of the B
following is true
i2
E r i1
O A
i3

R C
D
(A) i 1 + i2 (B) i2 + i3
(A) Potential difference across the terminals of the battery (C) i1 + i3 (D) i1 – i2 + i3
is maximum when R = r 259. When a resistance of 2 ohm is connected across the
(B) Power delivered to the resistor is maximum when terminals of a cell, the current is 0.5 A. When the resistance
R=r is increased to 5 ohm, the current is 0.25 A. The e.m.f. of
the cell is
(C) Current in the circuit is maximum when R = r
(A) 1.0 V (B) 1.5 V
(D) Current in the circuit is maximum when R > > r
(C) 2.0 V (D) 2.5 V
254. A dry cell has an e.m.f. of 1.5 V and an internal resistance 260. Two non-ideal identical batteries are connected in parallel.
of 0.05 . The maximum current obtainable from this cell Consider the following statements
for a very short time interval is (i) The equivalent e.m.f. is smaller than either of the two
(A) 30 A (B) 300 A e.m.f.s
(C) 3 A (D) 0.3 A (ii) The equivalent internal resistance is smaller than
either of the two internal resistances
255. Consider the circuit given here with the following
(A) Both (i) and (ii) are correct
parameters
(B) (i) is correct but (ii) is wrong
E.M.F. of the cell = 12 V. Internal resistance of the cell
(C) (ii) is correct but (i) is wrong
= 2 .. Resistance R = 2
E (D) Both (i) and (ii) are wrong
261. To convert a galvanometer into a voltmeter one should
connect a :–
(A) high resistance in series with galvanometer.
R (B) low resistance in series with galvanometer.
(C) high resistance in parallel with galvanometer.
Which one of the following statements is true
(D) low resistance in parallel with galvanometer.
(A) Rate of energy loss in the source is = 8 W 262. N identical cells whether joined together in series or in
(B) Rate of energy conversion in the source is 16 W parallel, give the same current, when connected to an
external resistance of ‘R’. The internal resistance of each
(C) Power output in is = 8 W
cell is :–
(D) Potential drop across R is = 16 V (A) r = nR (B) r = R
256. A current of two amperes is flowing through a cell of (C) r = (D) r = n2R
e.m.f. 5 volts and internal resistance 0.5 ohm from negative 263. The potential difference between the points A and B in
the following circuit shown in the figure:–
to positive electrode. If the potential of negative electrode
2 4
is 10V, the potential of positive electrode will be (A) volt (B) volt
3 5
(A) 5 V (B) 14 V 8
(C) volt (D) 2 volt
(C) 15 V (D) 16 V 9
264. In the potentiometer experiment if deflection in
257. 100 cells each of e.m.f. 5 V and internal resistance 1 ohm
galvanometer is measured zero then current will become
are to be arranged so as to produce maximum current in a
zero in:–
25 ohms resistance. Each row is to contain equal number
of cells. The number of rows should be (A) potentiometer wire
(B) galvanometer circuit
(A) 2 (B) 4 (C) main circuit
(C) 5 (D) 10 (D) cell
265. Length of a potentiometer wire is kept more and uniform 271. Four identical cells each having an electromotive force
to achive :– (e.m.f.) of 12V, are connected in parallel. The resultant
(A) uniform and more potential gradient electromotive force (e.m.f.) of the combination is
(B) non-uniform and more potential gradient (A) 48 V (B) 12 V
(C) uniform and less potential gradient (C) 4 V (D) 3 V
(D) non-uniform and less potential gradient 272. Electromotive force is the force, which is able to maintain
a constant
266. The balancing length is obtained at 78.4 cm length while
(A) Current (B) Resistance
measuring the potential difference at the ends of a
(C) Power (D) Potential difference
resistance wire. When same potential difference is
273. A cell of emf 6 V and resistance 0.5 ohm is short circuited.
measured with a voltmeter, it shows 1.20 volt. If a standard
The current in the cell is
cell of emf 1.018 volt is balanced at 63.2 cm, the error in
voltmeter reading in volt will be :– (A) 3 amp (B) 12 amp
(C) 24 amp (D) 6 amp
(A) –0.06 (B) +0.06
274. A storage cell is charged by 5 amp D.C. for 18 hours. Its
(C) –0.03 (D) +0.03
strength after charging will be
267. Consider four circuits shown in the figure below. In which (A) 18 AH (B) 5 AH
circuit power dissipated is greater (Neglect the internal (C) 90 AH (D) 15 AH
resistance of the power supply) :–
275. A battery having e.m.f. 5V and internal resistance 0.5 is
connected with a resistance of 4.5 then the voltage at
R the terminals of battery is
R R
(A) E (B) E (A) 4.5 V (B) 4 V
R (C) 0 V (D) 2 V
276. In the given circuit the current I1 is
R 30
R R
R
(C) E (D) E I1
R 40
R
I2 I3
268. The equivalent resistance across AB is :– 40V
40
A
80V
2 2
(A) 0.4 A (B) – 0.4 A
2 (C) 0.8 A (D) – 0.8 A
2 2 277. The internal resistance of a cell of e.m.f. 12V is 5 × 10–2 .
It is connected across an unknown resistance. Voltage
B
across the cell, when a current of 60 A is drawn from it, is
(A) 1 (B) 2 (A) 15 V (B) 12 V
(C) 3 (D) 4 (C) 9 V (D) 6 V
269. There are three voltmeters of the same range but of 278. The current in the given circuit is
resistance 10000 , 8000 and 4000 respectively. The 10 5V
best voltmeter among these is the one whose resistance
is :– A B
(A) 10000
(B) 8000 20
(C) 4000
2V
(D) all are equally good (A) 0.1 A (B) 0.2 A
270. When a voltmeter and an ammeter are connected (C) 0.3 A (D) 0.4 A
respectively across the terminals of a cell, measures 5 V 279. A current of 2.0 ampere passes through a cell of e.m.f.
and 10 A. Now only a resistance of 2 is connected 1.5 volts having internal resistance of 0.15 ohm. The
across the terminal of the cell. The current flowing through
potential difference measured, in volts, across both the
this resistance is :
ends of the cell will be
(A) 7.5 A (B) 5.0 A
(C) 2.5 A (D) 2.0 A (A) 1.35 (B) 1.50
(C) 1.00 (D) 1.20
280. A battery has e.m.f. 4 V and internal resistance r. When 286. The emf of a battery is 2 V and its internal resistance is
this battery is connected to an external resistance of 0.5 . The maximum power which it can deliver to any
2 ohms, a current of 1 amp. flows in the circuit. How much external circuit will be
current will flow if the terminals of the battery are connected (A) 8 Watt (B) 4 Watt
directly (C) 2 Watt (D) None of the above
(A) 1 amp (B) 2 amp 287. Kirchoff’s I law and II law of current, proves the
(C) 4 amp (D) Infinite (A) Conservation of charge and energy
281. Two batteries A and B each of e.m.f. 2 V are connected in (B) Conservation of current and energy
series to an external resistance R = 1 ohm. If the internal (C) Conservation of mass and charge
resistance of battery A is 1.9 ohms and that of B is (D) None of these
0.9 ohm, what is the potential difference between the 288. In the circuit, the reading of the ammeter is (assume internal
terminals of battery A resistance of the battery be zero)

A B
A
4 10V
5
R

(A) 2 V (B) 3.8 V 0 10


(A) A (B) A
(C) Zero (D) None of the above 29 9
282. When a resistor of 11 is connected in series with an
(C) A (D) 2 A
electric cell, the current flowing in it is 0.5 A. Instead, 3
when a resistor of 5 is connected to the same electric 289. In the above question, if the internal resistance of the
cell in series, the current increases by 0.4 A. The internal battery is 1 ohm, then what is the reading of ammeter
resistance of the cell is
40
(A) 1.5 (B) 2 (A) A (B) A
3 29
(C) 2.5 (D) 3.5
10
283. The internal resistance of a cell is the resistance of (C) A (D) 1 A
9
(A) Electrodes of the cell
290. Eels are able to generate current with biological cells called
(B) Vessel of the cell
electroplaques. The electroplaques in an eel are arranged
(C) Electrolyte used in the cell in 100 rows, each row stretching horizontally along the
(D) Material used in the cell body of the fish containing 5000 electroplaques. The
284. How much work is required to carry a 6 C charge from arrangement is suggestively shown below. Each
the negative terminal to the positive terminal of a 9 V electroplaques has an emf of 0.15 V and internal resistance
battery of 0.25
(A) 54 × 10–3 J (B) 54 × 10–6 J 0.15 V 0.25
+ – + – + –
(C) 54 × 10–9 J (D) 54 × 10–12J
285. Consider four circuits shown in the figure below. In which + – + – + –
circuit power dissipated is greatest (Neglect the internal
resistance of the power supply) 5000 electroplaques per
100

R
E R R + – + – + –
(A) (B) E
R

500
The water surrounding the eel completes a circuit between
R R R the head and its tail. If the water surrounding it has a
E resistance of 500 , the current an eel can produce in
R
(C) (D) E water is about
R R
(A) 1.5 A (B) 3.0 A
(C) 15 A (D) 30 A
291. Thirteen resistance each of resistance R ohm are connected 297. The length of a given cylindrical wire is increased by 100%.
in the circuit as shown in the figure. The effective Due to the consequent decrease in diameter the change
resistance between A and B is :– in the resistance of the wire will be :–
R R (A) 300% (B) 200%
(C) 100% (D) 50%
R R R 298. The length of a wire of a potentiometer is 100 cm, and the
R
emf of its standard cell is E volt. It is employed to measure
the e.m.f. of a battery whose internal resistance is 0.5 . If
A R B
the balance point is obtained at = 30 cm from the positive
R end, the e.m.f. of the battery is :–
R R R
30 E 30 E
(A) (B)
100 100.5
R R
30 E 30 E 0.5
4R (C) 100 0.5 (D)
(A) (B) 2R 100
3
299. Three unequal resistors in parallel are equivalent to a
2R resistant 1 ohm. If two of them are in the ratio 1 : 2 and if no
(C) R (D)
3 resistance value is fractional, the largest of the three
292. A group of N cells whose emf varies directly with the resistance in ohms is :–
internal resistance as per the equation EN = 1.5 rN are (A) 4 (B) 6
connected as shown in the figure. The current I in the
(C) 8 (D) 12
circuit is
300. Express which of the following setups can be used to
1
r2 verify Ohm’s law :–

N r1 2
Rh A
rN r3 V
r4 3 (A) E

4 V
(A) 5.1 A (B) 0.51 A
(C) 1.5 A (D) 0.15 A (B) E A
Rh
293. A galvanometer has resistance 36 . If a shunt of 4 is
added with this, then fraction of current that passes
through galvanometer is : A
Rh
1 1
(A) (B) (C) V
4 9 E
1 1
(C) (D)
10 40
294. If 106 electrons/s are flowing through an area of cross Rh V
section of 10–4 m2 then the current will be :– (D) A E
(A) 1.6 × 10–7 A (B) 1.6 × 10–13 A
–6
(C) 1 × 10 A (D) 1 × 102 A 301. For measurement of potential difference, potentiometer is
295. The terminal voltage is when a current of 2A is flowing preferred in comparison to voltmeter because
through 2 resistance, then the internal resistance of cell (A) Potentiometer is more sensitive than voltmeter
is :–
(B) The resistance of potentiometer is less than voltmeter
(A) 1 (B) 2
(C) Potentiometer is cheaper than voltmeter
(C) 3 (D) 4
(D) Potentiometer does not take current from the circuit
296. A 1 voltmeter has range 1V. Find the additional resistance
302. In order to pass 10% of main current through a moving
which has to join in series in voltmeter to increase the
coil galvanometer of 99 ohm, the resistance of the required
range of voltmeter to 100 V :–
shunt is
(A) 10 (B)
(A) 9.9 (B) 10
(C) 99 (D) 100
(C) 11 (D) 9
303. An ammeter of 5 ohm resistance can read 5 mA. If it is to 312. A potentiometer is an ideal device of measuring potential
be used to read 100 volts, how much resistance is to be difference because
connected in series (A) It uses a sensitive galvanometer
(A) 19.9995 (B) 199.995 (B) It does not disturb the potential difference it measures
(C) 1999.95 (D) 19995 (C) It is an elaborate arrangement
304. The potential gradient along the length of a uniform wire (D) It has a long wire hence heat developed is quickly
is 10 volt/metre. B and C are the two points at 30 cm and radiated
60 cm point on a meter scale fitted along the wire. The
313. A battery of 6 volts is connected to the terminals of a
potential difference between B and C will be
three metre long wire of uniform thickness and resistance
(A) 3 volt (B) 0.4 volt of the order of 100 . The difference of potential between
(C) 7 volt (D) 4 volt two points separated by 50cm on the wire will be
305. 100 mA current gives a full scale deflection in a (A) 1 V (B) 1.5 V
galvanometer of 2 resistance. The resistance connected
(C) 2 V (D) 3 V
with the galvanometer to convert it into a voltmeter to
314. A galvanometer of 10 ohm resistance gives full scale
measure 5V is
deflection with 0.01 ampere of current. It is to be converted
(A) 98 (B) 52
into an ammeter for measuring 10 ampere current. The
(C) 50 (D) 48
value of shunt resistance required will be
306. When a 12 resistor is connected with a moving coil
galvanometer then its deflection reduces from 50 divisions 10
(A) oh (B) 0.1 oh
to 10 divisions. The resistance of the galvanometer is 999
(A) 24 (B) 36 (C) 0.5 oh (D) 1.0 oh
(C) 48 (D) 60 315. A potentiometer is used for the comparison of e.m.f. of
307. A galvanometer can be used as a voltmeter by connecting two cells E1 and E2. For cell E1 the no deflection point is
a obtained at 20cm and for E2 the no deflection point is
(A) High resistance in series obtained at 30cm. The ratio of their e.m.f.’s will be
(B) Low resistance in series 2 1
(C) High resistance in parallel (A) (B)
3 2
(D) Low resistance in parallel
(C) 1 (D) 2
308. The tangent galvanometer, when connected in series with
316. Potential gradient is defined as
a standard resistance can be used as
(A) Fall of potential per unit length of the wire
(A) An ammeter
(B) A voltmeter (B) Fall of potential per unit area of the wire
(C) A wattmeter (C) Fall of potential between two ends of the wire
(D) Both an ammeter and a voltmeter (D) Potential at any one end of the wire
309. In Wheatstone’s bridge P = 9 ohm, Q = 11 ohm, R = 4ohm 317. In an experiment of meter bridge, a null point is obtained
and S = 6 ohm. How much resistance must be put in parallel at the centre of the bridge wire. When a resistance of
to the resistance S to balance the bridge 10 is connected in one gap, the value of resistance in
other gap is
44
(A) 24 ohm (B) ohm (A) 10 (B) 5
9
(C) 26.4 ohm (D) 18.7 ohm 1
310. A Daniel cell is balanced on 125cm length of a (C) (D) 500
5
potentiometer wire. Now the cell is short-circuited by a 318. If the length of potentiometer wire is increased, then the
resistance 2 ohm and the balance is obtained at 100cm. length of the previously obtained balance point will
The internal resistance of the Daniel cell is
(A) Increase (B) Decrease
(A) 0.5 ohm (B) 1.5 ohm
(C) Remain unchanged (D) Become two times
4
(C) 1.25 ohm (D) ohm 319. In potentiometer a balance point is obtained, when
5
311. Sensitivity of potentiometer can be increased by (A) The e.m.f. of the battery becomes equal to the e.m.f.
of the experimental cell
(A) Increasing the e.m.f. of the cell
(B) Increasing the length of the potentiometer wire (B) The p.d. of the wire between the +ve end to
jockey becomes equal to the e.m.f. of the experimental
(C) Decreasing the length of the potentiometer wire
cell
(D) None of the above
(C) The p.d. of the wire between +ve point and jockey (A) Equal to 5
becomes equal to the e.m.f. of the battery (B) Greater from 5
(D) The p.d. across the potentiometer wire becomes equal (C) Less than 5
to the e.m.f. of the battery (D) Greater or less than 5 depends on the material of R
320. In the experiment of potentiometer, at balance, there is no 327. A moving coil galvanometer has a resistance of 50 and
current in the gives full scale deflection for 10 mA. How could it be
(A) Main circuit converted into an ammeter with a full scale deflection for
(B) Galvanometer circuit 1A
(C) Potentiometer circuit
50 50
(D) Both main and galvanometer circuits (A) in series (B) in parallel
99 99
321. If in the experiment of Wheatstone’s bridge, the positions
(C) 0.01 in series (D) 0.01 in parallel
of cells and galvanometer are interchanged, then balance
points will 328. The current flowing through a coil of resistance 900 ohms
is to be reduced by 90%. What value of shunt should be
(A) Change
connected across the coil
(B) Remain unchanged
(A) 99 (B) 100
(C) Depend on the internal resistance of cell and
(C) 9 (D) 10
resistance of galvanometer
329. A galvanometer of resistance 25 gives full scale
(D) None of these
deflection for a current of 10 milliampere, is to be changed
322. The resistance of a galvanometer is 90 ohms. If only 10
into a voltmeter of range 100 V by connecting a resistance
percent of the main current may flow through the
of ‘R’ in series with galvanometer. The value of resistance
galvanometer, in which way and of what value, a resistor
R in is
is to be used
(A) 10000 (B) 10025
(A) 10 in series (B) 10 in parallel
(C) 975 (D) 9975
(C) 810 in series (D) 810 in parallel
330. In a potentiometer circuit there is a cell of e.m.f. 2 volt, a
323. Two cells when connected in series are balanced on 8m
resistance of 5 and a wire of uniform thickness of length
on a potentiometer. If the cells are connected with polarities
1000 cm and resistance 15 . The potential gradient in the
of one of the cell is reversed, they balance on 2m. The
wire is
ratio of e.m.f.’s of the two cells is
(A) 3 : 5 (B) 5 : 3 1 3
(A) V / cm (B) V / cm
(C) 3 : 4 (D) 4 : 3 500 2000
324. A voltmeter has a resistance of G and range V volts. 3 1
(C) V / cm (D) V / cm
The value of resistance used in series to convert it into a 5000 1000
voltmeter of range nV volts is 331. The resistance of a galvanometer is 25 and it requires
(A) nG (B) (n – 1)G 50 A for full deflection. The value of the shunt resistance
required to convert it into an ammeter of 5 is
G G
(C) (D) n 1 (A) 2.5 × 10–4 ohm (B) 1.25 × 10–3 ohm
n
(C) 0.05 ohm (D) 2.5 ohm
325. Which of the following statement is wrong
332. Which is a wrong statement
(A) Voltmeter should have high resistance
(A) The Wheatstone bridge is most sensitive when all
(B) Ammeter should have low resistance the four resistances are of the same order
(C) Ammeter is placed in parallel across the conductor in (B) In a balanced Wheatstone bridge, interchanging the
a circuit positions of galvanometer and cell affects the balance
(D) Voltmeter is placed in parallel across the conductor in of the bridge
a circuit (C) Kirchhoff’s first law (for currents meeting at a junction
326. In the diagram shown, the reading of voltmeter is 20 V and in an electric circuit) expresses the conservation of
that of ammeter is 4 A. The value of R should be (Consider charge
given ammeter and voltmeter are not ideal) (D) The rheostat can be used as a potential divider
V 333. A voltmeter having a resistance of 998 is connected to
20V a cell of e.m.f. 2 volt and internal resistance 2 . The error
in the measurement of e.m.f. will be
R
A (A) 4 × 10–1 volt (B) 2 × 10–3 volt
4A (C) 4 × 10–3 volt (D) 2 × 10–1 volt
334. For comparing the e.m.f.’s of two cells with a potentiometer, 339. In an experiment to measure the internal resistance of a
a standard cell is used to develop a potential gradient cell by potentiometer, it is found that the balance point is
along the wires. Which of the following possibilities at a length of 2m when the cell is shunted by a 5
would make the experiment unsuccessful resistance; and is at a length of 3m when the cell is shunted
(A) The e.m.f. of the standard cell is larger than the E by a 10 resistance. The internal resistance of the cell is,
e.m.f.’s of the two cells then
(B) The diameter of the wires is the same and uniform (A) 1.5 (B) 10
throughout (C) 15 (D) 1
(C) The number of wires is ten 340. A potentiometer circuit shown in the figure is set up to
(D) The e.m.f. of the standard cell is smaller than the measure e.m.f. of a cell E. As the point P moves from X to
e.m.f.’s of the two cells Y the galvanometer G shows deflection always in one
335. Which of the following is correct direction, but the deflection decreases continuously until
(A) Ammeter has low resistance and is connected in series Y is reached. In order to obtain balance point between X
and Y it is necessary to
(B) Ammeter has low resistance and is connected in
parallel V R
(C) Voltmeter has low resistance and is connected in
parallel
X P Y
(D) None of the above
336. An ammeter with internal resistance 90 reads 1.85 A
E
when connected in a circuit containing a battery and two G
resistors 700 and 410 in series. Actual current will be (A) Decreases the resistance R
(A) 1.85 A (B) Greater than 1.85 A (B) Increase the resistance R
(C) Less than 1.85 A (D) None of these (C) Reverse the terminals of battery V
337. AB is a wire of uniform resistance. The galvanometer G (D) Reverse the terminals of cell E
shows no current when the length AC = 20cm and CB = 80
341. A voltmeter of resistance 1000 gives full scale deflection
cm. The resistance R is equal to
when a current of 100 mA flow through it. The shunt
R 80 resistance required across it to enable it to be used as an
ammeter reading 1 A at full scale deflection is
(A) 10000
G
(B) 9000
A B
C (C) 222
(D) 111
(A) 2 (B) 8 342. The resistance of 10 metre long potentiometer wire is
(C) 20 (D) 40 1ohm/meter. A cell of e.m.f. 2.2 volts and a high resistance
box are connected in series to this wire. The value of
338. The circuit shown here is used to compare the e.m.f. of
resistance taken from resistance box for getting potential
two cells E1 and E2(E1 > E2). The null point is at C when
gradient of 2.2 millivolt/metre will be
the galvan ometer is connected to E 1. When the
galvanometer is connected to E2, the null point will be (A) 790
(B) 810
B
(C) 990
(D) 1000
C 343. We have a galvanometer of resistance 25 . It is shunted
A B by a 2.5 wire. The part of total current that flows through
E1 the galvanometer is given as
E2 G I I
(A) I 11 (B) I 10
0 0
(A) To the left of C
(B) To the right of C I 3 I 4
(C) I 11 (D) I 11
(C) At C itself 0 0
(D) Nowhere on AB
344. In the adjoining circuit, the e.m.f. of the cell is 2 volt and (C) Galvanometer shows zero deflection
the internal resistance is negligible. The resistance of the (D) Galvanometer and the cell must be interchanged to
voltmeter is 80 . The reading of the voltmeter will be balance
2V 351. The ammeter A reads 2 A and the voltmeter V reads 20V.
+ – The value of resistance R is (Assuming finite resistance’s
of ammeter and voltmeter)
80 R
V A
20 80
V
(A) 0.80 volt (B) 1.60 volt
(C) 1.33 volt (D) 2.00 volt (A) Exactly 10 (B) Less than 10
345. If the resistivity of a potentiometer wire be and area of (C) More than 10 (D) We cannot definitely say
cross-section be A, then what will be potential gradient 352. The resistance of a galvanometer coil is R. What is the
along the wire shunt resistance required to convert it into an ammeter of
range 4 times
I I
(A) (B) R R
A A (A) (B)
5 4
IA R
(C) (D) IA (C) (D) 4 R
3
346. A voltmeter has resistance of 2000 and it can measure 353. If an ammeter is connected in parallel to a circuit, it is
upto 2V. If we want to increase its range to 10 V, then the likely to be damaged due to excess
required resistance in series will be (A) Current (B) Voltage
(A) 2000 (B) 4000 (C) Resistance (D) All of these
(C) 6000 (D) 8000 354. In the given figure, battery E is balanced on 55 cm length
347. For a cell of e.m.f. 2V, a balance is obtained for 50 cm of the of potentiometer wire but when a resistance of 10 is
potentiometer wire. If the cell is shunted by a 2 resistor connected in parallel with the battery then it balances on
and the balance is obtained across 40 cm of the wire, then 50 cm length of the potentiometer wire then internal
the internal resistance of the cell is resistance r of the battery is
(A) 0.25 (B) 0.50 2V
(C) 0.80 (D) 1.00
348. The arrangement as shown in figure is called as 1m
B
A
E r
Total P.D.
(A) 1 (B) 3
(C) 10 (D) 5
Variable P.D. 355. A galvanometer with a resistance of 12 gives full scale
(A) Potential divider (B) Potential adder deflection when a current of 3 mA is passed. It is required
(C) Potential substracter (D) Potential multiplier to convert it into a voltmeter which can read up to 18 V.
349. A potentiometer wire of length 1 m and resistance 10 W is the resistance to be connected is
connected in series with a cell of emf 2V with internal (A) 6000 (B) 5988
resistance 1 and a resistance box including a resistance (C) 5000 (D) 4988
R. If potential difference between the ends of the wire is 356. The resistance of an ideal ammeter is
1 mV, the value of R is (A) Infinite (B) Very high
(A) 20000 (B) 19989 (C) Small (D) Zero
(C) 10000 (D) 9989 357. A galvanometer of 25 resistance can read a maximum current
350. In a balanced Wheatstone’s network, the resistances in of 6mA. It can be used as a voltmeter to measure a maximum
the arms Q and S are interchanged. As a result of this of 6 V by connecting a resistance to the galvanometer.
(A) Network is not balanced Identify the correct choice in the given answers
(B) Network is still balanced (A) 1025 in series (B) 1025 in parallel
(C) 975 in series (D) 975 in parallel
358. A galvanometer has a resistance of 25 ohm and a maximum V V
of 0.01 A current can be passed through it. In order to (A) (B)
R 2R
change it into an ammeter of range 10 A, the shunt
resistance required is 2V 3V
(C) (D)
R R
5 10
(A) (B) 364. A galvanometer of 50 resistance has 25 divisions. A
999 999 current of 4 × 10–4 ampere gives a deflection of one
20 25 division. To convert this galvanometer into a voltmeter
(C) (D)
999 999 having a range of 25 volts, it should be connected with a
359. In the circuit shown, a meter bridge is in its balanced resistance of :–
state. The meter bridge wire has a resistance 0.1 ohm/cm. (A) 245 in parallel (B) 2550 in series
The value of unknown resistance X and the current drawn (C) 2450 in series (D) 2500 in parallel
from the battery of negligible resistance is 365. A 6 volt battery is connected to the terminals of a three
X 6 metre long wire of uniform thickness and resistance of
100 . The difference of potential between two points on
G the wire separated by a distance of 50 cm will be :–
40 cm 60 cm (A) 3 volt (B) 1 volt
A B
(C) 1.5 volt (D) 2 volt
366. Resistance of an ideal voltmeter is :
5V (A) zero (B) less than
(A) 6 , 5 amp (B) 10 , 0.1 amp (C) more than (D) infinite
(C) 4 , 1.0 amp (D) 12 , 0.5 amp 367. Potentiometer is used for measuring :
360. A galvanometer has 30 divisions and a sensitivity (A) potential difference (B) current
16 A/div. It can be converted into a voltmeter to read (C) internal resistance (D) none of these
3 V by connecting
368. Find potential of J with :–
(A) Resistance nearly 6 k in series
respect to G –
(B) 6 k in parallel 60V
(C) 500 in series
64
(D) It cannot be converted
361. A bullet of mass 2 g. is having a charge of 2 C. Through J
what potential difference must it be accelerated, starting
32
from rest, to acquire a speed of 10 m/s ?
(A) 50 kV (B) 5 V 0V G
(C) 50 V (D) 5 kV (A) 40 V (B) 60 V
362. A battery is charged at a potential of 15V for 8 hours when (C) 20 V (D) 30 V
the current flowing is 10A. The battery on discharge 369. A galvanometer acting as a voltmeter will have :
supplies a current of 5 A for 15 hours. The mean terminal (A) a high resistance in series with its coil
voltage during discharge is 14 V. The “Watt hour”
(B) a low resistance in parallel with its coil
efficiency of the battery is :
(C) a low resistance in series with its coil
(A) 80% (B) 90%
(D) a high resistance in parallel with its coil
(C) 87.5% (D) 82.5%
370. Two batteries, one of emf 18 volts and internal resistance
363. Five equal resistances each of resistance R are connected
2 and the other of emf 12 volt and internal resistance
as shown in the Figure. A battery of V volts is connected
1 , are connected as shown. The voltmeter V will record
between A and B. The current flowing in AFCEB will be
a reading of :–
C
V
2 18V
R R R
1 12V
R F A
B
(A) 18 volt (B) 30 vol
B E (C) 14 volt (D) 15 volt
R
371. To convert a 800 mV range milli voltmeter of resistance 378. A 50 galvanometer gets full scale deflection when a
40 into a galvanometer of 100 mA range, the resistance current of 0.01 A passes through the coil. When it is
to be connected as shunt is converted to a 10 A ammeter, the shunt resistance is
(A) 10 (B) 20 (A) 0.01 (B) 0.05
(C) 30 (D) 40 (C) 2000 (D) 5000
372. A 100 galvanometer gives full scale deflection at 10 mA. 379. Resistance in the two gaps of a meter bridge are 10 and
How much shunt is required to read 100 mA 30 respectively. If the resistances are interchanged the
(A) 11.11 (B) 9.9 balance point shifts by
(C) 1.1 (D) 4.4 (A) 33.3 cm (B) 66.67cm
373. The potential difference across the 100 resistance in (C) 25 cm (D) 50 cm
the following circuit is measured by a voltmeter of 900 380. A potentiometer has uniform potential gradient. The
resistance. The percentage error made in reading the specific resistance of the material of the potentiometer
potential difference is wire is 10–7 ohm–meter and the current passing through it
is 0.1 ampere; cross-section of the wire is 10–6 m2. The
900
V potential gradient along the potentiometer wire is
(A) 10–4 V/m (B) 10–6 V/m
10
–2
(C) 10 V/m (D) 10–8 V/m
100 381. Two resistances of 400 and 800 are connected in series
with 6 volt battery of negligible internal resistance. A voltmeter
of resistance 10,000 is used to measure the potential
10 difference across 400 . The error in the measurement of
(A) (B) 0.1
9 potential difference in volts approximately is
(C) 1.0 (D) 10.0 (A) 0.01 (B) 0.02
374. A cell of internal resistance 3 and emf 10 volt is (C) 0.03 (D) 0.05
connected to a uniform wire of length 500 cm and resistance 382. A galvanometer, having a resistance of 50 gives a full
3 . The potential gradient in the wire is scale deflection for a current of 0.05 A. The length in meter
(A) 30 mV/cm (B) 10 mV/cm of a resistance wire of area of cross-section 2.97× 10–2
(C) 20 mV/cm (D) 4 mV/cm cm2 that can be used to convert the galvanometer into an
ammeter which can read a maximum of 5 A current is
375. An ammeter of 100 resistance gives full deflection for
(Specific resistance of the wire = 5 × 10–7 m)
the current of 10–5 amp. Now the shunt resistance required
to convert it into ammeter of 1 amp. range, will be (A) 9 (B) 6
(A) 10–4 (B) 10–5 (C) 3 (D) 1.5
(C) 10–3 (D) 10–1 383. An ammeter reads upto 1 ampere. Its internal resistance
is 0.81 . To increase the range to 10 A the value of the
376. A galvanometer of resistance 36 is changed into an
required shunt is
ammeter by using a shunt of 4 . The fraction f0 of total
current passing through the galvanometer is (A) 0.09 (B) 0.03
(C) 0.3 (D) 0.9
1 1
(A) (B) 384. The length of a wire of a potentiometer is 100 cm, and the
40 4 emf of its standard cell is E volt. It is employed to measure
1 1 the e.m.f of a battery whose internal resistance is 0.5 . If
(C) (D)
140 10 the balance point is obtained at l = 30 cm from the positive
377. If the ammeter in the given circuit reads 2 A, the resistance end, the e.m.f. of the battery is
R is 30 E
3 (A)
100
R 30 E
(B)
6 100.5
30 E
(C) 100 0.5
A
6V 30 E 0.5i
(D) , where i is th e current in the
(A) 1 (B) 2 100
(C) 3 (D) 4 potentiometer
385. Resistance of 100 cm long potentiometer wire is 10 , it is 392. For the network shown in the figure the value of the current
connected to a battery (2 volt) and a resistance R in series. i is :–
A source of 10 mV gives null point at 40 cm length, then 2
external resistance R is
(A) 490 (B) 790 4 4
(C) 590 (D) 990 3
6
386. The e.m.f. of a standard cell balances across 150 cm length
of a wire of potentiometer. When a resistance of 2 is i V
connected as a shunt with the cell, the balance point is
obtained at 100 cm. The internal resistance of the cell is
(A) 0.1 (B) 1 18V 5V
(A) (B)
(C) 2 (D) 0.5 5 9
387. What is the reading of voltmeter in the following figure 9V 5V
(C) (D)
10 V 35 18
393. In potentiometer experiment when terminals of the cell is
at distance of 52cm, then no current flows through it.
1000 When 5 shunt resistance is connected in it then balance
V length is at 40 cm. The internal resistance of the cell is :–
200
(A) 5 (B)
A 500 A 500 52
(A) 3 V (B) 2 V 52
(C) 5 V (D) 4 V (C) (D) 1·5
8
388. The current flowing in a coil of resistance 90 is to be 394. A potentiometer wire has resistance 40 and its length is
reduced by 90%. What value of resistance should be 10m. It is connected by a resistance of 760 in series. If
connected in parallel with it emf of battery is 2V then potential gradient is :–
(A) 9 (B) 90 (A) 0·5 × 10–6 V/m (B) 1 × 10–6 V/m
(C) 1000 (D) 10 –2
(C) 1 × 10 V/m (D) 2 × 10–6 V/m
389. The maximum current that can be measured by a
395. In the circuit shown, if a conducting wire is connected
galvanometer of resistance 40 is 10 mA. It is converted
between points A and B, the current in this wire will :–
into a voltmeter that can read upto 50 V. The resistance to A
be connected in series with the galvanometer is ... (in ohm)
40 40
(A) 5040 (B) 4960
(C) 2010 (D) 4050
390. For the post office box arrangement to determine the value
of unknown resistance the unknown resistance should 10 30
be connected between
B
B C D
V
(A) flow from A to B
(B) flow in the direction which will be decided by the
value of V
A
(C) be zero
B1 C1 (D) flow from B to A
(A) B and C (B) C and D 396. The resistance of an ammeter is 13 and its scale is
(C) A and D (D) B1 and C1 graduated for a current upto 100 am/pers. After and
391. When a wire of uniform cross-section a, length l and additional shunt has been connected to this ammeter it
resistance R is bent into a complete circle, resistance between becomes possible to measure currents upto 750 amperes
any two of diametrically opposite points will be :– by this meter. The value of shunt-resistance is :–
R R (A) 2 k (B) 20
(A) (B) (C) 2 (D) 0.2
2 4
R
(C) (D) 4R
8
397. A wire of resistance 12 ohms per meter is bent to form a 399. A galvanometer having a coil resistance of 60 shows
complete circle of radius 10 cm. The resistance between full scale deflection when a current of 1.0 amperes passes
its two diametrically opposite points, A and B as shown in through it. It can be converted into an ammeter to read
the figure, is :- currents upto 5.0 amperes by :-
(A) putting in parallel a resistance of 15
A B (B) putting in parallel a resistance of 240
(C) putting in series a resistance of 15
(A) 6 (B) 0.6 (D) putting in series a resistance of 240
(C) 3 (D) 6 400. A student measures the terminal potential difference (V)
398. See the electrical circuit shown in this figure. Which of of a cell (of emf and internal resistance r) as a function of
the following equations is a correct equation for it? the current (I) flowing through it. The slope, and intercept,
R of the graph between V and I, then respectively, equal :-
(A) – and r (B) and –r
i1 1 r1
(C) –r and (D) r and –
i2

r2 2
(A) 1
– (i1
+ i 2
)R + i r
1 1
= 0
(B) 1
– (i 1
+ i 2
)R – i r
1 1
= 0
(C) 2
– i2r2 – 1 – i1r1 = 0
(D) – 2 – (i1 + i2)R + i2r2 = 0
(Previous Year AIPMT Examination Questions)
1. Three resistances P, Q, R each of 2 and an unknown 7. See the electrical circuit shown in this figure. Which of
resistance S from the four arms of a Wheatstone’s bridge the following equations is a correct equation for it ?
circuit. When a resistance of 6 is connected in parallel [AIPMT 2009]
to S the bridge gets balanced.What is the value of S ? R
[AIPMT 2007]
i1 1 r1
(A) 2 (B) 3
(C) 6 (D) 1 i2
2. The power dissipated in the circuit shown in the figure is r2 2
30 watt. The value of R is [AIPMT-2007]
(A) 1 – (i1 + i2)R + i1r1 = 0
R
(B) 1 – (i1 + i2)R – i1r1 = 0
(C) 2 – i2r2 – e1 – i1r1 = 0
(D) – 2 – (i1 + i2)R + i2r2 = 0
10V 8. A wire of resistance 12 ohms per meter is bent to form a
complete circle of radius 10 cm. The resistance between
(A) 10 (B) 15 its two diametrically opposite points, A and B as shown in
(C) 20 (D) 30 the figure, is : [AIPMT 2009]
3. An electric kettle takes 4 A current at 220 V. How much
time will it take to boil 1 kg of water from temperature 20°C A B
? The temperature of boiling water is 100°C
[AIPMT 2008] (A) 6 (B) 0.6
(A) 126. min (B) 4.2 min (C) 3 (D) 6
(C) 6.3 min (D) 8.4 min
4. A cell can be balanced against 110 cm and 100 cm of 9. A student measures the terminal potential difference (V)
potentiometer wire, respectively with anbd without being of a cell (of emf and internal resistance r) as a function of
short circuited through a resistance of 10 . Its internal the current (I) flowing through it. The slope, and intercept,
resistance is [AIPMT-2008] of the graph between V and I, then respectively, equal :-
(A) 1.0 (B) 0.5 [AIPMT 2009]
(C) 2.0 (D) zero (A) – and r (B) and –r
5. A current of 3 A flows through the 2 resistor as shown (C) –r and (D) r and –
in th e circuit. Th e power dissipated in the 10. The mean free path of electrons in a metal is
5 resistor is [AIPMT-2008] 4 ×10–8 m. The electric field which can give on an average
2 eV energy to an electron in the metal will be in units
V/m [AIPMT 2009]
(A) 5 ×10 –11 (B) 8 ×10 –11

(C) 5 ×107 (D) 8 ×107


11. A potentiometer circuit is set up as shown. The potential
gradient across the potentiometer wire is k volt/cm and
the ammeter present in the circuit reads 1.0 A when two-
(A) 4 W (B) 2 W way key is switched off. The balance points, when the
(C) 1 W (D) 5 W key between the terminals (i) 1 and 2 (ii) 1 and 3, is plugged
6. A wire of a certain material is stretched slowly by ten in, are found to be at length l1 cm l2 cm respectively. The
percent. Its new resistances and specific resistance magnitudes, of the resistors r and X, in ohm, are then,
become respectively [AIPMT 2008] equal, respectively, to [AIPMT-2010]
(A) 1.2 times, 1.1 times (B) 1.21 times, same
(C) both remain the same (D) 1.1 times, 1.1 times
+ – 16. In the circuit shown the cells A and B have negligible
resistances. For VA = 12V, R1 = 500 and R = 100 , the
galvanometer (G) shows no deflection. The value of VB is
[AIPMT-2012]
A B R1
1 G 12 G
2
3
12v VA VB
– R X
A
+
o
(A) k(l2 – l1) and kl2 (B) kl1 and k(l2 – l1) (A) 6 V (B) 4 V
(C) k(l2 – l1) and kl1 (D) kl1 and kl2 (C) 2 V (D) 12 V
12. Consider the following two statements 17. A ring is made of a wire having a resistance R0 = 12 W.
(A) Kirch hoff’s junction law follows from the Find the points A and B, as shown in the figure, at which
conservation of charge. a current carrying conductor should be connected so that
the resistance R of the subcircuit between these points is
(B) Kirchhoff’s loop law follows from the
8
conservation of energy. [AIPMT 2010] equal to [AIPMT-2012]
3
(A) Both (A) and (B) are wrong
l1
(B) (A) is correct and (B) is wrong
(C) (A) is wrong and (B) is correct A B
(D) Both (A) and (B) are correct
13. A current of 2 A flows through a 2 resistor when
connected across a battery. The same battery supplies a
l2
current of 0.5 A when connected across a 9 resistor.
The internal resistance of the battery is [AIPMT 2011] l1 1 l1 5
(A) l 2 (B) l 8
1 1 2 2
(A) (B)
3 4
l1 1 l1 3
(C) 1 (D) 0.5 (C) l (D) l
2 3 2 8
14. If power dissipated in the 9 resistor in the circuit shown
is 36 W, the potential difference across the 2 resistor is 18. If voltage across a bulb rated 220 volt-100 watt drops by
[AIPMT-2011] 2.5% of its rated value, the percentage of the rated value
by which the power would decrease is [AIPMT-2012]
(A) 10% (B) 20%
(C) 2.5% (D) 5%
19. A cell having an emf and internal resistance r is connected
across a variable external resistance R. As the resistance
R is increased, the plot of potential difference V across R
V is given by [AIPMT-2012]
(A) 8 V (B) 10 V
(C) 2 V (D) 4 V
15. In the circuit shown in the figure,if potential at point A is (A) V (B) V
taken to be zero, the potential at point B is
[AIPMT 2011] R R
0 0
R1 D 2V
B
1A
R2 2A (C) V (D) V
1A 1A R R
A C
0 0
1V
(A) –1 V (B) + 2 V
(C) – 2 V (D) + 1 V
20. The resistances of the four arms P, Q, R and S in a 26. Across a metallic conductor of non-uniform cross-section
Wheatstone’s bridge are 10 , 30 , 30 and 90 , a constant potential difference is applied. The quantity
respectively. The e.m.f. and internal resistance of the cell which remains constant along the conductor is
are 7 volt and 5 respectively. If the galvanometer [AIPMT-2015]
resistanceis 50 ohm, the current drawn from the cell will (A) Current density (B) Current
be [AIPMT-2013] (C) Drift velocity (D) Electric field
(A) 1.0 A (B) 0.2 A 27. Two metal wires of identical dimensions connected in
(C) 0.1 A (D) 2.0 A series. If l and 2 are the conductivities of the metal wires
21. The internal resistance of a 2.1 V cell which gives a current respectively, the effective conductivity of the combination
of 0.2 A through a resistance of 10 is [AIPMT-2013] is [AIPMT-2015 Re]
(A) 0.2 (B) 0.5 1 2 2 1 2
(A) (B)
(C) 0.8 (D) 1.0 1 2 1 2
22. A wire of a resistance 4 is stretched to twice its original 1 2 1 2
length. The resistance of stretched wire would be (C) 2 1 2 (D)
1 2
[AIPMT-2013] 28. A, B and C are voltmeters of resistance R, 1.5 R and 3 R
(A) 2 (B) 4 respectively as shown in the figure. When some potential
(C) 8 (D) 16 difference is applied between X and Y, the voltmeter
23. Two cities are 150 km apart. Electric power is sent from readings are VA, VB and VC respectively, then
one city to another city through copper wires. The fall of [AIPMT-2015]
potential per km is 8 volt and the average resistance per B
km is 0.5 . The power loss in the wire is [AIPMT-2014] X A Y
(A) 19.2 W (B) 19.2 kW C
(C) 19.2 J (D) 12.2 kW (A) VA = VB = VC (B) VA VB = VC
24. The resistance in the two arms of the meter bridge are 5 (C) VA = VB VC (D) VA VB VC
and R , respectively. When the resistance R is shunted 29. A potentiometer wire of length L and a resistance r are
with an equal resistance, the new balance point is at 1.6 l1. connected in series with a battery of e.m.f. E0 and a
The resistance R is [AIPMT-2014] resistance r 1. An unknown e.m.f. E is balanced at a length
l of the potentiometer wire. The e.m.f. E will be given by
( ) [AIPMT-2015 Re]
LE0 r LE0 r
(A) (r r )l (B) Ir
1 1
E0 r I E0 r
(C) (r r ) L (D)
1 L
G 30. A circuit contains an ammeter, a battery of 30 V and a
resistance 40.8 ohm all connected in sereis. If the ammeter
A B has a coil of resistance 480 and a shunt of 20 the
l1 100 – l1 reading in the ammeter will be [AIPMT-2015 Re]
(A) 10 (B) 15 (A) 1 A (B) 0.5 A
(C) 20 (D) 25 (C) 0.25 A (D) 2 A
31. A potentiometer wire is 100 cm long and a constant
potential difference is maintained across it. Two cells are
25. A potentiometer circuit has been set up for finding the connected in series first to support one another and then
internal resistance of a given cell. The main battery used in opposite direction. The balance points are obtained at
across the potentiometer wire, has an emf of 2.0 V and a 50 cm and 10 cm from the positive end of the wire in the
negligible internal resistance. The potentiometer wire itself two cases. The ratio of emf's is :- [AIPMT 2016]
is 4 m long. When the resistance, R, connected across the (A) 5 : 1 (B) 5 : 4
given cell, has values of [AIPMT-2014] (C) 3 : 4 (D) 3 : 2
(i) Infinity 9 32. The potential difference (VA – VB) between the points A
(ii) the ‘balancing lengths’, on the potentiometer wire and B in the given figure is :- [NEET – II (2016)]
are found to be 3 m and 2.85 m, respectively.
The value of internal resistance of the cell is
(A) 0.25 (B) 0.95 (A) + 6 V (B) + 9 V
(C) – 3 V (D) + 3 V
(C) 0.5 (D) 0.75
33. A filament bulb (500 , 100 V) is to be used in a 230 V main
supply. When a resistance R is connected in series, it I I
works perfectly and the bulb consumes 500 W. The value
of R is :- [NEET – II (2016)] (C) (D)
(A) 26 (B) 13
(C) 230 (D) 46 O n O n
34. A potentiometer is an accurated and versatile device to
make electrical measurements of E.M.F. because the 39. In the circuits shown below, the readings of the
method involves : [NEET – (2017)] voltmeters and the ammeters will be : [NEET-2019]
(A) Cells 10 i 1
(B) potential gradients
(C) a condition of no current flow through the
galvanometer
V1 A1
(D) a combination of cells, galvanometer and resistance
35. The resistance of a wire is ‘R’ . If it is melted and stretched
to ‘n’ times its original length, its new resistance will be:
[NEET – 2017] 10V
R Circuit 1
(A) nR (B)
n 10 i2
R
(C) n 2R (D)
n2 10
36. A carbon resistor of (47 4.7) k is to be marked with V2 A2
rings of different colours for its identification. The colour
code sequence will be [NEET-2018]
(A) Yellow – Green – Violet – Gold
10V
(B) Yellow – Violet – Orange – Silver
Circuit 2
(C) Violet – Yellow – Orange – Silver
(A) V1 = V2 and i1 > i2 (B) V1 = V2 and i1 = i2
(D) Green – Orange – Violet – Gold
(C) V2 > V1 and i1 > i2 (D) V2 > V1 and i1 = i2
37. A set of ‘n’ equal resistors, of value ‘R’ each, are
40. Six similar bulbs are connected as shown in the figure
connected in series to a battery of emf ‘E’and internal
with a DC source of emf E, and zero internal resistance.
resistance ‘R’. The current drawn is I. Now, the ‘n’
resistors are connected in parallel to the same battery. The ratio of power consumption by the bulbs when (i)
Then the current drawn from battery becomes 10 I. The all are glowing and (ii) in the situation when two from
value of ‘n’ is [NEET-2018] section A and one from section B are glowing will be:
(A) 20 (B) 11
(C) 10 (D) 9
38. A battery consists of a variable number ‘n’ of identical
cells (having internal resistance ‘r’ each) which are
connected in series. The terminals of the battery are
short-circuited and the current I is measured. Which of
the graphs shows the correct relationship between I
and n? [NEET-2018]

I I

(A) (B)

n O n (A) 9 : 4 (B) 1 : 2 [NEET-2019]


O
(C) 2 : 1 (D) 4
(Previous Year AIIMS Questions)
1. The internal resistance of a cell of e.m.f. 2 volt is 0.1 . It 8. If a steady current of 4 A maintained for 40 minutes, deposits
is connected to a resistance of 3.9 . The voltage across 4.5 g of zinc at the cathode end, then its chemical
the cell will be (in volt) [1995] equivalent, is [1996]
(A) 1.95 V (B) 0.5 V (A) 47 × 10–5 g/C (B) 26 × 10–6 g/C
(C) 2 V (D) 1.9 V (C) 51 × 10–7 g/C (D) 36 × 10–6 g/C
2. Ten identical wires each having a resistance of 1 are 9. A battery of emf 10 V and internal resistance 3 is
connected in parallel. The combination will have a connected to a resistor. If the current in the circuit is 0.5 A,
resistance of [1995] what is the resistance of the resistor ? [1997]
(A) 0.1 (B) 10 (A) 13 (B) 15
(C) 0.01 (D) 10 (C) 17 (D) 19
3. Which of the following relations is called as current 10. What is the current (I) in the circuit, as shown in figure ?
density ? [1995] [1997]
l2 A I R2 = 2
(A) (A)
A I

I3 I
(C) (D) 3V R1 = 2 R3 = 2
2 A
A
4. What is the equivalent resistance between Aand B in the
given figure [1995] R4 = 2
20 (A) 1.2 A (B) 0.5 A
20
(C) 1 A (D) 2 A
20 11. In the figure, the equivalent resistance between the points
A B
A and B is [1998]

20 20 R2 = 4

(A) 40 (B) 10 R1 = 2 R4 = 2
(C) 50 (D) 20
5. Ten identical cells each of potential E and internal A B
R3 = 4
resistance r are connected in series to form a closed circuit.
An ideal voltmeter connected across three cells, will read
(A) 8 (B) 6
[1995]
(C) 2 (D) 4
(A) 10E (B) 3E
12. Electroplating does not help in [1998]
(C) 13E (D) 7E
(A) metals to become hard
6. The heat produced by a 100 W heater in 2 min is equal to E (B) shining appearance
[1995] (C) fine finish to be surface
E, r (D) protect metal against corrosion
13. Given a current carrying if wire of non uniform cross
section. Which of the following is constant throughout
the length of the wire ? [2000]

R B
(A) 10.5 kcal (B) 16.3 kcal 2 2
(C) 2.8 kcal (D) 14.2 kcal
7. Thermocouple thermometer is based on [1996] A 7 C
(A) Peltier effect (B) Seebeck effect 4 4
(C) Photoelectric effect (D) Compton effect
D
(A) Current electric field and drift speed 21. An electric bulb marked 40 W and 200 V, is used in a
(B) Drift speed only circuit of supply voltage 100 V. Now its power is [2002]
(C) Current and drift speed (A) 100 W (B) 20 W
(D) Current only (C) 40 W (D) 10 W
14. The resistance between the terminal points A and B of the 22. A wire of length L is drawn such that its diameter is
given infinitely long circuit will be [2000] reduced to half of its original diameter. If the initial
1 1 1 resistance of the wire was 10 , its new resistance would
be [2003]
A Up to infinity (A) 40 (B) 80
1 1 (C) 120 (D) 160
B
23. Eels are able to generate current with biological cells called
1 1 1 electroplaques. The electroplaques in an eel are arranged
(A) 3 1 (B) 1 3 in 100 rows, each row stretching horizontally along the
body of the fish containing 5000 electroplaques. The
(C) 1 3 (D) 2 3 arrangement is suggestively shown h ere. Each
15. Two resistance filaments of same length are connected electroplaques has an emf of 0.15 V and internal resistance
first in series and then in parallel. Find the ratio of power of 0.25 . The water surrounding the eel completes a
dissipated in both cases assuming that equal current flows circuit between the head and its tail. If the water
in the main circuit. [2000] surrounding it has a resistance of 500 , the current an
eel can produce in water is about [2004]
(A) 1 : 4 (B) 4 : 1
(C) 1 : 2 (D) 2 : 1. 0.15 V 0.25
+ –
16. A 60 W incandescent lamp operates at 120V. The number
of electrons passing through the filament per second will
be [2000] 5000 electroplaques per row
100 rows
(A) 1.61 × 1012 (B) 3.12 × 1018
(C) 7.21 × 1012 (D) 12.40 × 1013
17. The equivalent resistance between A and C of the given
circuit, is [1996, 2001]
+ – + –
32
(A) 8 (B)
12 500
4 5
(C) (D)
3 3 (A) 1.5 A (B) 3.0 A
r (C) 15 A (D) 30 A
18. When a wire is stretched and its radius becomes , then 24. For ensuring dissipation of same energy in all three
2
its resistance will be [1997, 2001] resistors (R1, R2, R3) connected as shown in figure, their
(A) 16R (B) 2R values must be related as [2005]
(C) 4R (D) R R1
19. Antimony and Bismuth are usually used in thermocouple,
because [1995, 2001]
(A) a constant thermo e.m.f. is produced Vin R2 R3
(B) higher thermo e.m.f. is produced
(C) a negative thermo e.m.f. is produced
(D) lower thermo e.m.f. is produced (A) R1 = R2 = R3
20. A metal rod consumes power P on passing current. If it is (B) R2 = R3 and R1 = 4R2
cut into two half and joined in parallel, it will consume
power [2001] 1
(C) R2 = R3 and R1 = R2
(A) P (B) 2P 4

P (D) R1 = R2 + R3.
(C) 4P (D)
4
25. Faraday law of electrolysis indirectly shows [2007] 29. Two heater wires, made of the same material and having
(A) quantisation of charge the same length and the same radius, are first connected
(B) quantisation of angular momentum in series and then in parallel to a constant potential
(C) quantisation of current difference. If the rates of heat produced in the two cases
(D) quantisation of viscosity are Hs, and Hp respectively, then Hs / Hp will be [2008]
26. The equivalent resistance between the points X and Y in 1
the following circuit diagram will be [2008]
(A) (B) 2
2
Z 1 T 1
(C) (D) 4
4
3 10 5 30. A bulb of 25 W, 200 V and another bulb of 100 W, 200 V are
connected in series with a supply line of 220 V. Then [2008]
(A) both bulbs will glow with same brightness
X Y
10 (B) both bulbs will get fused
(A) 10 (B) 5 (C) 25 W bulb will glow more brightly
(C) 7 (D) 3 (D) 100 W bulb will glow more brightly.
31. In the given circuit, the potential difference between A
J 1
27. The current curve between loge 2
and is [2008] and B is [2009]
T T
10 k A

2
(A) loge J/T
30 V 10 k 10 k
1/T
B
(A) 0 (B) 5 volt
2
(B) loge J/T (C) 10 volt (D) 15 volt.
32. When a current is passed in a conductor, 3°C rise in
1/T temperature is observed. If the strength of current is made
thrice, then rise in temperature will approximately be
[2009]
2 (A) 36°C (B) 27°C
(C) loge J/T
(C) 18°C (D) 9°C
33. In a metal with positive Thomson coefficient, current is
1/T
passed from the lower temperature to higher temperature
side. Then heat will be [2009]
(A) absorbed (B) constant
(D) loge J/T2
(C) evolved (D) either (B) or (C)
34. In the circuit shown, current flowing through 25 V cell is
1/T [2010]
28. A bulb and a condenser are connected in series with an
A.C. source. On increasing the frequency of the source 10V 5V 20 V 30 V 25 V
its brightness will [2008]
(A) increase
(B) decrease
(C) sometimes increase and sometimes decrease 5 10 5 11
(D) neither increase nor decrease.
(A) 7.2 A (B) 10 A
(C) 12 A (D) 14.2 A
35. Transmission lines transmit a voltage of V volt to our 40. What is the equivalent resistance between A and B in
houses from power stations, then the power P supplied given figure? [2015]
by them is proportional to [2010]
50 50
1
(A) (B) V
V
A 50 B
1
(C) V2 (D)
V2 50 50
36. Calculate I for the given circuit diagram. [2012]
B (A) 50 (B) 25
(C) 75 (D) 100
5 5 41. When a current is passed in a conductor, 5 °C rise in
temperature is observed. If the strength of current is made
5 thrice, rise in temperature will be [2015]
5 (A) 5°C (B) 20 °C
(C) 45°C (D) 15°C
5
42. Ten identical cells each of potential E and internal
D resistance r are connected in series to form a closed circuit.
An ideal voltmeter connected across three cells, will read
5 [2016]
(A) 10E (B) 3E
I 25V (C) 13E (D) 7E
(A) 10 A (B) 5 A 43. A steady current flow in a metallic conductor of non-
(C) 2.5 A (D) 20 A uniform cross-section. Which of these quantities is
37. Calculate the heat emitted by a bulb of 100 W in 1 min. constant along the conductor? [2016]
[2002] (A) Electric field (B) Drift velocity
(A) 100 J (B) 1000 J (C) Current (D) Current density
(C) 600 J (D) 6000 J 44. On increasing the temperature of a conductor, its
38. A circuit consisting of five resistors each of resistance R, resistance increases because the [2016]
forming a Wheatstone bridge. What is the equivalent (A) relaxation time increases
resistance of the circuit? [2013] (B) electron density decreases
(A) 2R (B) R (C) relaxation time decreases
2R R (D) relaxation time remains constant.
(C) (D) 45. The potential difference that must be applied to stop the
3 2
39. 2, 4 and 6 S are the conductances of three conductors. fastest photoelectrons emitted by a nickel surface, having
When they are joined in parallel, their equivalent work function 5.01 eV, when ultraviolet light of 200 nm
conductance will be [2014] falls on it, must be [2017]
(A) 2.4 V (B) – 1.2 V
1
(A) 12 S (B) S (C) – 2.4 V (D) 1.2 V
12
11 11
(C) S (D) S
12 12
(Assertion & Reason Type Questions)
These questions consist of two statements each, printed as 6. Assertion : Electric appliances with metallic body have
Assertion and Reason. While answering these questions you three connections, whereas an electric bulb
are required to choose any one of the following five responses. has a two pin connection.
(A) If both assertion and reason are true and the reason is Reason : Three pin connections reduce heating of
a correct explanation of the assertion. connecting wires.
(B) If both assertion and reason are true but reason is not 7. Assertion : The drift velocity of electrons in a metallic
a correct explanation of the assertion. wire will decrease, if the temperature of the
(C) If the assertion is true but reason is false. wire is increased.
(D) If both the assertion and reason are false. Reason : On increasing temperature, conductivity of
metallic wire decreases.
1. Assertion : The resistivity of a semicon ductor 8. Assertion : The electric bulbs glows immediately when
increases with temperature. switch is on.
Reason : The atoms of a semiconductor vibrate with Reason : The drift velocity of electrons in a metallic
larger amplitude at higher temperatures wire is very high.
thereby increasing its resistivity 9. Assertion : Bending a wire does not effect electrical
[AIIMS 2003] resistance.
2. Assertion : In a simple battery circuit the point of lowest Reason : Resistance of wire is proportional to
potential is positive terminal of the battery resistivity of material.
Reason : The current flows towards the point of the 10. Assertion : In meter bridge experimen t, a high
higher potential as it flows in such a circuit resistance is always connected in series with
from the negative to the positive terminal. a galvanometer.
[AIIMS 2002] Reason : As resistance increases current through the
3. Assertion : The temperature coefficient of resistance circuit increases.
is positive for metals and negative for p- 11. Assertion : A domestic electrical appliances, working
type semiconductor. on a three pin, will continue working even
Reason : The effective charge carriers in metals are if the top pin is removed.
negatively charged whereas in p-type Reason : The third pin is used only as a Safety
semiconductor they are positively charged. device. [1995]
[AIIMS 1996] 12. Assertion : Good conductors of heat are also good
4. Assertion : In the following circuit emf is 2V and internal conductors of electricity and vice versa.
resistance of the cell is 1 and R = 1 , Reason : Mainly electrons are responsible
then reading of the voltmeter is 1V. for these conductions. [2000]
13. Assertion : Electric appliances with metallic body e.g.,
V heaters, presses etc., have three pin
E=2 connections, whereas an electric bulb has
a two pin connection.
Reason : Three pin connections reduce heating of
r =1 connecting cables.
A
R=1 [1995,1996,2000]
14. Assertion : In a simple battery circuit, the point of the
lowest potential is positive terminal of the
2 battery.
Reason : V = E – ir where E = 2V, i 1A and
2 Reason : The current flows towards the point of the
R=1 [AIIMS 1995] higher potential, as it does in such a circuit
5. Assertion : There is no current in the metals in the from the negative to the positive terminal.
absence of electric field. [1996, 2002]
Reason : Motion of free electron are randomly. 15. Assertion : A larger dry cell has higher emf.
[AIIMS 1994] Reason : The emf of a dry cell is proportional to its
size. [2004]
16. Assertion : A curren t continues to follow in 25. Assertion : The e.m.f. of the driver cell in th e
superconducting coil even after switch is potentiometer experiment should be greater
off. than the e.m.f. of the cell to be determined.
Reason : Superconducting coils show Meissner Reason : The fall of potential across the
effect. [2007] potentiometer wire should not be less than
17. Assertion : Ohm’s law is applicable for all conducting the e.m.f. of the cell to be determined.
elements. 26. Assertion : A person touching a high power line gets
Reason : Ohm’s law is a fundamental law. stuck with the line.
[2007] Reason : The current carrying wires attract the man
18. Assertion : In the following circuit, emf is 2 V and towards it.
internal resistance of the cell is 1 W and 27. Assertion : The connecting wires are made of copper.
R = 1 , then reading of the voltmeter is 1 V. Reason : The electrical conductivity of copper is
high.
V
28. Assertion : It is advantageous to transmit electric power
E=2V at high voltage.
Reason : High voltage implies high current.
[2010]
r =1
A 29. Assertion : Electric appliances with metallic body. e.g.,
heaters, presses etc., have three pin
R=1 connections, whereas an electric bulb has
a two pin connection.
2
Reason : V = E – IR where E = 2 V, I = = I A. Reason : Three pin connections reduce heating of
2
connecting cables. [2013]
[2008]
30. Assertion : Total current entering a circuit is equal to
19. Assertion : A domestic electrical appliance, working on
leaving the circuit by Kirchhoff’s law.
a three pin will continue working even if
Reason : It is based on conservation of energy.
the top pin is removed.
[2013]
Reason : The third pin is used only as a safety
device. [2008] 31. Assertion : Current is passed through a metallic wire
so that it becomes red hot. When cold water
20. Assertion : A bird perches on a high power line and
is poured on half of its portion, the rest of
nothing happens to the bird.
the half portion becomes more hot.
Reason : The level of bird is very high from the
Reason : Resistance decreases due to decrease in
ground. [2008]
temperature. [2014]
21. Assertion : Voltameter measures current more
32. Assertion : The conductivity of an electrolyte is very
accurately than ammeter.
low as compared to a metal at room
Reason : Relative error will be small if measured from
temperature.
voltameter.
Reason : The number density of free ions in
22. Assertion : Electric field outside the conducting wire
electrolyte is much smaller as compared to
which carries a constant current is zero.
number density of free electrons in metals.
Reason : Net charge on conducting wire is zero. Further, ions drift much more slowly, being
23. Assertion : The resistance of super-conductor is zero. heavier. [2015]
Reason : The super-conductors are used for the
transmission of electric power.
24. Assertion : A potentiometer of longer length is used
for accurate measurement.
Reason : The potential gradient for a potentiometer
of longer length with a given source of
e.m.f. becomes small.
Exercise – I
1. (B) 2. (C) 3. (D) 4. (A) 5. (C) 6. (B) 7. (A) 8. (A) 9. (C) 10. (B)
11. (D) 12. (D) 13. (A) 14. (C) 15. (D) 16. (A) 17. (C) 18. (B) 19. (C) 20. (B)
21. (D) 22. (B) 23. (B) 24. (B) 25. (D) 26. (C) 27. (B) 28. (B) 29. (B) 30. (A)
31. (C) 32. (D) 33. (B) 34. (D) 35. (C) 36. (B) 37. (B) 38. (C) 39. (A) 40. (D)
41. (A) 42. (B) 43. (B) 44. (C) 45. (C) 46. (D) 47. (A) 48. (B) 49. (D) 50. (D)
51. (A) 52. (A) 53. (C) 54. (B) 55. (B) 56. (C) 57. (C) 58. (C) 59. (D) 60. (B)
61. (A) 62. (D) 63. (B) 64. (B) 65. (C) 66. (B) 67. (C) 68. (A) 69. (A) 70. (D)
71. (C) 72. (C) 73. (D) 74. (B) 75. (A) 76. (B) 77. (B) 78. (B) 79. (C) 80. (A)
81. (C) 82. (A) 83. (D) 84. (B) 85. (B) 86. (D) 87. (D) 88. (A) 89. (D) 90. (D)
91. (D) 92. (D) 93. (A) 94. (A) 95. (C) 96. (A) 97. (A) 98. (B) 99. (C) 100. (A)
101. (A) 102. (D) 103. (C) 104. (C) 105. (B) 106. (A) 107. (D) 108. (D) 109. (A) 110. (B)
111. (D) 112. (D) 113. (B) 114. (D) 115. (B) 116. (D) 117. (C) 118. (C) 119. (B) 120. (D)
121. (A) 122. (A) 123. (B) 124. (B) 125. (C) 126. (B) 127. (D) 128. (D) 129. (D) 130. (C)
131. (B) 132. (D) 133. (A) 134. (B) 135. (C) 136. (D) 137. (D) 138. (B) 139. (C) 140. (B)
141. (B) 142. (D) 143. (A) 144. (C) 145. (A) 146. (D) 147. (B) 148. (A) 149. (B) 150. (C)
151. (D) 152. (C) 153. (D) 154. (C) 155. (C) 156. (C) 157. (B) 158. (C) 159. (D) 160. (B)
161. (A) 162. (C) 163. (A) 164. (B) 165. (A) 166. (C) 167. (C) 168. (B) 169. (C) 170. (A)
171. (A) 172. (B) 173. (B) 174. (D) 175. (D) 176. (A) 177. (A) 178. (A) 179. (B) 180. (B)
181. (C) 182. (D) 183. (B) 184. (B) 185. (C) 186. (D) 187. (D) 188. (A) 189. (B) 190. (C)
191. (C) 192. (A) 193. (B) 194. (D) 195. (A) 196. (A) 197. (B) 198. (D) 199. (BC) 200. (B)
201. (D) 202. (C) 203. (A) 204. (A) 205. (D) 206. (A) 207. (D) 208. (C) 209. (D) 210. (C)
211. (B) 212. (B) 213. (B) 214. (C) 215. (B) 216. (A) 217. (C) 218. (B) 219. (C) 220. (A)
221. (C) 222. (C) 223. (B) 224. (B) 225. (D) 226. (C) 227. (D) 228. (D) 229. (B) 230. (A)
231. (C) 232. (D) 233. (A) 234. (D) 235. (B) 236. (C) 237. (C) 238. (C) 239. (D) 240. (B)
241. (C) 242. (C) 243. (B) 244. (D) 245. (A) 246. (A) 247. (B) 248. (B) 249. (A) 250. (B)
251. (A) 252. (C) 253. (B) 254. (A) 255. (A) 256. (B) 257. (A) 258. (B) 259. (B) 260. (C)
261. (A) 262. (B) 263. (A) 264. (B) 265. (C) 266. (A) 267. (A) 268. (A) 269. (A) 270. (D)
271. (B) 272. (D) 273. (B) 274. (C) 275. (A) 276. (B) 277. (C) 278. (A) 279. (D) 280. (B)
281. (C) 282. (C) 283. (C) 284. (B) 285. (A) 286. (C) 287. (A) 288. (D) 289. (B) 290. (A)
291. (D) 292. (C) 293. (C) 294. (B) 295. (B) 296. (C) 297. (A) 298. (A) 299. (B) 300. (B)
301. (D) 302. (C) 303. (D) 304. (A) 305. (D) 306. (C) 307. (A) 308. (B) 309. (C) 310. (A)
311. (B) 312. (B) 313. (A) 314. (A) 315. (A) 316. (A) 317. (A) 318. (A) 319. (B) 320. (B)
321. (B) 322. (B) 323. (B) 324. (B) 325. (C) 326. (C) 327. (B) 328. (B) 329. (D) 330. (B)
331. (A) 332. (B) 333. (C) 334. (D) 335. (A) 336. (B) 337. (C) 338. (A) 339. (B) 340. (A)
341. (D) 342. (C) 343. (A) 344. (C) 345. (A) 346. (D) 347. (B) 348. (A) 349. (B) 350. (A)
351. (C) 352. (C) 353. (A) 354. (A) 355. (B) 356. (D) 357. (C) 358. (D) 359. (C) 360. (A)
361. (A) 362. (C) 363. (B) 364. (C) 365. (B) 366. (D) 367. (A) 368. (C) 369. (A) 370. (C)
371. (A) 372. (A) 373. (C) 374. (B) 375. (C) 376. (D) 377. (A) 378. (B) 379. (D) 380. (C)
381. (D) 382. (C) 383. (A) 384. (A) 385. (B) 386. (B) 387. (D) 388. (D) 389. (B) 390. (C)
391. (B) 392. (D) 393. (D) 394. (C) 395. (D) 396. (C) 397. (B) 398. (B) 399. (A) 400. (C)
Exercise – II
(Previous Year AIPMT Examination Questions)
1. (B) 2. (B) 3. (C) 4. (A) 5. (D) 6. (B) 7. (B) 8. (B) 9. (C) 10. (C)
11. (B) 12. (D) 13. (A) 14. (B) 15. (D) 16. (C) 17. (A) 18. (D) 19. (C) 20. (B)
21. (B) 22. (D) 23. (B) 24. (B) 25. (C) 26. (B) 27. (B) 28. (A) 29. (C) 30. (B)
31. (D) 32. (B) 33. (A) 34. (C) 35. (C) 36. (B) 37. (C) 38. (C) 39. (B) 40. (A)

Exercise – III
(Previous Year AIIMS Questions)
1. (A) 2. (A) 3. (D) 4. (D) 5. (B) 6. (C) 7. (B) 8. (A) 9. (C) 10. (D)
11. (B) 12. (A) 13. (D) 14. (C) 15. (B) 16. (B) 17. (B) 18. (A) 19. (B) 20. (C)
21. (D) 22. (D) 23. (A) 24. (C) 25. (A) 26. (B) 27. (A) 28. (A) 29. (C) 30. (C)
31. (C) 32. (B) 33. (A) 34. (C) 35. (B) 36. (A) 37. (D) 38. (B) 39. (A) 40. (A)
41. (C) 42. (B) 43. (C) 44. (B) 45. (D)

Exercise – IV
1. (D) 2. (D) 3. (B) 4. (A) 5. (A) 6. (C) 7. (B) 8. (C) 9. (A) 10. (C)
11. (A) 12. (A) 13. (C) 14. (D) 15. (D) 16. (B) 17. (D) 18. (A) 19. (A) 20. (C)
21. (A) 22. (A) 23. (B) 24. (A) 25. (A) 26. (D) 27. (A) 28. (C) 29. (C) 30. (A)
31. (A) 32. (A)

Exercise - I ne 62.5×1018 ×1.6 ×10-19


19. (C) i = = = 10 ampere
11. (D) In case of stretching of wire R l2 t 1
If length becomes 3 times so Resistance becomes 20. (B) In twisted wire, two halves each of resistance 2
are in parallel, so equivalent resistance will be
9 times i.e. R´ = 9 × 20 = 180
12. (D) Resistivity is the property of the material. It does 2
=1 .
not depend upon size and shape. 2
13. (A) Because with rise in temperature resistance of 1
21. (D) In stretching of wire R
conductor increase, so graph between V and i r4
becomes non linear.
L ×1
14. (C) Because V-i graph of diode is non-linear. 22. (B) R = 0.7 =
A 22
e V e El (1×10 –3 ) 2
15. (A) vd = × or vd = . (Since V = El) 7
m l m l = 2.2 × 10–6 ohm-m.
vd E
1 1 1
16. (A) Resistance of conductor depends upon relation as 23. (B) R R [d = diameter of wire]
A r2 d2
1
R . With rise in temperature rms speed of free 24. (B) i = q v = 1.6 × 10 × 6.6 × 1015
–19

= 10.56 × 10–4 A = 1mA


electron inside the conductor increase, so relaxation
time decrease and hence resistance increases l
25. (D) R
q 4 r2
17. (C) i = = = 2 ampere
t 2 R1 l1 r22 1 5 2
2

3 = × = × l2 = 20m
18. (B) Volume = Al = 3 A= R 2 l 2 r12 1 l2 1
l
27. (B) In semiconductors charge carries are free electrons
l ×l l2 2 9 3 and holes
Now, R = 3= = l = =
A 3 /1 3
28. (B) Net current inet = i(+) + i(–) 2 2
R1 l1 d2 L 2d
+2e 1
R2 l2 d1 4L d
+
– R2 = R1 = R.
–e i 1.344
38. (C) vd 6 19
nAe 10 1.6 10 8.4 1022
inet
1.344
n (+) q (+) n (–) q (–) 0.01cm / s 0.1mm / s
= + 10 1.6 8.4
t t
1
n (+) n (–) 39. (A) Internal resistance
= × 2e + ×e Temperature
t t 40. (D) Charge = Current × Time =5 × 60 = 300 C
= 3.2 × 1018 × 2 × 1.6 × 10–19 + 3.6 × 1018 × 1.6 × 10–19
41. (A) Using R T2 R T1 [1 + (T2 – T1)]
= 1.6 A (towards right)
R100 = R50[1 + (100 – 50)]
29. (B) In the absence of external electric field mean velocity
of free electron (V rms) is gi ven by (7 5)
7 = 5[1+( × 50)] 0.008 / o C
3KT 250
Vrms = Vrms T
m 42. (B) This is because of secondary ionisation which is
30. (A) With rise in temperature specific resistance increases possible in the gas filled in it.
31. (C) For metallic conductors, temperature co-efficient of 44. (C) R1 (1 t1 ) 50 (1 3.92 10 3 20)
resistance is positive. R 2 (1 t 2 ) 76.8 (1 3.92 10 3 t)
33. (B) Length l = 1 cm 10–2 m t = 167° C
i
45. (C) From v d i vd A i v d r 2
neA
46. (D) Resistivity depends only on the material of the
conductor.
47. (A) A particular temperature, the resistance of a
100cm 1 1
superconductor is zero G
R 0
1 cm (n ) (q ) (n ) (q )
Area of cross-section A = 1 cm × 100 cm 48. (B) Net current i i i
t t
= 100 cm2 = 10–2 m2 –
– e + +
10 2 Ne
Resistance R = 3 10 = 3 × 10 –7 –7

10 2
i
34. (D) In the above question for calculating equivalent
resistance between two opposite square faces. (n ) (n )
i e e
l = 100 cm = 1 m, A = 1 cm2 = 10–4 m2, so resistance t t
= 2.9 × 1018 × 1.6 × 10–19 + 1.2 × 1018 × 1.6 × 10–19
1
R = 3 × 10–7 = 3 × 10–3 i = 0.66 A
10 4 49. (D) If E be electric field, then current density
i 20 j= E
35. (C) vd 29 6 19
nAe
10 10 1.6 10 i
Also we know that current density j
= 1.25 × 10 m/s
–3
A
E Hence j is different for different area of cross-sections.
36. (B) Specific resistance k When j is different, then E is also different. Thus E is
j not constant. The drift velocity v d is given by
l l j
37. (B) R vd = different for different j values. Hence only
A d2 ne
current i will be constant.
l 25 9 1 1
51. (A) R and mass m = volume (V) × density : : 25 : 3 : 125 :15 :1 .
A 1 3 5 5
(D) = (A l) d 4 4
Since wires have same material so and d is same R1 r2 R nr R .
61. (A) R2
for both. R2 r1 R2 r n4
Also they have same mass Al = constant
R1 (1 t1 ) 5 (1 50)
1 62. (D) R
l 2 (1 t2 ) 6 (1 100)
A
1
R1 l1 A2 A2
2
r2
4
per o C
200
R2 l2 A1 A1 r1 Again by Rt = R0(1 + t)
4 1
34 r 5 R0 1 50 R0 4 .
R2 = 544 200
R2 2r
R1 A2 Q 19
52. (A) R
l
( , L constant) 63. (B) i Q 1.6 10 5 1015 0.8mA .
A R2 A1 T
7
A1 R2 riron iron 1 10
2 64. (B) 2.4 .
rCopper 8
A2 R1 copper 1.7 10
Now, when a body dipped in water, loss of weight 65. (C) i = e v = 1.6 × 10–19 × 6.8 × 1015 = 1.1 × 10–3 amp.
= V Lg = AL Lg 66. (B) Resistivity of the material of the rod
(Loss of weight)1 A1 RA 3 10 3 (0.3 10 2 ) 2
So, 2; so A has more
(Loss of weight)2 A 2 l 1
loss of weight. = 27 × 10–9 ×m
53. (C) Q = it = 20 × 10–6 × 30 = 6× 10-–4 C
(Thickness)
54. (B) Ge is semiconductor and Na is a metal. The Resistance of disc R
conductivity of semiconductor increases and that (Area of cross section)
of the metals decreases with the rise in temperature.
9 (10 3 )
= 27 10 = 2.7 × 10–7 .
ne it 1.6 10 3 1 16 (1 10 2 )2
55. (B) i n 10 .
t e 1.6 10 19 67. (C) By using Rt = R0(1 + t)
i 1 1 3 × R0 = R0(1 + 4 × 10–3 t) t = 500° C.
56. (C) Drift velocity vd vd or vd 68. (A) i = 6 × 1015 × 1.6 × 10–19 = 0.96mA.
neA A d2
2 ne n 1.6 10 19
2
d 69. (A) i 16 10 3 n = 1017
vP dQ 1 1 t 1
2 vP vQ
vQ dP d 4 4 . 70. (D) R
V 100 0.5
10 0.25 .
i 10 0.2
57. (C) Human body, though has a large resistance of the l2 l12 l 22 l32
order, of K (say 10K ), is very sensitive to minute 83. (D) R R1 : R 2 : R 3 : :
m m1 m 2 m3
currents even as low as a few mA. Electrons, excites
and disorders the nervous system of the body and 9 4 1
R1 : R 2 : R 3 : : 27 : 6 :1 .
hence one fails to control the activity of the body. 1 2 3
58. (C) Rt = R0 (1 + t)
1 10 3
4.2 = R0(1 + 0.004 × 100) = 1.4R0 84. (B) n 6.25 1015 .
19
R0 = 3 . 1.6 10
2
l2 i i v i1 r2
59. (D) R 85. (B) vd vd
m ne r 2 r2 v' i2 r1
2 2 2
l1 l l v
R1 : R 2 : R 3 : 2 : 3 v'
m1 m2 m3 2
4 t2 5
R1 r2 idt Q i dt (1.2t 3) dt
86. (D) t1 0
R2 r1
5
4 1.2t 2
3r 3t 30C
R 81 256 2
4 R2 R
0
R2 r 256 81 4
R r1
100. (A) In stretching, 2
R1 r2
i
88. (A) vd 4
nAe R2 2
8 R 1
= 28 .
8 10 (2 10 3 ) 2 1.6 10 19 R2 = 16R
= 0.156 × 10–3 m/sec R1 1 l1 A2 2 3 5 5
89. (D) Specific resistance doesn’t depend upon length and R2 l2 A1 3 4 4 8
2
area.
111. (D) The circuit reduces to
90. (D) Heating effect of current.
3
R r2 4.2 3.14 (0.2 10 3 )2
91. (D) l 1.1m
48 10 8
92. (D) For conductors, resistance Temperature and for 3 3
1
semi-conductor, resistance
Temperature
A B
93. (A) If suppose initial length l1 = 100 then
l2 = 100 + 100 = 200
6
2 9 6 9 6 18
R1 l1 100
2 R AB 3.6
R2 = 4R1 9 6 15 5
R2 l2 200 112. (D) As resistance Length
R R 2 R1 4R1 R1 12
100 100 100 Resistance of each arm 4
R R1 R1 3
= 300%. 4 8 8
R effective
94. (A) Ammeter is always connected in series and Voltmeter 4 8 3
is always connected in parallel. 113. (B) Given circuit is equivalent to
95. (C) Same mass, same material i.e. volume is same or
6 3 A 3 C
Al = constant
2 4
l R1 l1 A2 A2 d2
Also, R 6
A R2 l2 A1 A1 d1 A 3
C
4 3
3 3
24 d
16 R2 = 1.5 . B B
R2 d
So the equivalent resistance between points A and
2
6 3
96. (A) I = neqe + npqp = 1mA towards right B is equal to R 2
97. (A) As steady current is flowing through the conductor, 6 3
hence the number of electrons entering from one 114. (D) Potential difference across all resistors in parallel
end and outgoing from the other end of any segment combination is same.
is equal. Hence charge will be zero. 115. (B) Current through each arm DAC and DBC = 1A
VD – VA = 2 and VD – VB = 3
1 A 1
98. (B) Conductance C C VA – VB = +1V
R l l
3r 5r
dQ 116. (D) Reffective = r
99. (C) i 2 2
dt
117. (C) If resistances are R1 and R2 then
R1R 2 6 2
...(i) 1
R1 R 2 8
Suppose R2 is broken then R1 = 2 ...(ii) i1

6 i2 3
On solving equations (i) and (ii) we get R 2 i
5
1.5V

i1 3
i1 = i 2 i2 = 0.5A = i1
118. (C) i2 3
6 12 6
125. (C) Vp Vq (0.5) (2 4) (0.5) 3V
3 12 6
119. (B) Because all the lamps have same voltage. 126. (B)
120. (D) Rseries = R1 + R2 + R3 + ....
121. (A) Current supplied by cell 4 24
2 1 8 20
i A
2 3 5 5 16 4 20
2 3 5 A B A B
16

6 6
i 9
6 12
6 18
2V
So potential difference across 3 will be 24 12
R AB 8
3 1 (24 12)
V 0.6V
5 127. (D) The network can be redrawn as follows
122. (A) According to the problem, we arrange four resistance 3 3 3
as follows A B
Req = 9
A 128. (D) Let the resistance of the wire be R, then we know
that resistance is proportional to the length of the
10 10
R
D B wire. So each of the four wires will have
4
resistance and they are connected in parallel. So the
effective resistance will be
10 10
1 4 R
C 4 R1
R1 R 16

20 20 4 4 6 6
Equivalent resistance 10 129. (D) Equivalent resistance 5 So the
40 4 4 6 6
R1R 2 20
2 current in the circuit 4 ampere Hence the
123. (B) R1 + R2 = 9 and R1R2 = 18 5
R1 R 2
current flowing through each resistance = 2 ampere.
R1 R2 (R1 R 2 )2 4R1R 2 81 72 3 130. (C) Let the resultant resistance be R. If we add one more
branch, then the resultant resistance would be the
R1 = 6 , R2 = 3 same because this is an infinite sequence.
1.5
124. (B) i1 i 2 1amp
3/ 2
R1 = 1 134. (B) The circuit will be as shown
X
A 10 V

R2 = 2 R

5
B A
Y
RR 2 10
R1 R 2R R 2 R2 2R i 2A
R R2 5
R2 – R – 2 = 0 R = –1 or R = 2 ohm 8 4
135. (C) The current in the circuit
131. (B) Cut the series from XY and let the resistance towards 5 1 3
right of XY be R0 whose value should be such that 4 4
when connected across AB does not change the Now VC VE
1 VE V
3 3
entire resistance. The combination is reduced to as
136. (D) According to the figure, (I – I1)R2 = I1R
shown below.
I – I1
E R X R R R C R2
A
I1
R R R R R R
I
B
F R Y R R R R D
E A
A + –
I
= R R0 Only two values satisfying the above relation are
2
and R
B 137. (D) Effective resistance between the points A and B is
F
32 8
R
The resistance across EF, = REF = (R0 + 2R) 12 3
10
(R 0 2R)R R 0 R 2R 2 138. (B) Req = 5 , Current i 2A and current in each
Thus R AB R0 5
R 0 2R R R 0 3R branch = 1A
R 20 2RR 0 2R 2 0 R 0 R( 3 1) 1 3
132. (D) The last two resistance are out of circuit. Now 8 is 3 A
in parallel with (1 + 1 + 4 + 1 + 1) .
3 1
8 i
R 8 || 8 4 i/2 B
2 10V
RAB = 4 + 2 + 2 = 8
133. (A) The given circuit can be simplified as follows
2 2 Potential difference between C and A,
18
4.5 VC – VA = 1 × 1 = 1V ...(i)
7
15V Potential difference between C and B,
15V
1 VC – VB = 1 × 3 = 3V ...(ii)
6 6 18
On solving (i) and (ii) VA – VB = 2 volt
0.5 0.5
10 Shart Trick :
i 2
8 8 (VA VB ) (R 2 R1 ) (3 1) 2V
2 2
On further solving equivalent resistance R = 15
15
Hence current from the battery i 1A
15
1 1 1 1 3 1 2R R
139. (C) R R' R R 2 RR 2R 2 0
R 1 1 1 1 3 (2R R )
Now such three resistance are joined in series, hence On solving the equation we get R’ = 2R.
1 1 1
total R 1 R 2 8 2
3 3 3 159. (D) R AB R = 2 2 .
3 3 3 3
140. (B) To obtain minimum resistance, all resistors must be
connected in parallel. E 10
160. (B) i 0.5 10 = 0.5R + 1.5
R r R 3
r
Hence equivalent resistance of combination R = 17 .
10
151. (D) After simplifying the network, equivalent resistance 3 6
161. (A) Equivalent resistance R 4 6 and main
obtained between A and B is 8 . 3 6
152. (C) The circuit consists of three resistances (2R, 2R and E 3
R) connected in parallel. current i 0.5A
R 6
153. (D) Resistance across the battery is Now potential difference across the combination of
1 1 1 2 1 3 3 6
3 and 6 , V 0.5 1Volt
Rp 3 6 6 6 3 6
The same potential difference also develops across
2 3 resistance.
Rp = 2 I
1A
2 162. (C)
154. (C) The voltmeter is assumed to have infinite resistance. i
Hence (1 + 2 + 1) + 4 = 8 .
R 1
155. (C) R ' 0.1 10
n 10 i/2
A
156. (C) The given circuit can be redrawn as follows i/2
30V 10
2 10
2 2 2 1 2
B

10
2 Equivalent resistance R 10 15 k
2
Req = 5 .
30
R 2R3 Current i = 2 10 3 A
4 4 15
157. (B) R AB R1 R4 = 2 2 6 .
R 2 R3 4 4 Hence, potential difference between A and B
158. (C) Let equivalent resistance between A and B is R’, so 3
2 10
given circuit can be reduced as follows V 10 103 10 Volt.
2
R R R
A 9
163. (A) Equivalent resistance R
9
R 2R 2R 2R
1A 1A 1A 1A 1A 1A 1A 1A 1A
+
B 9V 9 9 9 9 9 9 9 9 9
R –
A
A
R 2R R

B 9
Current i 9A
1
Current passes through the ammeter = 5A.
164. (B) The figure can be drawn as follows 169. (C) The given circuit can be simplified as follows
D 7 C 10
C
3
3 R Q
10
3 5
5 10
10
10
3
A P Q
A B B
10
10
C (3 + R)
10 10 (3 R) 30 10R
R 3 3
5 10 3 R 13 R
5 39 3R 30 10R 69 13R
R
13 R 13 R
A A B 13R + R2 = 69 + 13R R 69 .
B 10
170. (A) The circuit can be drawn as follows
RAB = 5 .
A
1l1 2l2
165. (A) R1 and R 2 . In series Req = R1 + R2
A A 3 3
eq. (l1 l 2 )
1l1 2l2 1l1 2l 2
eq i1
A A A l1 l 2 B C
166. (C) The figure can be drawn as follows i2 3
i
100 C
D
2
3 (3 3)
Equivalent resistance R 2
100 100 3 (3 3)
2 3 1
Current i 1A. So, i1 1 A.
2 3 6 3
100 B Potential difference between
A
1
200 200 A and B = 3 1volt.
R AC 100 . 3
200 200
1 1 1 1 4 2 1 8
171. (A) R eq .
R eq 2 4 8 8 7
1 1 1 1 1
A B 172. (B) The given figure is balance wheatstone bridge.
167. (C)
7 1 1
173. (B) R 3
12 4 R
1 1
R AB 2 2 . 174. (D) Suppose resistance of wires are R1 and R2 then
3 3
6 R1R 2
1 r1 . If R2 breaks then R1 = 2
168. (B) same, l same, A 2 A1 (as r2 ) 5 R1 R 2
4 2
By using 6 2 R2
Hence, R2 3 .
R1 A2 R1 5 2 R2
l 1
R R1 2 175. (D) Potential difference across PQ i.e. p.d. across the
A R2 A1 8 4
resistance of 20 , which is V = i × 20
R1R 2 2 8 8
Hence, R eq . 48
R1 R 2 (2 8) 5 and i 0.16A
(100 100 80 20)
V = 0.16 × 20 = 3.2V.
C Series 194. (D) The equivalent circuits are as shown below
100 + 60 = 160 C
C
100 60
176. (A) A B
A B A
40
160 40
R 32 .
160 40 D B D
177. (A)
C
A A 2 2
2
2 A
2 2 B
2
2 2
B B 2 2
2 2 D
R AB 1 .
2 2 Clearly, the circuit is a balanced Wheatstone bridge.
178. (A) Given circuit is a balance Wheatstone bridge circuit. So effective resistance between A and B is .
179. (B) All of th ree resistance are in parallel So,
R R 195. (A) By the concept of balanced Wheatstone bridge, the
R' .
n 3 given circuit can be redrawn as follows

180. (B) Req = R1 + R2 eff .2l 1l 2l 30


A A A 5 10 15
1 2
eff . A B
2
191. (C) Potential difference between B and D is zero, it means
Wheatstone bridge is in balanced condition
B 10 20 30
X 8X 60
21 6 3+
8 (8 + X)
15 30 60
3 R AB 20
(30 60)
A C 196. (A) The given circuit is a balanced Wheatstone bridge
6 type, hence it can be simplified as follows
18 15 4
6 4 5
6 4 2 3
D
P R 21 18 A B
So 8X X=8
Q S 6 4 6
3
(8 X)
192. (A) This is a balanced Wheatstone bridge. Therefore no 10
current will flow from the diagonal resistance 10
Equivalent resistance
(10 10) (10 10) 197. (B) Let current through 5 resistance be i. Then
= 10
(10 10) (10 10)
10
193. (B) This is a balanced Wheatstone bridge circuit. So i × 25 = (2.1 – i)10 i 2.1 0.6 A
35
potential at B and D will be same and no current
flows through 4R resistance.
198. (D) Let the value of shunt be r. Hence the equivalent A’(5 ) is obtained by connecting a 10 resistance
Sr in parallel with A.
resistance of branch containing S will be 205. (D) Given circuit is a balanced Wheatstone bridge circuit.
S r
So there will be no change in equivalent resistance.
Sr
Hence no further current will be drawn.
In balance condition, P (S r) .
206. (A) No current flow through vertical resistances
Q R
3 3 3
This gives r = 8 A B
199.(B, C)
C 2R
R R 3 3 3
B R D B R 9
D
A B
R R 2R
A
1 1 1 1 R
R BD 9
R BD 2R R 2R 2
9
Between A and C circuit becomes equivalent to R AB .
2
balanced Wheatstone bridge so RAC = R.
207. (D) The given circuit is a balanced Wheatstone bridge.
1 208. (C) The given circuit can be redrawn as follows
200. (B) i
R
A
201. (D) Equivalent resistance between P and Q R
1 1 1 1 48
R PQ
R PQ (6 2) 3 (4 12) 25 R R R
Current between P and Q; i = 1.5A
So, potential difference between P and Q
48 R
B
VPQ 1.52.88V .
25 Equivalent resistance between A to B is R.
202. (C) Given circuit is a balanced Wheatstone bridge i.e. 209. (D) Equivalent resistance of the given circuit is 3 .
potential difference between B and D is zero. Hence,
no current flows between B and D. 210. (C) R R
203. (A) The given circuit is a balanced Wheatstone bridge,
R R
hence it can be redrawn as follows R R
7 A R B
3 4
R R R
R

A B R R
6 8
14 2R/3 2R/3

R
7 14 14 A B
R eq .
(7 14) 3
204. (A) For a balance Wheatstone bridge. 2R/3 2R/3
A D 10 4
(Unbalanced) 2R
B C 5 4 Hence R eq .
A' D A' 4 3
A’ = 5
B C 5 4
P R i R2 l2 r1
2 2
212. (B) For balanced Wheatstone bridge 3 2 1
Q S 218. (B) 1
i2 R1 l1 r2 4 3 3
12 x 6
x=6
1 1 A A
2 2
213. (B) For maximum energy equivalent resistance of
combination should be minimum. R=4 R=4
10 R1 50 219. (C)
214. (C) For first balancing condition
R2 50
R2 = 10 + R1. For second balancing condition B B
R1 40 R1 2
R 2 60 10 R1 3 1 1 1 1 1 1 1 19
220. (A)
R1 = 2 R eq R1 R2 R3 2 4 5 20
215. (B) Given R = 6 . When resistor is cut into two equal
20
parts and connected in parallel, then R eq
19
R 231. (C) In charging V > E.
R 6
R eq 2 4 4
1.5 232. (D) In open circuit of a cell V = E
2
233. (A) Zero (Circuit open means no current and hence no
216.(A) Resistance between P and Q potential difference across resistance).
5 234. (D) Zero (No potential difference across voltmeter).
R R RR
6 5 235. (B) Let the e.m.f. of cell be E and internal resistance be r.
R PQ R || R
3 2 5 11 E E
R R Then 0.5 and 0.25
6 (r 5)
(r 2)
Resistance between Q and R
5 r
R 4R On dividing, 2 r = 10
R R 2 r
2 3 4
R QR || R R 236. (C) In short circuiting R = 0, so V = 0
2 3 R 4R 11
2 3 E 1.5
237. (C) Short circuit current iSC 3
Resistance between P and R r r
r = 0.5
R 3R
R R 3 2 3 50 50 5
R PR || R R 238. (C) i r 10 1.1
3 2 R 3R 11 R r 4.5 4.5
3 2 239. (D) (4 + r)i = 2.2 ...(i)
Hence it is clear that RPQ is maximum. 1
and 4i = 2 i
217. (C) Given circuit can be redrawn as follows 2
2 Putting the value of i in (i),
1.5
we get r = 0.4 ohm.
240. (B) Let the internal resistance of cell be r, then
6
6V 3 6V 3 E 1.5
i 15 0.06
R r 0.04 r
241. (C) The voltage across cell terminal will be given by
E 2
R 3.9 1.95V
R r (3.9 0.1)
242. (C) E = 2.2 volt, V = 1.8 volt, R = 5R
6V 3 3
E 2.2
r 1 R 1 5 1.1
V 1.8
6
i 4A
1.5
243. (B) In parallel, equivalent resistance is low E 1.5
254. (A) i 30A
r 0.05
E
i 12
r 255. (A) i 2A
R (4 2)
n
Energy loss inside the source
244. (D) Internal resistance distance
= i2r = (2)2 × 2 = 8W
1 256. (B) V2 – V1 = E – ir = 5 – 2 × 0.5 = 4 volt
concentration
Area V2 = 4 + V1 = 4 + 10 = 14 volt
245. (A) Total e.m.f. = nE, Total resistance R + nr 257. (A) If m = Number of rows
nE and n = Number of cells in a row
i
R nr Then m × n = 100 ...(i)
246. (A) Current through R is maximum when total internal nr
resistance of the circuit is equal to external resistance. Also condition of maximum current is R
m
247. (B) Cells are joined in parallel when internal resistance
is higher then a external resistance. (R <<r) 1 n
25 n = 25 m ...(ii)
m
E
i On solving (i) and (ii) m = 2
r
R 258. (B) According to Kirchhoff’s law
n
iCD = i2 + i3
2E E
248.(B) In series , i1 259. (B) Since i
2 2r , we get
R r
E 2E
In parallel, I2 E
r 4 r 0.5 ...(i)
2 2 r
2
2E 2E E
Since i1 = i2 r=2 0.25 ...(ii)
4 r 2 2r 5 r
249. (A) Applying Kirchhoff law 5 r
Dividing (i) by (ii), we get 2 r=1
20 2 r
(2 + 2) = (0.1 + 0.3 + 0.2)i i A
3 E
Hence potential difference across A 0.5 E = 1.5V
2 1
20 4 r
2 0.1 V (less than 2V) 260. (C) Because Eeq = E and req
3 3 2
20 271. (B) In parallel combination Eeq = E = 12V
Potential difference across B 2 0.3 0
3 E 6
273. (B) i 12amp.
250. (B) Here two cells are in series. r 0.5
Therefore total emf = 2E. 274. (C) Strength = 5 × 18 = 90AH.
Total resistance = R + 2r
E 5
275. (A) i 1A
R r 4.5 0.5
2E 2 1.45 2.9 29 V = E – ir = 5 – 1 × 0.5 = 4.5 Volt
i 1.611amp
R 2r 1.5 2 0.15 1.8 18 276. (B) The circuit can be simplified as follows
251. (A) E = V + ir
B C
After short-circuiting,V = 0;
i1 30
E 2 i3
r 0.5 i3
i 4 A D
252. (C) By Kirchhoff’s current law. 40 40V
i2
253. (B) For power to be maximum
External resistance = Equivalent internal resistance F E
of the circuit 40 80V
Applying KCL at junction A 288. (D) Applying Kirchhoff law in the first mesh
i3 = i1 + i2 ...(i) 10
Applying Kirchoff’s voltage law for the loop ABCDA 10 = 5 × i 2A i
5
–30i1 – 40i3 + 40 = 0 289. (B) Applying Kirchhoff law in the first mesh
–30i 1 – 40(i1 + i2) + 40 = 0 10 = 5i1 + i ...(i)
7i1 + 4i2 = 4 ...(ii)
i – i1 i
Applying Kirchoff’s voltage law for the loop ADEFA.
–40i2 – 40i3 + 80 + 40 = 0 i1
10V
–40i2 – 40(i1 + i2) = –120 A
i1 + 2i2 = 3 ...(iii) 4
On solving equation (ii) and (iii) i1 = –0.4A. 5
277. (C) V = E – ir = 12 – 60 × 5 × 10–2 = 9V.
278. (A) Applying Kirchoff’s voltage law in the loop Applying in the second mesh
10 5V 5i1 = 4i – 4i1 ...(ii)
40
i Solving equation (i) and (ii), we get i1 A
29
290. (A) Given problem is the case of mixed grouping of cells
A B
i nE
So total current produced i
nr
2V 20 R
m
–10i + 5 – 20i – 2 = 0 i = 0.1A
Here m = 100, n = 5000, R = 500
279. (D) V = E – ir = 1.5 – 2 × 0.15 = 1.20Volt.
E = 0.15V and r = 0.25
E 4
280. (B) i 1 r 2 5000 0.15 750
R r 2 r i 1.5A
5000 0.25 512.5
Short circuit when terminals of battery connected 500
100
directly then current flows which is
301. (D) Potentiometer works on null deflection method. In
E 4 balance condition no current flows in secondary
iSC 2A .
r 2 circuit.
2 2 4 ig G 10 99
281. (C) i A 302. (C) Shunt resistances S 11
1 1.9 0.9 3.8 (i i g ) (100 10)
4 V
For cell A E = V + ir V 2 1.9 0. 303. (D) By using R
3.8 G
Ig
E
282. (C) By using i
R r 100
R 5 19,995
E 5 10 3
0.5 E = 5.5 + 0.5r (i) 304. (A) Potential gradient = Change in voltage per unit length
11 r
E V2 V1
and 0.9 E = 4.5 + 0.9r (ii) 10 V2 V1 3 volt
30
5 r
On solving these equation, we have r = 2.5 100
V 5 5000
305. (D) R G 2 2 48
284. (B) W = qV = 6 × 10 × 9 = 54 × 10 J.
–6 –6
ig 100 100

V2 103
285. (A) P ; for P to be maximum Req should be less.
R eq iS 50 12
306. (C) i g 10 12 + G = 60
Hence option (A) is correct. S G 12 G
G = 48
E2 (2)2 307. (A) To convert a galvanometer into a voltmeter, a high
286. (C) Pmax 2W
4r 4 0.5 value resistance is to be connected in series with it.
318. (A) When the length of potentiometer wire is increased,
P R
309. (C) (For balancing bridge) the potential gradient decreases and the length of
Q S previous balance point is increased.
B 321. (B) The actual circuit is same.

P=9 Q = 11 i
322. (B) ig = 10% of i
10
A C
G 90
6 S 10
(n 1) (10 1)
4=4
r E1 l1 l 2 (8 2) 5
D S 323. (B)
E2 l1 l 2 (8 2) 3
324. (B) Suppose resistance R is connected in series with
4 11 44 voltmeter as shown.
S
9 9
ig ig R
1 1 1 G
S r 6
V (n– 1)V
9 1 1
nV
44 6 r
By Ohm’s law
132 ig.R = (n – 1)V
r 26.4
5
V
l1 l2 25 R = (n – 1)G (where i g )
310. (A) r R 2 0.5 G
l2 100 325. (C) Ammeter is always connected in series with circuit.
311. (B) The sensitivity of potentiometer can be increased 326. (C) If resistance of ammeter is r then
by decreasing the potential gradient i.e. by increasing 20 = (R + r)4 R + r = 5 R < 5
the length of potentiometer wire.
ig G 3
10 10 50 50
1 327. (B) S in parallel.
(Sensitivity Length ) i ig 1 10 3
10 99
P.G.
312. (B) In balance condition, potentiometer doesn’t take the i
current from secondary circuit. 328. (B) ig = (100 – 90)% of i
10
313. (A) Here same current is passing throughout the length
G 900
of the wire, hence V R l Required shunt S 100
(n 1) (10 1)
V1 l1 6 300
V2 = 1 V.. V 100
V2 l2 V2 50 329. (D) R G 25 9975
ig 3
10 10
ig G 10 0.01 10
314. (A) S V iR
i ig 10 0.01 999 330. (B) Potential gradient x
L L
315. (A) Ratio will be equal to the ratio of no deflection lengths
2 15 3
E l1 2 x volt / cm
i.e. 1 (15 5) 10 2000
E2 l2 3
G 25
331. (A) S
Potential difference i 5
316. (A) Potential gradient 1 1
Length ig 50 10 6

317. (A) Wheatstone bridge is balanced, therefore 25 25 4


5
2.5 10
P R 10 10 1 105
or 1 S = 10
Q S S
332. (B) In balanced Wheatstone bridge, the arms of i G S ig S 2.5 1
galvanometer and cell can be interchanged without 343. (A)
ig S i G S 27.5 11
affecting the balance of the bridge.
333. (C) Error in measurement = Actual value – Measured 80
value 344. (C) Total resistance of the circuit 20 60
2
998 2 1
Main current i A
V 60 30
Combination of voltmeter and 80 resistance is
connected in series with 20 , so current through
i 1
20 and this combination will be same A.
30
Since the resistance of voltmeter is also 80 , so this
–
+ current is equally distributed in 80 resistance and
2V 2
1
voltmeter (i.e. A through each)
Actual value = 2V 60
2 1 1
i A P.D. across 80 resistance 80 1.33 V
998 2 500 60
Since E = V + ir
L
1 998 i
V E ir 2 2 V V iR A i
345. (A) Potential gradient x
500 500 L L L A
998 10
Measured value V 346. (D) Here n 5
500 2
998 R = (n – 1)G = (5 – 1)2000 = 8000
Error 2 4 10 3 volt
500 l1 l2
334. (D) The emf of the standard cell must be greater than 347. (B) r R 0.5 .
l1
that of experimental cells, otherwise balance point is
not obtained.
e
336. (B) In general, ammeter always reads less than the actual
349. (B) V i.R. .R 10 3
(R Rh r)
value because of its resistance.
2
R AC 20 10
337. (C) By Wheatstone bridge, (10 R r)
80 BC 80
R = 20 R = 19,989 .
338. (A) E l (balancing length)
351. (C) 2R > 20 R > 10 .
l1 l2 l1 2
339. (B) r R' 5 ... (i) i G 4 R R
l2 2 352. (C) 1 1 S
ig S 1 S 3
l1 3 353. (A) When ammeter is connected in parallel to the circuit,
and r 10 ... (ii)
3 net resistance of the circuit decreases. Hence more
On solving (i) and (ii) r = 10 current is drawn from the battery, which damages
the ammeter.
i G l1 l2 55 50
341. (D) By using 1 354. (A) r R' r 10 1
ig S l2 50

i 1000 1000 V 18
1 S 111 355. (B) R G
3
12 5988
100 10 3 S 9 ig 3 10
V e R
342. (C) Potential gradient x
L (R Rh r) L V 6
357. (C) R G 25 975 (In series).
3 2.2 ig 6 10 3
2.2 10 1 R’ = 990
(10 R h )
S S 100 10
358. (D) i g i 0.01 10 Vi V V
G S 25 S (100 10) 11
25 Finally after connecting voltmeter across 100
1000S = 25 + S S .
999 100 900
Equivalent resistance 90
X 6 (100 900)
Final potential difference

G 900

4 6
A B
C 10 100
359. (C)
Vf

5V
Resistance of the part AC
RAC = 0.1 × 40 = 4 and RCB = 0.1 × 60 = 6 V
X 4 90 9
In balanced condition X 4 Vf V V
6 6 (90 10) 10
Equivalent resistance Req = 5 Vi Vf
% error = 100
5 Vi
so current drawn from battery i 1A .
5 10 9
V V
V 11 10 100 1.0.
360. (A) (R + G)ig = V (R G) 10
ig V
11
ig R
e.R 10 3
G 374. (B) Potential gradient = = .
(R r).L (3 3) 5
3 = 1V/m = 10 mV/cm.
6.25k
30 16 10 6 i G 1 100
375. (C) 1 1
Value of R is nearly equal to 6k ig S 5 S
10
This is connected in series in a voltmeter.
100 3
i G S 10 .
371. (A) 1 105
ig S
ig 4 4S 1
3
376. (D)
i.G G 100 10 40 40 i G S 36 4 40 10
1 1
Vg S 3 S V 6 6
800 10 377. (A) i 2 R=1 .
R 6 3 2 R
S = 10 . R
6 3
S 3 S 3
372. (A) i g i 10 10 100 10 S 0.01 5
G S 100 S 378. (B) i g i
G S 10 50 S
1000
90 S = 1000 11.11 .
S 50
90 S 0.05 .
999
373. (C) Before connecting the voltmeter, potential difference
across 100 resistance 100 l
379. (D) S .R
10 100 l
100 l
Initially, 30 10 l 25cm
l
Vi
100 l
Finally, 10 30 l 75cm
l
So, shift = 50cm.
V
380. (C) Potential gradient
l1 150
7 386. (B) Using r R 1 2 1 1
i 0.1 10 l2 100
(x) 10 2 V/m
A 6
10 387. (D) Resistance between
381. (D) Before connecting voltmeter potential difference 1000 500 1000
across 400 resistance is A and B
(1500) 3
10,000 So, equivalent resistance of the circuit
V
10V

A 400 B 800
1000
V

6V A B C
500 500
400
Vi 6 2V
(400 800)
After connecting voltmeter equivalent resistance 1000 2500
400 10,000 500
between A and B 384.6 3 3
(400 10,000)
Current drawn from the cell
Hence, potential difference measured by voltmeter
384.6 10 3
Vf 6 1.95V i A
(384.6 800) 2500 250
Error in measurement = Vi – Vf = 2 – 1.95 3
= 0.05V. Reading of voltmeter i.e.
i G 5 50
382. (C) 1 1 3 1000
ig S 0.05 S potential difference across AB 4V
250 3
50 l
S i
99 A 388. (D) i g
10
50 2.97 10 2 10 4
l 3m . G 90
99 5 10 7 Required shunt S 10
(n 1) (10 1)
i G 10 0.81
383. (A) 1 1 S = 0.09 .
ig S 1 S 50
389. (B) i g 40 4960
3
384. (A) From the principle of potentiometer V l 10 10
V l 390. (C) Post office box is based on the prin ciple of
; where V = emf of battery, E = emf of Wheatstone’s bridge
E L
standard cell, L = Length of potentiometer wire
El 30E . Exercise - II
V
L 100 (Previous Year AIPMT Examination Questions)
e R 1. (B) For balanced wheatstone bridge circuit
385. (B) E . l
(R R h r) L
2 2
S 6 3S S 3
3 2 10 2 6S .
10 10 0.4
(10 R 0) 1 S+6
R = 790
R 11l
2. (A) 10 l (11) 2
So R' 1.21R
10A A (10)2
11
Now resistance becomes 1.21 times of initial and
10V
specific resistance is the intrinsic property so
remains same.
The equivalent resistance of the given circuit is
7. (B)
5R
Req
5 R A B
Power dissipated in the given circuit is

V2 (10)2 E F
P 30
R eq = 5R
5 R
D C
150 R = 100(5 + R)
150 R = 500 + 100 R
50 R = 500 Applying Kirchoff’s equation to the loop ABFE,
500 – (i1 + i2)R – i1R1 + 1 = 0
R 10 .
50 (i1 + i2)R – i, r + 1 = 0
3. (C) Power = 220 V × 4A = 880 watts. = 880 J/s. or 1 – (i1 + i2)R – i1r1 = 0.
Heat needed to raise the temperature of 1 kg water 8. (B) Total resistance of wire = 12 × 2 × 10–1
through 80° C = 2.4
ms. T 4.2 J/cal 2.4
Resistance of each half = 1.2
1000 g × 1 cal/g 80 4.2 J/cal. 2
1000 1 80 4.2 and as about diameter both parts are in parallel
time taken
880 R
Req. =
2
336 103
= 382 s = 6.3 min. 1.2
880 Req. 0.6 .
2
E
4. (A) E l1 and E l2 V
R r
r+R l1 l1
r 1 R 9. (C)
R l2 l2
110 100 I
= 10 1 V = – Ir, comparing with y = c – mx
100
5. (D) Voltage across all three branches are same, i.e., Slope = –r, internal resistance.
V1 = V2 Vmax = emf . This is intercept on the y–axis
3 × 2 = i × (5 + 1) Slope is negative
i = 1A I decreases as R increases.
Power dissipated in 5
P = i2 R 10. (C) qV = 2eV
= (A)2 × 5 = 5 watt. 1.6 × 10–19 V = 2 × 1.6 × 10–19 V
l V = 2V
6. (B) R
A V
E=
11l d
Now, l l
10 10 2V
E= 5 107 .
and therefore, A
10A 4×10 –8
11
11. (B) R k 1 and R X k 2. V
16. (C) I =
Req
12. (D) Both (A) and (B) are correct 1
E I= A
50
13. (A) I
R+r VB = I R
E 1
2 ...(i) = × 100 = 2 V
2+r 50
E
0.5 ...(ii)
9+r 17. (A) SinceR l.
(i) divided by (ii) According to problem R1 + R2 = 12 and
9 r R1 R 2 8
4 = 8 + 4r = 9 + r or 3r = 1
2 r R1 R 2 3
1 On solving this we get R1 = 4 and R2 = 8 therefore
r .
3 l 1
the ratio 1 .
14. (B) Ig2 R ,P = I2 R l2 2

36 = I2 (9) V2
18. (D) Power P
I9 = 2A R
1 P 2 V
In parallel I For small variation, 100% 100%
R P V
I9 6 2 6 therefore power would decrease by 5 %
I6 9 = I6 9 r
I6 = 3A
Ickt = 2 + 3 = 5A I I
V2 = IR = (5)(2) = 10 volt. 19. (C)
R1 1A D 2A 2V R
15. (D) B
1A 1A
2
Current in the circuit, I
R2 2A R r
Potential difference across R,
A
1A C 2A V IR R
1V R r
Applying Kirchhoff voltage law in the circuit as When R = 0, V = 0
shown in the figure below. R = , V=
2A 2V Hence, option (c) represents the correct graph.
D
B
1A 20. (B) The situation is as shown in the figure.
2 B

P Q

A
1A C A G C
1V
VA + 1 + 2(1) – 2 = VB
R S
I I
0 + 1 = VB ( VA = 0 V (Given)) D

VB = +1V
For a balanced Wheatstones’s bridge 23. (B) Resistance (0.5 / km) (150 km) = 75
P R Total Voltage drop
Q S = (8 V/km) (150 km) = 1200 V
10 30 1 1 ( V) 2 (1200) 2
30 90 or Power loss = W
3 3 R 75
It is a balanced Wheatstone’s bridge. Hence no = 19200 W = 19.2 kW
current flows in the galvanometer arm. Hence,
5 1 5 1.6 1
resistance 50 becomes ineffective. 24. (B) and
R 100 1 R 100 1.6 1
The equivalent resistance of the circuit is
2
R1R 2 R = 15
Req =
R1 R 2 25. (C) Internal resistance,
(10 30 ) (30 90 ) E V
R eq 5 r R= 1 2
R
(10 30 ) (30 90 ) V 2
(40 ) (120 ) 3 2.85
5 = (9.5) 0.5
40 120 2.85
= 5 + 30 = 35 26. (B) As all the cross sections of the conductor are
Current drawn from the cell is considered to be in series, the current should be
7V 1 constant.
I A = 0.2 A
35 5 27. (B)
21. (B) I Req = R1 + R2
R r
R 2 2 1 2
eq .
eq A 1A 2A 1 2
I I 28. (A) VA = VB = VC

r ir E0 r
29. (C) Potential gradient x = =
or IR + Ir = L (r1 r) L
Here, R = 10 , r = ?
E0 r r
= 2.1 V, I = 0.2 A e.m.f. E xl
(r r1 ) L
0.2 × 10 + 0.2 × r = 2.1
30. (B)
2 + 0.2r = 2.1
1
0.2r = 0.1 or r 0.5 .
2
22. (D) Resistance of a wire,
l
R 4 ...(i)
A
When wire is streched twice, its new length be l’.
Then
l’ = 2l
On stretching volume of the wire remains constant. 480 20
R eff 40.8 40.8 19.2 60
lA = l’ A’ where A’ is the new cross – sectional area 480 20
l l A Veff
or A' A A I= 0.5 A .
l' 2l 2 R eff
Resistance of the stretched wire is
l' 2l 1 30 1
R' 4 I= = A
A' A A 60 2
2
= 4(4 ) = 16 (Using (i))
31. (D) E1 + E2 = K(50) 38. (C)
E1 – E2 = K(10)
E1
E2 1
E2
E1 E 2 5 5
E1 =
E1 E 2 1 E2 1 1 n
E2 i= = independent of n
nr r
E1 3 39. 10 is in series with ideal voltmeter. Therefore it will
not affect the circuit.
E2 2.
10
3V (O) i1 = = 1A
32. (B) 10
9V 2A 5V 2V VB 10
VA – VB = 9 volt. i2 = = 1A
10
33. (A) v1 = 10v
500W
v2 = 10v
+(100V)–+(130V)– I 40. A B

230V
500
i 5A
100
so 130 = 5R
R = 26
34. (C) Reading of potentiometer is accurate because during
taking reading it does not draw any current from the
circuit.
E
R2 l22 n2l12 R2 Case – I
35. (C) R 2 = R
n2 2R
1 l12 l1 1 R eq1 =
3
= R2 = n2R1.
36. (B) 4 Yellow, 7 Violet, 103 Orange, E 2 3P
Peq1 = =
2R 2
4.7
tolerance = ×100 = 10% (silver) 3
47
37. (C) E R

E E
=I = (n +1)I
R+nR R
E E (n +1)10I
= 10 I = E
R R n
R+ Case – II
n
R 3R
(n +1)10 I R eq 2 = +R =
(n +1) I = n = 10 2 2
n
E 2 2P
Peq2 = =
3R 3
2
Exercise - III 8. (A) Given: Current (I) = 4 A; Time (t) = 40min = 2400 sec
and weight deposited on cathode (m) = 4.5 g. From
(Previous Year AIIMS Questions) the Faraday’s law of electrolysis.
1. (A) Given: Internal resistance of the cell (r) = 0.1 ; E.M.F. The weight deposited (m) = 4.5 = z × I × t or
of the cell (E) = 2 V and external resistance (R) = 3.9
4.5 4.5
. Voltage (V) = E – Ir z= = 47 10 –5 g/C.
I× t 4 2400
E 2
= E– .r = 2 – ×0.1 = 1.95 V. (where z is the electrochemical equivalent of zinc).
R+r 3.9 0.1 9. (D) Given: E.M.F. of the battery (E) = 10volts;
2. (A) Given: No. of wires = 10 and resistance of each wire
Internal resistance (r) = 3 and
= 1 . Relation for equivalent resistance of wires
connected in parallel E
current (I) = 0.5 A. Current (I) =
1 1 1 1 1 R+r
..... 1 10
R eq 1 1 1 1 R eq = = 01. . or 0.5 or 0.5R + 1.5 = 10
10 terms 10 R 3
3. (D) If a current I in a conductor is uniformly distributed or 0.5R = 8.5 or R = 8.5 = 17 .
over the area of cross-section (A) of theconductor, 0.5
I 10. (D) As R2, R3 and R4 are in series, their equivalent
then the ratio is called the currentdensity at any resistance is R2 + R3 + R4 = 6 . Now the 6 resistance
A is in parallel with R1 = 2 whose equivalent
point on the area. Its unit is amp/m2 and it is a vector
quantity. 2×6 3
resistance will be =
4. (D) This is a case of balanced Wheatstone bridge. Since 2+6 2
the resistances in upperand lower arms are in series, I
therefore equivalent resistance in upperarm (R) = R1
+ R2= 20 + 20 = 40 and equivalent resistance of
lower arm
R’= R3 + R4= 20 + 20 =40 . Now the equivalent 3V 3/2
resistances R and R’ are in parallel. Therefore, their

resultant resistance R"=


40× 40 1600
= = 20 .
40 + 40 80
3
5. (B) Given: Initial number of cells (n1) = 10; Potential of The current through the circuit, I =2A .
each cell = E; Internal resistance of each cell = r and 3
final number of cells (n2) = 3. 2
From the Ohm’s law, total voltage of ten cells 11. (B) Since the resistances R2 and R3 are in parallel, their
= 10 × E= 10 E and total resistance in ten cells = 10 × 4× 4
equivalent resistance will be R23= =2
10E E 2× 4
r = 10r. Therefore current in the circuit (I) = = Now the equivalent circuit will be
10r r
or potential difference across three cells R1 = 2 R4 = 2
A B
E R23 = 2
= I×3r = ×3r = 3E.
r There are three resistances in series.
(Since the voltmeter is ideal, therefore it will read Therefore the equivalent resistance will be
3E).
RAB = 2 + 2 + 2 = 6 .
6. (C) Given: Power of heater (P) = 100 W and time
12. (A) Metals do not become hard when electroplated.
(t) = 2 min = 120 sec.
13. (D) In steady state condition the current at each cross-
Work done to produce heat (W) = P × t
section will be the same. Drift velocity and electric
= 100 × 120 = 12000 J. field depend on the current density. Current density
12000 which varies from one cross section to another.
W= = 2.8 kcal.
4.2×1000 14.(C) The equivalent circuit may be shown as :
7.(B) Seebeck discovered that on heating one of the 1
junctions of a thermocouple, when keeping the other
cold, a current is produced in the circuit. This effect 1 R
is known as Seebeck effect. Therefore,thermo couple
thermometer is based on Seebeck effect.
1
where R may be treated as the total resistance.
1× R l1 A2 r22 0.5r
2
1
Hence 1 + +1= R. or = = = = .
1 R l2 A1 r12 r 4
This gives R = 1 ± 3
1 1
The resistance (R) = .
Since 1 – 3 gives –ve value,therefore A A
R = 1+ 3 . R1 l1 A2 1 1 1
Therefore
15. (B) Required ratio R2 l2 A1 4 4 16
or R2 = 16R1 = 16R.
i2 R + i2 R 19. (B) Antimony and bismuth are the pair of metals
2 2
4 :1.
1 1 producing a higher thermo e.m.f. in comparison with
R+ R other pairs of metals. Therefore, this pair is usually
2 2
used in thermocouple.
R 20. (C) Let the resistance of the rod be R. If the rod is divided
into two equal parts, then each part of the rod will
i/2 R
have a resistance of . Now these two parts of the
i i 2
i/2 rod are connected in parallel and a voltage V is
applied across them.
Parallel Combination
Hence th e total power consumed is
R R
V2 V2 V2 V2
P' = + =4 = 4P where P = is the
R R R R
Series Combination
2 2
16. (B) The power of the incandescent lamp
power consumed by the original rod when voltage V
P = 60 W is applied across it.
Operating voltage V = 120 V 21. (D) Actual power of bulb (P1) = 40 W
Current through the filament Actual voltage of bulb (V1) = 200 V and supply
P 60 1 voltage (V2) = 100 V.
I= = = A
V 120 2 V2 P1 V12
Power (P) = V 2 . Therfore
1 R p2 V22
Now Q = I × t. For t = 1 sec, Q = I = C
2
1 40 (200) 2 40
or, 4 or P2 10W
ne = where e = electronic charge, and n = P2 (100) 2 4
2
number of electrons. (Where P2 = power when voltage is 100.V)
1 1 22. (D) Let the original diameter of the wire be D.
n= = =3.12×1018
2e 2×1.6×10–19 D
Therefore the new diameter is .
17. (B) From the given circuit, we find that RBD is ineffective 2
as it is a balanced Wheatstone bridge. Since both D2
the upper resistances are in a series combination, Original area of cross-section is and the final
4
therefore their equivalent resistance (RU) = 2 + 2 = 4
. Similarly both the lower resistances are in a series area of cross - section is D2 .
combination, therefore their equivalent resistance 16
(RL) = 4 + 4 = 8 . Now we see that both the resistance The new length of the wire is given by
RU and RI are in a parallel combination. Therefore
equivalent resistance between D2 D2 16
L× = L'× L'
L = 4L
RU RL 4 8 32 4 16 4
A and C . L
RU RL 4 8 12 Resistance is given by R = .
18. (A) Initial radius of wire, r 1 = r and final radius of wire,
A
r L' 4L D2 A
r2 0.5r. Since volume of the wire after stretch R' = 16R. = =
2 A A 16 4
remains constant, therefore l1A1 = l2A2 4
R’ = 16× 10 = 160 .
23. (A) In each row all the 5000 electro plaques are 10
connected in series. Therefore,
equivalent emf = 0.15 × 5000 = 750V.
X Y
Equivalent resistance = 0.25 × 5000 = 1250 . 10
All 100 rows are connected in parallel. 10 10
Therefore voltage = 750 V. R XY 5 .
10 10
1250 b
Equivalent resistance = = 12.50
100 27. (A) We know that J AT 2 e T
Thus resultant circuit of eel becomes –b
J
= Ae T
750 V 12.50 T2
Taking log on both side, we get
b
J T
loge loge A.e
T2
500 J b
loge 2
loge A –
V 750 T T
Current across 500 = = =1.46A Comparing with y = mx + C
R 500 +12.50
I 1.5 A. 28. (A) With increase in frequency, reactance of LC circuit
24. (C) As the voltage in R2 and R3 is same therefore will increases. As a result, voltage increases. So,
according to, brightness will increase.
29. (C) When heater wires are connected in series, then
V2 equivalent resistance.
H= .t , R2 = R3
R Rs = R1+ R2 = 2R( R1 = R2)
Also the energy in all resistance is same.
R1 V2
i A B Rate of heat produced, Hs
Rs
i1 i2 V2
Hs ... (i)
2R
Vin
R2 R3 R 1R 2 R.R
In second case, R p = =
R1 + R 2 2R
R
D C or R p =
2
i 2 R1t = i12 R2 t
V2 2V 2
R3 R3 1 Rate of heat produced, H p = Hp =
Using i1 = i= i= i Rp R
R 2 + R3 R3 + R3 2
Dividing (i) by (ii), we get
2 i2 R2
Thus i R1 t = R 2 t or, R1 V2
4 4
25. (A) Hs 2R v2 R 1
=
T Hp 2V 2 2R 2V 2 4.
10 R
5 30. (C) Resistance of 25W, 200 V bulb
26. (B) 10
X Y 200 2
R1 = .
10 100
T Resistance of 100 W, 200 V bulb
5 200 2
5 R2 = .
100
X Clearly, R1 > R2. H1 > H2.
Y
10 25 W bulb will glow more brightly.
31. (C) Here p-n junction is forward biased. If p-n junction B
is ideal, its resistance is zero. The effective resistance
across A and B 5
5
10×10 5
= =5k A C
10 +10
5
30 2 5
Current in the circuit I = = 3A
3
10 + 5 10 10
D
I 1
Current in each arm = 3 A.
2 10 5

1
Pot. diff. across A and B 3
10 103 10V .
10 25V
32. (B) H = I2Rt = mc I2 R AB R AD 5 5
2 2 R BC R DC 5 5
2 I2 2 3
Hence or Resistance of arm ABC = 5 + 5 = 10
1 I1 3
Resistance of arm ADC = 5 + 5 = 10
or 2
= 9 × 3 = 27°C. The effective resistance between A and C is
33. (A) According to positive Thomson’s effect the potential 1 1 1
at a point of lower temperature is lower than at the or R AC 5
R AC 10 10
point at higher temperature which is at higher
potential. When current is passed from lower Now the equivalent circuit becomes
temperature to higher temperature of a metal, work is The equivalent resistance of the circuit
to be done, hence energy is absorbed. 1 1 1
or R 2.5
I H E D A R 5 5
V 25
10V 5V 20V 30V 25V Current in the circuit, I = = = 10A .
R 2.5
5
i4 i3 i2 i1

5 10 5 11
34. (C)
I 5
J G F C B
25V
i1 + i2 + i3 + i4 37. (D) Here, P = 100 W, t = 10 min = 60 s
Applying KVL in loop ABCDA, ABFEA, ABGHA Heat developed in time t
and ABJIA, we get H = P × t = (100 W)(60 s) = 6000 J.
30 – i1 × 11 = –25 ... (i) 38. (B)
20 + i2 × 5 = 25 ...(ii) R R
5 – i3 × 10 = –25 ...(iii) A B
10 + i4 × 5 = 25 ...(iv)
Solving equations (i), (ii), (iii) and (iv) we get i1 = 5A, R R
i2 = 1A, i3 = 3A and i4 = 3A.
Hence current flowing through 25 V cell is 12A. Equivalent circuit,
36. (A) It is a balanced Wheatstone bridge. Hence no current R R
flows through resistance of arm BD and the resistance A B
of arm BD is ineffective.
R R

A B
R
1 1 1 Exercise - IV
39. (A) Here, R1 , R2 ; R3
2 4 6 1. (D) Resistivity of a semiconductor decreases with the
1 1 1 1 temperature. The atoms of a semiconductor vibrate
Inparallel; 2 4 6 12 with larger amplitudes at higher temperatures thereby
RP R1 R2 R3
increasing it’s conductivity not resistivity.
1 2. (D) It is quite clear that in a battery circuit, the point of
Or equivalent conductance, G 12 S
RP lowest potential is the negative terminal of the
40. (A) Given circuit is a balanced is a Wheatstone bridge. battery and the current flows from higher potential
So circuit becomes to lower potential.
50 50 3. (B) The temperature co-efficient of resistance for metal
is positive and that for semiconductor is negative.
In metals free electrons (negative charge) are charge
A B carriers while in P-type semiconductors, holes
50 50 (positive charge) are majority charge carriers.
2
Equivalent resistance, 4. (A) Here, E = 2V, 1 1A and r = 1
2
50 50 50 50 100 100
R 50 Therefore, V = E – ir = 2 – 1 × 1 = 1V
50 50 50 50 200 5. (A) It is clear that electrons move in all directions
41. (C) Joule heating in a wire, haphazardly in metals. When an electric field is
h = I2 Rt applied, each free electron acquire a drift velocity.
For given R and t, H I2 There is a net flow of charge, which constitute
H1 I12 current. In the absence of electric field this is
(i) impossible and hence, there is no current.
H2 I 22
6. (C) The metallic body of the electrical appliances is
Also, H = ms T connected to the third pin which is connected to the
For given m and s, H T
earth. This is a safety precaution and avoids eventual
H1 T1 electric shock. By doing this the extra charge flowing
H2 T2 (ii) through the metallic body is passed to earth and
For equation (i) and (ii) avoid shocks. There is nothing such as reducing of
the heating of connecting wires by three pin
I12 T1 connections.
I 22 T2 7. (B) On increasing temperature of wire the kinetic energy
Here, I1 = I, T1 = 5°C, I2 = 3I, T2 = ? of free electrons increase and so they collide more
2 rapidly with each other and hence their drift velocity
I 5 decreases. Also when temperature increases,
or T2 = 45°C
3I T2 resistivity increase and resistivity is inversely
42. (B) Net resistance = 10r proportional to conductivity of material.
Net potential = 10E 8. (C) In a conductor there are large number of free electrons.
Current in the circuit When we close the circuit, the electric field is
10E E established instantly with the speed of
I
10r r electromagnetic wave which cause electron drift at
Potential difference across three cells every portion of the circuit. Due to which the current
= I × 3r is set up in the entire circuit instantly. The current
E which is set up does not wait for the electrons flow
3r 3E from one end of the conductor to the another end. It
r
is due to this reason, the electric bulb glows
hc hc immediately when switch is on.
45. (D) K max W 5.01
l
9. (A) Resistance wire R . where is resistivity of
12375 A
5.01 hc 12375
inÅ) material which does not depend on the geometry of
wire. Since when wire is banded, resistivity, length
12375 and area of cross-section do not change, therefore
5.01 6.1875 5.01
2000 resistance of wire also remain same.
1.17775 1.2V
10. (C) The resistance of the galvanometer is fixed. In meter the electrical circuit is completed through the
bridge experiments, to protect the galvanometer from ground. The hands of man are at high potential and
a high current, high resistance is connected to the his feets are at low potential. Hence large amount of
galvanometer in order to protect it from damage. current flows through the body of the man and person
11. (A) The third pin is used for grounding purposesso that there fore gets a fatal shock.
it leaves the user safe whi1ehandling the appliance 21. (A) Voltameter measures current indirectly in terms of
by making the extra charge on it getdischarged. mass of ions deposited an d electrochemical
12. (A) Metals are good conductors of electricity. It is m
equivalent of the substance I . Since value
because of the presence of a large number of free Zt
electrons in metals. And for metals electrons are the of m and Z are measured to 3rd decimal place and 5th
main cause for thermal conduction. That’s why all decimal place respectively. The relative error in the
good conductors of heat are also good conductors measurement of current by voltmeter will be very
of electricity. small as compared to that when measured by ammeter
directly.
13. (C) In case the circuit inside becomes faulty, an electric
22. (A) When current flows through a conductor it always
appliance might end with some charge. Because of
remains uncharged, hence no electric field is
which on touching the appliance, the user might get
produced outside it.
a shock. The third pin provided is for grounding
purpose so that all accumulated charges might get 23. (B) Here assertion and reason both are correct but the
discharged to the ground and the appliance remains reason is not the correct explanation of assertion.
safe. Now appliances like heater, where there can be 24. (A) Sensitivity
more charge accumulation than the electric bulb, 1
grounding is a must. That’s why, electric appliances (Length of wire)
Potential gradient
with metallic body, such as heaters have three pin
connections. 25. (A) If either the e.m.f. of the driver cell or potential
difference across the whole potentiometer wire is
14. (D) In a battery circuit, the point of lowest potential is
lesser than the e.m.f. of the experimental cell, then
the negative terminal of the battery. And the current
balance point will not obtained.
flows from higher potential to lower potential.
26. (D) Because there is no special attractive force that keeps
15. (D) The electrical force of a cell or battery called electro
a person stuck with a high power line. The actual
motive force (emf). This force, which makes electrons
reason is that a current of the order of 0.05 A or even
flow around a circuit. Each kind of cell has a particular
less is enough to bring disorder in our nervous
emf. The size of a cell has no effect on its emf. The
system. As a result of it, the affected person may
chemicals in the cell determines its emf, but large
lose temporarily his ability to exercise his nervous
cells last longer than small cells of same basic type.
control to get himself free from the high power line.
This means that from a large cell,we get a particular
emf for more time as compared to small cell. 27. (A) Due to high electrical conductivity of copper, it
conducts th e current without offering much
16. (B) The assertion is true. Even when the emf is switched
resistance. The copper being diamagnetic material
off, the current continues to flow because its
does not get magnetised due to current through it
resistance becomes zero. The Meissner effect is
and hence does not disturb the current in the circuit.
another property of the super conductor and not the
28. (C) As P = Vi, hence for the transmission of same power,
reason for the assertion. Meissner effect repels the
high voltage implies less current. There fore heat
magnetic field lines from the interior of the super
Rt
conductor. energy loses (H i 2 ) are minimized if power is
17. (D) Both the assertion and reason are false. Ohm’s law is 4.2
transmitted at high voltage.
obeyed by metals for a certain range of temperature,
not obeyed by super conductors, valves, diodes and 30. (A) The sum of all the currents directed towards a point
semiconductors. It is not auniversal law but it is in circuit is equal to the sum of all the currents directed
purely empirical. away from that point. It is based on conservation of
electrical energy.
18. (A) V = E – I(R + r)
31. (A) The resistance of that half portion of the metallic
20. (C) Electric shock is due to the electric current flowing
wire ( heated red hot) on which cold water is poured
through a living body. When the bird perches on a
decreases due to fall in its temperature. This results
single high power line, no current passes through
in a decreases in the resistance of the entire wire. As
its body because its body is atequipotential surface
a result, the current in the wire increases and the
i.e., there is no potential difference. While when man
other half portion becomes hotter than before.
touches the same line, standing bare foot on ground

You might also like